Anda di halaman 1dari 390

Table of Contents

1. THE APPLICABLE LAWS .................................................................................................................. 1


2. BASIC PRINCIPLES ............................................................................................................................ 1
CHARLIE JAO
VS. BCC PRODUCTS SALES INC., AND TERRANCE TY ......................................... 1
LEGEND HOTEL (MANILA) VS HERNANI S. REALUYO .................................................................... 4
THE NEW PHILIPPINE SKYLANDERS, INC. VS FRANCISCO N. DAKILA...................................... 7
3. RIGHT TO HIRE .................................................................................................................................. 9
4. WAGES & WAGE RATIONALIZATION ACT................................................................................. 9
ROYAL PLANT WORKERS UNION VS COCA-COLA BOTTLERS PHILIPPINES, INC.-
CEBU PLANT. ................................................................................................................................................ 9
RICARDO E. VERGARA, JR. VS. COCA-COLA BOTTLERS PHILIPPINES, INC. .......................... 13
5. VIOLATION OF WAGE ORDERS ................................................................................................... 14
6. WAGE ENFORCEMENT AND RECOVERY .................................................................................. 14
PEOPLE’S BROADCASTING (BOMBO RADYO PHILS.) VS. SECRETARY OF DOLE ............... 14
SUPERIOR PACKAGING CORPORATION VS. ARNEL BALAGSAY, ET AL. ................................ 17
7. WAGE PROTECTION PROVISIONS & PROHIBITIONS REGARDING WAGES ................. 18
NIÑA JEWELRY MANUFACTURING OF METAL ARTS, INC. VS MONTECILLO ....................... 18
8. PAYMENT OF WAGES ..................................................................................................................... 21
9. CONDITIONS OF EMPLOYMENT ................................................................................................. 21
10. MINIMUM LABOR STANDARD BENEFITS ............................................................................. 21
RADIO MINDANAO NETWORK, INC. AND ERIC S. CANOY, PETITIONERS, VS.
DOMINGO Z. YBAROLA, JR. AND ALFONSO E. RIVERA, JR., RESPONDENTS. ........................ 21
11. OTHER SPECIAL BENEFITS ........................................................................................................ 22
RADIO MINDANAO NETWORK, INC. AND ERIC S. CANOY VS. DOMINGO Z.
YBAROLA, JR. AND ALFONSO E. RIVERA, JR. . ................................................................................. 23
ELEAZAR S. PADILLO VS. RURAL BANK OF NABUNTURAN, INC. AND MARK S.
OROPEZA .................................................................................................................................................... 24
12. RIGHT TO SECURITY OF TENURE............................................................................................ 27
JAIME N. GAPAYAO VS. ROSARIO FULO, SOCIAL SECURITY SYSTEM AND SOCIAL
SECURITY COMMISSION ........................................................................................................................ 27
CONCRETE SOLUTIONS, INC, ET AL VS ARTHUR CABUSAS ....................................................... 30
D.M. CONSUNJI CORPORATION VS ROGELIO P. BELLO ............................................................... 33
COLEGIO DEL SANTISIMO ROSARIO AND SR. ZANAIDA S. MOFADA, OP VS.
EMMANUEL ROJO .................................................................................................................................... 35
13. MANAGEMENT PREROGATIVE................................................................................................. 37

1
BARBA VS LICEO DE CAGAYAN UNIVERSITY.................................................................................. 37
BEST WEAR GARMENTS AND/OR WARREN PARDILLA VS ADELAIDA B. DE
LEMOS AND CECILE M. OCUBILLO...................................................................................................... 40
ROYAL PLANT WORKERS UNION VS COCA-COLA BOTTLERS PHILIPPINES, INC.-
CEBU PLANT .............................................................................................................................................. 42
PECKSON VS ROBINSONS ...................................................................................................................... 43
14. TERMINATION OF EMPLOYMENT ........................................................................................... 45
RAMIREZ ET.ALVS MAR FISHING CO. INC. ....................................................................................... 45
PRUDENTIAL GUARANTEE AND ASSURANCE EMPLOYEE LABOR UNION AND
SANDY T. VALLOTA VS NLRC ............................................................................................................... 47
ROMEO E. PAULINO VS NLRC ............................................................................................................... 51
EVER ELECTRICAL MANUFACTURING, INC., (EEMI) VS. SAMAHANG
MANGGAGAWA NG EVER ELECTRICAL............................................................................................. 53
WATERFRONT CEBU CITY HOTEL VS. JIMENEZ. ........................................................................... 57
MANILA ELECTRIC COMPANY (MERALCO) VS. HERMINIGILDO H. DEJAN ........................... 61
COSMOS BOTTLING VS FERMIN.......................................................................................................... 65
REYES-RAYEL VS. PHILIPPINE LUEN THAI HOLDINGS ............................................................... 67
VERDADERO VS. BARNEY AUTOLINES GROUP OF COMPANIES TRANSPORT, INC., ......... 72
ALEX NARANJO, DONNALYN DE GUZMAN. RONALD V. CRUZ, ROSEMARIE P.
PIMENTEL AND ROWENA B. BARDAJE V.S. BIOMEDICAL HEALTH CARE, INC. AND
KAREN J. MOTOL ...................................................................................................................................... 75
THE NEW PHILIPPINE SKYLANDERS INC V.S DAKILA ................................................................ 85
MORALES VS METROPOLITAN BANK & TRUST CO. ..................................................................... 87
MIRANT PHILIPPINES CORPORATION VS DANILO SARIO......................................................... 90
MA. MERCEDES L. BARBA VS. LICEO DE CAGAYAN UNIVERSITY ............................................. 93
SAMPAGUITA AUTO TRANSPORT CORPORATION VS. NATIONAL LABOR
RELATIONS COMMISSION AND EFREN I. SAGAD .......................................................................... 97
SAMPAGUITA AUTO TRANSPORT CORPORATION VS. NLRC .................................................... 99
GENERAL MILLING CORPORATION VS. VIAJAR ........................................................................... 101
JONATHAN SANG-AN VS EQUATORN KNIGHTS DETECTIVE AND SECURITY
AGENCY, INC. ........................................................................................................................................... 103
LEOPARD SECURITY AND INVESTIGATION AGENCY VS. TOMAS QUITOY, RAUL
SABANG AND DIEGO MORALES......................................................................................................... 105
PHILIPPINE PLAZA HOLDINGS, INC., PETITIONER, VS. MA. FLORA M.
EPISCOPE, RESPONDENT .................................................................................................................... 107
ROLANDO DS.TORRES VS. RURAL BANK OF SAN JUAN, INC., ANDRES CANO
CHUA, JOBEL GO CHUA, JESUS CANO CHUA, MEINRADO DALISAY, JOSE
MANALANSAN III, OFELIA GINA BE AND NATY ASTRERO ....................................................... 111
THE ORCHARD GOLF AND COUNTRY CLUB VS. AMELIA R. FRANCISCO ............................. 115
ALEXANDER B. BAÑARES VS. TABACO WOMEN’S TRANSPORT SERVICE
COOPERATIVE (TAWTRASCO), ET. AL. .......................................................................................... 116
REYES, ET AL. VS. RP GUARDIANS SECURITY AGENCY, INC. ................................................... 117
CELDRAN VS. FORZA INTEGRATED SERVICES, ET AL. .............................................................. 119

2
SURIGAO DEL NORTE ELECTRIC COOPERATIVE, INC., AND/OR DANNY Z.
ESCALANTE
VS. TEOFILO GONZAGA............................................................................................. 121
UNIVAC DEVELOPMENT, INC., VS. WILLIAM M. SORIANO. ...................................................... 126
UNILEVER PHILS VS RIVERA.............................................................................................................. 128
SAMAR-MED DISTRIBUTION VS NLRC ........................................................................................... 131
ALEX Q. NARANJO, DONNALYN DE GUZMAN, RONALD V. CRUZ, ROSEMARIE P.
PIMENTEL, AND ROWENA B. BARDAJE VS BIOMEDICA HEALTH CARE, INC. AND
CARINA "KAREN" J. MOTOL, RESPONDENTS. ................................................................................ 133
MANILA JOCKEY CLUB, INC., VS AIMEE O. TRAJANO. ................................................................. 141
ALFONSO L. FIANZA VS. NATIONAL LABOR RELATIONS COMMISSION (SECOND
DIVISION), BINGA HYDROELECTRIC PLANT, INC., ANTHONY C. ESCOLAR,
ROLAND M. LAUTCHANG .................................................................................................................... 147
ROY D. PASOS VS PHILIPPINE NATIONAL CONSTRUCTION CORPORATION. .................... 148
UNIVERSAL ROBINA CORPORATION AND LANCE Y. GOKONGWEI VS. WILFREDO
Z. CASTILLO. ............................................................................................................................................ 150
JESSIE G. MARTINEZ VS.
CENTRAL PANGASINAN ELECTRIC COOPERATIVE, INC.
(CENPELCO).............................................................................................................................................. 153
ZUELLIG PHARMA CORPORATION VS.
ALICE M. SIBAL. ET. AL ............................................ 155
ZUELLIG FREIGHT AND CARGO SYSTEMS VS. NATIONAL LABOR RELATIONS
COMMISSION AND RONALDO V. SAN MIGUEL. ............................................................................. 157
ABBOTT LABORATORIES, PHILS, ET. AL VS. PEARLIE ANN F. ALCARAZ ............................. 159
MANILA POLO CLUB EMPLOYEES' UNION (MPCEU) FUR-TUCP VS. MANILA POLO
CLUB, INC.,................................................................................................................................................. 165
LUCIANO P. CANEDO VS.
KAMPILAN SECURITY AND DETECTIVE AGENCY, INC.
AND RAMONCITO L. ARQUIZA. .......................................................................................................... 170
VICENTE ANG, VS.
CEFERINO SAN JOAQUIN, JR., AND DIOSDADO FERNANDEZ. ............ 171
SANOH FULTON PHILS., INC. AND MR. EDDIE JOSE VS. EMMANUEL BERNARDO
AND SAMUEL TAGHOY ......................................................................................................................... 174
JEROME M. DAABAY VS. COCA-COLA BOTTLERS PHILS., INC.. ................................................ 176
MZR INDUSTRIES, MARILOU R. QUIROZ AND LEA TIMBAL VS. MAJEN COLAMBOT ....... 178
INTEGRATED MICROELECTRONICS INC. VS. ADONIS A. PIONILLA ....................................... 179
ASIA BREWERY, INC. VS TUNAY NA PAGKAKAISA NG MGA MANGGAGAWA SA
ASIA (TPMA). ........................................................................................................................................... 181
15 SUSPENSION OF BUSINESS OPERATION ............................................................................. 183
MINDANAO TERMINAL & BROKERAGE SERVICE ET. AL. VS NAGKAHIUSANG
MAMUMUO SA MINTERBRO-SOURTHERN PHILS FED. OF LABOR ....................................... 183
LEOPARD SECURITY AND INVESTIGATION AGENCY VS QUITOY ......................................... 185
16 DISEASE AS GROUND FOR TERMINATION ......................................................................... 186
17 OTHER CAUSES OF SEVERANCE OF EMPLOYMENT RELATION ................................... 186
18 PRESCRIPTION OF CLAIMS ...................................................................................................... 186
19JURISDICTION OF THE LABOR ARBITER ............................................................................. 186

3
PORTILLO VS RUDOLF LIETZ ............................................................................................................ 186
ACE NAVIGATION CO., INC., VELA INTERNATIONAL MARINE LTD., AND/OR
RODOLFO PAMINTUAN VS. TEODORICO FERNANDEZ, ASSISTED BY GLENITA
FERNANDEZ. ............................................................................................................................................ 189
20 . 2011 NLRC RULES OF PROCEDURE .................................................................................... 193
RODOLFO LUNA VS. ALLADO CONSTRUCTION CO., INC., AND/OR RAMON
ALLADO. .................................................................................................................................................... 193
BANAHAW BROADCASTING CORPORATION VS. PACANA III, ET. AL................................... 196
SOCIAL SECURITY COMMISSION VS. RIZAL POULTRY AND LIVESTOCK
ASSOCIATION, INC., BSD AGRAO INDUSTRIAL DEVELOPMENT CORPORATION
AND BENJAMIN SAN DIEGO ................................................................................................................ 197
UNIVERSITY PLANS INC. VS. SOLANO ............................................................................................. 199
BPI EMPLOYEES UNION-METRO MANILA VS. BANK OF THE PHIL. ISLANDS .................... 202
DUP SOUND PHILS. AND/OR MANUEL TAN VS. COURT OF APPEALS AND CIRILO
A. PIAL. ...................................................................................................................................................... 206
HYPTE R. AUJERO VS. PHILIPPINE COMMUNICATIONS SATELLITE
CORPORATION. ...................................................................................................................................... 208
TIMOTEO H. SARONA VS. NATIONAL LABOR RELATIONS COMMISSION, ROYALE
SECURITY AGENCY AND CESAR S. TAN........................................................................................... 211
SALENGA, ET. AL. VS. COURT OF APPEALS.................................................................................... 212
LOCKHEED DETECTIVE AND WATCHMAN AGENCY, INC. VS. UNIVERSITY OF THE
PHILIPPINES............................................................................................................................................ 214
3RD ALERT SECURITY & DETECTIVE SERVICES INC. VS. NAVIA.............................................. 218
ESTATE OF NELSON R. DULAY, REPRESENTED BY HIS WIFE MERRIDY JANE P.
DULAY VS. ABOITIZ JEBSEN MARITIME, INC. AND GENERAL CHARTERERS, INC.. .......... 220
PRUDENTIAL GUARANTEE AND ASSURANCE EMPLOYEE LABOR UNION AND
SANDY T. VALLOTA VS. NATIONAL LABOR RELATIONS COMMISSION,
PRUDENTIAL GUARANTEE AND ASSURANCE INC., AND/OR JOCELYN RETIZOS ............. 223
RADIO PHILIPPINES NETWORK INC. ET AL. VS. YAP ET AL. ................................................... 227
ACE NAVIGATION CO. INC. VS. FERNANDEZ, ET AL. ................................................................... 229
PORTILLO VS. RUDOLF LIETZ, INC. ................................................................................................. 233
GONZALES VS. SOLID CEMENT CORPORATION AND ALLEN QUERUBIN ............................ 236
FELIX MARTOS, ET AL VS. NEW SAN JOSE BUILDERS, INC. ...................................................... 238
BUILDING CARE CORPORATION VS. MACARAEG........................................................................ 242
KAPISANANG PANGKAUNLARAN NG KANANAIHANG POTRERO, INC. VS.
BARRENO ET AL. .................................................................................................................................... 244
PHILIPPINE TRANSMARINE CARRIERS, INC. VS. LEANDRO LEGASPI .................................. 245
PASOS VS. PHIL. NATIONAL CONSTRUCTION CORP. ................................................................. 248
DARIO NACAR VS. GALLERY FRAMES AND/OR FELIPE BORDEY, JR. ................................... 252
MARIA LOURDES D. CASTELLS AND SHALIMAR CENTI-MANDANAS VS. SAUDI
ARABIAN AIRLINES ............................................................................................................................... 254
DONGON VS. RAPID MOVERS AND FORWARDERS CO INC., ET AL. ....................................... 256

4
MALAYANG MANGGAGAWA NG STAYFAST PHILS., INC. VS. NATIONAL
RELATIONS LABOR COMMISSION, STAYFAST PHILIPPINES, INC./ MARIA
ALMEIDA................................................................................................................................................... 259
HERNANDO BORRA, ET AL. VS. COURT OF APPEALS SECOND AND NINETEENTH
DIVISIONS AND HAWAIIAN PHILIPPINE COMPANY .................................................................. 261
21. RIGHT TO SELF-ORGANIZATION .......................................................................................... 264
NATIONAL UNION OF BANK EMPLOYEES (NUBE) VS. PHILNABANK EMPLOYEES
ASSOCIATION (PEMA) AND PHILIPPINE NATIONAL BANK .................................................... 265
CARLOS L. OCTAVIO VS. PHILIPPINE LONG DISTANCE TELEPHONE COMPANY.............. 269
22. RIGHTS OF LEGITIMATE LABOR ORGANIZATION .......................................................... 271
DIGITAL TELECOMMUNICATIONS PHILIPPINES, INC. VS. DIGITEL EMPLOYEES
UNION (DEU), ARCELO RAFAEL A. ESPLANA, ALAN D. LICANDO, FELICITO C.
ROMERO, JR., ARNOLD D. GONZALES, REYNEL FRANCISCO B. GARCIA, ZOSIMO B.
PERALTA, REGINO T. UNIDAD AND JIM L. JAVIER. ..................................................................... 271
AUTOMOTIVE ENGINE REBUILDERS VS. PROGRESIBONG UNYON NG MGA
MANGGAGAWA SA AER RESOLUTION ON THE MAIN DECISION OF JULY 13, 2011............ 273
HOLY CHILD CATHOLIC SCHOOL VS. HON. STO. TOMAS, ET AL. ........................................... 276
23. REVISED GUIDELINES OF THE NCMB FOR THE CONDUCT OF VOLUNTARY
ARBITRATION PROCEEDINGS ...................................................................................................... 279
7K CORPORATION VS. ALBARICO .................................................................................................... 279
LEPANTO CONSOLIDATED MINING COMPANY VS LEPANTO ................................................. 283
24. UNFAIR LABOR PRACTICE ...................................................................................................... 286
GOYA INC. VS. GOYA INC. EMPLOYEES UNION-FFW ................................................................... 286
PARK HOTEL, ET AL. VS. SORIANO ET AL., .................................................................................... 290
MINETTE BAPTISTA, BANNIE EDSEL SAN MIGUEL, AND MA. FEDAYON VS.
ROSARIO VILLANUEVA, JANETTE ROLDAN, DANILO OLAYVAR, ONOFRE
ESTRELLA, CATALINO LEDDA, MANOLO GUBANGCO, GILBERT ORIBIANA,
CONSTANCIO SANTIAGO, RUTH BAYQUEN, RUBY CASTANEDA, ALFRED LANDAS,
JR., ROSELYN GARCES, EUGENE CRUZ, MENANDRO SAMSON, FEDERICO MUNOZ
AND SALVADOR DIWA. ........................................................................................................................ 292
25. OTHER IMPORTANT LABOR PROVISIONS ........................................................................ 297
A. CONTRACTING ARRANGEMENT.............................................................................................. 297
MARIALY O. SY, ET AL VS. FAIRLAND KNITCRAFT CO., INC.. .................................................. 297
SUSAN T. DE LEON VS. FAIRLAND KNITCRAFT CO., INC., ET AL............................................. 297
POLYFOAM-RGC INTERNATIONAL, CORPORATION AND PRECILLA A. GRAMAJE
VS EDGARDO CONCEPCION, ............................................................................................................... 305
SUPERIOR PACKAGING CORP. VS BALAGSAY .............................................................................. 306
DIGITAL TELECOMMUNICATIONS PHILIPPINES, INC. VS. DIGITEL EMPLOYEES
UNION (DEU), ARCEO RAFAEL A. ESPLANA, ALAN D. LICANDO, FELICITO C.

5
ROMERO, JR., ARNOLD D. GONZALES, REYNEL FRANCISCO B. GARCIA, ZOSIMO B.
PERALTA, REGINO T. UNIDAD AND JIM L. JAVIER . .................................................................... 308
NORKIS TRADING CORPORATION VS. JOAQUIN BUENAVISTA, HENRY FABROA,
RICARDO CAPE, BERTULDO TULOD, WILLY DONDOYANO AND GLEN
VILLARIASA. ............................................................................................................................................ 311
GOYA, INC. VS GOYA, INC. EMPLOYEES UNION-FFW .................................................................. 313
BENIGNO M. VIGILLA, ET AL VS. PHILIPPINE COLLEGE OF CRIMINOLOGY INC.
AND/OR GREGORY ALAN F. BAUTISTA .......................................................................................... 316
BPI EMPLOYEES UNION-DAVAO CITY-FUBU (BPIEU-DAVAO CITY-FUBU) VS.
BANK OF THE PHILIPPINE ISLANDS (BPI), AND BPI OFFICERS CLARO M. REYES,
CECIL CONANAN AND GEMMA VELEZ. ............................................................................................ 320
B. WORKER’S PREFERENCE ........................................................................................................... 324
C. ATTORNEY’S FEES & APPEARANCE OF LAWYERS ............................................................ 324
KAISAHAN AT KAPATIRAN NG MGA MANGGAGAWA AT KAWANI SA MWC-EAST
ZONE UNION AND EDUARDO BORELA, REPRESENTING ITS MEMBERS VS MANILA
WATER COMPANY, INC.. ...................................................................................................................... 324
CZARINA T. MALVAR VS. KRAFT FOOD PHILS., INC. AND/OR BIENVENIDO
BAUTISTA, KRAFT FOODS INTERNATIONAL ............................................................................... 328
26. MISCELLANEOUS PROVISIONS .............................................................................................. 331
A. SPECIAL TYPES OF WORKERS ................................................................................................. 331
B. EMPLOYMENT OF WOMEN ....................................................................................................... 331
C. EMPLOYMENT OF CHILDREN ................................................................................................... 331
D. EMPLOYMENT OF HOUSEHELPER ......................................................................................... 331
FERNANDO CO (FORMERLY DOING BUSINESS UNDER THE NAME “NATHANIEL
MAMI HOUSE”)VS. LINA B. VARGAS ................................................................................................. 331
E. EMPLOYMENT OF HOMEWORKERS ....................................................................................... 334
F. EMPLOYMENT OF NON-RESIDENT ALIENS.......................................................................... 334
G. EMPLOYMENT OF STUDENTS & WORKING SCHOLAR..................................................... 334
H. EMPLOYMENT OF ACADEMIC/NON-ACADEMIC PERSONNEL IN PRIVATE
EDUCATIONAL INSTITUTION ....................................................................................................... 334
COLEGIO DEL SANTISIMO ROSARIO, ET. AL. VS. ROJO.............................................................. 334
CITING MERCADO, ET.AL. VS AMA- PARANAQUE CITY GR. NO. 183572, APRIL 13,
2010 ........................................................................................................................................................... 334
UNIVERSITY OF THE EAST VS. PEPANIO ....................................................................................... 336
I. MEDICAL, DENTAL AND OCCUPATIONAL SAFETY (ARTS. 156 TO 165)..................... 338
J. MIGRANT WORKER’S ACT/ RECRUITMENT AND PLACEMENT ..................................... 338

6
CLAUDIO S. YAP VS. THENAMARIS SHIP'S MANAGEMENT AND INTERMARE
MARITIME AGENCIES, INC. ................................................................................................................. 338
SKIPPERS UNITED PACIFIC, INC. AND SKIPPERS MARITIME SERVICES, INC., LTD.
VS. NATHANIEL DOZA, NAPOLEON DE GRACIA, ISIDRO L. LATA, AND CHARLIE
APROSTA .................................................................................................................................................. 339
PERT/CPM MANPOWER EXPONENT CO. VS VINUYA ET. AL ................................................... 343
INTERNATIONAL MANAGEMENT SERVICES/MARILYN PASCUAL VS ROEL
LOGARTA .................................................................................................................................................. 347
HON. STO. TOMAS VS. SALAC ............................................................................................................. 350
27. SOCIAL LEGISLATION ............................................................................................................... 352
SOCIAL SECURITY COMMISSION VS. FAR S. ALBA ......................................................................... 352
RODRIN VS GSIS ET AL ............................................................................................................................ 354
GSIS VS CASCO ........................................................................................................................................... 357
SSS VS. GLORIA DE LOS SANTOS .......................................................................................................... 358
BECMEN SERVICE EXPORTER AND PROMOTION, INC. VS. SPOUSES SIMPLICIO AND
MILA CUARESMA (FOR AND IN BEHALF OF THEIR DAUGHTER, JASMIN G.
CUARESMA), WHITE FALCON SERVICES, INC. AND JAIME ORTIZ
(PRESIDENT,WHITE FALCON SERVICES, INC.). .............................................................................. 360
GREAT SOUTHERN MARITIME SERVICES CORP. AND IMC SHIPPING CP., PTE LTD
VS. LEONILA SURIGAO ............................................................................................................................ 363
GOVERNMENT SERVICE INSURANCE SYSTEM VS. SALVADOR DE CASTO ............................. 366
KESTREL SHIPPING CO., INC./ CAPT. AMADOR P. SERVILLON AND ATLANTIC
MANNING LTD. VS. FRANCISCO D. MUNAR. ..................................................................................... 370
SUSANA R. SY VS. PHILIPPINE TRANSMARINE CARRIERS, INC., AND/OR SSC SHIP
MANAGEMENT PTE., LTD.. .................................................................................................................... 372
PHILIPPINE JOURNALISTS, INC. VS. JOURNAL EMPLOYEES UNION (JEU), FOR ITS
UNION MEMBER, MICHAEL ALFANTE. .............................................................................................. 374
RAMON G. NAZARENO VS. MAERSK FILIPINAS CREWING INC., AND ELITE
SHIPPING A/S............................................................................................................................................. 376
MITSUBISHI MOTORS PHILIPPINES SALARIED EMPLOYEES UNION (MMPSEU) VS.
MITSUBISHI MOTORS PHILIPPINES INCORPORATED ................................................................. 378
PHILMAN MARINE AGENCY, INC. (NOW DOHLE-PHILMAN MANNING AGENCY, INC.)
AND/OR DOHLE (10M) LIMITED VS. ARMANDO S. CABANBAN, RESPONDENT.................. 379

7
1. THE APPLICABLE LAWS

2. BASIC PRINCIPLES

G.R. No. 163700 April 18, 2012

CHARLIE JAO, PETITIONER, 



VS.

BCC PRODUCTS SALES INC., AND TERRANCE TY, RESPONDENTS.

Facts:

Petitioner maintained that respondent BCC Product Sales Inc. (BCC) and its President, respondent
Terrance Ty (Ty), employed him as comptroller starting from September 1995 with a monthly
salary of P20,000.00 to handle the financial aspect of BCC’s business;2 that on October 19,1995, the
security guards of BCC, acting upon the instruction of Ty, barred him from entering the premises of
BCC where he then worked; that his attempts to report to work in November and December 12,
1995 were frustrated because he continued to be barred from entering the premises of BCC;3 and
that he filed a complaint dated December 28, 1995 for illegal dismissal, reinstatement with full
backwages, non-payment of wages, damages and attorney’s fees.4
Respondents countered that petitioner was not their employee but the employee of Sobien Food
Corporation (SFC), the major creditor and supplier of BCC; and that SFC had posted him as its
comptroller in BCC to oversee BCC’s finances and business operations and to look after SFC’s
interests or investments in BCC.
The labor arbiter rendered decision on favor of BCC however later on reversed by NLRC. BCC
appealed to the Court of Appeals, which rendered a decision on their favor on the basis that there is
no employer-employee relationship.

Issue:
The sole issue is whether or not an employer-employee relationship existed between petitioner and
BCC. A finding on the existence of an employer-employee relationship will automatically warrant a
finding of illegal dismissal, considering that respondents did not state any valid grounds to dismiss
petitioner.

Ruling:
The petition lacks merit.
The existence of an employer-employee relationship is a question of fact.
To prove his employment with BCC, petitioner offered the following: (a) BCC Identification Card

1
(ID) issued to him stating his name and his position as "comptroller," and bearing his picture, his
signature, and the signature of Ty; (b) a payroll of BCC for the period of October 1-15, 1996 that
petitioner approved as comptroller; (c) various bills and receipts related to expenditures of BCC
bearing the signature of petitioner; (d) various checks carrying the signatures of petitioner and Ty,
and, in some checks, the signature of petitioner alone; (e) a court order showing that the issuing
court considered petitioner’s ID as proof of his employment with BCC; (f) a letter of petitioner dated
March 1, 1997 to the Department of Justice on his filing of a criminal case for estafa against Ty for
non-payment of wages; (g) affidavits of some employees of BCC attesting that petitioner was their
co-employee in BCC; and (h) a notice of raffle dated December 5, 1995 showing that petitioner,
being an employee of BCC, received the notice of raffle in behalf of BCC.
Respondents denied that petitioner was BCC’s employee. They affirmed that SFC had installed
petitioner as its comptroller in BCC to oversee and supervise SFC’s collections and the account of
BCC to protect SFC’s interest; that their issuance of the ID to petitioner was only for the purpose of
facilitating his entry into the BCC premises in relation to his work of overseeing the financial
operations of BCC for SFC; that the ID should not be considered as evidence of petitioner’s
employment in BCC;19 that petitioner executed an affidavit in March 1996,20 stating, among others,
as follows:

1. I am a CPA (Certified Public Accountant) by profession but presently associated with, or


employed by, Sobien Food Corporation;
2. 2. In the course of my association with, or employment by, Sobien Food Corporation (SFC,
for short), I have been entrusted by my employer to oversee and supervise collections on account of
receivables due SFC from its customers or clients; for instance, certain checks due and turned over
by one of SFC’s customers is BCC Product Sales, Inc., operated or run by one Terrance L. Ty,
(President and General manager), pursuant to, or in accordance with, arrangements or agreement
thereon;

Our perusal of the affidavit of petitioner compels a conclusion similar to that reached by the CA and
the Labor Arbiter to the effect that the affidavit actually supported the contention that petitioner
had really worked in BCC as SFC’s representative. It does seem more natural and more believable
that petitioner’s affidavit was referring to his employment by SFC even while he was reporting to
BCC as a comptroller in behalf of SFC. As respondents pointed out, it was implausible for SFC to still
post him to oversee and supervise the collections of accounts receivables due from BCC beyond
December 1995 if, as he insisted, BCC had already illegally dismissed him and had even prevented
him from entering the premises of BCC. Given the patent animosity and strained relations between
him and respondents in such circumstances, indeed, how could he still efficiently perform in behalf
of SFC the essential responsibility to "oversee and supervise collections" at BCC? Surely,
respondents would have vigorously objected to any arrangement with SFC involving him.
We note that petitioner executed the affidavit in March 1996 to refute a statement Ty himself made
in his own affidavit dated December 11, 1995 to the effect that petitioner had illegally appropriated
some checks without authority from BCC.22 Petitioner thereby sought to show that he had the
authority to receive the checks pursuant to the arrangements between SFC and BCC. This showing
would aid in fending off the criminal charge respondents filed against him arising from his

2
mishandling of the checks. Naturally, the circumstances petitioner adverted to in his March 1996
affidavit concerned those occurring before December 11, 1995, the same period when he actually
worked as comptroller in BCC.
Moreover, in determining the presence or absence of an employer-employee relationship, the Court
has consistently looked for the following incidents, to wit: (a) the selection and engagement of the
employee; (b) the payment of wages; (c) the power of dismissal; and (d) the employer’s power to
control the employee on the means and methods by which the work is accomplished. The last
element, the so-called control test, is the most important element.24
Hereunder are some of the circumstances and incidents occurring while petitioner was supposedly
employed by BCC that debunked his claim against respondents.
It can be deduced from the March 1996 affidavit of petitioner that respondents challenged his
authority to deliver some 158 checks to SFC. Considering that he contested respondents’ challenge
by pointing to the existing arrangements between BCC and SFC, it should be clear that respondents
did not exercise the power of control over him, because he thereby acted for the benefit and in the
interest of SFC more than of BCC.
In addition, petitioner presented no document setting forth the terms of his employment by BCC.
The failure to present such agreement on terms of employment may be understandable and
expected if he was a common or ordinary laborer who would not jeopardize his employment by
demanding such document from the employer, but may not square well with his actual status as a
highly educated professional.
Moreover, in determining the presence or absence of an employer-employee relationship, the Court
has consistently looked for the following incidents, to wit: (a) the selection and engagement of the
employee; (b) the payment of wages; (c) the power of dismissal; and (d) the employer’s power to
control the employee on the means and methods by which the work is accomplished. The last
element, the so-called control test, is the most important element.24
Hereunder are some of the circumstances and incidents occurring while petitioner was supposedly
employed by BCC that debunked his claim against respondents.
It can be deduced from the March 1996 affidavit of petitioner that respondents challenged his
authority to deliver some 158 checks to SFC. Considering that he contested respondents’ challenge
by pointing to the existing arrangements between BCC and SFC, it should be clear that respondents
did not exercise the power of control over him, because he thereby acted for the benefit and in the
interest of SFC more than of BCC.
In addition, petitioner presented no document setting forth the terms of his employment by BCC.
The failure to present such agreement on terms of employment may be understandable and
expected if he was a common or ordinary laborer who would not jeopardize his employment by
demanding such document from the employer, but may not square well with his actual status as a
highly educated professional.

G.R. No. 153511 July 18, 2012

3
LEGEND HOTEL (MANILA), OWNED BY TITANIUM CORPORATION, AND/OR, NELSON NAPUD,
IN HIS CAPACITY AS THE PRESIDENT OF PETITIONER CORPORATION, PETITIONER, 

VS.

HERNANI S. REALUYO, ALSO KNOWN AS JOEY ROA, RESPONDENT.

Facts:
This labor case for illegal dismissal involves a pianist employed to perform in the restaurant of a
hotel. On August 9, 1999, respondent, whose stage name was Joey R. Roa, filed a complaint for
alleged unfair labor practice, constructive illegal dismissal, and the underpayment/nonpayment of
his premium pay for holidays, separation pay, service incentive leave pay, and 13th month pay. He
prayed for attorney's fees, moral damages off P100,000.00 and exemplary damages for
P100,000.00.

Respondent averred that he had worked as a pianist at the Legend Hotel’s Tanglaw Restaurant from
September 1992 with an initial rate of P400.00/night that was given to him after each night’s
performance; that his rate had increased to P750.00/night; and that during his employment, he
could not choose the time of performance, which had been fixed from 7:00 pm to 10:00 pm for
three to six times/week. He added that the Legend Hotel’s restaurant manager had required him to
conform with the venue’s motif; that he had been subjected to the rules on employees’
representation checks and chits, a privilege granted to other employees; that on July 9, 1999, the
management had notified him that as a cost-cutting measure his services as a pianist would no
longer be required effective July 30, 1999; that he disputed the excuse, insisting that Legend Hotel
had been lucratively operating as of the filing of his complaint; and that the loss of his employment
made him bring his complaint.2

In its defense, petitioner denied the existence of an employer-employee relationship with


respondent, insisting that he had been only a talent engaged to provide live music at Legend Hotel’s
Madison Coffee Shop for three hours/day on two days each week; and stated that the economic
crisis that had hit the country constrained management to dispense with his services.
Labor arbiter and NLRC ruled that there is no employer-employee relationship while the Court of
Appeals says there is.

Issues:

1. Whether or not, there is an employer-employee relationship.


2. if respondent was petitioner’s employee, whether he was validly terminated.

Ruling:

Employer-employee relationship existed between the parties. The issue of whether or not
an employer-employee relationship existed between petitioner and respondent is essentially a
question of fact.9 The factors that determine the issue include who has the power to select the
employee, who pays the employee’s wages, who has the power to dismiss the employee, and who

4
exercises control of the methods and results by which the work of the employee is accomplished. 10
Although no particular form of evidence is required to prove the existence of the relationship, and
any competent and relevant evidence to prove the relationship may be admitted,11 a finding that the
relationship exists must nonetheless rest on substantial evidence, which is that amount of relevant
evidence that a reasonable mind might accept as adequate to justify a conclusion.

A review of the circumstances reveals that respondent was, indeed, petitioner’s employee. He was
undeniably employed as a pianist in petitioner’s Madison Coffee Shop/Tanglaw Restaurant from
September 1992 until his services were terminated on July 9, 1999.

First of all, petitioner actually wielded the power of selection at the time it entered into the service
contract dated September 1, 1992 with respondent. This is true, notwithstanding petitioner’s
insistence that respondent had only offered his services to provide live music at petitioner’s
Tanglaw Restaurant, and despite petitioner’s position that what had really transpired was a
negotiation of his rate and time of availability. The power of selection was firmly evidenced by,
among others, the express written recommendation dated January 12, 1998 by Christine Velazco,
petitioner’s restaurant manager, for the increase of his remuneration.14

Petitioner could not seek refuge behind the service contract entered into with respondent. It is the
law that defines and governs an employment relationship, whose terms are not restricted to those
fixed in the written contract, for other factors, like the nature of the work the employee has been
called upon to perform, are also considered. The law affords protection to an employee, and does
not countenance any attempt to subvert its spirit and intent. Any stipulation in writing can be
ignored when the employer utilizes the stipulation to deprive the employee of his security of
tenure. The inequality that characterizes employer-employee relations generally tips the scales in
favor of the employer, such that the employee is often scarcely provided real and better options.
Respondent’s remuneration, albeit denominated as talent fees, was still considered as included in
the term wage in the sense and context of the Labor Code, regardless of how petitioner chose to
designate the remuneration. Anent this, Article 97(f) of the Labor Code clearly states:
xxx wage paid to any employee shall mean the remuneration or earnings, however designated,
capable of being expressed in terms of money, whether fixed or ascertained on a time, task, piece,
or commission basis, or other method of calculating the same, which is payable by an employer to
an employee under a written or unwritten contract of employment for work done or to be done, or
for services rendered or to be rendered, and includes the fair and reasonable value, as determined
by the Secretary of Labor, of board, lodging, or other facilities customarily furnished by the
employer to the employee.
the power of the employer to control the work of the employee is considered the most significant
determinant of the existence of an employer-employee relationship.18 This is the so-called control
test, and is premised on whether the person for whom the services are performed reserves the
right to control both the end achieved and the manner and means used to achieve that end.19
respondent performed his work as a pianist under petitioner’s supervision and control. Specifically,
petitioner’s control of both the end achieved and the manner and means used to achieve that end
was demonstrated by the following, to wit:

5
a. He could not choose the time of his performance, which petitioners had fixed from 7:00 pm to
10:00 pm, three to six times a week;
b. He could not choose the place of his performance;
c. The restaurant’s manager required him at certain times to perform only Tagalog songs or music,
or to wear barong Tagalog to conform to the Filipiniana motif; and
d. He was subjected to the rules on employees’ representation check and chits, a privilege granted
to other employees.

Relevantly, it is worth remembering that the employer need not actually supervise the performance
of duties by the employee, for it sufficed that the employer has the right to wield that power.
Substantive Issue No. 2:
Validity of the Termination
Retrenchment is one of the authorized causes for the dismissal of employees recognized by the
Labor Code. It is a management prerogative resorted to by employers to avoid or to minimize
business losses. On this matter, Article 283 of the Labor Code states:
Article 283. Closure of establishment and reduction of personnel. – The employer may also
terminate the employment of any employee due to the installation of labor-saving devices,
redundancy, retrenchment to prevent losses or the closing or cessation of operation of the
establishment or undertaking unless the closing is for the purpose of circumventing the provisions
of this Title, by serving a written notice on the workers and the Ministry of Labor and Employment
at least one (1) month before the intended date thereof. xxx. In case of retrenchment to prevent
losses and in cases of closures or cessation of operations of establishment or undertaking not due to
serious business losses or financial reverses, the separation pay shall be equivalent to one (1)
month pay or at least one-half (1/2) month pay for every year of service, whichever is higher. A
fraction of at least six (6) months shall be considered one (1) whole year.

The Court has laid down the following standards that an employer should meet to justify
retrenchment and to foil abuse, namely:
(a) The expected losses should be substantial and not merely de minimis in extent;
(b) The substantial losses apprehended must be reasonably imminent;
(c) The retrenchment must be reasonably necessary and likely to effectively prevent the expected
losses; and
(d) The alleged losses, if already incurred, and the expected imminent losses sought to be
forestalled must be proved by sufficient and convincing evidence.

In termination cases, the burden of proving that the dismissal was for a valid or authorized cause
rests upon the employer. Here, petitioner did not submit evidence of the losses to its business
operations and the economic havoc it would thereby imminently sustain. It only claimed that
respondent’s termination was due to its "present business/financial condition." This bare
statement fell short of the norm to show a valid retrenchment. Hence, we hold that there was no
valid cause for the retrenchment of respondent.
The Court realizes that the lapse of time since the retrenchment might have rendered respondent's
reinstatement to his former job no longer feasible. If that should be true, then petitioner should

6
instead pay to him separation pay at the rate of one. month pay for every year of service computed
from September 1992 (when he commenced to work for the petitioners) until the finality of this
decision, and full backwages from the time his compensation was withheld until the finality of this
decision.

G.R. No. 199547. September 24, 2012

THE NEW PHILIPPINE SKYLANDERS, INC. AND/OR JENNIFER M. ENANO-BOTE


VS.
FRANCISCO N. DAKILA

Facts:

Dakila, a former employee by The New Philippine Skylanders, Inc., was rehired as consultant by the
petitioners under a Contract for Consultancy Services in April 1997. In a letter dated April 19, 2007,
Dakila informed petitioners of his compulsory retirement effective May 2, 2007 and sought for the
payment of his retirement benefits pursuant to the Collective Bargaining Agreement. His request,
however, was not acted upon. Instead, he was terminated from service effective May 1, 2007.

Dakila filed a complaint for constructive illegal dismissal, non-payment of retirement benefits,
under/non-payment of wages and other benefits of a regular employee, and damages against
petitioners before the NLRC. He averred, among others, that the consultancy contract was a scheme
to deprive him of the benefits of regularization, claiming to have assumed tasks necessary and
desirable in the trade or business of petitioners and under their direct control and supervision. In
support of his claim, he submitted, among others, copies of his time cards, Official Business
Itinerary Slips, Daily Attendance Sheets and other documents prescribing the manner in which his
tasks were to be accomplished under the control of the petitioners and acknowledging his status as
a regular employee of the corporation.

On the other hand, petitioners, in their position paper, asserted that Dakila was a consultant and
not their regular employee. The latter was not included in petitioners' payroll and paid a fixed
amount under the consultancy contract. He was not required to observe regular working hours and
was free to adopt means and methods to accomplish his task except as to the results of the work
required of him. Hence, no employer-employee relationship existed between them. Moreover,
Dakila terminated his contract in a letter dated April 19, 2007, thus, negating his dismissal.

Ruling of the Labor Arbiter


Dakila was declared illegally dismissed and ordered his reinstatement with full backwages
computed from the time of his dismissal on May 1, 2007 until his actual reinstatement as well as the
payment of his unpaid benefits under the Collective Bargaining Agreement (CBA). Dakila is a

7
regular employee on the basis of the unrebutted documentary evidence showing that he was under
the petitioners' direct control and supervision and performed tasks that were either incidental or
usually desirable and necessary in the trade or business of petitioner corporation for a period of ten
years. Having been dismissed without cause and notice, respondent Dakila was awarded moral and
exemplary damages in the amount of P 50,000.00 each. He is also entitled to avail of
thecorporation's retirement benefits upon his reinstatement.

Ruling of the NLRC


Dakila was a regular employee and that his dismissal was illegal. However, it noted that since he
was already beyond the retirement age, his reinstatement was no longer feasible. As such, it
ordered the payment of his retirement pay to be computed from 1997 until the date of the decision.
Moreover, it found respondent Dakila entitled to reinstatement wages from the time petitioners
received a copy of the LA’s Decision on July 7, 2008 up to the date of the NLRC's decision. Thus, it
ordered the petitioners to pay respondent Dakila the additional amount
of P 278,508.33representing reinstatement wages and retirement pay.

Ruling of the CA
Declared that the factual findings of the LA and the NLRC to be supported by substantial evidence
and thus, should be accorded respect and finality.

Issues:

(1) whether or not Dakila is a regular employee.


(2) whether or not Dakila was not illegally dismissed as it was the respondent who resigned.

Ruling:

The issue of illegal dismissal is premised on the existence of an employer-employee relationship


between the parties herein. It is essentially a question of fact, beyond the ambit of a petition for
review on certiorari under Rule 45 of the Rules of Court unless there is a clear showing of palpable
error or arbitrary disregard of evidence which does not obtain in this case. Records reveal that both
the LA and the NLRC, as affirmed by the CA, have found substantial evidence to show that
respondent Dakila was a regular employee who was dismissed without cause.

Following Article 279 of the Labor Code, an employee who is unjustly dismissed from work is
entitled to reinstatement without loss of seniority rights and other privileges and to his full
backwages computed from the time he was illegally dismissed. However, considering that
respondent Dakila was terminated on May 1, 2007, or one (1) day prior to his compulsory
retirement on May 2, 2007, his reinstatement is no longer feasible. Accordingly, the NLRC correctly
held him entitled to the payment of his retirement benefits pursuant to the CBA. On the other hand,
his backwages should be computed only for days prior to his compulsory retirement which in this

8
case is only a day. Consequently, the award of reinstatement wages pending appeal must be deleted
for lack of basis.

3. RIGHT TO HIRE

4. WAGES & WAGE RATIONALIZATION ACT

G.R. No.198783. April 15, 2013

ROYAL PLANT WORKERS UNION


VS.
COCA-COLA BOTTLERS PHILIPPINES, INC.-CEBU PLANT.

Facts:

Petitioner Coca-Cola Bottlers Philippines, Inc. (CCBPI) – a domestic corporation engaged in the
manufacture, sale and distribution of softdrink products – has several bottling plants all over the
country, one of which is located in Cebu City. Under the employ of each bottling plant are bottling
operators. In the case of the plant in Cebu City, there are 20 bottling operators who work for its
Bottling Line 1 while there are 12-14 bottling operators who man its Bottling Line 2. All of them are
male and they are members of herein respondent Royal Plant Workers Union (ROPWU).

The bottling operators work in two shifts. The first shift is from 8 a.m. to 5 p.m. and the second shift
is from 5 p.m. up to the time production operations is finished. Each shift has rotations of work time
and break time. Prior to September 2008, the rotation is this: after two and a half (2 ½) hours of
work, the bottling operators are given a 30-minute break and this goes on until the shift ends. In
September 2008 and up to the present, the rotation has changed and bottling operators are now
given a 30-minute break after one and one half (1 ½) hours of work.
In 1974, the bottling operators of then Bottling Line 2 were provided with chairs upon their
request. In 1988, the bottling operators of then Bottling Line 1 followed suit and asked to be
provided also with chairs. Their request was likewise granted. Sometime in September 2008, the
chairs provided for the operators were removed pursuant to a national directive of petitioner. This
directive is in line with the "I Operate, I Maintain, I Clean" program of petitioner for bottling
operators, wherein every bottling operator is given the responsibility to keep the machinery and
equipment assigned to him clean and safe. The program reinforces the task of bottling operators to
constantly move about in the performance of their duties and responsibilities. The bottling
operators took issue with the removal of the chairs.

Grievance machinery Conciliation/mediation Arbitration CA


of CBA: deadlock proceedings before the
between parties NCMB: failed to arrive at an Petition
for
amicable settlement review 9
under
Rule 43
Arbitration:

Issue:Whether the removal of chairs of the operators assigned at the production/manufacturing


line while performing their duties and responsibilities is valid or not.

Petitioner’s stand - removal of the chairs is valid as it is a legitimate exercise of management


prerogative, it does not violate the Labor Code and it does not violate the CBA it contracted with
respondent.

Respondent’s stand - the bottling operators have been performing their assigned duties
satisfactorily with the presence of the chairs; the removal of the chairs constitutes a violation of the
Occupational Health and Safety Standards, the policy of the State to assure the right of workers to
just and humane conditions of work as stated in Article 3 of the Labor Code and the Global
Workplace Rights Policy.

Ruling of the Arbtration Committee - rendered a decision in favor of the Royal Plant Workers Union
(the Union) and against CCBPI. This because the use of chairs by the operators was considered as
a company practice for 34 years in Bottling Line 2 and 20 years in Bottling Line 1 which then
ripened into a benefit after it had been enjoyed by it; that any benefit being enjoyed by the
employees could not be reduced, diminished, discontinued, or eliminated by the employer in
accordance with Article 100 of the Labor Code, which prohibited the diminution or elimination
by the employer of the employees’ benefit; and that jurisprudence had not laid down any rule
requiring a specific minimum number of years before a benefit would constitute a voluntary
company practice which could not be unilaterally withdrawn by the employer.

Ruling of the CA:


Nullified and set aside the decision of the Arbitration Committee. The removal of the chairs from
the manufacturing/production lines by CCBPI is within the province of management prerogatives;
that it was part of its inherent right to control and manage its enterprise effectively; and that since
it was the employer’s discretion to constantly develop measures or means to optimize the efficiency
of its employees and to keep its machineries and equipment in the best of conditions, it was only
appropriate that it should be given wide latitude in exercising it.

Issues:
(1) Is an appeal to the CA via a petition for review under Rule 43 of the 1997 Rules of Civil
Procedure a proper remedy to question the decision of the Arbitration Committee?
(2) Was the removal of the bottling operators’ chairs from CCBPI’s production/manufacturing
lines a valid exercise of a management prerogative?
(3) Is the provision of chairs to the bottling operators covered by Article 100 of the Labor
Code on elimination or diminution of benefits?

10
Ruling:
The Court sustains the ruling of the CA on both issues.

A PETITION FOR REVIEW UNDER RULE 43 IS THE PROPER REMEDY.


It is a well-settled rule that a decision or award of a voluntary arbitrator is appealable to the CA via
petition for review under Rule 43.

A VALID EXERCISE OF MANAGEMENT PREROGATIVE.


The Court has held that management is free to regulate, according to its own discretion and
judgment, all aspects of employment, including hiring, work assignments, working methods, time,
place, and manner of work, processes to be followed, supervision of workers, working regulations,
transfer of employees, work supervision, lay-off of workers, and discipline, dismissal and recall of
workers. The exercise of management prerogative, however, is not absolute as it must be exercised
in good faith and with due regard to the rights of labor.

In the present controversy, it cannot be denied that CCBPI removed the operators’ chairs pursuant to
a national directive and in line with its "I Operate, I Maintain, I Clean" program, launched to enable
the Union to perform their duties and responsibilities more efficiently. The chairs were not removed
indiscriminately. They were carefully studied with due regard to the welfare of the members of the
Union. The removal of the chairs was compensated by: a) a reduction of the operating hours of the
bottling operators from a two-and-one-half (2 ½)-hour rotation period to a one-and-a-half (1 ½) hour
rotation period; and b) an increase of the break period from 15 to 30 minutes between rotations.

Apparently, the decision to remove the chairs was done with good intentions as CCBPI wanted to
avoid instances of operators sleeping on the job while in the performance of their duties and
responsibilities and because of the fact that the chairs were not necessary considering that the
operators constantly move about while working. In short, the removal of the chairs was designed to
increase work efficiency. Hence, CCBPI’s exercise of its management prerogative was made in good
faith without doing any harm to the workers’ rights.
The fact that there is no proof of any operator sleeping on the job is of no moment. There is no
guarantee that such incident would never happen as sitting on a chair is relaxing. Besides, the
operators constantly move about while doing their job. The ultimate purpose is to promote work
efficiency.

NO VIOLATION OF ARTICLE 100 OF THE LABOR CODE


The operators’ chairs cannot be considered as one of the employee benefits covered in
Article 100 of the Labor Code. In the Court’s view, the term "benefits" mentioned in the non-
diminution rule refers to monetary benefits or privileges given to the employee with
monetary equivalents. Such benefits or privileges form part of the employees’ wage, salary
or compensation making them enforceable obligations.

This Court has already decided several cases regarding the non-diminution rule where the benefits
or privileges involved in those cases mainly concern monetary considerations or privileges with

11
monetary equivalents. Some of these cases involve the payment of 14th, 15th and 16th month
bonuses; the 13th month pay, legal/special holiday pay, night premium pay and vacation and sick
leaves; salary wage increases; and service awards with cash incentives, premium pay, Christmas
party with incidental benefits and promotional increase.

In this regard, the Court agrees with the CA when it resolved the matter and wrote:
Let it be stressed that the aforequoted article speaks of non-diminution of supplements and other
employee benefits. Supplements are privileges given to an employee which constitute as extra
remuneration besides his or her basic ordinary earnings and wages. From this definition, we can
only deduce that the other employee benefits spoken of by Article 100 pertain only to those which are
susceptible of monetary considerations. Indeed, this could only be the most plausible conclusion
because the cases tackling Article 100 involve mainly with monetary considerations or privileges
converted to their monetary equivalents.

Without a doubt, equating the provision of chairs to the bottling operators Ds something
within the ambit of "benefits'' in the context of Article 100 of the Labor Code is unduly
stretching the coverage of the law. The interpretations of Article 100 of the Labor Code do
not show even with the slightest hint that such provision of chairs for the bottling operators
may be sheltered under its mantle.

NO VIOLATION OF LABOR LAWS


The rights of the Union under any labor law were not violated. There is no law that requires
employers to provide chairs for bottling operators. The CA correctly ruled that the Labor Code,
specifically Article 132 thereof, only requires employers to provide seats for women. No similar
requirement is mandated for men or male workers. It must be stressed that all concerned bottling
operators in this case are men.
There was no violation either of the Health, Safety and Social Welfare Benefit provisions under
Book IV of the Labor Code of the Philippines. As shown in the foregoing, the removal of the chairs
was compensated by the reduction of the working hours and increase in the rest period. The
directive did not expose the bottling operators to safety and health hazards.

NO VIOLATION OF THE CBA


The CBA between the Union and CCBPI contains no provision whatsoever requiring the
management to provide chairs for the operators in the production/manufacturing line while
performing their duties and responsibilities. The CBA even expressly provides that benefits and/or
privileges, not expressly given therein but which are presently being granted by the company and
enjoyed by the employees, shall be considered as purely voluntary acts by the management and
that the continuance of such benefits and/or privileges, no matter how long or how often, shall not
be understood as establishing an obligation on the company’s part. Since the matter of the chairs is
not expressly stated in the CBA, it is understood that it was a purely voluntary act on the part of
CCBPI and the long practice did not convert it into an obligation or a vested right in favor of the
Union.

12
NO VIOLATION OF THE GENERAL PRINCIPLES OF JUSTICE AND FAIR PLAY
The Court completely agrees with the CA ruling that the removal of the chairs did not violate the
general principles of justice and fair play because the bottling operators’ working time was
considerably reduced from two and a half (2 ½) hours to just one and a half (1 ½) hours and the
break period, when they could sit down, was increased to 30 minutes between rotations. The
bottling operators’ new work schedule is certainly advantageous to them because it greatly
increases their rest period and significantly decreases their working time. A break time of thirty
(30) minutes after working for only one and a half (1 ½) hours is a just and fair work schedule.
Jurisprudence recognizes the exercise of management prerogatives. Labor laws also discourage
interference with an employer's judgment in the conduct of its business. For this reason, the Court
often declines to interfere in legitimate business decisions of employers. The law must protect not
only the welfare of the employees, but also the right of the employers.

G.R. No. 176985, April 1, 2013

RICARDO E. VERGARA, JR.


VS.
COCA-COLA BOTTLERS PHILIPPINES, INC.

FACTS:
PETITIONER Ricardo E. Vergara Jr. worked an employee of respondent Coca-Cola Bottlers
Philippines, Inc. from May 1968 until he retired on Jan. 31, 2002 as a district sales
supervisor (DSS) for Las Piñas City, Metro Manila.
Pursuant to the respondent company’s Retirement Plan Rules and Regulations at the time,
the Annual Performance Incentive Pay of DSSs shall be considered in the computation of retirement
benefits.
Claiming his entitlement to an additional PhP474,600 as sales management incentives and
to the amount of P496,016.67 that the company allegedly deducted illegally, Vergara filed a
complaint before the National Labor Relations Commission (NLRC) on June 11, 2012. He asked for
his “full retirement benefits, merit increase, commission/incentives, length of service, actual, moral
and exemplary damages, and attorney’s fees.”

ISSUE:
Whether or not the Sales Management Incentives (SMI) should be included in the
computation of petitioner’s retirement benefits on the ground of consistent company practice

RULING:
No.

13
Upon review of the entire case records, the Supreme Court found no substantial evidence to prove
that the grant of sales management incentives to all retired DSSs, regardless of whether or not they
qualified for the same, had ripened into company practice.
Despite more than sufficient opportunity given him while his case was pending before the NLRC,
the Court of Appeals and even this Court, the petitioner utterly failed to adduce proof to establish
his allegation that the incentive has been consistently, deliberately and voluntarily granted to all
retired DSSs without any qualification or conditions whatsoever.
The only two pieces of evidence that he stubbornly presented throughout the entirety of this case
are the sworn statements of Renato C. Hidalgo and Ramon V. Velazquez, former DSSs of respondent
who retired in 2000 and 1998, respectively. They claimed that the sales management incentive was
included in their retirement package even if they did not meet the sales and collection qualifiers.
However, juxtaposing these with the evidence presented by respondent would reveal the frailty of
their statements.

The respondent’s isolated act of including the sales management incentive in the retirement
package of Velazquez could hardly be classified as a company practice that may be considered an
enforceable obligation.
To repeat, the principle against diminution of benefits is applicable only if the grant or benefit
is founded on an express policy or has ripened into a practice over a long period of time that is
consistent and deliberate. It presupposes that a company practice, policy and tradition favorable
to the employees has been clearly established; and that the payments made by the company
pursuant to it have ripened into benefits enjoyed by them.
Certainly, a practice or custom is, as a general rule, not a source of a legally demandable or
enforceable right. Company practice, just like any other fact, habits, customs, usage or patterns of
conduct must be proven by the offering party, who must allege and establish specific, repetitive
conduct that might constitute evidence of habit or company practice

5. VIOLATION OF WAGE ORDERS

6. WAGE ENFORCEMENT AND RECOVERY

G.R. 179652
PEOPLE’S BROADCASTING (BOMBO RADYO PHILS.)
VS.
SECRETARY OF DOLE

FACTS:

14
Based on Juezan’s complaint against People’s Broadcasting Service, Inc., the DOLE
conducted a plant level inspection on 23 September 2003. Petitioner was required to
rectify/restitute the violations within five (5) days from receipt. No rectification was effected by
petitioner; thus, summary investigations were conducted, with the parties eventually ordered to
submit their respective position papers.

In his Order dated 27 February 2004, DOLE Regional Director Atty. Rodolfo M. Sabulao (Regional
Director) ruled that respondent is an employee of petitioner, and that the former is entitled to his
money claims amounting to P203, 726.30.

ISSUE:

Whether or not the Secretary of Labor has the power to determine the existence of an employer-
employee relationship

RULING:

Secretary of Labor has the power to determine the existence of an employer-employee relationship.

Clearly the law accords a prerogative to the NLRC over the claim when the employer-employee
relationship has terminated or such relationship has not arisen at all. The existence of an
employer-employee relationship is a matter which is not easily determinable from an ordinary
inspection, necessarily so, because the elements of such a relationship are not verifiable from a
mere ocular examination. The intricacies and implications of an employer-employee
relationship demand that the level of scrutiny should be far above the cursory and the
mechanical. While documents, particularly documents found in the employer’s office are the
primary source materials, what may prove decisive are factors related to the history of the
employer’s business operations, its current state as well as accepted contemporary practices in the
industry. More often than not, the question of employer-employee relationship becomes a battle of
evidence, the determination of which should be comprehensive and intensive and therefore best
left to the specialized quasi-judicial body that is the NLRC. It can be assumed that the DOLE in
the exercise of its visitorial and enforcement power somehow has to make a determination
of the existence of an employer-employee relationship. Such prerogatival determination,
however, cannot be coextensive with the visitorial and enforcement power itself. Indeed,
such determination is merely preliminary, incidental and collateral to the DOLE’s primary
function of enforcing labor standards provisions. The determination of the existence of
employer-employee relationship is still primarily lodged with the NLRC. This is the meaning
of the clause “in cases where the relationship of employer-employee still exists” in Art. 128
(b). Thus, before the DOLE may exercise its powers under Article 128, two important questions
must be resolved: (1) Does the employer-employee relationship still exist, or alternatively, was

15
there ever an employer-employee relationship to speak of; and (2) Are there violations of the Labor
Code or of any labor law?

The existence of an employer-employee relationship is a statutory prerequisite to and


a limitation on the power of the Secretary of Labor, one which the legislative branch is
entitled to impose. The rationale underlying this limitation is to eliminate the prospect of
competing conclusions of the Secretary of Labor and the NLRC, on a matter fraught with questions
of fact and law, which is best resolved by the quasi-judicial body, which is the NRLC, rather than an
administrative official of the executive branch of the government. If the Secretary of Labor
proceeds to exercise his visitorial and enforcement powers absent the first requisite, as the dissent
proposes, his office confers jurisdiction on itself which it cannot otherwise acquire.

Reading of Art. 128 of the Labor Code reveals that the Secretary of Labor or his authorized
representatives was granted visitorial and enforcement powers for the purpose
of determining violations of, and enforcing, the Labor Code and any labor law, wage order, or
rules and regulations issued pursuant thereto. Necessarily, the actual existence of an employer-
employee relationship affects the complexion of the putative findings that the Secretary of Labor
may determine, since employees are entitled to a different set of rights under the Labor Code from
the employer as opposed to non-employees. Among these differentiated rights are those accorded
by the “labor standards” provisions of the Labor Code, which the Secretary of Labor is mandated
to enforce. If there is no employer-employee relationship in the first place, the duty of the employer
to adhere to those labor standards with respect to the non-employees is questionable.

At least a prima facie showing of such absence of relationship, as in this case, is needed to preclude
the DOLE from the exercise of its power. The Secretary of Labor would not have been precluded
from exercising the powers under Article 128 (b) over petitioner if another person with better-
grounded claim of employment than that which respondent had. Respondent, especially if he were
an employee, could have very well enjoined other employees to complain with the DOLE, and, at
the same time, petitioner could ill-afford to disclaim an employment relationship with all of the
people under its aegis.

The most important consideration for the allowance of the instant petition is the
opportunity for the Court not only to set the demarcation between the NLRC’s jurisdiction
and the DOLE’s prerogative but also the procedure when the case involves the fundamental
challenge on the DOLE’s prerogative based on lack of employer-employee relationship.
As exhaustively discussed here, the DOLE’s prerogative hinges on the existence of employer-
employee relationship, the issue is which is at the very heart of this case. And the evidence
clearly indicates private respondent has never been petitioner’s employee.

G.R. No. 178909 : October 10, 2012

16
SUPERIOR PACKAGING CORPORATION
VS.
ARNEL BALAGSAY, ET AL.

FACTS:

The petitioner engaged the services of Lancer to provide reliever services to its business, which
involves the manufacture and sale of commercial and industrial corrugated boxes. According to
petitioner, the respondents were engaged for four (4) months from February to June 1998 and their
tasks included loading, unloading and segregation of corrugated boxes.

Pursuant to a complaint filed by the respondents against the petitioner and its President, Cesar Luz
(Luz), for underpayment of wages, non-payment of premium pay for worked rest, overtime pay and
non-payment of salary, the Department of Labor and Employment (DOLE) conducted an inspection
of the petitioners premises and found several violations, to wit: (1) non-presentation of payrolls
and daily time records; (2) non-submission of annual report of safety organization; (3) medical and
accident/illness reports; (4) non-registration of establishment under Rule 1020 of Occupational
and Health Standards; and (5) no trained first aide.

ISSUE:

WON Superior Packaging Corporation (petitioner) may be held solidarily liable with Lancer Staffing
& Services Network, Inc. (Lancer) for respondents unpaid money claims.

RULING OF THE COURT:

The petition is bereft of merit.

Under Art. 128(b) of the Labor Code, as amended by RA 7730, the DOLE is fully empowered to
make a determination as to the existence of an employer-employee relationship in the exercise of
its visitorial and enforcement power, subject to judicial review, not review by the NLRC.22Ï‚r

It was the consistent conclusion of the DOLE and the CA that Lancer was not an independent
contractor but was engaged in "labor-only contracting"; hence, the petitioner was considered an
indirect employer of respondents and liable to the latter for their unpaid money claims.

At the time of the respondents employment in 1998, the applicable regulation was DOLE
Department Order No. 10, Series of 1997.25ςrνll Under said Department Order, labor-only
contracting was defined as follows:

Sec. 9. Labor-only contracting. (a) Any person who undertakes to supply workers to an employer
shall be deemed to be engaged in labor-only contracting where such person:

17
(1) Does not have substantial capital or investment in the form of tools, equipment, machineries,
work premises and other materials; and
(2) The workers recruited and placed by such persons are performing activities which are directly
related to the principal business or operations of the employer in which workers are habitually
employed.
Labor-only contracting is prohibited and the person acting as contractor shall be considered merely
as an agent or intermediary of the employer who shall be responsible to the workers in the same
manner and extent as if the latter were directly employed by him.26ςrνll

The ratio of Lancers authorized capital stock of P 400,000.00 as against its subscribed and paid-up
capital stock of P 25,000.00 shows the inadequacy of its capital investment necessary to maintain
its day-to-day operations. And while the Court does not set an absolute figure for what it considers
substantial capital for an independent job contractor, it measures the same against the type of work
which the contractor is obligated to perform for the principal.27ςrνll Moreover, the nature of
respondents work was directly related to the petitioners business. The marked disparity between
the petitioners actual capitalization (P 25,000.00) and the resources needed to maintain its
business, i.e., "to establish, operate and manage a personnel service company which will conduct
and undertake services for the use of offices, stores, commercial and industrial services of all
kinds," supports the finding that Lancer was, indeed, a labor-only contractor. Aside from these is
the undisputed fact that the petitioner failed to produce any written service contract that might
serve as proof of its alleged agreement with Lancer.28ςrνll

Finally, a finding that a contractor is a "labor-only" contractor is equivalent to declaring that there
is an employer-employee relationship between the principal and the employees of the supposed
contractor, and the "labor only" contractor is considered as a mere agent of the principal, the real
employer.29ςrνll The former becomes solidarily liable for all the rightful claims of the
employees.30ςrνll The petitioner therefore, being the principal employer and Lancer, being the
labor-only contractor, are solidarily liable for respondents unpaid money claims.

7. WAGE PROTECTION PROVISIONS & PROHIBITIONS REGARDING


WAGES

G.R. NO. 188169 NOVEMBER 28, 2011


NIÑA JEWELRY MANUFACTURING OF METAL ARTS, INC.
VS.
MONTECILLO

FACTS:

18
Madeline Montecillo (Madeline) and Liza Trinidad (Liza), hereinafter referred to collectively as the
respondents, were first employed as goldsmiths by the petitioner Niña Jewelry Manufacturing of
Metal Arts, Inc. (Niña Jewelry) in 1996 and 1994, respectively. Madeline's weekly rate was
P1,500.00 while Liza's was P2,500.00. Petitioner Elisea Abella (Elisea) is Niña Jewelry's president
and general manager.

On August 13, 2004, Niña Jewelry imposed a policy for goldsmiths requiring them to post cash
bonds or deposits in varying amounts but in no case exceeding 15% of the latter's salaries per
week. The deposits were intended to answer for any loss or damage which Niña Jewelry may
sustain by reason of the goldsmiths' fault or negligence in handling the gold entrusted to them. The
deposits shall be returned upon completion of the goldsmiths' work and after an accounting of the
gold received.

Niña Jewelry alleged that the goldsmiths were given the option not to post deposits, but to sign
authorizations allowing the former to deduct from the latter's salaries amounts not exceeding 15%
of their take home pay should it be found that they lost the gold entrusted to them. The respondents
claimed otherwise insisting that Niña Jewelry left the goldsmiths with no option but to post the
deposits. The respondents alleged that they were constructively dismissed by Niña Jewelry as their
continued employments were made dependent on their readiness to post the required deposits.

Niña Jewelry averred that on August 14, 2004, the respondents no longer reported for work and
signified their defiance against the new policy which at that point had not even been implemented
yet.

Respondents filed against Niña Jewelry complaints for illegal dismissal and sought reinstatement
and payment of backwages, attorney's fees and 13th month pay.

ISSUE:

WON respondents were constructively dismissed by the implementation of the new policy.

RULING OF THE COURT:

The instant petition is partially meritorious.

Constructive dismissal occurs when there is cessation of work because continued employment is
rendered impossible, unreasonable or unlikely; when there is a demotion in rank or diminution in
pay or both; or when a clear discrimination, insensibility, or disdain by an employer becomes

unbearable to the employee.[43]

In the case now under our consideration, the petitioners did not whimsically or arbitrarily impose
the policy to post cash bonds or make deductions from the workers' salaries. As attested to by the

19
respondents' fellow goldsmiths in their Joint Affidavit, the workers were convened and informed of
the reason behind the implementation of the new policy. Instead of airing their concerns, the
respondents just promptly stopped reporting for work.

Although the propriety of requiring cash bonds seems doubtful for reasons to be discussed
hereunder, we find no grounds to hold that the respondents were dismissed expressly or even
constructively by the petitioners. It was the respondents who merely stopped reporting for work.
While it is conceded that the new policy will impose an additional burden on the part of the
respondents, it was not intended to result in their demotion. Neither is a diminution in pay
intended because as long as the workers observe due diligence in the performance of their tasks, no
loss or damage shall result from their handling of the gold entrusted to them, hence, all the amounts
due to the goldsmiths shall still be paid in full. Further, the imposition of the new policy cannot be
viewed as an act tantamount to discrimination, insensibility or disdain against the respondents. For
one, the policy was intended to be implemented upon all the goldsmiths in Niña Jewelry's employ
and not solely upon the respondents. Besides, as stressed by the petitioners, the new policy was
intended to merely curb the incidences of gold theft in the work place. The new policy can hardly be
said to be disdainful or insensible to the workers as to render their continued employment
unreasonable, unlikely or impossible.

Article 113 of the Labor Code is clear that there are only three exceptions to the general rule that no
deductions from the employees' 
salaries can be made. The exception which finds application in
the instant petition is in cases where the employer is authorized by law or regulations issued by the
Secretary of Labor to effect the deductions. On the other hand, Article 114 states that generally,
deposits for loss or damages are not allowed except in cases where the employer is engaged in such
trades, occupations or business where the practice of making deposits is a recognized one, or is
necessary or desirable as determined by the Secretary of Labor in appropriate rules or regulations.

While employers should generally be given leeways in their exercise of management prerogatives,
we agree with the respondents and the CA that in the case at bar, the petitioners had failed to prove
that their imposition of the new policy upon the goldsmiths under Niña Jewelry's employ falls
under the exceptions
In view of the foregoing, we hold that no dismissal, constructive or otherwise, occurred. The
findings of the NLRC and the LA that it was the respondents who stopped reporting for work are
supported by substantial evidence.

20
8. PAYMENT OF WAGES

9. CONDITIONS OF EMPLOYMENT

10. MINIMUM LABOR STANDARD BENEFITS

G.R. No. 198662 September 12, 2012

RADIO MINDANAO NETWORK, INC. AND ERIC S. CANOY, PETITIONERS,


VS.
DOMINGO Z. YBAROLA, JR. AND ALFONSO E. RIVERA, JR., RESPONDENTS.

Facts:
Domingo Ybarola, Jr. and Alfonso Rivera, Jr. were hired by RMN on June 15, 1977 and June 1, 1983,
respectively. They eventually became account managers, soliciting advertisements and servicing
various clients of RMN. On September 15, 2002, the respondents’ services were terminated as a
result of RMN’s reorganization/restructuring, they were given separation pay; P631,250.00 for
Ybarola and P481,250.00 for Rivera. They both executed release/quitclaim affidavits in December
2002. Dissatisfied with their separation pay, they filed a complaint against RMN and its President
Eric Canoy, for illegal dismissal with money claims.

In the Compulsory Arbitration Proceedings Ybarola and Rivera argued that the release/quitclaim
should not be a bar to the recovery of the full benefits due them. They also said that they only sign it
because of dire necessity. The petitioners denied liability saying that it was fair and reasonable
settlement which they executed voluntarily and freely. The LA dismissed the illegal dismissal case
but ordered payment of additional separation pay P490,066.00 for Ybarola and P429,517.55 for
Rvera. But later on LA adjusted the award because of the presentation of Certificates of
Compensation Payment/Tax Withheld. On appeal by RMN to the NLRC, the NLRC set aside the LA’s
decision. It ruled that the withholding tax certificate cannot be the basis of the computation of the
respondents’ separation pay as the tax document included the respondents’ cost-of-living
allowance and commissions; as a general rule, commissions cannot be included in the base figure
for the computation of the separation pay because they have to be earned by actual market
transactions attributable to the respondents, as held in previous cases and upheld the validity of the
quitclaim for failure to show that they were force to sign it.

CA, overruled the NLRC’s decision. SC denied the petition, thus this Motion for Reconsideration.

Issues:
Were the commissions earned through actual market transactions attributable to them?

21
Were the quitclaims valid?
Is President Canoy personally liable in this case?

Ruling:

The motion for reconsideration is bereft of merit.

RMN insists that the respondents’ commissions were not part of their salaries, because they failed
to present proof that they earned the commission due to actual market transactions attributable to
them. They submit that the commissions are profit-sharing payments which do not form part of
their salaries. We are not convinced. If these commissions had been really profit-sharing bonuses to
the respondents, they should have received the same amounts, yet, as the NLRC itself noted,
Ybarola and Rivera received P 372,173.11 and P 586,998.50 commissions, respectively, in 2002.
The variance in amounts the respondents received as commissions supports the CA’s finding that
the salary structure of the respondents was such that they only received a minimal amount as
guaranteed wage; a greater part of their income was derived from the commissions they get from
soliciting advertisements; these advertisements are the "products" they sell. As the CA aptly noted,
this kind of salary structure does not detract from the character of the commissions being part of
the salary or wage paid to the employees for services rendered to the company.

In this case, as the CA noted, the separation pay the respondents each received was deficient by at
least P400,000.00; thus, they were given only half of the amount they were legally entitled to. To be
sure, a settlement under these terms is not and cannot be a reasonable one, given especially the
respondents’ length of service – 25 years for Ybarola and 19 years for Rivera. The CA was correct
when it opined that the respondents were in dire straits when they executed the release/quitclaim
affidavits. Without jobs and with families to support, they dallied in executing the quitclaim
instrument, but were eventually forced to sign given their circumstances.
Lastly, the petitioners are estopped from raising the issue of Canoy's personal liability. They did not
raise it before the NLRC in their appeal from the labor arbiter's decision, nor with the CA in their
motion for reconsideration of the appellate court's judgment. The risk of having Canoy's personal
liability for the judgment award did not arise only with the filing of the present petition, it had been
there all along - in the NLRC, as well as in the CA.

11. OTHER SPECIAL BENEFITS

G.R. No. 198662 September 12, 2012

22
RADIO MINDANAO NETWORK, INC. AND ERIC S. CANOY, PETITIONERS,
VS.
DOMINGO Z. YBAROLA, JR. AND ALFONSO E. RIVERA, JR., RESPONDENTS.

Facts:

Domingo Ybarola, Jr. and Alfonso Rivera, Jr. were hired by RMN on June 15, 1977 and June 1, 1983,
respectively. They eventually became account managers, soliciting advertisements and servicing
various clients of RMN. On September 15, 2002, the respondents’ services were terminated as a
result of RMN’s reorganization/restructuring, they were given separation pay; P631,250.00 for
Ybarola and P481,250.00 for Rivera. They both executed release/quitclaim affidavits in December
2002. Dissatisfied with their separation pay, they filed a complaint against RMN and its President
Eric Canoy, for illegal dismissal with money claims.

In the Compulsory Arbitration Proceedings Ybarola and Rivera argued that the release/quitclaim
should not be a bar to the recovery of the full benefits due them. They also said that they only sign it
because of dire necessity. The petitioners denied liability saying that it was fair and reasonable
settlement which they executed voluntarily and freely. The LA dismissed the illegal dismissal case
but ordered payment of additional separation pay P490,066.00 for Ybarola and P429,517.55 for
Rvera. But later on LA adjusted the award because of the presentation of Certificates of
Compensation Payment/Tax Withheld. On appeal by RMN to the NLRC, the NLRC set aside the LA’s
decision. It ruled that the withholding tax certificate cannot be the basis of the computation of the
respondents’ separation pay as the tax document included the respondents’ cost-of-living
allowance and commissions; as a general rule, commissions cannot be included in the base figure
for the computation of the separation pay because they have to be earned by actual market
transactions attributable to the respondents, as held in previous cases and upheld the validity of the
quitclaim for failure to show that they were force to sign it.

CA, overruled the NLRC’s decision. SC denied the petition, thus this Motion for Reconsideration.

Issues:
Were the commissions earned through actual market transactions attributable to them?
Were the quitclaims valid?
Is President Canoy personally liable in this case?

Ruling:
The motion for reconsideration is bereft of merit.
RMN insists that the respondents’ commissions were not part of their salaries, because they
failed to present proof that they earned the commission due to actual market transactions
attributable to them. They submit that the commissions are profit-sharing payments which do not
form part of their salaries. We are not convinced. If these commissions had been really profit-
sharing bonuses to the respondents, they should have received the same amounts, yet, as the NLRC
itself noted, Ybarola and Rivera received P 372,173.11 and P 586,998.50 commissions, respectively,
in 2002. The variance in amounts the respondents received as commissions supports the CA’s

23
finding that the salary structure of the respondents was such that they only received a minimal
amount as guaranteed wage; a greater part of their income was derived from the commissions they
get from soliciting advertisements; these advertisements are the "products" they sell. As the CA
aptly noted, this kind of salary structure does not detract from the character of the commissions
being part of the salary or wage paid to the employees for services rendered to the company.

In this case, as the CA noted, the separation pay the respondents each received was deficient by at
least P400,000.00; thus, they were given only half of the amount they were legally entitled to. To be
sure, a settlement under these terms is not and cannot be a reasonable one, given especially the
respondents’ length of service – 25 years for Ybarola and 19 years for Rivera. The CA was correct
when it opined that the respondents were in dire straits when they executed the release/quitclaim
affidavits. Without jobs and with families to support, they dallied in executing the quitclaim
instrument, but were eventually forced to sign given their circumstances.

Lastly, the petitioners are estopped from raising the issue of Canoy's personal liability. They did not
raise it before the NLRC in their appeal from the labor arbiter's decision, nor with the CA in their
motion for reconsideration of the appellate court's judgment. The risk of having Canoy's personal
liability for the judgment award did not arise only with the filing of the present petition, it had been
there all along - in the NLRC, as well as in the CA

ELEAZAR S. PADILLO+
VS.
RURAL BANK OF NABUNTURAN, INC. AND MARK S. OROPEZA

Facts:

Petitioner, the late Eleazar Padillo, was employed by Rural Bank of Nabunturan, Inc. as its SA
Bookkeeper. Due to liquidity problems, the Bank took out retirement/insurance plans for all its
employees in anticipation of its possible closure and the concomitant severance of its personnel. In
this regard, the Bank procured a Philam Life Plan in favor of Padillo for a benefit amount of
P100,000.00 and which was set to mature on July 11, 2009.

On October 14, 2004, respondent Mark S. Oropeza (Oropeza), the President of the Bank, bought
majority shares of stock in the Bank and took over its management which brought about its gradual
rehabilitation. The Bank's finances improved and eventually, its liquidity was regained.

During the latter part of 2007, Padillo suffered a mild stroke due to hypertension which
consequently impaired his ability to effectively pursue his work, the nature of which had been
classified as a total disability. He wrote a letter addressed to Oropeza expressing his intention to
avail of an early retirement package. Despite several follow-ups, his request remained unheeded.

24
On October 3, 2007, Padillo was separated from employment due to his poor and failing health. Not
having received his claimed retirement benefits, Padillo filed with the Labor Arbiter a complaint for
the recovery of unpaid retirement benefits. He asserted that the Bank had adopted a policy of
granting its aging employees early retirement packages, pointing out that one of his co-employees,
Nenita Lusan, was accorded retirement benefits when she retired at the age of only fifty-three (53).

The LA dismissed Padillo's complaint but directed the Bank to pay him the amount of P100,000.00
as financial assistance, treated as an advance from the amounts receivable under the Philam Life
Plan. It found Padillo disqualified to receive any benefits under Article 300 of the Labor Code as he
was only 55 years old when he resigned, while the law specifically provides for an optional
retirement age of 60 and compulsory retirement age of 65.

Dissatisfied, Padillo elevated the matter to the NLRC which reversed and set aside the LA's ruling.
The NLRC applied the Labor Code provision on termination on the ground of disease —
particularly, Article 297— holding that while Padillo did resign, he did so only because of his poor
health condition.

The CA held that Padillo could not, absent any agreement with the Bank, receive any retirement
benefits pursuant to Article 300 of the Labor Code considering that he was only 55 years old when
he retired. It likewise found the evidence insufficient to prove that the Bank has an existing
company policy of granting retirement benefits to its aging employees. Finally, citing the case
of Villaruel v. Yeo Han Guan, it pronounced that separation pay on the ground of disease under
Article 297 of the Labor Code should not be given to Padillo because he was the one who initiated
the severance of his employment and that even before September 10, 2007, he already stopped
working due to his poor and failing health.

Issue: WON petitioner is entitled to (1) separation pay and (2) retirement benefits

Ruling:
At the outset, it must be maintained that the Labor Code provision on termination on the ground of
disease does not apply in this case, considering that it was the petitioner and not the Bank who
severed the employment relations. As borne from the records, the clear import of Padillo's
letter and the fact that he stopped working before the foregoing date and never reported for work
even thereafter show that it was Padillo who voluntarily retired and that he was not terminated by
the Bank.

As held in Villaruel v. Yeo Han Guan, Article 297 [now renumbered] of the Labor Code contemplates
a situation where the employer, and not the employee, initiates the termination of employment on
the ground of the latter's disease or sickness, viz.:

A plain reading of the [Article 297 of the Labor Code] clearly presupposes that it is the employer
who terminates the services of the employee found to be suffering from any disease and
whose continued employment is prohibited by law or is prejudicial to his health as well as to

25
the health of his co-employees. It does not contemplate a situation where it is the employee
who severs his or her employment ties.

Thus, given the inapplicability of [Article 297 of the Labor Code] to the case at bar, it necessarily
follows that petitioners' claim for separation pay anchored on such provision must be denied.

What remains applicable, however, is the Labor Code provision on retirement [Article 300]. Simply
stated, in the absence of any applicable agreement, an employee must (1) retire when he is at least
sixty (60) years of age and (2) serve at least (5) years in the company to entitle him/her to a
retirement benefit of at least one-half (1/2) month salary for every year of service, with a fraction
of at least six (6) months being considered as one whole year. Notably, these age and tenure
requirements are cumulative and non-compliance with one negates the employee's entitlement to
the retirement benefits under the Labor Code.

In this case, it is undisputed that there exists no retirement plan, collective bargaining agreement or
any other equivalent contract between the parties which set out the terms and condition for the
retirement of employees, with the sole exception of the Philam Life Plan which premiums had
already been paid by the Bank. Neither was it proven that there exists an established company
policy of giving early retirement packages to the Bank's aging employees.

It has been pronounced that to be considered a company practice, the giving of the benefits should
have been done over a long period of time, and must be shown to have been consistent and
deliberate.

Neither can the grant of an early retirement package to Lusan show that Padillo was unfairly
discriminated upon. Records show that the same was merely an isolated incident and petitioners
have failed to show that any bad faith or motive attended such disparate treatment between Lusan
and Padillo. Irrefragably also, there is no showing that other Bank employees were accorded the
same benefits as that of Lusan which thereby dilutes the soundness of petitioners' imputation of
discrimination and bad faith.

All told, in the absence of any applicable contract or any evolved company policy, Padillo should
have met the age and tenure requirements set forth under Article 300 of the Labor Code to be
entitled to the retirement benefits provided therein. Unfortunately, while Padillo was able to
comply with the five (5) year tenure requirement — as he served for twenty-nine (29) years — he,
however, fell short with respect to the sixty (60) year age requirement given that he was only fifty-
five (55) years old when he retired. Therefore, without prejudice to the proceeds due under the
Philam Life Plan, petitioners' claim for retirement benefits must be denied.

Nevertheless, the Court concurs with the CA that financial assistance should be awarded but at an
increased amount. The Court, in light of the dictates of social justice, holds that the CA's financial
assistance award should be increased from P50,000.00 to P75,000.00, still exclusive of the

26
P100,000.00 benefit receivable by the petitioners under the Philam Life Plan which remains
undisputed.

12. RIGHT TO SECURITY OF TENURE

JAIME N. GAPAYAO
VS.
ROSARIO FULO, SOCIAL SECURITY SYSTEM AND SOCIAL SECURITY COMMISSION

Facts:
On 4 November 1997, Jaime Fulo died of "acute renal failure secondary to 1st degree burn 70%
secondary electrocution" while doing repairs at the residence and business establishment of
petitioner Gapayao.

Allegedly moved by his Christian faith, Gapayao extended some financial assistance to private
respondent Rosario, wife of Jaime Fulo. Rosario then executed an Affidavit of Desistance stating that
she was not holding them liable for the death of her late husband, and was waiving her right and
desisting from filing any criminal or civil action against petitioner. Both parties then executed a
Compromise Agreement.

Thereafter, Rosario filed a claim for social security benefits with the SSS. However, upon
verification and evaluation, it was discovered that the deceased was not a registered member of the
SSS. Upon the insistence of Rosario that her late husband had been employed by petitioner, the SSS
conducted a field investigation to clarify his status of employment. In its field investigation report, it
found that Jaime Fulo is an employee of Mr. & Mrs. Jaime Gapayao.

Consequently, the SSS demanded that petitioner remit the social security contributions of the
deceased. Gapayao denied that the deceased was his employee.

In asserting the existence of an employer-employee relationship, Rosario alleges that her late
husband had been in the employ of petitioner for 14 years, from 1983 to 1997. During that period,
he was made to work as a laborer in the agricultural landholdings, a harvester in the abaca
plantation, and a repairman/utility worker in several business establishments owned by
petitioner. This view is bolstered by the admission of petitioner himself in the Compromise
Agreement that he was the deceased's employer.

27
This position was similarly espoused by the SSC, which insists that pakyaw workers are considered
employees, as long as the employer exercises control over them. The SSC further asserts that the
deceased rendered services essential for the petitioner's harvest. While these services were not
rendered continuously (in the sense that they were not rendered every day throughout the year),
still, the deceased had never stopped working for petitioner from year to year until the day the
former died.

Petitioner, on the other hand, insists that the deceased was not his employee. Supposedly, the latter,
during the performance of his function, was not under petitioner's control. Granting without
admitting that petitioner gave rules or guidelines to the deceased in the process of the latter's
performing his work, the situation cannot be interpreted as control, because it was only intended to
promote mutually desired results.

Alternatively, petitioner insists that the deceased was hired by Adolfo Gamba, the contractor whom
he had hired to construct their building; and by Amado Gacelo, the tenant whom petitioner
instructed to manage the latter's farm.

Anent the Compromise Agreement, petitioner clarifies that it was executed to buy peace, because
"respondent kept on pestering them by asking for money." Petitioner allegedly received threats
that if the matter was not settled, private respondent would refer the matter to the
NPA. http://www.cdasiaonline.com/search/show_article/57129?search=%28gr+no+193493%29 -
footnotes Likewise, petitioner maintains that he shouldered the hospitalization and burial expenses
to express his "compassion and sympathy to a distressed person and his family," and not to admit
liability.

Lastly, petitioner alleges that the deceased is a freelance worker. Since he was engaged on
a pakyaw basis and worked for a short period of time, in the nature of a farm worker every season,
he was not precluded from working with other persons and in fact worked for them.

Issue:
WON there exists an employer-employee relationship between Jaime Fulo and petitioner that
would merit an award of benefits in favor of private respondent under social security laws.

Ruling: YES.

Farm workers may be considered regular seasonal employees.


Undet Article 280 of the Labor Code, jurisprudence has identified the three types of employees
mentioned in the provision: (1) regular employees or those who have been engaged to perform
activities that are usually necessary or desirable in the usual business or trade of the employer; (2)
project employees or those whose employment has been fixed for a specific project or undertaking,
the completion or termination of which has been determined at the time of their engagement, or
those whose work or service is seasonal in nature and is performed for the duration of the season;
and (3) casual employees or those who are neither regular nor project employees.

28
Farm workers generally fall under the definition of seasonal employees. The Court has consistently
held that seasonal employees may be considered as regular employees. Regular seasonal employees
are those called to work from time to time. The nature of their relationship with the employer is
such that during the off season, they are temporarily laid off; but reemployed during the summer
season or when their services may be needed. They are in regular employment because of the
nature of their job, and not because of the length of time they have worked.

This rule, however, is not absolute. Seasonal workers who have worked for one season only may
not be considered regular employees. Also when seasonal employees are free to contract their
services with other farm owners, then the former are not regular employees.

For regular employees to be considered as such, the primary standard used is the reasonable
connection between the particular activity they perform and the usual trade or business of the
employer. This test has been explained thoroughly in De Leon v. NLRC:

The primary standard, therefore, of determining a regular employment is the reasonable


connection between the particular activity performed by the employee in relation to the usual
business or trade of the employer. The test is whether the former is usually necessary or desirable
in the usual business or trade of the employer. The connection can be determined by considering
the nature of the work performed and its relation to the scheme of the particular business or trade
in its entirety. Also if the employee has been performing the job for at least one year, even if the
performance is not continuous or merely intermittent, the law deems the repeated and continuing
need for its performance as sufficient evidence of the necessity if not indispensability of that
activity to the business. Hence, the employment is also considered regular, but only with respect to
such activity and while such activity exists.

The records reveal that the deceased was indeed a farm worker who was in the regular employ of
petitioner. From year to year, the deceased had been working on petitioner's land by harvesting
abaca and coconut, processing copra, and clearing weeds. His employment was continuous in the
sense that it was done for more than one harvesting season. Moreover, no amount of reasoning
could detract from the fact that these tasks were necessary or desirable in the usual business of
petitioner. The other tasks allegedly done by the deceased outside his usual farm work only bolster
the existence of an employer-employee relationship. As found by the SSC, the deceased was a
construction worker in the building and a helper in the bakery, grocery, hardware, and piggery —
all owned by petitioner. This fact only proves that even during the off season, the deceased was still
in the employ of petitioner.

The most telling indicia of this relationship is the Compromise Agreement executed by petitioner
and private respondent. Petitioner entered into the agreement with full knowledge that he was
described as the employer of the deceased. This knowledge cannot simply be denied by a
statement that petitioner was merely forced or threatened into such an agreement.

29
Pakyaw workers are considered employees, provided they are subject to the control of
petitioner.
In Legend Hotel Manila v. Realuyo, the Court held that "the power of the employer to control the
work of the employee is considered the most significant determinant of the existence of an
employer-employee relationship. This is the so-called control test and is premised on whether the
person for whom the services are performed reserves the right to control both the end achieved
and the manner and means used to achieve that end." It should be remembered that the control test
merely calls for the existence of the right to control, and not necessarily the exercise thereof. It is
not essential that the employer actually supervises the performance of duties by the employee. It is
enough that the former has a right to wield the power.

In this case, we agree with the CA that petitioner wielded control over the deceased in the discharge
of his functions. Being the owner of the farm on which the latter worked, petitioner — on his own
or through his overseer — necessarily had the right to review the quality of work produced by his
laborers. It matters not whether the deceased conducted his work inside petitioner's farm or not
because petitioner retained the right to control him in his work, and in fact exercised it through his
farm manager Amado Gacelo. The latter himself testified that petitioner had hired the deceased as
one of the pakyaw workers whose salaries were derived from the gross proceeds of the harvest.

The right of an employee to be covered by the Social Security Act is premised on the existence of an
employer-employee relationship. That having been established, the Court hereby rules in favor of
private respondent.

G.R. No. 177812; June 19, 2013

CONCRETE SOLUTIONS, INC, ET AL


VS
ARTHUR CABUSAS

Gist: A project EE was illegally dismissed prior to the expiration of his employment. Also, he was
dismissed without valid or just cause, as there was no clear showing of abandonment, as alleged by ER.
SC awarded EE back wages but it ruled that EE could no longer be reinstated as the project he was
assigned to was already finished.

FACTS: Respondent Arthur Cabusas (respondent) was hired by petitioner Primary Structures
Corporation (PSC) as transit mixer driver for petitioner Concrete Solutions Inc. (CSI) - Batching
Plant Project. The appointment provided, among others: that respondent was hired for the period
from June 28, 2000 until June 23, 2001; the status of his employment was that of a project employee
and, as such, his employment was co-terminus with the completion of the project or any phase
thereof.

30
In the course of his work, Cabusas was alleged to have committed theft on several instances(ie, not
returning excess concrete mix to plant but instead selling them to residents where he unloaded the
mix; selling an empty plastic drum). In each instance, CSI Admin Assistant made an incident report;
CSI Manager required an explanation in writing from Cabusas. Cabusas was placed under
preventive suspension from April 20, 2001 to April 27, 2001 pending investigation of his case.
The administrative investigation which was scheduled on April 26, 2001 was postponed to May 4,
2001 and respondent's preventive suspension was extended up to May 5, 2001. Respondent alleged
that after the investigation on May 4, 2001, he and his counsel had asked for the result of the
investigation and were waiting for such result. In the course of deliberation, CSI noticed that the SSS
records of Cabusas were under the name of Alex (instead of Arthur but could not clarify the issue
due to Cabusas’ absence since May 6, 2001.

On May 25, CSI asked employee a telegram and asked for a notification regarding the latter’s
absence. On June 12, CSI through its Manager sent Cabusas a termination letter. (Earlier, however,
on May 30, 2001, respondent had filed with Regional Arbitration Branch No. VII of Cebu City a
Complaint for unfair labor practice, illegal dismissal, non-payment of holiday pay, premium pay for
holiday, rest day, night shift premium, separation pay and moral damages against petitioners.)

Labor Arbiter declared Cabusas was validly dismissed. NLRC modified decision and ordered CSI to
reinstate complainant and pay his full backwages, saying there was no abandonment of work on the
part of Cabusas. NLRC also ruled that Cabusas was not a project employee, applying the principle
that where from circumstances it is apparent that periods have been imposed to preclude
acquisition of tenurial security by the employee. CA affirmed NLRC in toto.

ISSUE #1: Whether respondent deliberately abandoned his work which is a just cause for his
dismissal or whether he was illegally dismissed by petitioners.

RULING # 1: It is well settled that in termination cases, the burden of proof rests upon the
employer to show that the dismissal was for a just and valid cause, and failure to discharge the
same would mean that the dismissal is not justified and, therefore, illegal.

To constitute abandonment, two elements must concur, to wit: (1) the failure to report for work or
absence without valid or justifiable reason; and (2) a clear intention to sever the employer-
employee relationship, with the second element as the more determinative factor and being
manifested by some overt acts. Abandonment is a matter of intention and cannot lightly be
presumed from certain equivocal acts. To be a valid cause for dismissal for abandonment, there
must be clear proof of deliberate and unjustified intent to sever the employer-employee
relationship. Clearly, the operative act is still the employee's ultimate act of putting an end to his
employment.

We find that the elements of abandonment are lacking. The CA did not commit any reversible error
in affirming the NLRC's decision that respondent was illegally dismissed for petitioners' failure to
substantiate their claim that the former abandoned his work.

31
The circumstances obtaining in this case do not indicate abandonment. There is no showing of
respondent's intent to sever the employer-employee relationship. It is also notable that when
respondent was refused entry to petitioners' premises and the letter of former's counsel was
refused acceptance by the latter, there is already constructive dismissal which led respondent to
seek recourse by filing an illegal dismissal case against petitioners on May 30, 2001. The proximity
of respondent's filing of the complaint from the time he received the telegram and was refused
entry to petitioners' premises showed that he had the least intention of abandoning his job. Well-
settled that the filing by an employee of a complaint for illegal dismissal with a prayer for
reinstatement is proof enough of his desire to return to work, thus, negating the employer’s charge
of abandonment.

ISSUE # 2: Whether the CA committed a reversible error in affirming the NLRC's award of
respondent's reinstatement and backwages.

RULING # 2: Petitioners contend that respondent was a project employee and the project to which
he was hired was already completed, thus he could not be reinstated anymore.

Project employee is one whose employment has been fixed for a specific project or undertaking the
completion or termination of which has been determined at the time of the engagement of the
employee or where the work or services to be performed is seasonal in nature and the employment
is for the duration of the season. We held that the length of service of a project employee is not the
controlling test of employment tenure but whether or not the employment has been fixed for a
specific project or undertaking the completion or termination of which has been determined at the
time of the engagement of the employee.

We rule that respondent is a project employee. His appointment letter showed that he was hired as
transit mixer driver for the Concrete Solutions Inc. (CSI) – Batching Plant Project for the period
from June 28, 2000 until June 23, 2001. The same letter provided that he was a project employee
whose employment was co-terminus with the completion of the project or any phase thereof and
upon completion of the particular project or phase, he was free to seek other employment of his
choice. There is no evidence showing that respondent did not sign the conforme part of the
appointment letter voluntarily. Hence, respondent was bound by the provisions in the appointment
letter. Moreover, there is also no showing that the period fixed in the appointment letter was
imposed to preclude acquisition of tenurial security by the employee and should be disregarded for
being contrary to public policy as ruled by the NLRC since no evidence exists on the record to
support such conclusion.

Considering that respondent was dismissed prior to the expiration of the duration of his
employment and without a valid or just cause, his termination was therefore illegal. However,
respondent could no longer be reinstated since the project he was assigned to was already
completely finished. However, we find that he is entitled to the salary corresponding to the
unexpired portion of his employment. Respondent is entitled to the payment of his salary from the

32
time he was not admitted back to work on May 26, 2001 up to June 23, 2001, the expiration of his
employment contract.

G.R. No. 159371; July 29, 2013

D.M. CONSUNJI CORPORATION


VS
ROGELIO P. BELLO

Gist: A mason who started out as a project employee was continuously hired in the construction
company’s other projects. He became sick with TB and had to absent himself from work. Upon
recovery, he was terminated by the construction company when he returned for work. SC ruled that
the extension of the employment of a project employee long after the supposed project has been
completed removes the employee from the scope of a project employee and makes him a regular
employee.

FACTS: Bello brought a complaint for illegal dismissal and damages against DMCI and/or Rachel
Consunji. In his position paper, he claimed that DMCI had employed him as a mason without any
interruption from February 1, 1990 until October 10, 1997 at an hourly rate of P25.081; that he had
been a very diligent and devoted worker and had served DMCI as best as he could and without any
complaints; that he had never violated any company rules; that his job as a mason had been
necessary and desirable in the usual business or trade of DMCI; that he had been diagnosed to be
suffering from pulmonary tuberculosis, thereby necessitating his leave of absence; that upon his
recovery, he had reported back to work, but DMCI had refused to accept him and had instead
handed to him a termination paper; that he had been terminated due to "RSD" effective November
5, 1997; that he did not know the meaning of "RSD" as the cause of his termination; that the cause
had not been explained to him; that he had not been given prior notice of his termination; that he
had not been paid separation pay as mandated by law; that at that time of his dismissal, DMCI’s
projects had not yet been completed; and that even if he had been terminated due to an authorized
cause, he should have been given at least one month pay or at least one-half month pay for every
year of service he had rendered, whichever was higher.

The Executive Labor Aribter (ELA) held DMCI guilty of illegal dismissal and it is hereby ordered to
reinstate complainant to his former position without loss of seniority rights and to pay him full
backwages reckoned from the time of his dismissal up to his actual reinstatement which as of this
date is in the amount of P232,648,81.

NLRC set aside the decision of the ELA. CA reversed NLRC and declared Bello to have acquired the
status of a regular employee although he had started as a project employee of DMCI.

ISSUE: Whether or not private respondent was a regular employee

33
RULING: The petition for review lacks merit. The provision that governs the first issue is Article
280 of the Labor Code.

A project employee is one who is hired for a specific project or undertaking, and the completion or
termination of such project or undertaking has been determined at the time of engagement of the
employee. In the context of the law, Bello was a project employee of DMCI at the beginning of their
employer-employee relationship.

However, the history of Bello’s appointment and employment showed that he performed his tasks
as a mason in DMCI’s various constructions projects.

Based on the foregoing, we affirm the CA’s conclusion that Bello acquired in time the status of a
regular employee by virtue of his continuous work as a mason of DMCI. The work of a mason like
him – a skilled workman working with stone or similar material – was really related to building or
constructing, and was undoubtedly a function necessary and desirable to the business or trade of
one engaged in the construction industry like DMCI. His being hired as a mason by DMCI in not one,
but several of its projects revealed his necessity and desirability to its construction business.

It is settled that the extension of the employment of a project employee long after the supposed
project has been completed removes the employee from the scope of a project employee and makes
him a regular employee.http://www.lawphil.net/judjuris/juri2013/jul2013/gr_159371_2013.html
- fnt17 In this regard, the length of time of the employee’s service, while not a controlling
determinant of project employment, is a strong factor in determining whether he was hired for a
specific undertaking or in fact tasked to perform functions vital, necessary and indispensable to the
usual business or trade of the employer.

Verily, the principal test for determining whether an employee is a project employee, as
distinguished from a regular employee, is whether or not he is assigned to carry out a specific
project or undertaking, the duration and scope of which are specified at the time he is engaged for
the project.

Still, DMCI contends that Bello’s services as a mason were deemed necessary and desirable in its
usual business only for the period of time it had taken it to complete the project. The contention
may be correct if each engagement of Bello as a mason over the span of eight years was to be
treated separately. The contention cannot be upheld, however, simply because his successive
reengagement in order to perform the same kind of work as a mason firmly manifested the
necessity and desirability of his work in DMCI’s usual business of construction.

G.R. No. 170388, September 4, 2013

34
COLEGIO DEL SANTISIMO ROSARIO AND SR. ZANAIDA S. MOFADA, OP, PETITIONERS,
VS.
EMMANUEL ROJO, RESPONDENT.

Facts:
Petitioner Colegio del Santisimo Rosario (CSR) hired respondent Rojo as a high school teacher on
probationary basis for the school years 1992-1993, 1993-1994 and 1994-1995. On April 5, 1995,
CSR, through petitioner Sr. Zenaida S. Mofada, OP (Mofada), decided not to renew respondent’s
services.

Thus, on July 13, 1995, respondent filed a Complaint for illegal dismissal. He alleged that since he
had served three consecutive school years which is the maximum number of terms allowed for
probationary employment, he should be extended permanent employment. On the other hand,
petitioners argued that respondent knew that his Teacher’s Contract for school year 1994-1995
with CSR would expire on March 31, 1995. Accordingly, respondent was not dismissed but his
probationary contract merely expired and was not renewed.

The LA found that the non-renewal of Rojo’s contract for school year 1995-1996 constitutes illegal
dismissal and done in bad faith. On appeal, the NLRC affirmed the LA’s Decision with modification.
It held that after serving three school years, respondent had attained the status of regular
employment especially because CSR did not make known to respondent the reasonable standards
he should meet. The NLRC also agreed with the LA that respondent’s termination was done in bad
faith. It held that respondent is entitled to reinstatement, if viable; or separation pay, if
reinstatement was no longer feasible, and backwages. Likewise, upon a petition for Certiorari with
the CA, the court ruled for the respondent and denied the motion for reconsideration.

Issue:
Whether a basic education teacher hired for three consecutive school years as a probationary
employee by law becomes a permanent employee upon completion of his third year of probation
notwithstanding (a) the pronouncement of the Supreme Court that a probationary teacher aquires
permanent status only when he is allowed to work after the probationary period and (b) DOLE-
DECS-CHED-TESDA Order No. 01, S. 1996 which provide that teachers who have served the
probationary period shall be made regular or permanent if allowed to work after such probationary
period.

Ruling:
Petition is denied. Respondent was illegally dismissed. With regard to the probationary period,
Section 92 of the 1992 Manual of Regulations for Private Schools provides: Section 92. Probationary
Period. – Subject in all instances to compliance with the Department and school
requirements, the probationary period for academic personnel shall not be more than three
(3) consecutive years of satisfactory service for those in the elementary and secondary levels
xxx

35
At the end of the third year, the employer may now decide whether to extend a permanent
appointment to the employee, primarily on the basis of the employee having met the
reasonable standards of competence and efficiency set by the employer. For the entire
duration of this three-year period, the teacher remains under probation. Upon the expiration
of his contract of employment, being simply on probation, he cannot automatically claim security of
tenure and compel the employer to renew his employment contract. As such, “no vested right to a
permanent appointment shall accrue. Mere rendition of service for three consecutive years does
not automatically ripen into a permanent appointment. It is also necessary that the employee be a
full-time teacher, and that the services he rendered are satisfactory.”

However, this scheme “of fixed-term contract is a system that operates during the probationary
period and for this reason is subject to Article 281 of the Labor Code, which provides: x x x The
services of an employee who has been engaged on a probationary basis may be terminated for a
just cause or when he fails to qualify as a regular employee in accordance with reasonable
standards made known by the employer to the employee at the time of his engagement. An
employee who is allowed to work after a probationary period shall be considered a regular
employee.

When fixed-term employment is brought into play under the above probationary period rules, the
situation – as in the present case – may at first blush look muddled as fixed-term employment is in
itself a valid employment mode under Philippine law and jurisprudence. The conflict, however, is
more apparent than real when the respective nature of fixed-term employment and of employment
on probationary status are closely examined.

An example given of a fixed-term contract specifically used for the fixed term it offers is a
replacement teacher contracted for a period of one year to temporarily take the place of a
permanent teacher who is on leave. The expiration of the reliever’s fixed-term contract does not
have probationary status implications as he or she was never employed on probationary basis. This
is because his or her employment is for a specific purpose with particular focus on the term. There
exists an intent to end his or her employment with the school upon expiration of this term.

However, for teachers on probationary employment, it is incumbent upon the school to have
not only set reasonable standards to be followed by said teachers in determining
qualification for regular employment, the same must have also been communicated to the
teachers at the start of the probationary period, or at the very least, at the start of the period
when they were to be applied. These terms, in addition to those expressly provided by the Labor
Code, would serve as the just cause for the termination of the probationary contract. The specific
details of this finding of just cause must be communicated to the affected teachers as a matter of
due process. Corollarily, should the teachers not have been apprised of such reasonable
standards at the time specified above, they shall be deemed regular employees.

36
In this case, glaringly absent from petitioners’ evidence are the reasonable standards that
respondent was expected to meet that could have served as proper guidelines for purposes of
evaluating his performance. Nowhere in the Teacher’s Contract could such standards be found.
Neither was it mentioned that the same were ever conveyed to respondent.

If the termination is brought about by the completion of a contract or phase thereof, or by


failure of an employee to meet the standards of the employer in the case of probationary
employment, it shall be sufficient that a written notice is served the employee, within a
reasonable time from the effective date of termination. As a matter of due process, teachers on
probationary employment, just like all probationary employees, have the right to know whether
they have met the standards against which their performance was evaluated. Should they fail, they
also have the right to know the reasons therefor.

It should be pointed out that absent any showing of unsatisfactory performance on the part of
respondent, it can be presumed that his performance was satisfactory, especially taking into
consideration the fact that even while he was still more than a year into his probationary
employment, he was already designated Prefect of Discipline. In such capacity, he was able to
uncover the existence of a drug syndicate within the school and lessen the incidence of drug use
therein. Yet despite respondent’s substantial contribution to the school, petitioners chose to
disregard the same and instead terminated his services.

13. MANAGEMENT PREROGATIVE

GR NO 193857, November 28, 2012

BARBA
VS
LICEO DE CAGAYAN UNIVERSITY

FACTS:
Petitioner Dr. Ma. Mercedes L. Barba was the Dean of the College of Physical Therapy of respondent
Liceo de Cagayan University, Inc., a private educational institution with school campus located at
Carmen, Cagayan de Oro City. Petitioner started working for respondent on July 8, 1993 as medical
officer/school physician for a period of one school year or until March 31, 1994. In July 1994, she
was chosen by respondent to be the recipient of a scholarship grant to pursue a three-year
residency training in Rehabilitation Medicine at the Veterans Memorial Medical Center (VMMC).
TheScholarship Contract provides:

37
That the SCHOLAR after the duration of her study and training shall serve the SCHOOL in whatever position the
SCHOOL desires related to the SCHOLAR’s studies for a period of not less than ten (10) years;

After completing her residency training with VMMC in June 1997, petitioner returned to continue working for
respondent. She was appointed as Acting Dean of the College of Physical Therapy and at the same time
designated as Doctor-In-Charge of the Rehabilitation Clinic of the Rodolfo N. Pelaez Hall, City
Memorial Hospital. On June 19, 2002, petitioner’s appointment as Doctor -In-Charge of the
Rehabilitation Clinic was renewed and she was appointed as Dean of the College of Physical
Therapy by respondent’s President, Dr. Jose Ma. R. Golez. Petitioner accepted her appointment and
assumed the position of Dean of the College of Physical Therapy.

In the school year 2003 to 2004, the College of Physical Therapy suffered a dramatic decline in the
number of enrollees from a total of 1,121 students in the school year 1995 to 1996 to only 29
students in the first semester of school year2003 to 2004. This worsened in the next year or in
school year 2004 to 2005 where a total of only 20 students enrolled. Due to the low number of
enrollees, respondent decided to freeze the operation of the College of Physical Therapy
indefinitely. Respondent’s President wrote petitioner a letter informing her that her services as
dean of the said college will end at the close of the school year. Thereafter, the College of Physical
Therapy ceased operations on March 31, 2005, and petitioner went on leave without pay starting
on April 9, 2005. Subsequently, respondent’s Executive Vice President, Dr. Lerin, sent petitioner a
letter instructing petitioner to return to work on June 1, 2005 and report to Palomares, the Acting
Dean of the College of Nursing, to receive her teaching load and assignment as a full-time faculty
member in that department for the school year 2005-2006. In reply, petitioner informed Dr. Lerin
that she had not committed to teach in the College of Nursing and that as far as she can recall, her
employment is not dependent on any teaching load. She then requested for the processing of her
separation benefits in view of the closure of the College of Physical Therapy. Dr. Magdale, Vice-
President for Academic Affairs, sent another letter to petitioner ordering her to report for work
asshe was still bound by the Scholarship Contract to serve respondent for two more years. But petitioner did not do
so. Hence, on June 28, 2005, Dr. Magdale sent petitioner a notice terminating her services on the
ground of abandonment.

Meanwhile, on June 22, 2005, prior to the termination of her services, petitioner filed a complaint
before the Labor Arbiter for illegal dismissal, payment of separation pay and retirement benefits
against respondent, Dr. Magdale and Dr. Golez. She alleged that her transfer to the College of
Nursing as a faculty member is a demotion amounting to constructive dismissal. Labor Arbiter (LA)
found that respondent did not constructively dismiss petitioner; therefore, she was not entitled to
separation pay. The Labor Arbiter held that petitioner’s assignment as full-time professor in the
College of Nursing was not a demotion tantamount to constructive dismissal. Petitioner appealed
the above decision to the NLRC. NLRC reversed the LA’s decision and held that petitioner was
constructively dismissed. The NLRC held that petitioner was demoted when she was assigned as a
professor in the College of Nursing because there are functions and obligations and certain
allowances and benefits given to a College Dean but not to an ordinary professor. Respondent went to the
CA on a petition for certiorari alleging that the NLRC committed grave abuse of discretion

38
when it declared that petitioner’s transfer to the College of Nursing as full-time professor but
without diminution of salaries and without loss of seniority rights amounted to constructive
dismissal because there was a demotion involved in the transfer and because petitioner was
compelled to accept her new assignment. The CA reversed and set aside the NLRC resolutions and
reinstated the decision of the Labor Arbiter. The CA did not find merit in respondent’s assertion in
its Supplemental Petition that the position of petitioner as College Dean was a corporate office.
Instead, the appellate court held that petitioner was respondent’s employee. CA held that contrary
to the allegation of petitioner, the position of Dean does not appear to be the same as that of a
College Director. Aside from the obvious disparity in name, the By-Laws of LDCU provides for only
one College Director. But as shown by LDCU itself, numerous persons have been appointed as Deans. If it is
indeed the intention of LDCU to give its many Deans the rank of College Director, then it exceeded the authority given
to it by its By-Laws because only one College Director is authorized to be appointed. It must amend its By-Laws.
Petitioner filed a motion for reconsideration from the above decision, but her motion was denied by
the CA. Hence, petitioner filed the present petition.

ISSUE:
Whether petitioner was constructively dismissed

RULING:
We grant the petition. The Supreme Court ruled in favor of the University, finding that Barba was
not constructively dismissed, but was instead validly transferred. On the issue of constructive
dismissal, we agree with the Labor Arbiter and the appellate court’s earlier ruling that petitioner
was not constructively dismissed. Petitioner’s letter of appointment specifically appointed her as
Dean of theCollege of Physical Therapy and Doctor-in-Charge of the Rehabilitation Clinic “ for a
period of three years effective July 1, 2002unless sooner revoked for valid cause or causes.” Evidently,
petitioner’s appointment as College Dean was for a fixed term, subject to reappointment and
revocation or termination for a valid cause. When respondent decided to close its College of
Physical Therapy due to drastic decrease in enrollees, petitioner’s appointment as its College Dean
was validly revoked and her subsequent assignment to teach in the College of Nursing was justified
as it is still related to her scholarship studies in Physical Therapy. As we observed in Brent School, Inc.
v. Zamora, also cited by the CA, it is common practice in educational institutions to have fixed-term
contracts in administrative positions, thus:

Some familiar examples may be cited of employment contracts which may be neither for seasonal work nor for
specific projects, but to which a fixed term is an essential and natural appurtenance:

overseas employment contracts, for one, to which, whatever the nature of the engagement, the concept of regular
employment with all that it implies does not appear ever to have been applied, Article 280 of the Labor Code
notwithstanding; also appointments to the positions of dean, assistant dean, college secretary,
principal, and other administrative offices in educational institutions, which are by practice or
tradition rotated among the faculty members, and where fixed terms are a necessity without which
no reasonable rotation would be possible. x x x (Emphasis supplied)

39
In constructive dismissal cases, the employer has the burden of proving that its conduct and action
or the transfer of an employee are for valid and legitimate grounds such as genuine business
necessity. Particularly, for a transfer not to be considered a constructive dismissal, the employer
must be able to show that such transfer is not unreasonable, inconvenient, or prejudicial to the
employee. In this case, petitioner’s transfer was not unreasonable, inconvenient or prejudicial to
her. On the contrary, the assignment of a teaching load in the College of Nursing was undertaken by
respondent to accommodate petitioner following the closure of the College of Physical Therapy.
Respondent further considered the fact that petitioner still has two years to serve the university
under the Scholarship Contract. Petitioner’s subsequent transfer to another department or college
is not tantamount to demotion as it was a valid transfer. There is therefore no constructive
dismissal to speak of. That petitioner ceased to enjoy the compensation, privileges and benefits as
College Dean was but a logical consequence of the valid revocation or termination of such fixed-
term position. Indeed, it would be absurd and unjust for respondent to maintain a deanship
position in a college or department that has ceased to exist. Under the circumstances, giving
petitioner a teaching load in another College/Department that is related to Physical Therapy thus –
enabling her to serve and complete her remaining two years under the Scholarship Contract – is a
valid exercise of management prerogative on the part of respondent.

[G.R. No. 191281. December 5, 2012.]

BEST WEAR GARMENTS AND/OR WARREN PARDILLA, PETITIONERS,


VS.
ADELAIDA B. DE LEMOS AND CECILE M. OCUBILLO, RESPONDENTS.

FACTS:

Petitioner Best Wear Garments is a sole proprietorship represented by its General Manager Alex
Sitosta. Respondents Cecile M. Ocubillo and Adelaida B. De Lemos were hired as sewers on piece-
rate basis by petitioners. Both filed a complaint for illegal dismissal. Sistosa arbitrarily transferred
them in other areas of operation which they alleged to be tantamount to constructive dismissal as it
resulted to less earnings for them.

De Lemos claimed that after two months in her new assignment, she was able to adjust but Sitosta
again transferred her to a "different operation where she could not earn [as] much as before
because by-products require long period of time to finish." She averred that the reason for her
transfer was her refusal "to render [overtime work] up to 7:00 p.m." Her request to be returned to
her previous assignment was rejected and she was "constrained not to report for work as Sitosta
had become indifferent to her since said transfer of operation." She further alleged that her last
salary was withheld by petitioner company.

40
On her part, Ocubillo alleged that her transfer was precipitated by her having "incurred excessive
absences since 2001." Her absences were due to the fact that her father became very sick since
2001 until his untimely demise on November 9, 2003; aside from this, she herself became very
sickly. She claimed that from September to October 2003, Sitosta assigned her to different machines
"whichever is available" and that "there were times, she could not earn for a day because there was
no available machine to work for [sic]." Sitosta also allegedly required her to render overtime work
up to 7:00 p.m. which she refused "because she was only paid up to 6:25 p.m."

Petitioners asserted that respondents are piece-rate workers and hence they are not paid according
to the number of hours worked.

ISSUES:

WON there is a valid exercise of management prerogative

HELD:

Yes.

The right of employees to security of tenure does not give them vested rights to their positions to
the extent of depriving management of its prerogative to change their assignments or to transfer
them. Thus, an employer may transfer or assign employees from one office or area of operation to
another, provided there is no demotion in rank or diminution of salary, benefits, and other
privileges, and the action is not motivated by discrimination, made in bad faith, or effected as a
form of punishment or demotion without sufficient cause.

Being piece-rate workers assigned to individual sewing machines, respondents' earnings depended
on the quality and quantity of finished products. That their work output might have been affected
by the change in their specific work assignments does not necessarily imply that any resulting
reduction in pay is tantamount to constructive dismissal. Workers under piece-rate employment
have no fixed salaries and their compensation is computed on the basis of accomplished tasks. As
admitted by respondent De Lemos, some garments or by-products took a longer time to finish so
they could not earn as much as before. Also, the type of sewing jobs available would depend on the
specifications made by the clients of petitioner company. Under these circumstances, it cannot be
said that the transfer was unreasonable, inconvenient or prejudicial to the respondents. Such
deployment of sewers to work on different types of garments as dictated by present business
necessity is within the ambit of management prerogative which, in the absence of bad faith, ill
motive or discrimination, should not be interfered with by the courts.

[G.R. No. 198783. April 15, 2013.]

41
ROYAL PLANT WORKERS UNION, PETITIONER,
VS.
COCA-COLA BOTTLERS PHILIPPINES, INC.-CEBU PLANT RESPONDENT.

FACTS:

Petitioner Coca-Cola Bottlers Philippines, Inc. (CCBPI) is a domestic corporation engaged in the
manufacture, sale and distribution of softdrink products. In the case of the plant in Cebu City, there
are 20 bottling operators who work for its Bottling Line 1 while there are 12-14 bottling operators
who man its Bottling Line 2. All of them are male and they are members of herein respondent Royal
Plant Workers Union (ROPWU).

The bottling operators work in two shifts. The first shift is from 8 a.m. to 5 p.m. and the second shift
is from 5 p.m. up to the time production operations is finished. Thus, the second shift varies and
may end beyond eight (8) hours. However, the bottling operators are compensated with overtime
pay if the shift extends beyond eight (8) hours. For Bottling Line 1, 10 bottling operators work for
each shift while 6 to 7 bottling operators work for each shift for Bottling Line 2.

In 1974, the bottling operators of then Bottling Line 2 were provided with chairs upon their
request. In 1988, the bottling operators of then Bottling Line 1 followed suit and asked to be
provided also with chairs. Their request was likewise granted. Sometime in September 2008, the
chairs provided for the operators were removed pursuant to a national directive of petitioner. This
directive is in line with the "I Operate, I Maintain, I Clean" program of petitioner for bottling
operators, wherein every bottling operator is given the responsibility to keep the machinery and
equipment assigned to him clean and safe. The program reinforces the task of bottling operators to
constantly move about in the performance of their duties and responsibilities. A

ISSUES:

WON the removal of the chairs is a valid exercise of management prerogative

HELD:

Yes.
The Court has held that management is free to regulate, according to its own discretion and
judgment, all aspects of employment, including hiring, work assignments, working methods, time,
place, and manner of work, processes to be followed, supervision of workers, working regulations,
transfer of employees, work supervision, lay-off of workers, and discipline, dismissal and recall of
workers. The exercise of management prerogative, however, is not absolute as it must be exercised
in good faith and with due regard to the rights of labor.

42
In the present controversy, it cannot be denied that CCBPI removed the operators' chairs pursuant
to a national directive and in line with its "I Operate, I Maintain, I Clean" program, launched to
enable the Union to perform their duties and responsibilities more efficiently. The chairs were not
removed indiscriminately. They were carefully studied with due regard to the welfare of the
members of the Union. The removal of the chairs was compensated by: a) a reduction of
the operating hours of the bottling operators from a two-and-one-half (2 1/2)-hour rotation
period to a one-and-a-half (1 1/2) hour rotation period; and b) an increase of the break
period from 15 to 30 minutes between rotations.
Apparently, the decision to remove the chairs was done with good intentions as CCBPI wanted to
avoid instances of operators sleeping on the job while in the performance of their duties and
responsibilities and because of the fact that the chairs were not necessary considering that the
operators constantly move about while working. In short, the removal of the chairs was designed to
increase work efficiency. Hence, CCBPI's exercise of its management prerogative was made in good
faith without doing any harm to the workers rights.

The rights of the Union under any labor law were not violated. There is no law that requires
employers to provide chairs for bottling operators. The CA correctly ruled that the Labor Code,
specifically Article 132 thereof, only requires employers to provide seats for women. No similar
requirement is mandated for men or male workers. It must be stressed that all concerned bottling
operators in this case are men.

(GR 198534 July 2013)

PECKSON
VS.
ROBINSONS

Facts:
The petitioner Jenny Peckson started working with Robinsons Supermarket Corp. (RSC) as a sales
clerk in 1987. In 2006, while holding the position of Category Buyer she was reassigned by
respondent Roena Sarte (RSC Assistant VP for Merchandising) to the position of Provincial
Coordinator effective November 1, 2006. However, the petitioner refused to turn over her Category
Buyer responsibilities or accept her new position as the Provincial Coordinator claiming that the
new assignment was a demotion because it was non-supervisory and clerical in nature. Because of
such refusal or non-acceptance, RSC sent two memorandums to the petitioner. First was on Nov 13,
2006 demanding from her a written explanation within 48hrs for her refusal to accept her new
assignment despite written and verbal demands. It cited company rule, Offenses subject to
Disciplinary Action 4.07 which states that "disobedience, refusal or failure to do assigned task or to
obey superior’s/official’s orders/instructions, or to follow established procedures or practices
without valid reason" would be meted the penalty of suspension. Since this was ignored by the
petitioner, the second memorandum was issued on November 23, 2006 reiterating respondent’s
demand for an explanation and warning the petitioner that it’s her final chance to present her side

43
otherwise, it will be deemed waived. The petitioner replied maintaining her stand that such
assignment is a demotion.

But prior to the issuance of the memorandum, petitioner already filed on Nov. 9, 2006 a complaint
for constructive dismissal against the respondents. She argued before the Labor Arbiter that the
organizational chart of RSC showed that the position of Category Buyer was one level above that of
the Provincial Coordinator and that the latter’s job description was of clerical in nature and did not
require her to analyse stock levels and order points or source new local and international suppliers,
or monitor stock level per store and recommend items for replenishment, or negotiate better items
and discounts from suppliers, duties which only a Category Buyer could perform.

The respondents on the other hand, denied the petitioner’s argument and maintained that such
transfer was not a demotion since Provincial Coordinator occupied a Level 5 similar to that of the
Category Buyer with the same work conditions, salary and benefits. Although both positions had no
significant disparity in the required skill, experience and aptitude, the position of a Category Buyer
demanded traits of punctuality , diligence and attentiveness because it is a frontline position in the
daily business operations of RSC which the petitioner lack. Respondent presented petitioner’s
record of habitual tardiness since 1999 and his poor performance in 2005 to support its claim. The
respondents further asserted that such transfer was in the exercise of management prerogrative in
view of the crucial and sensitive position of a Category Buyer in RSC’s daily business operations vis-
à-vis the petitioner’s below expectation performance and habitual tardiness.

Issue: WON the reassignment of petitioner from a Category Buyer to a Provincial Coordinator is a
valid exercise of respondent’s management prerogrative?

Ruling:
The respondent validly exercised its management prerogrative.
Under the doctrine of management prerogative, every employer has the inherent right to regulate,
according to his own discretion and judgment, all aspects of employment, including hiring, work
assignments, working methods, the time, place and manner of work, work supervision, transfer of
employees, lay-off of workers, and discipline, dismissal, and recall of employees. The only
limitations to the exercise of this prerogative are those imposed by labor laws and the principles of
equity and substantial justice.

The respondents are justified in moving the petitioner to the position of a Provincial Coordinator,
which would be likely less affected by her habitual tardiness or inconsistent attendance than if she
continued as a Category Buyer,a "frontline position" in the daily business operations of RSC. . An
employee’s right to security of tenure does not give him such a vested right in his position as would
deprive the company of its prerogative to change his assignment or transfer him where he will be
most useful. But the managerial prerogative to transfer personnel has also its limitations. It must be
exercised without grave abuse of discretion, bearing in mind the basic elements of justice and fair
play. Indeed, management prerogrative may not be used as a subterfuge by the employer to rid
himself of an undesirable worker. The employer has the burden of proof to show that the transfer is

44
not unreasonable, inconvenient or prejudicial to the employee; nor does it involve a demotion in
rank or a diminution of his salaries, privileges and other benefits. The failure to do so will
tantamount to constructive dismissal which is defined as a quitting because continued employment
is rendered impossible, unreasonable or unlikely; as an offer involving a demotion in rank and
diminution in pay and it also exist when an act of clear discrimination, insensibility or disdain by an
employer has become so unbearable to the employee leaving him with no option but to forego with
his continued employment.

The SC agrees with the CA that the transfer of the petitioner from Category Buyer to Provincial
Coordinator was not unreasonable, inconvenient, or prejudicial to her. This court also agrees with
the NLRC that the Provincial Coordinator’s position does not involve mere clerical functions but
requires the exercise of discretion from time to time, as well as independent judgment, since the
Provincial Coordinator gives appropriate recommendations to management and ensures the
faithful implementation of policies and programs of the company. It even has influence over a
Category Buyer because of its recommendatory function that enables the Category Buyer to make
right decisions on assortment, price and quantity of the items to be sold by the store. It must be
noted that judicial review of labor cases does not go beyond the evaluation of the sufficiency of the
evidence upon which labor officials’ findings rest. The findings of facts and conclusion of the NLRC
are generally accorded not only great weight and respect but even clothed with finality and deemed
binding on this Court as long as they are supported by substantial evidence. Especially when passed
upon and upheld by the Court of Appeals, they are binding and conclusive upon the Supreme Court
and will not normally be disturbed.

14. TERMINATION OF EMPLOYMENT

(GR 168208, June 13, 2012)

RAMIREZ ET.AL
VS.
MAR FISHING CO. INC.

Facts:
Respondent Mar Fishing Co. Inc. is engaged in fishing and canning tuna. On June 28, 2001, Miramar
(co-respondent) bought its principal assets and the proceeds of the sale were paid to Trade and
Investment Corp. of the Philippines (TIDCORP) to pay the former’s obligation. Mar Fishing issued a
Memorandum on Oct 23, 2001 informing its workers that the company would cease to operate by
the end of the month and it was only 2 days before its cessation that it notified DOLE.

The petitioner Mar Fishing’s Union - NFL and co –respondent Miramar entered into an agreement
that Miramar as the acquiring company shall absorb Mar Fishing’s regular rank and file employees

45
of satisfactory performance without loss of seniority rights and privileges. But petitioners were not
hired or given separation pay which prompted them to file for illegal dismissal before the
Arbitration branch of the NLRC. The Labor Arbiter held that Mar Fishing closed its operations
because Miramar already bought the former’s tuna canning plant and the petitioners were legally
dismissed for authorized cause. However, NLRC modified the LA’s decision and held that the
dismissal was ineffectual because notification to DOLE was made only 2 days before the closure of
business. It imposed liability on Mar Fishing for the money claims and held that Miramar is not
liable since labor contracts cannot be enforced against the transferee of an enterprise in the
absence of a stipulation in the contract that the transferee assumes the obligation of the transferor.
Petitioners then filed Rule 65 before the CA and argued that both Mar Fishing and Miramar must be
liable for their separation pay and back wages up to the time of their actual reinstatement. CA
dismissed the petition because only 3 of the 228 petitioners signed the Verification and
Certification against forum shopping. Petitioners filed for a reconsideration and invoked the rule
that technical rules do not strictly apply to labor cases. CA ruled that although litigation is not a
game of technicalities and that rules of procedure should not be enforced strictly at the cost of
substantial justice , still the Rules of Court may not be ignored at will and at random to the
prejudice of the orderly presentation, assessment and just resolution of the issues. But this court
recognized the merits of the case and addressed the following issues.

Issues: 1. WON Mar Fishing and Miramar are solidarily liable?


2. What is the reckoning period for the award of back wages?

Ruling:

A dismissal based on the closure of business must comply with the following requirements:

1. The cessation of or withdrawal from business operations must be bona fide in character.
2. There must be payment to the employees of termination pay amounting to at least one-half
(1/2) month pay for each year of service, or one (1) month pay, whichever is higher.
3. The company must serve a written notice on the employees and on the DOLE at least one
(1) month before the intended termination.

The petition for review on certiorari filed by Mari Fishing Union only questioned the holding that
only Mar Fishing was liable for their monetary claims. They did not dispute the authorized cause of
their dismissal made by Mar Fishing.

This Court sustains the ruling of the LA as affirmed by the NLRC that Miramar and Mar Fishing are
separate and distinct entities, based on the marked differences in their stock ownership. Also, the
fact that Mar Fishing’s officers remained as such in Miramar does not by itself warrant a conclusion
that the two companies are one and the same. As held in Sesbreño v. Court of Appeals, the mere
showing that the corporations had a common director sitting in all the boards without more does
not authorize disregarding their separate juridical personalities.

46
Neither can the veil of corporate fiction between the two companies be pierced. To pierce the veil of
corporate fiction it must be clearly establish that the separate and distinct personalities of the
corporations are set up to justify a wrong, protect a fraud, or perpetrate a deception. This,
unfortunately, petitioners have failed to do. Having been found by the trial courts to be a separate
entity, Mar Fishing – and not Miramar – is required to compensate petitioners. Indeed, the back
wages and retirement pay earned from the former employer cannot be filed against the new
owners or operators of an enterprise. This court will no longer disturb on the exact computation of
the back wages which was sufficiently threshed out by the LA and the NLRC. Judicial review of labor
cases does not go beyond an evaluation of the sufficiency of the evidence upon which labor officials'
findings rest. Generally, the findings of fact and the conclusion of the labor courts are not only
accorded great weight and respect, but are even clothed with finality and deemed binding on this
Court, as long as they are supported by substantial evidence.

G.R. No. 185335, June 13, 2012

PRUDENTIAL GUARANTEE AND ASSURANCE EMPLOYEE LABOR UNION AND SANDY T.


VALLOTA
VS.
NLRC

Facts of the case:

 Petitioner Vallota is Junior Programmer assigned to the Electronic Data Processing (EDP)
Department with respondent Prudential Guarantee and Assurance, Inc. (PGAI).
 On November 11, 2005, PGAI’s Human Resource Manager, Atty. Joaquin R. Rillo (Atty. Rillo),
together Union President, Mike Apostol (Apostol) and other conducted an on-the-spot security
check in the Information and Technology (IT) Department. The inspection team proceeded to the IT
Department, PGAI network administrator Angelo Gutierrez (Gutierrez), initiated the spot check of
IT Department computers, beginning with the one assigned to Vallota. After exploring the contents
of all the folders and subfolders a folder named "MAA” was found.
 On November 14, 2005, Vallota received a memorandum directing him to explain within 72
hours why highly confidential files were stored in his computer.
 In a letter dated December 6, 2005, Vallota requested a conference, to be attended by a
Union representative and counsel. The Union sent a letter to PGAI President requesting that a
grievance committee be convened, which was ignored.
 On December 21, 2005, Vallota was given a notice of termination of his employment
effective January 10, 2006 on the ground of loss of trust and confidence.
 Thus, the petitioners filed a complaint for illegal dismissal with claims for full backwages,
moral and exemplary damages with the Labor Arbiter with a favourable judgment which was later
reversed by the NLRC.
 The Court of Appeals dismissed the appeal of the petitioners for lack of merit and hence the
case reached the Supreme Court.

47
Issues of the Case:

 Whether or not petitioner was validly dismissed on the ground of trust and confidence.
 Whether or not the procedural due process requirements for termination were observed.
 Whether or not petitioner should be reinstated.

Ruling of the Court:

First issue: Petitioner was not validly dismissed on the ground of loss of trust and confidence

Loss of trust and confidence and its requisites

 Loss of confidence as a just cause for dismissal was never intended to provide employers
with a blank check for terminating their employees. Loss of confidence should ideally apply only to
cases involving employees occupying positions of trust and confidence or to those situations where
the employee is routinely charged with the care and custody of the employer's money or property.
To the first class belong managerial employees, i.e., those vested with the powers or prerogatives
to lay down management policies and/or to hire, transfer, suspend, lay-off, recall, discharge, assign
or discipline employees or effectively recommend such managerial actions; and to the second class
belong cashiers, auditors, property custodians, etc., or those who, in the normal and routine
exercise of their functions, regularly handle significant amounts of money or property.
 To be a valid ground for dismissal, loss of trust and confidence must be based on a willful
breach of trust and founded on clearly established facts. A breach is willful if it is done intentionally,
knowingly and purposely, without justifiable excuse, as distinguished from an act done carelessly,
thoughtlessly, heedlessly or inadvertently. It must rest on substantial grounds and not on the
employer’s arbitrariness, whims, caprices or suspicion; otherwise, the employee would remain
eternally at the mercy of the employer. Further, in order to constitute a just cause for dismissal, the
act complained of must be work-related and show that the employee concerned is unfit to continue
working for the employer. Such ground for dismissal has never been intended to afford an occasion
for abuse because of its subjective nature.
 The first requisite for dismissal on the ground of loss of trust and confidence is that
the employee concerned must be one holding a position of trust and confidence.
 The second requisite is that there must be an act that would justify the loss of trust and
confidence. Loss of trust and confidence to be a valid cause for dismissal must be based on a willful
breach of trust and founded on clearly established facts.

First requisite applied to the facts of the case

48
 Vallota’s position as Junior Programmer is analogous to the second class of positions of
trust and confidence. Though he did not physically handle money or property, he became privy to
confidential data or information by the nature of his functions. At a time when the most sensitive of
information is found not printed on paper but stored on hard drives and servers, an employee who
handles or has access to data in electronic form naturally becomes the unwilling recipient of
confidential information.

Second requisite applied to the facts of the case

 The act alleged to have caused the loss of trust and confidence of PGAI in Vallota was the
presence in his computer’s hard drive of a folder named "MAA" allegedly containing files with
information on MAA Mutual Life Philippines, a domestic corporation selling life insurance policies
to the buying public, and files relating to PGAI’s internal affairs.

Sufficiency of the act that would justify the loss of trust and confidence and burden of proof
on the employer.

 While the law and this Court recognize the right of an employer to dismiss an employee
based on loss of trust and confidence, the evidence of the employer must clearly and convincingly
establish the facts upon which the loss of trust and confidence in the employee is based. The
employer’s case succeeds or fails on the strength of its evidence and not the weakness of that
adduced by the employee, in keeping with the principle that the scales of justice should be tilted in
favor of the latter in case of doubt in the evidence presented by them. Often described as more than
a mere scintilla, the quantum of proof is substantial evidence which is understood as such relevant
evidence as a reasonable mind might accept as adequate to support a conclusion, even if other
equally reasonable minds might conceivably opine otherwise. Failure of the employer to discharge
the foregoing onus would mean that the dismissal is not justified and, therefore, illegal.
 In this case, there was no other evidence presented to prove fraud in the manner of securing
or obtaining the files found in Vallota’s computer. In fact, aside from the presence of these files in
Vallota’s hard drive, there was no other evidence to prove any gross misconduct on his part. There
was no proof either that the presence of such files was part of an attempt to defraud his employer
or to use the files for a purpose other than that for which they were intended. If anything, the
presence of the files reveals some degree of carelessness or neglect in his failure to delete them, but
it is an extremely farfetched conclusion bordering on paranoia to state that it is part of a larger
conspiracy involving corporate espionage.
 Moreover, contrary to the respondents’ allegations, the MAA files found in Vallota’s
computer, the prospectus and corporate profile, are not sensitive corporate documents. These are
documents routinely made available to the public, and serve as means to inform the public about
the company and to disseminate information about the products it sells or the services it provides.

49
 Such act is not of the same class, degree or gravity as the acts that have been held to be of
such character. While Vallota’s act or omission may have been done carelessly, it falls short of the
standard required for termination of employment. It does not manifest either that the employee
concerned is unfit to continue working for his employer.

Second Issue: The procedural due process requirements for termination were not observed

 The following are the guiding principles in connection with the hearing requirement in
dismissal cases:
 "Ample opportunity to be heard" means any meaningful opportunity (verbal or written)
given to the employee to answer the charges against him and submit evidence in support of his
defense, whether in a hearing, conference or some other fair, just and reasonable way.
 A formal hearing or conference becomes mandatory only when requested by the employee
in writing or substantial evidentiary disputes exist or a company rule or practice requires it, or
when similar circumstances justify it.
 The "ample opportunity to be heard" standard in the Labor Code prevails over the "hearing
or conference" requirement in the implementing rules and regulations.
 In this case, the two-notice requirement was complied with. By the petitioners’ own
admission, PGAI issued to Vallota a written Notice of Charges & Preventive Suspension dated
November 14, 2005. After an exchange of memoranda, PGAI then informed Vallota of his dismissal
in its decision dated December 21, 2005.
 Given, however, that the petitioners expressly requested a conference or a convening of a
grievance committee, following the Court’s ruling in the Perez case, such formal hearing became
mandatory. After PGAI failed to affirmatively respond to such request, it follows that the hearing
requirement was not complied with and, therefore, Vallota was denied his right to procedural due
process.

Third Issue: Reinstatement is no longer a feasible relief

 An illegally dismissed employee is entitled to two reliefs: backwages and reinstatement. The
two reliefs provided are separate and distinct. In instances where reinstatement is no longer
feasible because of strained relations between the employee and the employer, separation pay is
granted. In effect, an illegally dismissed employee is entitled to either reinstatement, if viable, or
separation pay if reinstatement is no longer viable, and backwages.
 The normal consequences of respondents’ illegal dismissal, then, are reinstatement without
loss of seniority rights, and payment of backwages computed from the time compensation was
withheld up to the date of actual reinstatement. Where reinstatement is no longer viable as an
option, under the doctrine of strained relations, the, separation pay equivalent to one (1) month

50
salary for every year of service should be awarded as an alternative. The payment of separation pay
is in addition to payment of backwages.
 In view of the strained relations between Vallota and PGAI, it is not in the best interest of
the parties, nor is it advisable or practical to order reinstatement therefore, petitioner must be
awarded separation pay.

G.R. No. 17618 June 13, 2012

ROMEO E. PAULINO
VS.
NLRC

FACTS OF THE CASE:

Petitioner, Romeo Paulino, was then an employee of private respondent Philippine Long Distance
Telephone Company, Inc. (PLDT) as Cable Splicer III. Petitioner brought home some plant materials
belonging to respondent which stayed with the former for 1 month and 11 days, until Members of
the Philippine National Police (PNP), retrieved it by virtue of a search warrant. Petitioner did not
present any documents that would justify his possession of the materials.

Respondent then caused the filing of an information for qualified theft against petitioner.
Respondent then received a security report stating that petitioner had engaged in the illicit disposal
of its plant materials, which were recovered during the search conducted at his residence.

Respondent then issued an Inter-Office Memo requiring petitioner to explain why he should not be
terminated from employment for serious misconduct (theft of company property). In reply,
petitioner requested that the proceedings be held in abeyance until the criminal case against him
had been concluded. Respondent then informed petitioner in writing that since his reply did not
provide any clarification whatsoever that would have warranted an evaluation of his case, the
company was terminating his services effective on the said date.

Three years later, after the criminal case for qualified theft had been terminated for failure of the
prosecution to prove his guilt beyond reasonable doubt, petitioner filed a Complaint for Illegal
Dismissal with the Labor Arbiter (LA) which was dismissed. Appeal to the NLRC and the Court of
Appeals concluded the same result, hence the case reached the Supreme Court.

ISSUES OF THE CASE:

51
 Whether or not Petitioner, Romeo Paulino, was validly terminated from service.

RULING OF THE COURT:

Romeo Paulino was validly terminated, hence the petition was dismissed.

Right of the Employer to terminate an employee

 The Labor Code recognizes that an employer, for just cause, may validly terminate the
services of an employee for serious misconduct or willful disobedience of the lawful orders of the
employer or representative in connection with the employee’s work. Fraud or willful breach by the
employee of the trust reposed by the employer in the former, or simply loss of confidence, also
justifies an employee’s dismissal from employment.

On the dismissal of the criminal case for theft

 The Supreme Court held that it was correct for the respondent tribunals to have dismissed
the illegal dismissal case despite the dismissal of the criminal case for theft since proof beyond
reasonable doubt of an employee’s misconduct is not required in dismissing an employee. Rather,
as opposed to the "proof beyond reasonable doubt" standard of evidence required in criminal cases,
labor suits require only substantial evidence to prove the validity of the dismissal.

Willful breach of trust or loss of confidence as grounds for dismissal

 Willful breach of trust or loss of confidence requires that the employee (1) occupied a
position of trust or (2) was routinely charged with the care of the employer’s property. Here,
petitioner was charged with the care and custody of PLDT’s property.
 To warrant dismissal based on loss of confidence, there must be some basis for the loss of
trust or the employer must have reasonable grounds to believe that the employee is responsible for
misconduct that renders the latter unworthy of the trust and confidence demanded by his or her
position
 Petitioner argues that there was no sufficient basis for loss of trust and confidence by PLDT
as he did not steal the plant materials considering that he had lawful possession thereto. The
Supreme Court buttressed his contention saying that, Assuming that petitioner lawfully possessed
the materials, PLDT still had ample reason or basis to already distrust petitioner. For more than a
month, he did not even inform PLDT of the whereabouts of the plant materials. Instead, he stocked
these materials at his residence even if they were needed in the daily operations of the company. In
keeping with the honesty and integrity demanded by his position, he should have turned over these
materials to the plant’s warehouse. The fact that petitioner did not present any documents or
requisition slips at the time that the PNP took the plant materials logically excites suspicion. In
addition, PLDT received a security report stating that petitioner had engaged in the illicit disposal
of its plant materials, which were recovered during the search conducted at his residence

52
 Thus, PLDT reasonably suspected petitioner of stealing the company’s property. At that
juncture, the employer may already dismiss the employee since it had reasonable grounds to
believe or to entertain the moral conviction that the latter was responsible for the misconduct, and
the nature of his participation therein rendered him absolutely unworthy of the trust and
confidence demanded by his position.

Significance of company rules

 Petitioner claims that he could only be faulted for breaching PLDT’s rules and regulations
which prohibited the employees from bringing home company materials. The Supreme Court held
that by admitting that he breached company rules, he buttressed his employer’s claim that he
committed serious misconduct.
 The Supreme Court then held that employees cannot take company rules for granted,
especially in this case where petitioner’s breach involved various plant materials that may cause
major disruption in the company’s operations. Indeed, an employer may discharge an employee for
refusal to obey a reasonable company rule.

G.R. No. 194795. June 13, 2012

EVER ELECTRICAL MANUFACTURING, INC., (EEMI)


VS.
SAMAHANG MANGGAGAWA NG EVER ELECTRICAL

FACTS:
The controversy started when EEMI closed its business operations on October 11, 2006 resulting in
the termination of the services of its employees. Aggrieved, respondents filed a complaint for illegal
dismissal with prayer for payment of 13th month pay, separation pay, damages, and attorney's fees.
Respondents alleged that the closure was made without any warning, notice or memorandum and
in full disregard of the requirements of the Labor Code.

In its defense, EEMI explained that it had closed the business due to various factors. In 1995, it
invested in Orient Commercial Banking Corporation (Orient Bank) the sum of P500,000,000.00 and
during the Asian Currency crises, various economies in the South East Asian Region were hurt
badly. EEMI was one of those who suffered huge losses. In November 1996, it obtained a loan in the
amount of P121,400,000.00 from United Coconut Planters Bank (UCPB). As security for the loan,
EEMI's land and its improvements, including the factory, were mortgaged to UCPB.

EEMI's business suffered further losses due to the continued entry of cheaper goods from China and
other Asian countries. Adding to EEMI's financial woes was the closure of Orient Bank where most

53
of its resources were invested. As a result, EEMI was not able to meet its loan obligations with
UCPB. TEcADS

In an attempt to save the company, EEMI entered into a dacion en pago arrangement with UCPB
which, in effect, transferred ownership of the company's property to UCPB as reflected in TCT No.
429159. Originally, EEMI wanted to lease the premises to continue its business operation but under
UCPB's policy, a previous debtor who failed to settle its loan obligation was not eligible to lease its
acquired assets. Thus, UCPB agreed to lease it to an affiliate corporation, EGO Electrical Supply Co.,
Inc. (EGO), for and in behalf of EEMI. On February 2, 2002, a lease agreement was entered into
between UCPB and EGO. 4 The said lease came to a halt when UCPB instituted an unlawful detainer
suit against EGO before the Metropolitan Trial Court, Branch 5, Makati City (MeTC) docketed as Civil
Case No. 88602. On August 11, 2006, the MeTC ruled in favor of UCPB and ordered EGO to vacate
the leased premises and pay rentals to UCPB in the amount of P21,473,843.65. 5 On September 19,
2006, a writ of execution was issued. 6 Consequently, on October 11, 2006, the Sheriff implemented
the writ by closing the premises and, as a result, EEMI's employees were prevented from entering
the factory.

On April 25, 2007, the Labor Arbiter (LA) ruled that respondents were not illegally dismissed. It,
however, ordered EEMI and its President, Vicente Go (Go), to pay their employees separation pay
and 13th month pay respectively. 7 The decretal portion of the LA decision, reads:
On September 15, 2008, the NLRC reversed and set aside the decision of the LA. The NLRC
dismissed the complaint for lack of merit and ruled that since EEMI's cessation of business
operation was due to serious business losses, the employees were not entitled to separation pay. 8
Respondents moved for reconsideration of the NLRC decision, but the NLRC denied the motion in
its March 23, 2009 Resolution. 9

Unperturbed, respondents elevated the case before the CA via a petition for certiorari under Rule
65. 10

The CA held that respondents were entitled to separation pay and 13th month pay because the
closure of EEMI's business operation was effected by the enforcement of a writ of execution and not
by reason of business losses. The CA, citing Restaurante Las Conchas v. Lydia Llego, 12 upheld the
solidary liability of EEMI and Go, declaring that "when the employer corporation is no longer
existing and unable to satisfy the judgment in favor of the employees, the officers should be held
liable for acting on behalf of the corporation." 13

EEMI and Go filed a motion for reconsideration but it was denied in the CA Resolution dated
December 16, 2010. 14
Hence, this petition. 15

Issues:
1.Whether the CA erred in finding that the closure of EEMI's operation was not due to
business losses; and

54
2.Whether the CA erred in finding Vicente Go solidarily liable with EEMI.

HELD:

The petition is partly meritorious.


Article 283 of the Labor Code provides:
Art. 283.Closure of establishment and reduction of personnel. — The employer may also terminate
the employment of any employee due to the installation of labor saving devices, redundancy,
retrenchment to prevent losses or the closing or cessation of operation of the establishment or
undertaking unless the closing is for the purpose of circumventing the provisions of this Title, by
serving a written notice on the workers and the Ministry of Labor and Employment at least one (1)
month before the intended date thereof. In case of termination due to the installation of labor
saving devices or redundancy, the worker affected thereby shall be entitled to a separation pay
equivalent to at least his one (1) month pay or to at least one (1) month pay for every year of
service, whichever is higher. In case of retrenchment to prevent losses and in cases of closures or
cessation of operations of establishment or under taking not due to serious business losses or
financial reverses, the separation pay shall be equivalent to one (1) month pay or at least one-half
(1/2) month pay for every year of service, whichever is higher. A fraction of at least six (6) months
shall be considered one (1) whole year. TSADaI

Article 283 of the Labor Code identifies closure or cessation of operation of the establishment as
an authorized cause for terminating an employee. Similarly, the said provision mandates that
employees who are laid off from work due to closures that are not due to business insolvency
should be paid separation pay equivalent to one-month pay or to at least one-half month pay for
every year of service, whichever is higher. A fraction of at least six months shall be considered one
whole year.

Although business reverses or losses are recognized by law as an authorized cause, it is still
essential that the alleged losses in the business operations be proven convincingly; otherwise, this
ground for termination of employment would be susceptible to abuse by conniving employers, who
might be merely feigning business losses or reverses in their business ventures in order to ease out
employees. 16

In this case, EEMI failed to establish that the main reason for its closure was business
reverses. As aptly observed by the CA, the cessation of EEMI's business was not directly
brought about by serious business losses or financial reverses, but by reason of the
enforcement of a judgment against it. Thus, EEMI should be required to pay separation pay
to its affected employees.

As to whether or not Go should be held solidarity liable with EEMI, the Court agrees with the
petitioner.

55
As a general rule, corporate officers should not be held solidarily liable with the corporation for
separation pay for it is settled that a corporation is invested by law with a personality separate and
distinct from those of the persons composing it as well as from that of any other legal entity to
which it may be related. Mere ownership by a single stockholder or by another corporation of all or
nearly all of the capital stock of a corporation is not of itself sufficient ground for disregarding the
separate corporate personality. 17

In two subsequent cases, the Court's ruling in Restaurante Las Conchas was invoked but the Court
refused to consider it reasoning out that it was the exception rather than the rule. The two cases
were Mandaue Dinghow Dimsum House, Co., Inc. and/or Henry Uytengsu v. National Labor Relations
Commission 25 andPantranco Employees Association (PEA-PTGWO) v. National Labor Relations
Commission. 26

In Mandaue Dinghow Dimsum House, Co., Inc., the Court declined to apply the ruling in Restaurante
Las Conchas because there was no evidence that the respondent therein, Henry Uytengsu, acted in
bad faith or in excess of his authority. It stressed that a corporation is invested by law with a
personality separate and distinct from those of the persons composing it as well as from that of any
other legal entity to which it may be related. For said reason, the doctrine of piercing the veil of
corporate fiction must be exercised with caution. 27 Citing Malayang Samahan ng mga
Manggagawa sa M. Greenfield v. Ramos, 28 the Court explained that corporate directors and officers
are solidarily liable with the corporation for the termination of employees done with malice or bad
faith. It stressed that bad faith does not connote bad judgment or negligence; it imports a dishonest
purpose or some moral obliquity and conscious doing of wrong; it means breach of a known duty
through some motive or interest or ill will; it partakes of the nature of fraud. CIDaTc

Clearly, what can be inferred from the earlier cases is that the doctrine of piercing the corporate
veil applies only in three (3) basic areas, namely: 1) defeat of public convenience as when the
corporate fiction is used as a vehicle for the evasion of an existing obligation; 2) fraud cases or
when the corporate entity is used to justify a wrong, protect fraud, or defend a crime; or 3) alter ego
cases, where a corporation is merely a farce since it is a mere alter ego or business conduit of a
person, or where the corporation is so organized and controlled and its affairs are so conducted as
to make it merely an instrumentality, agency, conduit or adjunct of another corporation. In the
absence of malice, bad faith, or a specific provision of law making a corporate officer liable,
such corporate officer cannot be made personally liable for corporate
liabilities. 29 [Emphasis supplied]

Similarly, in the case at bench, the records do not warrant an application of the exception. The rule,
which requires the presence of malice or bad faith, must still prevail. In the recent case of Wensha
Spa Center and/or Xu Zhi Jie v. Yung, 30 the Court absolved the corporation's president from liability
in the absence of bad faith or malice. In the said case, the Court stated:

In labor cases, corporate directors and officers may be held solidarily liable with the
corporation for the termination of employment only if done with malice or in bad

56
faith. 31 Bad faith does not connote bad judgment or negligence; it imports a dishonest
purpose or some moral obliquity and conscious doing of wrong; it means breach of a known
duty through some motive or interest or ill will; it partakes of the nature of fraud. 32

In the present case, Go may have acted in behalf of EEMI but the company's failure to operate
cannot be equated to bad faith. Cessation of business operation is brought about by various causes
like mismanagement, lack of demand, negligence, or lack of business foresight. Unless it can be
shown that the closure was deliberate, malicious and in bad faith, the Court must apply the general
rule that a corporation has, by law, a personality separate and distinct from that of its owners. As
there is no evidence that Go, as EEMI's President, acted maliciously or in bad faith in handling their
business affairs and in eventually implementing the closure of its business, he cannot be held jointly
and solidarily liable with EEMI.

WHEREFORE, the petition is PARTIALLY GRANTED. The August 31, 2010 Decision of the Court of
Appeals is AFFIRMED with MODIFICATION that Vicente Go is not solidarily liable with Ever
Electrical Manufacturing, Inc. aDICET
SO ORDERED.

G.R. No 174214. June 13, 2012

WATERFRONT CEBU CITY HOTEL


VS.
JIMENEZ.

FACTS:

Respondents Ma. Melanie P. Jimenez, Jacqueline C. Baguio, Lovella V. Carillo, and Maila G. Roble
were hired for Club Waterfront (the Club), a division under petitioner Waterfront Cebu City Hotel
(the Hotel) which catered to foreign high stakes gamblers, 3 for different positions and dates as
indicated below:

NAME POSITIONS DATE HIRED MONTHLY


SALARY
Ma. Melanie P. Guest Services Sept. 4, 1996 P15,148.67
Jimenez Assistant
Jacqueline Cosep Treasury June 1, 1996 P16,082.22
Baguio Supervisor
Lovella V. Carillo Guest Services May 26, 1998 P15,652.00
Assistant
Maila G. Roble Pit Supervisor Sept. 1, 1999 P16,452.00[4]

57
On 12 May 2003, respondents received identical letters of termination from petitioner's Director of
Human Resources informing them of the temporary suspension of business of the Club. A total of 45
employees were notified of the imminent closure.

On the following day, petitioner served the notice of suspension of business with the Department of
Labor and Employment (DOLE).

The dismissed employees were offered separation pay equivalent to half-month pay for every year
of service. The Club's closure took effect on 15 June 2003.

On 26 June 2003, respondents filed a complaint before the Labor Arbiter for illegal dismissal, illegal
suspension, and non-payment of salaries and other monetary benefits. They likewise prayed for
damages and attorney's fees. 5

Respondents refused to believe that the Club was suffering from losses because they knew exactly
the number of arrivals as well as junket clients of the Club. They presented documents 6 to show
the arrival of foreign guests at the Club.

Respondents maintained that upon the other hand, they are employees of petitioner assigned to the
Club, hence they should have been allowed to work in other departments of the hotel.

Oppositely, petitioner averred that since April 2002, the Club has been incurring losses that it had
to temporarily cease its operations effective 15 June 2003. To support the allegations of losses,
petitioner presented financial statements of Waterfront Promotion, Ltd. Petitioner argued that
pursuant to Article 286 of the Labor Code, the temporary suspension of business operations does
not terminate employment. Thus, respondents have no cause of action against them.

On 12 December 2003, Labor Arbiter Ernesto F. Carreon ruled in favor of petitioner and upheld the
closure of the Club's business operations as a management prerogative. The petitioner was,
however, directed to comply with Article 283 of the Labor Code and to pay complainants their
separation pay equivalent to one-half month pay for every year of service, a fraction of at least 6
months being considered as one year.

The other claims and the case against individual respondents are dismissed for lack of merit. 7
Respondents appealed to the NLRC 8 which issued a Decision affirming the ruling of the Labor
Arbiter. The NLRC observed that petitioner was able to substantiate the losses suffered by the Club
through financial statements properly audited by an independent auditor.

After the denial of respondents' motion for reconsideration, they elevated the case to the Court of
Appeals.

Respondents argued that the NLRC should have considered the financial statements of the
petitioner Hotel and not merely of the Club, which is only a division of the Hotel. According to

58
respondents, the permanent closure of the Club resulted in retrenchment but petitioner failed to
prove that it complied with the standards for retrenchment.

The appellate court found that petitioner Hotel is the actual employer of respondents, thus the
evidence of losses and closure of the Club is immaterial and irrelevant. The appellate court stated
that there is no independent evidence on record that petitioner Hotel incurred losses sufficient to
sustain the termination of respondents. Absent a clear, valid and legal cause for the termination of
employment, the appellate court opined that there is illegal dismissal. The appellate court
disregarded the audited financial statement of Waterfront Promotions, Ltd. on the ground that said
statement does not prove that the Club has become a losing proposition because it was not shown
that the Club is a division of Waterfront Promotions. Neither was it proven that Waterfront
Promotions and petitioner are one and the same. 10

Petitioner filed a motion for reconsideration but it was denied in a Resolution dated 15 August
2006.

ISSUE: WON there is retrenchment; WON it is validly;

HELD:
At the outset, it should be stated that the respondents cannot be accommodated in other
departments of the Hotel. The duties and functions they perform are peculiar to the positions they
hold in the Club. It is likewise undisputed that the Club remained closed and there is no other
department in the Hotel similar to the Club and which catered to foreign high stakes gamblers.
Verily, reinstatement cannot be and could not have been an option for petitioner Hotel.
For the purpose of proving financial losses, petitioner presented the financial statements of
Waterfront Promotion, Ltd. which petitioner describes as the company which promotes, markets
and finances the Club. 13

Waterfront Promotion, Ltd. is a wholly-owned subsidiary of Waterfront Philippines. Petitioner


Hotel, as shown in the official records, is also another subsidiary of Waterfront
Philippines. 15 Strictly speaking, the Club is not related to petitioner except to say that they are two
different subsidiaries of one parent corporation, i.e., Waterfront Philippines. Petitioner, then, could
have right at the beginning avoided the conflict with respondents by setting itself apart from them.
Petitioner could have invoked the separateness from the Hotel of the Club which employed
respondents. Petitioner did not do so. Instead, and at the outset, it formally presented itself as the
respondents' employer when, through its Director of Human Resources, it informed respondents
about the temporary suspension of the business of the Club and forthwith served the notices of
suspension of business on DOLE.

We find the consolidated financial statements that were prepared in the name of Waterfront
Promotion refer to the casino operations of the Club. A consolidated financial statement is usually
prepared for a parent company and its subsidiaries, the purpose of which is to provide an overview

59
of the financial condition of the group of companies as a single entity. The Club, being a wholly-
owned subsidiary of Waterfront Promotion, Ltd. operates under the management, supervision and
control of Waterfront Promotion, Ltd. The relationship between these two companies is so
intertwined that the Club is practically considered a department or division of Waterfront
Promotion, Ltd.

A review of the consolidated financial statement shows that for the fiscal years 2002 and 2003, the
parent company and the consolidated companies reflect the same amounts of losses: United States
(U.S.) $2,791,104.00 for 2002 and U.S. $765,222.00 for 2003. This proves petitioner's assertion that
the losses there reflected refer to the losses of the Club.

The consolidated financial statement and the corporate relationships it indicates, cannot, however,
be relied upon by petitioner to avoid this particular labor dispute because, as already stated,
petitioner itself has been claiming from the very beginning that the Club is only a
division/department of the hotel.

Verily, retrenchment and not closure was effected to warrant the valid dismissal of respondents.
Petitioner has not totally ceased its operations. It merely closed down a department.
Retrenchment is the termination of employment initiated by the employer through no fault of and
without prejudice to the employees. It is resorted to during periods of business recession, industrial
depression, or seasonal fluctuations or during lulls occasioned by lack of orders, shortage of
materials, conversion of the plant for a new production program or the introduction of new
methods or more efficient machinery or of automation. 16 It is an act of the employer of dismissing
employees because of losses in the operation of a business, lack of work, and considerable
reduction on the volume of his business. 17 TSIEAD

In case of retrenchment, proof of financial losses becomes the determining factor in proving its
legitimacy. In establishing a unilateral claim of actual or potential losses, financial statements
audited by independent external auditors constitute the normal method of proof of profit and loss
performance of a company. The condition of business losses justifying retrenchment is normally
shown by audited financial documents like yearly balance sheets and profit and loss statements as
well as annual income tax returns. 18

Retrenchment is subject to faithful compliance with the substantative and procedural requirements
laid down by law and jurisprudence. 19 For a valid retrenchment, the following elements must be
present:
(1)That retrenchment is reasonably necessary and likely to prevent business losses which, if
already incurred, are not merely de minimis, but substantial, serious, actual and real, or if only
expected, are reasonably imminent as perceived objectively and in good faith by the employer;
(2)That the employer served written notice both to the employees and to the Department of Labor
and Employment at least one month prior to the intended date of retrenchment;
(3)That the employer pays the retrenched employees separation pay equivalent to one (1) month
pay or at least 1/2 month pay for every year of service, whichever is higher;

60
(4)That the employer exercises its prerogative to retrench employees in good faith for the
advancement of its interest and not to defeat or circumvent the employees' right to security of
tenure; and
(5)That the employer used fair and reasonable criteria in ascertaining who would be dismissed and
who would be retained among the employees, such as status, efficiency, seniority, physical fitness,
age, and financial hardship for certain workers. 20

All these elements were successfully proven by petitioner. First, the huge losses suffered by the
Club for the past two years had forced petitioner to close it down to avert further losses which
would eventually affect the operations of petitioner. Second, all 45 employees working under the
Club were served with notice of termination. The corresponding notice was likewise served to the
DOLE one month prior to retrenchment. 21 Third, the employees were offered separation pay, most
of whom have accepted and opted not to join in this complaint. Fourth, cessation of or withdrawal
from business operations was bona fide in character and not impelled by a motive to defeat or
circumvent the tenurial rights of employees. 22 As a matter of fact, as of this writing, the Club has
not resumed operations. Neither is there a showing that petitioner carried out the closure of the
business in bad faith. No labor dispute existed between management and the employees when the
latter were terminated.

Finally, we affirm the NLRC's award and computation of separation pay in favor of respondents.
WHEREFORE, the petition is hereby GRANTED. The 5 July 2006 Decision and 15 August 2006
Resolution of the Court of Appeals in CA-G.R. SP No. 01548 areREVERSED and SET ASIDE. The 27
June 2005 Decision and 18 November 2005 Resolution of the National Labor Relations Commission
in NLRC NCR Case No. V-000482-2004 are REINSTATED. IaD
TES

G.R. No. 194106 June 18, 2012

MANILA ELECTRIC COMPANY (MERALCO),


VS.
HERMINIGILDO H. DEJAN

(summary-Dejan, a branch rep, no ordinary EE, required to act with highest degree of honesty,
integrity, sincerity, released meter sockets without following the company rules-authorization,
alleging that his act is an accepted practice in the office-once deposit is made, meter sockets can be
released. Meralco dismissed him on the basis of Art282 LC(just causes of termination-serious
misconduct or wilful disobedience by the EE of the lawful orders of ER in connection with his work and
loss of trust and confidence. SC upheld Meralco)

Facts:

61
Respondent Herminigildo Dejan commenced employment with the Manila Electric Company
(Meralco) on July 7, 1992. He was then Meralco's branch representative in its San Pedro, Laguna
branch, with a monthly salary of P30,500.00. His work consisted of accepting payments of the
required fees from applicants for electric service installation and issuing the corresponding meter
sockets/bases after payment of a deposit, preceded by an inspection of the premises to be
energized by a Meralco field personnel.

In the mid-afternoon of March 18, 2005, the security guard on duty at the branch, Warlito Silverio,
noticed a certain Estanislao Gozarin a.k.a. Mang Islao, a private electrician, take out from the branch
premises 20 pieces of meter sockets which were then loaded into a parked Meralco contracted jeep
belonging to one Cesar Reyes. Reyes brought the meter sockets to his house. The meter sockets
were thereafter allegedly picked up by Gil Duenas, a Meralco field representative. Dejan was asked
to explain the incident.
In his letter-explanation, dated March 23, 2005, to a certain Emilia SJ Reaso, Dejan admitted that he
released the meter sockets in question because the deposit fees had already been paid. He alleged
that he released the items even without authorization as it had been the accepted practice in the
office, provided the deposit fee had been paid.
Marcelino Rosario, head of Meralco's Investigation-Paralegal Services, charging him with the
unauthorized taking of 20 meter sockets, in violation of Section 7, paragraphs 4 and 11 of the
Company Code of Employee Discipline, in relation to Article 282 of the Labor Code. After
investigation, Meralco served Dejan a letter on April 6, 2006, terminating his employment
effective the following day, with forfeiture of all rights and privileges.

On April 20, 2006, Dejan filed his complaint with the National Labor Relations Commission (NLRC).

Labor Arbiter Macam declared Dejan's dismissal as a valid exercise of Meralco's management
prerogative. Dejan appealed the labor arbiter's decision to the NLRC. On April 24, 2008, the NLRC
rendered a decision reversing the labor arbiter. 12 It found that Dejan's release of the meter sockets
did not constitute an unauthorized taking or stealing of company property. It believed that the
release of the meter sockets was done in good faith as it was in accordance with an accepted
company practice. NLRC ordered his reinstatement without loss of seniority rights, but without
backwages. It penalized Dejan, however, with a one-month suspension for his negligence. Both
Meralco and Dejan went to CA.
Like the NLRC, the CA believed that Dejan acted in good faith as the release of the meter sockets
was upon the request, of the owner of the meter sockets. The CA pointed out that under Article 279
of the Labor Code, a dismissed employee is entitled to backwages, in addition to reinstatement

Issue:
Is Dejan liable for the stealing or unauthorized taking (constituting dishonesty) of company
property under Section 7 (4) of the Company Code of Employee Discipline, which warrants his
dismissal from the service

Ruling

62
Dejan is liable as charged. More specifically, he is liable for violation of Section 7, paragraphs 4 and
11 of the Company Code of Employee Discipline, constituting serious misconduct, fraud and willful
breach of trust of the employer, just causes for termination of employment under the law.
There is also no question that Dejan released the meter sockets to Gozarin without the written
authority or SPA from the customer or customers who applied for electric connection (as a matter
of company policy).

Dejan himself admitted or was aware that as a matter of branch procedure, field representatives
are prohibited from personally taking meter sockets from the branch and delivering them directly
to customers who applied for electrical connection or to their authorized agents or representatives.
. Meter sockets are issued only after payment of the required meter deposit fee and submission of
the required documents. In case a meter socket is to be issued to the customer's authorized
representative, a letter of authority or special power of attorney (SPA) from the owner/customer is
required, together with the customer's valid ID. After the meter socket is issued, a field inspection is
conducted to determine whether the meter socket and service entrance have been installed. Once
the service entrance is ready, the branch then issues a "field order" for the installation of the meter
and to energize the account. The procedure is mandatory for all branch employees.
He released the meter sockets with intent to defraud the company. We cannot blame Meralco for
losing its trust and confidence in Dejan. He is no ordinary employee. As branch representative, "he
was principally charged with the function and responsibility to accept payment of fees required for
the installation of electric service and facilitate issuance of meter sockets." 32 The duties of his
position require him to always act with the highest degree of honesty, integrity and sincerity, as the
company puts it. In light of his fraudulent act, Meralco, an enterprise imbued with public interest,
cannot be compelled to continue Dejan's employment, as it would be inimical to its interest. 33
Needless to say, "[t]he law, in protecting the rights of the laborer, authorizes neither oppression nor
self-destruction of the employer." 34 For sure, Dejan was validly dismissed for serious
misconduct, and loss of trust and confidence.

G.R. No. 176671 June 20, 2012


APO CEMENT CORPORATION,
vs.
ZALDY E. BAPTISMA

(summary: Baptisma,power plant manager of Naga, “end user” of items procured by the company,
received kickbacks from suppliers. Another EE complained about the kickbacks, supported by a
supplier. Apo dismissed Baptisma on ground of loss of trust and confidence. SC upheld APO.)

Facts:
On June 16, 1998, respondent Zaldy E. Baptisma was employed by petitioner Apo Cement
Corporation, a duly registered corporation maintaining and operating a cement manufacturing
plant in Tinaan, Naga, Cebu. 4

63
Sometime in September 2003, petitioner received information from one of its employees, Armando
Moralda (Moralda), that some of its personnel, including respondent who was then the manager of
petitioner's Power Plant Department, were receiving commissions or "kickbacks" from suppliers. 5
To ascertain the veracity of the information given by Moralda, the top management of petitioner
conducted an investigation during which Jerome Lobitaña (Lobitaña), one of petitioner's accredited
suppliers, doing business under the name and style "Precision Process," came forward to
corroborate the statement of Moralda.

Having been implicated in the irregularities, respondent, on November 3, 2003, received a Show
Cause Letter with Notice of Preventive Suspension 10 from Plant Director Ariel Mendoza. To
further afford respondent ample opportunity to defend himself, petitioner conducted a series of
administrative investigation hearings during which respondent was able to face his accusers. On
March 22, 2004, respondent received the Notice of Termination 19 dated March 19, 2004 informing
him of his dismissal from employment effective immediately on the ground of loss of trust and
confidence

On March 31, 2004, respondent filed with the Regional Arbitration Branch VII of the National Labor
Relations Commission (NLRC) in Cebu City a complaint for illegal dismissal with claims for non-
payments of salaries, 13th month pay, service incentive leave, damages, and attorney's fees. LA
ruled in favour of Baptisma. the NLRC reversed the ruling of the Labor Arbiter. It ruled that
respondent's "personal and direct involvement in the irregularities complained of renders him
unworthy of the trust and confidence demanded [of] his position. CA reinstated the Decision of the
Labor Arbiter. APO elevated the case to SC.

Issue:
WON Baptisma was dismissed legally dismissed.

Ruling:
The petition has merit. To validly dismiss an employee on the ground of loss of trust and
confidence under Article 282 (c) 53 of the Labor Code of the Philippines, the following guidelines
must be observed: "1) loss of confidence should not be simulated; 2) it should not be used as
subterfuge for causes which are improper, illegal or unjustified; 3) it may not be arbitrarily asserted
in the face of overwhelming evidence to the contrary; and 4) it must be genuine, not a mere
afterthought to justify earlier action taken in bad faith." 54 More important, it "must be based on a
willful breach of trust and founded on clearly established facts."

To begin with, we find no inconsistencies between the first and the second affidavits of Lobitaña. If
at all, the only difference between the two is that the second affidavit is more detailed than the first
one. Also, there appears to be no ill-motive on the part of Lobitaña to falsely accuse respondent of
accepting commissions and/or "kickbacks." In fact, it was not Lobitaña but Moralda who reported
the irregularities to petitioner. Lobitaña came forward only during petitioner's initial investigation
to confirm the testimony of Moralda that some personnel were indeed receiving commissions
and/or "kickbacks."

64
Likewise erroneous is the reasoning of the Labor Arbiter and the CA that since respondent was not
involved in the procurement process, he could not be guilty of violating Section 2.04 57 of
petitioner's Company Rules and Regulations, which prohibits employees from:

Obtaining or accepting money or anything of value by entering into unauthorized arrangements(s)


with supplier (s), client(s) or other outsiders(s).

This is a non sequitur. As aptly pointed out by the NLRC, although he was not directly involved in
the procurement process, respondent, as the then Power Plant Manager, had some power or
authority "vital and indispensable to the procurement process.

One of the end-users that often generates purchase requests is the Power Plant, ofwhich
[respondent] was then the manager. Being then the manager of the Power Plant, it was
[respondent's] duty to approve purchase requisition[s] and prepare or caused to be prepared the
desired specifications of the item sought to be procured for the Power Plant, especially on the
technical side of the items. In performing this function, [respondent] would exercise some
discretion either to accept the items delivered if he finds them to have complied with the desired
specifications or reject the same if to his judgment the items delivered failed to meet the desired
specifications. In fact, [respondent] himself categorically admitted during the administrative
investigation that in the event the item is rejected, the end-user has the right to demand for
replacement:

Thus, to the mind of any supplier, the role of the end-user, like the Power Plant then headed by
[respondent], in the entire procurement process is as important and indispensable as that of the
procurement personnel. Since the final acceptance of the items and/or equipment
delivered/supplied by a supplier lies with the "end-user," the "end-user" equally wields the power
to "make or break" a supplier, and therefore, the suppliers have all the reasons in the world to
"bribe" the "end-users" if only to smoothen the acceptance of the items supplied/delivered. It is
obviously because of this inherent power to accept or reject any item delivered that Mr. Lobitaña
had to give 10% kickbacks to the [respondent].

Jurisprudence consistently holds that for managerial employees "the mere existence of a basis for
believing that such employee has breached the trust of his employer would suffice for his
dismissal." As we then see it, respondent's termination was for a just and valid cause

GR 193676 June 20, 2012

COSMOS BOTTLING
VS
FERMIN

Facts:
-Wilson B. Fermin (Fermin) was a forklift operator at Cosmos Bottling Corporation (COSMOS),
where he started his employment on 27 August 1976. 4 On 16 December 2002, he was accused of

65
stealing the cellphone of his fellow employee, Luis Braga (Braga). 5 Fermin was then given a Show
Cause Memorandum, requiring him to explain why the cellphone was found inside his locker. In
compliance therewith, he submitted an affidavit the following day, explaining that he only hid the
phone as a practical joke and had every intention of returning it to Braga. Braga executed a
handwritten narration of events.

After conducting an investigation, COSMOS found Fermin guilty of stealing Braga's phone in
violation of company rules and regulations. Consequently, on 2 October 2003, the company
terminated Fermin from employment after 27 years of service.

Meanwhile, Fermin filed a Complaint for Illegal Dismissal, which the Labor Arbiter (LA) dismissed
for lack of merit on the ground that the act of taking a fellow employee's cellphone amounted to
gross misconduct. Further, the LA likewise took into consideration Fermin's other infractions,
namely: (a) committing acts of disrespect to a superior officer, and (b) sleeping on duty and
abandonment of duty. This was affirmed by the Commission.Court of Appeals reversed the rulings
of the LA and the NLRC and awarded him his full retirement benefits. Although the CA accorded
with finality the factual findings of the lower tribunals as regards Fermin's commission of theft, it
nevertheless held that the penalty of dismissal from service was improper on the ground that the
said violation did not amount to serious misconduct or willful disobedience.

Issue: Whether or not the imposition of the penalty of dismissal was appropriate.

Ruling: Affirmative. Theft committed against a co-employee is considered as a case analogous to


serious misconduct, for which the penalty of dismissal from service may be meted out to the erring
employee, viz.:

Article 282 of the Labor Code provides:

Article 282.Termination by Employer. — An employer may terminate an employment for any of the
following causes:
(a)Serious misconduct or willful disobedience by the employee of the lawful orders of his
employer or his representatives in connection with his work.
Misconduct involves "the transgression of some established and definite rule of action, forbidden
act, a dereliction of duty, willful in character, and implies wrongful intent and not mere error in
judgment."

In this case, the LA has already made a factual finding, which was affirmed by both the NLRC and
the CA, that Fermin had committed theft when he took Braga's cellphone. Thus, this act is deemed
analogous to serious misconduct, rendering Fermin's dismissal from service just and valid.
Further, the CA was correct in ruling that previous infractions may be cited as justification for
dismissing an employee only if they are related to the subsequent offense. However, it must be
noted that such a discussion was unnecessary since the theft, taken in isolation from Fermin's other
violations, was in itself a valid cause for the termination of his employment.

66
Finally, it must be emphasized that the award of financial compensation or assistance to an
employee validly dismissed from service has no basis in law. Therefore, considering that Fermin's
act of taking the cellphone of his co-employee is a case analogous to serious misconduct, this Court
is constrained to reverse the CA's ruling as regards the payment of his full retirement benefits. In
the same breath, neither can this Court grant his prayer for backwages.

GR 174893 July 11, 2012

REYES-RAYEL
VS.
PHILIPPINE LUEN THAI HOLDINGS

Facts:
In February 2000, PLTHC hired petitioner as Corporate Human Resources (CHR) Director for
Manufacturing for its subsidiary/affiliate company, L&T. In the employment contract, 5 petitioner
was tasked to perform functions in relation to administration, recruitment, benefits,
audit/compliance, policy development/structure, project plan, and such other works as may be
assigned by her immediate superior, Frank Sauceda (Sauceda), PLTHC's Corporate Director for
Human Resources.

On September 6, 2001, petitioner received a Prerequisite Notice from Sauceda and the Corporate
Legal Counsel of PLTHC, Ma. Lorelie T. Edles with reference to her failure to perform in accordance
with management directives in various instances, which collectively have resulted in loss of
confidence because on numerous occasions. Also, in the presence of colleagues and subordinates,
she made statements that serve to undermine the Company's efforts at pursuing the HR2 Program
of which the other colleagues complained about her.
She explained that her alleged failure to perform management directives could be attributed to the
lack of effective communication with her superiors due to malfunctioning email system. This caused
her to miss certain directives coming from her superiors and likewise, for her superiors to overlook
the reports she was submitting. She denied uttering negative comments about the HR2 Program
and instead claimed to have intimated her support for it.
Petitioner was dismissed from the service for loss of confidence on her ability to promote the
interests of the company. This led petitioner to file a Complaint for illegal dismissal, payment of
separation pay, 13th month pay, moral and exemplary damages, attorney's fees, and other unpaid
company benefits against respondents and its officers.

Issue: Whether or not there was a valid dismissal.

Ruling:
There exists a valid ground for petitioner's termination fro employment.
Jurisprudence provides that an employer has a distinct prerogative and wider latitude of discretion
in dismissing a managerial personnel who performs functions which by their nature require the

67
employer's full trust and confidence. As distinguished from a rank and file personnel, mere
existence of a basis for believing that a managerial employee has breached the trust of the employer
justifies dismissal. Loss of confidence as a ground for dismissal does not require proof beyond
reasonable doubt as the law requires only that there be at least some basis to justify it.

Petitioner was L&T's CHR Director for Manufacturing. As such, she was directly responsible for
managing her own departmental staff. It is therefore without question that the CHR Director for
Manufacturing is a managerial position saddled with great responsibility. Because of this, petitioner
must enjoy the full trust and confidence of her superiors. Not only that, she ought to know that she
is "bound by more exacting work ethics" and should live up to this high standard of responsibility.
records show that petitioner indeed unreasonably failed to effectively communicate with her
immediate superior. There was an apparent neglect in her obligation to maintain constant
communication with Sauceda in order to ensure that her work is up to par. Second, the affidavits of
petitioner's co-workers revealed her negative attitude and unprofessional behavior towards them
and the company. The third and most important is petitioner's display of inefficiency and ineptitude
in her job.

An employer has the right to regulate, according to its discretion and best judgment, all aspects of
employment, including work assignment, working methods, processes to be followed, working
regulations, transfer of employees, work supervision, lay-off of workers and the discipline,
dismissal and recall of workers soo long as they are exercised in good faith.

[G.R. No. 174893. July 11, 2012.]


REYES-RAYEL
vs.
PHILIPPINE LUEN THAI HOLDINGS, CORPORATION

FACTS:
PLTHC hired petitioner as Corporate Human Resources (CHR) Director for Manufacturing for its
subsidiary/affiliate company, L&T. In the employment contract, 5 petitioner was tasked to perform
functions in relation to administration, recruitment, benefits, audit/compliance, policy
development/structure, project plan, and such other works as may be assigned by her immediate
superior, Frank Sauceda (Sauceda), PLTHC's Corporate Director for Human Resources.

In a Termination Notice 10 dated September 12, 2001, respondents, through Sauceda and Edles,
dismissed petitioner from the service for loss of confidence on her ability to promote the interests
of the company. This led petitioner to file a Complaint 11 for illegal dismissal, payment of
separation pay, 13th month pay, moral and exemplary damages, attorney's fees, and other unpaid
company benefits against respondents and its officers, namely, Sauceda, Edles and Willie Tan (Tan),
the Executive Vice-President of PLT.
The Labor Arbiter declared petitioner to have been illegally dismissed. Respondents appealed to
the NLRC which found merit in respondents' appeal. To the NLRC, respondents have sufficiently

68
established the validity of petitioner's dismissal on the ground of loss of trust and confidence
through the various emails, affidavits and other documents attached to the records.

Petitioner filed a Motion for Reconsideration 23 which was granted by the NLRC. In a
Resolution 25 dated July 21, 2004, the NLRC resolved to dismiss respondents' motion for
reconsideration.
Proceedings before the Court of Appeals
Respondents thus filed with the CA a Petition for Certiorari with Urgent Motion for Issuance of
Temporary Restraining Order (TRO) or Writ of Preliminary Injunction. The CA rendered a
Decision 29 finding merit in the petition.

ISSUES:

1. WHETHER OR NOT THERE IS A VALID GROUND FOR PETITIONER'S TERMINATION


FROM EMPLOYMENT.
2. WHETHER OT NOT PETITIONER IS ACCORDED DUE PROCESS

1. YES. Jurisprudence provides that an employer has a distinct prerogative and wider latitude of
discretion in dismissing a managerial personnel who performs functions which by their nature
require the employer's full trust and confidence. 34 As distinguished from a rank and file personnel,
mere existence of a basis for believing that a managerial employee has breached the trust of the
employer justifies dismissal. 35 "[L]oss of confidence as a ground for dismissal does not require
proof beyond reasonable doubt as the law requires only that there be at least some basis to justify
it." 36

Petitioner, in the present case, was L&T's CHR Director for Manufacturing. As such, she was directly
responsible for managing her own departmental staff. It is therefore without question that the CHR
Director for Manufacturing is a managerial position saddled with great responsibility. Because of
this, petitioner must enjoy the full trust and confidence of her superiors. Not only that, she ought to
know that she is "bound by more exacting work ethics" 37 and should live up to this high standard
of responsibility. However, petitioner delivered dismal performance and displayed poor work
attitude which constitute sufficient reasons for an employer to terminate an employee on the
ground of loss of trust and confidence. Respondents also impute upon petitioner gross negligence
and incompetence which are likewise justifiable grounds for dismissal. 38 The burden of proving
that the termination was for a valid cause lies on the employer. 39Here, respondents were able to
overcome this burden as the evidence presented clearly support the validity of petitioner's
dismissal.

First, records show that petitioner indeed unreasonably failed to effectively communicate with her
immediate superior. There was an apparent neglect in her obligation to maintain constant
communication with Sauceda in order to ensure that her work is up to par. This is evident from the
various emails 40 showing that she failed to update Sauceda on the progress of her important
assignments on several occasions. While petitioner explained in her written reply to the
Prerequisite Notice that such failure to communicate was due to the company's computer system

69
breakdown, respondents however were able to negate this as they have shown that the computer
virus which affected the company's system only damaged some email addresses of certain
employees which did not include that of Sauceda's. On the other hand, petitioner failed to present
any concrete proof that the said computer virus also damaged Sauceda's email account as to
effectively disrupt their regular communication. Moreover, we agree with respondents' stance that
petitioner could still reach Sauceda through other means of communication and should not
completely rely on the web. EcHAaS

Second, the affidavits of petitioner's co-workers revealed her negative attitude and unprofessional
behavior towards them and the company. In her affidavit, 41 Agnes Suzette Pasustento, L&T's
Manager for the Corporate Communications Department, attested to petitioner's "badmouthing" of
Sauceda in one of their meetings abroad and of discussing with her about filing a labor case against
the company. Also, in the affidavits of Rizza S. Esplana 42 (Sauceda's Executive Assistant), Cynthia
Yñiguez 43 (Corporate Human Resources Manager of an affiliate of L&T), and Ana Wilma
Arreza 44 (Human Resources and Administration Division Manager of an affiliate of L&T), they
narrated several instances which demonstrated petitioner's notoriously bad temper. They all
described her to have an "irrational" behavior and "superior and condescending" attitude in the
workplace. Unfortunately for petitioner, these sworn statements which notably remain
uncontroverted and unrefuted, militate against her innocence and strengthen the adverse
averments against her. 45 It is well to state that as a CHR Director tasked to efficiently manage the
company's human resource team and practically being considered the "face" of the Human
Resource, petitioner should exhibit utmost concern for her employer's interest. She should likewise
establish not only credibility but also respect from co-workers which can only be attained if she
demonstrates maturity and professionalism in the discharge of her duties. She is also expected to
act as a role model who displays uprightness both in her own behavior and in her dealings with
others.

The third and most important is petitioner's display of inefficiency and ineptitude in her job as a
CHR Director. In the affidavit 46 of Ornida B. Calma, Chief Accountant of L&T's affiliate company,
petitioner, on two occasions, gave wrong information regarding issues on leave and holiday pay
which generated confusion among employees in the computation of salaries and wages. Due to the
nature of her functions, petitioner is expected to have strong working knowledge of labor laws and
regulations to help shed light on issues and questions regarding the same instead of complicating
them. Petitioner obviously failed in this respect. No wonder she received a less than par
performance in her performance evaluation conducted in June 2001, contrary to her assertion that
an 80.2% rating illustrates good and dependable work performance. As can be gleaned in the
performance appraisal form, petitioner received deficient marks and low ratings on areas of
problem solving and decision making, interpersonal relationships, planning and organization,
project management and integrity notwithstanding an overall passing grade. As aptly remarked by
the CA, these low marks revealed the "degree of [petitioner's] work handicap" and should have
served as a notice for her to improve on her job. However, she appeared complacent and remained
lax in her duties and this naturally resulted to respondents' loss of confidence in her managerial
abilities.

70
Taking all these circumstances collectively, the Court is convinced that respondents have sufficient
and valid reasons in terminating the services of petitioner as her continued employment would be
patently inimical to respondents' interest. An employer "has the right to regulate, according to its
discretion and best judgment, all aspects of employment, including work assignment, working
methods, processes to be followed, working regulations, transfer of employees, work supervision,
lay-off of workers and the discipline, dismissal and recall of workers." 47 "[S]o long as they are
exercised in good faith for the advancement of the employer's interest and not for the purpose of
defeating or circumventing the rights of the employees under special laws or under valid
agreements," 48 the exercise of this management prerogative must be upheld.

Anent petitioner's imputation of bad faith upon respondents, the same deserves no credence. That
she was publicly embarrassed when she was coerced by Sauceda and Edles to vacate her office,
return the company car and take all her personal belongings on the day she was dismissed, are all
mere allegations not substantiated by proof. And since it is hornbook rule that he who alleges must
prove, we could not therefore conclude that her termination was tainted with any malice or bad
faith without any sufficient basis to substantiate this bare allegation. Moreover, we are more
inclined to believe that respondents' offer of settlement immediately after petitioner's termination
was more of a generous offer of financial assistance rather than an indication of ill-motive on
respondents' part. DTEcSa

2. YES. We have examined the Prerequisite Notice and contrary to petitioner's assertion, find the
same to be free from any ambiguity. The said notice properly advised petitioner to explain through
a written response her failure to perform in accordance with management directives, which
deficiency resulted in the company's loss of confidence in her capability to promote its interest. As
correctly explained by the CA, the notice cited specific incidents from various instances which
showed petitioner's "repeated failure to comply with work directives, her inclination to make
negative remarks about company goals and her difficult personality," that have collectively
contributed to the company's loss of trust and confidence in her. Indeed, these specified acts, in
addition to her low performance rating, demonstrated petitioner's neglect of duty and
incompetence which support the termination for loss of trust and confidence.

Neither can there be any denial of due process due to the absence of a hearing or investigation at
the company level. It has been held in a plethora of cases that due process requirement is met when
there is simply an opportunity to be heard and to explain one's side even if no hearing is
conducted. 49 In the case of Perez v. Philippine Telegraph and Telephone Company, 50this Court
pronounced that an employee may be afforded ample opportunity to be heard by means of any
method, verbal or written, whether in a hearing, conference or some other fair, just and reasonable
way, in that: SIDEaA
xxx xxx xxx

After receiving the first notice apprising him of the charges against him, the employee may submit a
written explanation (which may be in the form of a letter, memorandum, affidavit or position
paper) and offer evidence in support thereof, like relevant company records (such as his 201 file

71
and daily time records) and the sworn statements of his witnesses. For this purpose, he may
prepare his explanation personally or with the assistance of a representative or counsel. He may
also ask the employer to provide him copy of records material to his defense. His written
explanation may also include a request that a formal hearing or conference be held. In such a case,
the conduct of a formal hearing or conference becomes mandatory, just as it is where there exist
substantial evidentiary disputes or where company rules or practice requires an actual hearing as
part of employment pretermination procedure. To this extent, we refine the decisions we have
rendered so far on this point of law.
xxx xxx xxx
In sum, the following are the guiding principles in connection with the hearing requirement in
dismissal cases:
(a)'ample opportunity to be heard' means any meaningful opportunity (verbal or written) given to
the employee to answer the charges against him and submit evidence in support of his defense,
whether in a hearing, conference or some other fair, just and reasonable way.
(b)a formal hearing or conference becomes mandatory only when requested by the employee in
writing or substantial evidentiary disputes exist or a company rule or practice requires it, or when
similar circumstances justify it.
(c)the 'ample opportunity to be heard' standard in the Labor Code prevails over the 'hearing or
conference' requirement in the implementing rules and regulations. 51

In this case, petitioner's written response to the Prerequisite Notice provided her with an avenue to
explain and defend her side and thus served the purpose of due process. That there was no hearing,
investigation or right to appeal, which petitioner opined to be a violation of company policies, is of
no moment since the records is bereft of any showing that there is an existing company policy that
requires these procedures with respect to the termination of a CHR Director like petitioner or that
company practice calls for the same. There was also no request for a formal hearing on the part of
petitioner.

As she was served with a notice apprising her of the charges against her, and also a subsequent
notice informing her of the management's decision to terminate her services after respondents
found her written response to the first notice unsatisfactory, petitioner was clearly afforded her
right to due process

[G.R. No. 195428. August 29, 2012.]

VERDADERO, PETITIONER,
VS.
BARNEY AUTOLINES GROUP OF COMPANIES TRANSPORT, INC.,

FACTS:

72
Respondent Barney Autolines Group of Companies Transport, Inc. (BALGCO) hired Verdadero as
bus conductor and paid him a salary on commission basis at the rate of 12% of the gross ticket sales
per day. 5

On January 27, 2008, an altercation took place between Verdadero and respondent Atty. Gerardo
Gimenez (Gimenez), BALGCO's Disciplinary Officer. Gimenez was on board BALGCO Bus. No. 55455,
together with his wife and four other companions, travelling from Mulanay to Macalelon, Quezon.
Verdadero was then the assigned bus conductor. BALGCO has a company policy of granting free
rides to company employees and their wives. The story started when Verdadero began issuing fare
tickets to passengers, including the wife of Gimenez. The wife informed Verdadero who she
was 6 and the incidents thereafter took two versions as both parties told a different story.

Gimenez filed an unverified complaint for serious misconduct against Verdadero before the
BALGCO Management. He requested Barney D. Chito (Barney) and Rosela F. Chito (Rosela), owners
of BALGCO, to preside over the conciliation proceedings. Verdadero, accompanied by his father,
appeared at the BALGCO Office on February 8, 2008. Verdadero was said to have shown willingness
to be penalized for his misconduct provided no record of the proceedings would be made. Gimenez,
on the other hand, was willing to waive the imposition of any penalty if Verdadero would give a
simple letter of apology, which the latter supposedly agreed with his father guaranteeing the
same. 7

On February 16, 2008, Verdadero, instead, submitted his counter-affidavit refuting all allegations in
the written complaint against him. Rosela told Verdadero she was not expecting that piece of paper,
to which the latter was said to have replied, "Sabi mo papel, yan papel yan!" 8

Thereafter, Verdadero furtively reported for work for fear of having another confrontation with
Gimenez. Rosela sent Verdadero a letter, dated February 25, 2008, requiring him to immediately
report for work and finish the pending disciplinary proceedings against him. On March 28, 2008,
Verdadero submitted his Letter-Reply, explaining that he had been receiving threats. He likewise
believed he was already illegally dismissed as he was not given any work assignment since January
28, 2008. Rosela responded to Verdadero's letter and reminded him of the letter of apology which
he was yet to submit as compliance. On April 15, 2008, however, Verdadero filed a complaint for
illegal dismissal before the Labor Arbiter (LA), claiming, as well, non-payment of holiday pay,
premium on holiday, 13th month pay, separation pay, retirement benefits, moral and exemplary
damages, and reinstatement plus backwages. 9 HTScEI

The LA rendered a Decision dismissing Verdadero's complaint and declaring that no dismissal took
place but merely an administrative investigation. Aggrieved, Verdadero filed an appeal before the
NLRC which partially granted the appeal. It ruled that Verdadero was illegally dismissed, but
affirmed the LA insofar as the holiday and overtime pays were concerned.

73
The CA ruled that there was no constructive dismissal neither was there abandonment on the part
of Verdadero, reiterating the well-settled rule that the filing of a complaint for illegal dismissal is
inconsistent with a charge of abandonment.

ISSUES:
1. Whether or not petitioner Verdadero was constructively dismissed.
2. If so, whether or not he is entitled to reinstatement and backwages.

HELD:
1. No. Verdadero cannot be deemed constructively dismissed.
Constructive dismissal exists where there is cessation of work, because "continued employment is
rendered impossible, unreasonable or unlikely, as an offer involving a demotion in rank or a
diminution in pay" and other benefits. Aptly called a dismissal in disguise or an act amounting to
dismissal but made to appear as if it were not, constructive dismissal may, likewise, exist if an act of
clear discrimination, insensibility, or disdain by an employer becomes so unbearable on the part of
the employee that it could foreclose any choice by him except to forego his continued
employment. 26

Records do not show any demotion in rank or a diminution in pay made against him. Neither was
there any act of clear discrimination, insensibility or disdain committed by BALGCO against
Verdadero which would justify or force him to terminate his employment from the company.
To support his contention of constructive dismissal, Verdadero considers the verbal abuse by
Gimenez against him as an act which rendered his continued employment impossible, unreasonable
or unlikely. The claimed abuse was corroborated by the sworn written statement executed by
Mascariña, which was given credence by the NLRC and the CA. With the alleged threats of Gimenez,
Verdadero believed that he could no longer stay and work for BALGCO.

It is to be emphasized that the abovementioned acts should have been committed by the employer
against the employee. Unlawful acts committed by a co-employee will not bring the matter within
the ambit of constructive dismissal.

Assuming arguendo that, Gimenez did commit the alleged unlawful acts, still, this fact will not
suffice to conclude that constructive dismissal was proper. Contrary to the arguments of Verdadero,
Gimenez is not the employer. He may be the "disciplinary officer," but his functions as such, as can
be gleaned from the BALGCO Rules and Regulations, 27 do not involve the power or authority to
dismiss or even suspend an employee. Such power is exclusively lodged in the BALGCO
management. Gimenez remains to be a mere employee of BALGCO and, thus, cannot cause the
dismissal or even the constructive dismissal of Verdadero. HA

Moreover, it was not established that BALGCO itself or its owners had been, in any way, forcing
Verdadero to resign from his employment.

74
Furthermore, records are bereft of any showing that Verdadero was no longer allowed to report for
work starting January 28, 2008, 30 when Gimenez lodged a complaint for serious misconduct
against him before the BALGCO management. 2

It was Verdadero himself who terminated his employment. It was, in fact, his position that the
January 27, 2008 bus incident gave rise to constructive dismissal.

Well-settled is the rule in illegal dismissal case that while the employer bears the burden of proving
that the termination was for a valid or authorized cause, the employee must first establish by
substantial evidence the fact of his dismissal from service. 33 In this case, however, the employer
should not be belabored to prove a valid dismissal as BALGCO itself has not terminated the
employment of Verdadero.

2. Reinstatement and backwages are reliefs available to an illegally dismissed employee.


Reinstatement restores the employee who was unjustly dismissed to the position from which he
was removed, that is, to his status quo ante dismissal, while the grant of backwages allows the same
employee to recover from the employer that which he had lost by way of wages as a result of his
dismissal. These twin remedies — reinstatement and payment of backwages — make the dismissed
employee whole who can then look forward to continued employment. Thus, do these two
remedies give meaning and substance to the constitutional right of labor to security of tenure. 34
In the case at bench, considering that there has been no dismissal at all, there can be no
reinstatement. One cannot be reinstated to a position he is still holding. As there is no
reinstatement to speak of, Verdadero cannot invoke the doctrine of strained relations. It is only
applied when there is an order for reinstatement that is no longer feasible. In the same vein, no
separation pay can be awarded as it is given only in lieu of reinstatement. Consequently, there is
likewise no justification for the award of backwages. The CA was correct in ruling against the
payment of backwages following the "no work, no pay" principle.

GR NO. 193789

ALEX NARANJO, DONNALYN DE GUZMAN. RONALD V. CRUZ, ROSEMARIE P. PIMENTEL AND


ROWENA B. BARDAJE
V.S
BIOMEDICAL HEALTH CARE, INC. AND KAREN J. MOTOL

FACTS:
Biomedica Health Care, inc. was , during the material period, engaged in the distribution of
medical equipment. Respondent Karen J. Motol ( motol ) was then its President.
Petitioners were employees of Biomedica, Naranjo as Laiason Officer, Cruz as Service
engineer, Bardaje as Administration Clerk, De guzman as Sales Representative and Pimentel as
Accounting Clerk.

75
On November 7, 2006, petitioners were all absent for various personal reasons. Notably,
these are the same employees who filed a letter-complaint dated October 31, 2006 addressed to the
national Director of DOLE ( national capital region ) against Biomedica for lack of salary increases,
failure to remit SSS and Pag-ibig contributions and violations of the minimum wage law among
others.
Later that day, petitioners reported for work after receiving text
messages for them to proceed to Biomedica. They were, however, refused entry and told to start
looking for another workplace.
The next day, November 8, 2006, petitioners allegedly came in for work but were not
allowed to enter the premises.13 Motol purportedly informed petitioners, using foul language, to
just find other employment.
Correspondingly, on November 9, 2006, Biomedica issued a notice of
preventive suspension and notices to explain within 24 hours (Notices)14 to
petitioners. In the Notices, Biomedica accused the petitioners of having conducted an illegal strike
and were accordingly directed to explain why they should not be held guilty of and dismissed for
violating the company policy against illegal strikes under Article XI, Category Four, Sections 6, 8, 12,
18 and 25 of the Company Policy.
On November 20, 2006, petitioners filed a Complaint with the NLRC for constructive
dismissal and nonpayment of salaries, overtime pay, 13th month pay as well as non-remittance of
SSS, Pag-IBIG and Philhealth contributions as well as loan payments.
The labor arbiter dismissed the comlaint for illegal dismissal. The Labor Arbiter found that,
indeed, petitioners engaged in a mass
leave akin to a strike. He added that, assuming that petitioners were not aware of the company
policies on illegal strikes, such mass leave can sufficiently be deemed as serious misconduct under
Art. 282 of the Labor Code. Thus, the Labor Arbiter concluded that petitioners were validly
dismissed.

On appeal to the NLRC, it found that there was indeed illegal dismissal on the part of
Biomedica.
The CA ruled that the dismissal was valid.

ISSUE
Whether or not the dismissal of petitioners were illegl

HELD:

Petitioner were illegally dismissed

The Labor Code promotes the right of the worker to


security of tenure protecting them against illegal dismissal:

ARTICLE 279. Security of Tenure. - In cases of regular

76
employment, the employer shall not terminate the services of an employee except for a just cause
or when authorized by this Title. An Employee who is unjustly dismissed from work shall be
entitled to reinstatement without loss of seniority rights and other privileges and to his full
backwages, inclusive of allowances, and to his other benefits or their monetary equivalent
computed from the time his compensation was withheld from him up to the time of his actual
reinstatement.

It bears pointing out that in the dismissal of an employee, the law requires that due process
be observed. Such due process requirement is two-fold, procedural and substantive, that is, “ the
termination of employement must be based on just or quthorized cause of dismissal and the
dismissal must be effected after due notice and hearing.” In the instant case, petitioners were not
afforded both procedural and substantive due process.

Petitioners were not afforded Procedural due process

Art. 277(b) of the Labor Code contains the procedural due process requirements in the
dismissal of an employee:

Art. 277. Miscellaneous Provisions. – x x x

(b) Subject to the constitutional right of workers to security of


tenure and their right to be protected against dismissal except for a just and authorized cause
without prejudice to the requirement of notice under Article 283 of this Code, the employer shall
furnish the worker whose employment is sought to be terminated a written notice containing a
statement of the causes for termination and shall afford the latter ample opportunity to be heard
and to defend himself with the assistance of his representative if he so desires in accordance with
company rules and regulations promulgated pursuant to guidelines set by the Department of Labor
and Employment. Any decision taken by the employer shall be without prejudice to the right of the
worker to contest the validity or legality of his dismissal by filing a complaint with the regional
branch of the National Labor Relations Commission. The burden of proving that the termination
was for a valid or authorized cause shall rest on the employer.

On the other hand, Rule XIII, Book V, Sec. 2 I (a) of the


Implementing Rules and Regulations of the Labor Code states:

SEC. 2. Standards of due process; requirements of notice.––In all


cases of termination of employment, the following standards of due
process shall be substantially observed:

I. For termination of employment based on just causes as


defined in Article 282 of the Code:

77
(a) A written notice served on the employee
specifying the ground or grounds for termination, and
giving said employee reasonable opportunity within
which to explain his side.

Thus, the Court elaborated in King of Kings Transport, Inc. v.


Mamac that a mere general description of the charges against an employee by the employer is
insufficient to comply with the above provisions of the law:
x x x Moreover, in order to enable the employees to intelligently
prepare their explanation and defenses, the notice should contain a
detailed narration of the facts and circumstances that will serve as
basis for the charge against the employees. A general description of
the charge will not suffice. Lastly, the notice should specifically mention which company rules, if
any, are violated and/or which among the grounds under Art. 282 is being charged against the
employees.

In the instant case, the notice specifying the grounds for termination
dated November 9, 2006 states:

Effective upon receipt hereof, you are placed under preventive suspension for willfully organizing
and/or engaging in illegal strike on November 7, 2006. Your said illegal act-in conspiracy with your
other co-employees, paralyzed the company operation on that day and resulted to undue damage
and prejudice to the company and is direct violation of Article XI, Category Four Section 6, 8, 12, 18
& 25 of our Company Policy, which if found guilty, you will be meted a penalty of dismissal.

Please explain in writing within 24 hours from receipt hereof why you
should not be held guilty of violating the company policy considering
further that you committed and timed such act during the birthday of our Company president.28

Clearly, petitioners were charged with conducting an illegal strike, not


a mass leave, without specifying the exact acts that the company considers as constituting an illegal
strike or violative of company policies. Such allegation falls short of the requirement in King of
Kings Transport, Inc. of “a detailed narration of the facts and circumstances that will serve as basis
for the charge against the employees.” A bare mention of an “illegal strike” will not suffice.

It is incumbent upon respondent company to show that


petitioners were duly informed of said company policies at the time of their employment and were
given copies of these policies. No such proof was presented by respondents. There was even no
mention at all that such requirement was met. Worse, respondent Biomedica did not even quote or
reproduce the company policies referred to in the notice as pointed out by the CA stating:

78
It must be noted that the company policy which the petitioner was
referring to was not quoted or reproduced in the petition, a copy of which is not also appended in
the petition, as such we cannot determine the veracity of the existence of said policy.29

Without a copy of the company policy being presented in the CA or


the contents of the pertinent policies being quoted in the pleadings, there is no way by which one
can determine whether or not there was, indeed, a violation of said company policies.

Moreover, the period of 24 hours allotted to petitioners to answer the


notice was severely insufficient and in violation of the implementing rules of the Labor Code. Under
the implementing rule of Art. 277, an employee should be given “reasonable opportunity” to file a
response to the notice.
King of Kings Transport, Inc. elucidates in this wise:

To clarify, the following should be considered in terminating the


services of employees:

(1) The first written notice to be served on the employees should


contain the specific causes or grounds for termination against them, and a directive that the
employees are given the opportunity to submit their written explanation within a reasonable
period. “Reasonable opportunity” under the Omnibus Rules means every kind of assistance that
management must accord to the employees to enable them to prepare adequately for their defense.
This should be construed as a period of at least five (5)calendar days from receipt of the notice to
give the employees an opportunity to study the accusation against them, consult a union official or
lawyer, gather data and evidence, and decide on the defenses they will raise against the complaint.
30 (Emphasis supplied.)

Following King of Kings Transport, Inc., the notice sent out by


Biomedica in an attempt to comply with the first notice of the due process requirements of the law
was severely deficient.

In addition, Biomedica did not set the charges against petitioners for
hearing or conference in accordance with Sec. 2, Book V, Rule XIII of the Implementing Rules and
Regulations of the Labor Code and in line with ruling in King of Kings Transport, Inc., where the
Court explained:

(2) After serving the first notice, the employers should schedule
and conduct a hearing or conference wherein the employees will be given the opportunity to: (1)
explain and clarify their defenses to the charge against them; (2) present evidence in support of
their defenses; and (3) rebut the evidence presented against them by the management. During the
hearing or conference, the employees are given the chance to defend themselves personally, with

79
the assistance of a representative or counsel of their choice. Moreover, this conference or hearing
could be used by the parties as an opportunity to come to an amicable settlement.31

While petitioners did not submit any written explanation to the


charges, it is incumbent for Biomedica to set the matter for hearing or
conference to hear the defenses and receive evidence of the employees.
More importantly, Biomedica is duty-bound to exert efforts, during said hearing or conference, to
hammer out a settlement of its differences with petitioners. These prescriptions Biomedica failed to
satisfy.

Lastly, Biomedica again deviated from the dictated contents of a


written notice of termination as laid down in Sec. 2, Book V, Rule XIII of the Implementing Rules
that it should embody the facts and circumstances to support the grounds justifying the
termination. As amplified in King of Kings Transport, Inc.:

(3) After determining that termination of employment is justified,


the employers shall serve the employees a written notice of
termination indicating that: (1) all circumstances involving the charge
against the employees have been considered; and (2) grounds have been established to justify the
severance of their employment.32

The November 26, 2006 Notice of Termination issued by


Biomedica miserably failed to satisfy the requisite contents of a valid notice of termination, as it
simply mentioned the failure of petitioners to submit their respective written explanations without
discussing the facts and circumstances to support the alleged violations of Secs. 6, 8, 12, 18 and 25
of Category Four, Art. XI of the alleged company rules.

Petitoners were denied Substantive due process


In any event, petitioners were also not afforded substantive due
process, that is, they were illegally dismissed.

The just causes for the dismissal of an employee are exclusively found
in Art. 282(a) of the Labor Code, which states:

ARTICLE 282. Termination by employer. – An employer may


terminate an employment for any of the following causes:

(a) Serious misconduct or willful disobedience by the


employee of the lawful orders of his employer or representative in
connection with his work

It was on this ground that the CA upheld the dismissal of petitioners

80
from their employment. Serious misconduct, as a justifying ground for the dismissal of an
employee, has been explained in Aliviado v. Procter & Gamble, Phils., Inc.:

Misconduct has been defined as improper or wrong conduct; the


transgression of some established and definite rule of action, a
forbidden act, a dereliction of duty, unlawful in character implying
wrongful intent and not mere error of judgment. The misconduct to be
serious must be of such grave and aggravated character and not merely
trivial and unimportant. To be a just cause for dismissal, such misconduct
(a) must be serious; (b) must relate to the performance of the employee’s
duties; and (c) must show that the employee has become unfit to continue
working for the employer.

Clearly, to justify the dismissal of an employee on the ground of


serious misconduct, the employer must first establish that the employee is guilty of improper
conduct, that the employee violated an existing and valid company rule or regulation, or that the
employee is guilty of a wrongdoing.
In the instant case, Biomedica failed to even establish that petitioners indeed violated company
rules, failing to even present a copy of the rules and to prove that petitioners were made aware of
such regulations. In fact, from the records of the case, Biomedica has failed to prove that petitioners
are guilty of a wrongdoing that is punishable with termination from employment. Art. 277(b) of the
Labor Code states, “The burden of proving that the termination was for a valid or authorized cause
shall rest on the employer.” In the instant
case, Biomedica failed to overcome such burden. As will be shown,
petitioners’ absence on November 7, 2006 cannot be considered a mass leave, much less a strike
and, thus, cannot justify their dismissal from employment.

There was no MASS LEAVE on the part of petitioners


The accusation is for engaging in a mass leave tantamount to an illegal strike.

The term “Mass Leave” has been left undefined by the Labor Code. Plainly, the legislature intended
that the term’s ordinary sense be used. “Mass” is defined as “participated in, attended by, or
affecting a large number of individuals; having a large-scale character.”34 While the term “Leave” is
defined as “an authorized absence or vacation from duty or employment usually with pay.”35

Thus, the phrase “mass leave” may refer to a simultaneous availment of authorized leave benefits
by a large number of employees in a company.

It is undeniable that going on leave or absenting one’s self from work for personal reasons when
they have leave benefits available is an employee’s right. In Davao Integrated Port Stevedoring

81
Services v. Abarquez,the Court acknowledged sick leave benefits as a legitimate economic benefit of
an employee, carrying a purpose that is at once legal as it is practical:

Sick leave benefits, like other economic benefits stipulated in the


CBA such as maternity leave and vacation leave benefits, among others, are by their nature,
intended to be replacements for regular income which otherwise would not be earned because an
employee is not working during the period of said leaves. They are non-contributory in nature, in
the sense that the employees contribute nothing to the operation of the benefits. By their nature,
upon agreement of the parties, they are intended to alleviate the economic condition of the
workers.

In addition to sick leave, the company, as a policy or practice or as


agreed to in a CBA, grants vacation leave to employees. Lastly, even the Labor Code grants a service
incentive leave of 5 days to employees.

In the factual milieu at bar, Biomedica did not submit a copy of the
CBA or a company memorandum or circular showing the authorized sick or vacation leaves which
petitioners can avail of. Neither is there any document to show the procedure by which such leaves
can be enjoyed. Absent such pertinent documentary evidence, the Court can only conclude that the
availment of petitioners of their respective leaves on November 7, 2006 was authorized, valid and
in accordance with the company or CBA rules on entitlement to and availment of such leaves.

Here, the five (5) petitioners were absent on November 7, 2006. The
records are bereft of any evidence to establish how many workers are
employed in Biomedica. There is no evidence on record that 5 employees constitute a substantial
number of employees of Biomedica. And, as earlier stated, it is incumbent upon Biomedica to prove
that petitioners were dismissed for just causes, this includes the duty to prove that the leave was
large-scale in character and unauthorized. This, Biomedica failed to prove.

Having failed to show that there was a mass leave, the Court
concludes that there were only individual availment of their leaves by
petitioners and they cannot be held guilty of any wrongdoing.

Petitioners did not go on strike


Granting for the sake of argument that the absence of the 5 petitioners on November 7,
2006 is considered a mass leave, still, their actions cannot be considered a strike.

Art. 212(o) of the Labor Code defines a strike as “any temporary


stoppage of work by the concerted action of employees as a result of any industrial or labor
dispute.”

“Concerted” is defined as “mutually contrived or planned” or

82
“performed in unison.”38 In the case at bar, the 5 petitioners went on leave for various reasons.
Petitioners were in different places on November 7, 2006 to attend to their personal needs or
affairs. They did not go to the company premises to petition Biomedica for their grievance.

To demonstrate their good faith in availing their leaves, petitions reported for work and were at the
company premises in the afternoon after they received text messages asking them to do so. This
shows that there was NO intent to go on strike. Unfortunately, they were barred from entering the
premises and were told to look for new jobs. Surely the absence of petitioners in the morning of
November 7, 2006 cannot in any way be construed as a concerted action, as their absences are
presumed to be for valid causes, in good faith, and in the exercise of their right to avail themselves
of CBA or company benefits.

Notably, the CA still ruled that petitioners went on strike as evidenced


by the explanation letters of Angeles and Casimiro sent by Biomedica. They stated in the letters that
they, along with petitioners, agreed to go on leave on the birthday on Motol to stress their demands
against the company.

These statements do not deserve much weight and credit.

Sec. 11(c) of the 2011 NLRC Rules of Procedure relevantly provides:

SECTION 11. SUBMISSION OF POSITION PAPER AND


REPLY. – x x x

xxxx

c) The position papers of the parties shall cover only those claims
and causes of action stated in the complaint or amended complaint,
accompanied by all supporting documents, including the affidavits of
witnesses, which shall take the place of their direct testimony,
excluding those that may have been amicably settled. (Emphasis
supplied.)

In the instant case, the CA accepted as evidence the explanation letters issued by Angeles and
Casimiro when these are not notarized. While notarization may seem to be an inconsequential
requirement considering that the Labor Arbiter and the NLRC are not strictly bound by technical
rules of evidence, however, mere explanation letters submitted to the company that the authors
issued even before the case was filed before the NLRC cannot be accepted as direct testimony of the
authors. The requirement that the direct testimony can be contained in an affidavit is to ensure that
the affiant swore under oath before an administering officer that the statements in the affidavit are
true. The affiant knows that he or she can be charged criminally for perjury under solemn
affirmation or at least he or she is bound to his or her oath. Thus, the affidavits or sworn statements
of these employees should have been presented. At the very least, the workers should have been

83
summoned to testify on such letters. Ergo, these letters cannot be the sole basis for the finding that
petitioners conducted a strike against Biomedica and for the termination of their employment.
Lastly, the explanation letters cannot overcome the clear and categorical statements made by the
petitioners in their verified positions papers. As between the verified statements of petitioners and
the unsworn letters of Angeles and Casimiro, clearly, the former must prevail and are entitled to
great weight and value.

Finally, it cannot be overemphasized that in case of doubt, a case


should be resolved in favor of labor. As aptly stated in Century Canning Corporation v. Ramil:

x x x Unsubstantiated suspicions, accusations, and conclusions of


employers do not provide for legal justification for dismissing employees. In case of doubt, such
cases should be resolved in favor of labor, pursuant to the social justice policy of labor laws and the
Constitution.

Biomedica has failed to adduce substantial evidence to prove that


petitioners’ dismissal from their employment was for a just or authorized cause. The conclusion is
inescapable that petitioners were illegally dismissed.

Given the illegality of their dismissal, petitioners are entitled to


reinstatement and backwages as provided in Art. 279 of the Labor Code, which states:

An employee who is unjustly dismissed from work shall be entitled to


reinstatement without loss of seniority rights and other privileges and to his full backwages,
inclusive of allowances, and to his other benefits or their monetary equivalent computed from the
time his compensation was withheld from him up to the time of his actual reinstatement.

Thus, the Court ruled in Golden Ace Builders v. Talde,


41 citing Macasero v. Southern Industrial Gases Philippines:

Thus, an illegally dismissed employee is entitled to two reliefs:


backwages and reinstatement. The two reliefs provided are separate and distinct. In instances
where reinstatement is no longer feasible because of strained relations between the employee and
the employer, separation pay is granted. In effect, an illegally dismissed employee is entitled to
either reinstatement, if viable, or separation pay if reinstatement is no longer viable, and
backwages.

The normal consequences of respondents’ illegal dismissal,


then, are reinstatement without loss of seniority rights, and payment
of backwages computed from the time compensation was withheld up
to the date of actual reinstatement. Where reinstatement is no longer
viable as an option, separation pay equivalent to one (1) month salary
for every year of service should be awarded as an alternative. The

84
payment of separation pay is in addition to payment of backwages.

Petitioners were terminated in swift


fashion and in gross violation of their right to due process revealing that they are no longer wanted
in the company. The convergence of these facts coupled with the filing by petitioners of their
complaint with the DOLE shows a relationship governed by antipathy and antagonism as to justify
the award of separation pay in lieu of reinstatement. Thus, in addition to backwages, owing to the
strained relations between the parties, separation pay in lieu of reinstatement would be proper. In
Golden Ace Builders, We explained why:

Under the doctrine of strained relations, the payment of separation


pay is considered an acceptable alternative to reinstatement when the latter
option is no longer desirable or viable. On one hand, such payment
liberates the employee from what could be a highly oppressive work
environment. On the other hand, it releases the employer from the grossly
unpalatable obligation of maintaining in its employ a worker it could no
longer trust.

Strained relations must be demonstrated as a fact, however, to be


adequately supported by evidence — substantial evidence to show that the
relationship between the employer and the employee is indeed strained as
a necessary consequence of the judicial controversy.43

And in line with prevailing jurisprudence,44 petitioners are entitled to


nominal damages in the amount of PhP 30,000 each for Biomedica’s
violation of procedural due process.

GR NO . 1999547

THE NEW PHILIPPINE SKYLANDERS INC


V.S
DAKILA

FACTS:
Respondent Dakila was employed by Skylanders Inc. as early as 1987 and was terminated
for cause in April of 1997 when the company was sold.
In May of 1997, Respondent Dakila was rehired as a consultant by the petitoners under a
Contract of consultancy under a Contract for consultancy Services dated April 30, 1997.
Respondent Dakia then filed a complaint for constructive illegal dismissal against
petitioners including non-payment of retirement benefits, under/non-payment of wages and other
benefits of a refular employee and damages against petitioners before the NLRC. He averred that
the consultancy contract was a scheme to deprive him of the benefits of reglarization, claiming to

85
have assumed tasks necessary and desirable in the trade or busines of petitioners and under their
direct control and supervision. In support of his claim, he submitted, among others, copies of his
time cards, official business slips, daily attendance sheets and other documents prescribing the
manner in which his tasks were to be accomplished under the control of petitioners and
acknowledging his status as a regular employee of the corporation.
On the other hand, petitioners, in their position paper,8ςrνll asserted that respondent
Dakila was a consultant and not their regular employee. The latter was not included in petitioners'
payroll and paid a fixed amount under the consultancy contract. He was not required to observe
regular working hours and was free to adopt means and methods to accomplish his task except as
to the results of the work required of him. Hence, no employer-employee relationship existed
between them. Moreover, respondentDakila terminated his contract in a letter dated April 19, 2007,
thus, negating his dismissal.
The Labor arbiter, NLRC and the Court of Appeals were one in ruling that Dakila was a
regular employee and that his dismissal was illegal. However, it noted that since he was already
beyond the retirement age, his reinstatement was no longer feasible. As such, it ordered the
payment of his retirement pay to be computed from 1997 until the date of the decision.

ISSUE:
Whether or not Respondent Dakila was an Employee of Petitioner Skylanders, inc.?
Whether or not there was constructive illegal dismissal of respondent by petitioner?

HELD:
YES, there was an emloyee and employer relationship and that petitoners illegally
dismissed petitioners.

Following Article 279 of the Labor Code, an employee who is unjustly dismissed from work is
entitled to reinstatement without loss of seniority rights and other privileges and to his full
backwages computed from the time he was illegally dismissed. However, considering that
respondent Dakila was terminated on May 1, 2007, or one (1) day prior to his compulsory
retirement on May 2, 2007, his reinstatement is no longer feasible. Accordingly, the NLRC correctly
held him entitled to the payment of his retirement benefits pursuant to the CBA. On the other hand,
his backwages should be computed only for days prior to his compulsory retirement which in this
case is only a day. Consequently, the award of reinstatement wages pending appeal must be deleted
for lack of basis
Similarly, the Court finds no basis to hold petitioner Jennifer M. Eno-Bote, President and
General Manager of The New Philippine Skylanders, Inc., jointly and severally liable with the
corporation for the payment of the monetary awards. The mere lack of authorized or just cause to
terminate one's employment and the failure to observe due process do not ipso facto mean that the
corporate officer acted with malice or bad faith.15ςrνll There must be independent proof of malice
or bad faith which was not established in this case. Perforce, petitioner Jennifer M. Eno-Bote cannot
be made personally liable for the liabilities of the corporation which, by legal fiction, has a
personality separate and distinct from its officers, stockholders and members.

86
(Nov 21, 2012)

MORALES
VS
METROPOLITAN BANK & TRUST CO.

Facts:
Solidbank merged with respondent Metropolitan Bank & Trust Company (Metrobank) in
September 2000, the latter, as surviving entity, absorbed Morales(teller of Solidbank) and assigned
him to its Customer Service Relations-Reserve Pool (CSR-RP) which was composed of employees
who, with no permanent places of assignment, acted as relievers whenever temporary vacancies
arise in other branches.

From a job with a grade four rank, Morales was subsequently promoted in April 20034 to the
position of Customer Service Representative (CSR), with a job grade 6 rank and a gross monthly
salary of P16,250.00. While occupying the latter position that Morales was informed by Federico
Mariano, the Senior Manager of Metrobank’s Tacloban City Main Branch, that he was covered by the
bank’s Special Separation Program (SSP) and that, in accordance therewith, his employment was
going to be terminated on the ground of redundancy.

Assured that his termination was through no fault of his own but mainly due to business exigencies
and developments in the banking industry, Morales was notified that he shall be paid the following:
(a) a redundancy premium/separation pay, on top of his entitlements under the bank’s retirement
plan; (b) proportionate 13th month pay; (c) cash conversion of his outstanding vacation and sick
leave credits; and, if applicable, (d) the return of his Provident Fund contributions; and, (e) cash
surrender value of his Insurance. Having signed a form on the same day signifying his unqualified
and unconditional acceptance of Metrobank’s decision to terminate his employment, Morales
executed on 10 November 2003 a Release, Waiver and Quitclaim acknowledging receipt of the sum
of P158,496.95 as full payment of his monetary entitlements.

Morales filed against Metrobank a complaint for illegal dismissal, separation pay, backwages, moral
and exemplary damages as well as attorney’s fees. He alleged that his actions were blown out of
proportions and was singled out for termination, aside from that the directives to explain his
unauthorized absences were also just a result of miscommunication, lastly that he was also forced
to sign the Release, Waiver and Quitclaim.

On the other hand, Metrobank averred that it adopted the Special Separation Program to address
the worsening economic conditions and stiff competition to make operations efficient but cost
effective. The company also claims that it had served not only Morales a one month prior notice of
termination but also DOLE. Likewise, Morales unqualifiedly and unconditionally accepted his
termination by receiving his separation benefits and signing the Release, Waiver and Quitclaim.
Labor Arbiter found the dismissal illegal, but was reversed by the NLRC who was likewise affirmed
by the CA.

87
Issue: WON Morales’ termination was valid.

Held:
The dismissal was valid and so was the Release, Waiver and Quitclaim signed by petitioner Morales.
One of the authorized causes for the dismissal of an employee, redundancy exists when the service
capability of the workforce is in excess of what is reasonably needed to meet the demands of the
business enterprise. A position is redundant when it is superfluous, and superfluity of a position or
positions could be the result of a number of factors, such as the overhiring of workers, a decrease in
the volume of business or the dropping of a particular line or service previously manufactured or
undertaken by the enterprise. Time and again, it has been ruled that an employer has no legal
obligation to keep more employees than are necessary for the operation of its business. For the
implementation of a redundancy program to be valid, however, the employer must comply with the
following requisites: (1) written notice served on both the employees and the DOLE at least one
month prior to the intended date of termination of employment; (2) payment of separation pay
equivalent to at least one month pay for every year of service; (3) good faith in abolishing the
redundant positions; and (4) fair and reasonable criteria in ascertaining what positions are to be
declared redundant and accordingly abolished.

Contrary to the first and second errors Morales imputes against the CA, our perusal of the record
shows that Metrobank has more than amply proven compliance with the third and fourth of the
above-enumerated requisites for the validity of his termination from service on the ground of
redundancy. Under the SSP which Metrobank adopted in 1995, employees who voluntarily gave up
their employment were paid the amount of separation pay they were entitled under the law and a
premium equivalent to 50%-75% of their salaries. It appears that employees “whose work
evaluation showed consistent poor performance and/or those who had not been promoted for five
years” were also considered primary candidates for optional separation from service. In order to
meet the challenges of the business and to make its operations efficient and cost effective, however,
it was shown that Metrobank further conducted a bank-wide operational review and study which
resulted in the adoption in March 2003 of the HRP, a major component of the SSP which was
designed to reduce its workforce by 10%. Entailing various initiatives like conversion of regular
branches into mini-branches, consolidation of branches, centralization of loans processing and
branch headcount reduction, the HRP yielded 291 employees who could no longer be redeployed,
fifteen (15) of whom belonged to Visayas Region III.

In implementing a redundancy program, it has been ruled that the employer is required to adopt a
fair and reasonable criteria, taking into consideration such factors as (a) preferred status; (b)
efficiency; and (c) seniority, among others. Consistent with this principle, Metrobank established
that, as a direct result of the adoption of the HRP, it was determined that the volume of
transactions in Visayas Region III required the further reduction of its eight-man reserve pool by
two employees. As these employees had no permanent place of assignment and merely acted as
relievers whenever temporary vacancies arise in other branches, they were the most logical
candidates for inclusion in the SSP. Already lacking preferred status in Metrobank’s hierarchy of

88
positions, Morales was included in the SSP because of his poor work performance which reportedly
caused complaints from the branches where he was temporarily assigned as reliever. To our mind,
the foregoing circumstances contradict Morales’ claim that he was arbitrarily singled out for
termination by Metrobank which, having validly determined the surplus in its manpower
complement appears to have appropriately identified him as a candidate for the SSP on account of
his work attitude.

Given Morales’ previous record of not reporting for work for one whole week without prior leave
of absence while assigned as reliever in its Borongan, Samar Branch, we find that Metrobank
cannot be faulted for including him in the list of employees covered by the SSP. The rule is settled
that “the determination that the employee’s services are no longer necessary or sustainable and,
therefore, properly terminable for being redundant is an exercise of business judgment of the
employer.”

“While it is true that management may not, under the guise of invoking its prerogative, ease out
employees and defeat their constitutional right to security of tenure,” the wisdom and soundness of
such characterization or decision is not subject to discretionary review unless a violation of law or
arbitrary or malicious action is shown. Against Morales’ bare assertion that he was arbitrarily and
maliciously terminated from service, Metrobank was able to establish that its action was based on
the fair application of a criterion established in connection with the implementation of a well-
thought redundancy program. For these reasons, we find that the CA cannot be faulted for
upholding the NLRC’s finding that Morales’ termination pursuant to the SSP was valid. Morales
next insists that Metrobank failed to comply in good faith with the notice requirement under
Article 283 of the Labor Code which allows the employer to terminate the employment of any
employee due to redundancy by serving a written notice on the worker and the DOLE at least one
(1) month before the intended date thereof. Intended to enable the employee to prepare himself
for the legal battle to protect his tenure of employment and to find other means of employment and
ease the impact of the loss of his job and his income, said notice requirement is also designed to
allow the DOLE to ascertain the verity of the cause for the termination. As correctly determined by
the CA, Metrobank’s compliance with this requirement is evident from its service of the 27 August
2003 notice of termination upon Morales on the same date, effective 1 October 2003 or 30 days
after the date of said notice.

On 29 August 2003, Metrobank similarly served the DOLE with an Establishment Termination
Report, together with a list of the 43 employees about to be terminated on the ground of
redundancy, effective 1 October 2003.

By and of themselves, the notices of termination Metrobank served to the DOLE and Morales one
month before their intended effectivity date significantly belie the latter’s claim that he was told not
to report for work anymore immediately upon receipt thereof. As proof of the bad faith and malice
which supposedly attended his separation from service, Morales asserted that Mariano caused him
great embarrassment by announcing that he was no longer required to report for work, within
hearing distance of his colleagues. For one who claims to have been immediately terminated from

89
employment, however, Morales quite distinctly indicated in his 18 February 2004 complaint that he
was dismissed on 30 September 2003. Reckoned from the service of notice of termination upon
Morales on 27 August 2003, said admitted date of dismissal clearly confirms Metrobank’s
compliance with the above-discussed one-month prior notice that the law requires for severance
from service on the ground of redundancy. Neither are we inclined to entertain Morales’ belated
argument that the real cause for his termination was retrenchment to prevent losses and that
Metrobank failed to establish the requirements therefor. For one, said theory contradicts Morales’
claim that he was dismissed from employment for personal reasons, in a manner amounting to
constructive dismissal. For another, not having been raised before the Labor Arbiter, the NLRC and
the CA, it stands to reason that Morales’ theory of termination to preserve the viability of
Metrobank’s business cannot be entertained for the first time in connection with the petition at
bench. Consistent with the principle that issues not raised a quo cannot be raised for the first time
on appeal, points of law, theories and arguments not brought to the attention of the CA need not –
and ordinarily will not – be considered by this Court. For a reviewing court to allow otherwise
would be offensive to the basic rules of fair play, justice and due process.

Morales, finally, argues that the CA erred in upholding the validity of the 10 November 2003
Release, Waiver and Quitclaim which he supposedly signed out dire economic necessity. While “it
may be accepted as ground to annul [a] quitclaim if the consideration is unconscionably low and the
employee was tricked into accepting it, [dire necessity is not, however,] an acceptable ground for
annulling the release when it is not shown that the employee has been forced to execute it.”

Not having sufficiently proved that he was forced to sign said Release, Waiver and Quitclaim,
Morales cannot expediently argue that quitclaims are looked upon with disfavor and considered
ineffective to bar claims for the full measure of a worker’s legal rights. This Court has held that not
all quitclaims are per se invalid or against public policy, except (1) where there is clear proof that
the waiver was wangled from an unsuspecting or gullible person, or (2) where the terms of
settlement are unconscionable on their face.

These two instances are not present in this case.

(Nov 21 2012)

MIRANT PHILIPPINES CORPORATION


VS
DANILO SARIO

Facts:
Defendant Sario sued for illegal dismissal, backwages, damages and attorney’s fees against
petitioner Mirant and its officers. Defendant worked as procurement officer for petitioner, where
their duties as procurement officers were guided by certain manuals for compliance in soliciting
bid quotations and proposals from vendors, suppliers and contractors. Defendant received a Show
Cause Notice advising him that he committed several violations of the manual based on internal

90
audit. He was given 10 days to explain why no disciplinary action should be taken against him and
he was notified that investigations were being conducted on the matter.

On October 6, 2005, and administrative hearing was held where Sario argued that his non
compliance was due to the improper dissemination of the manual and such failure to comply was
likewise due to his desire to meet the quotas. On October 25, 2005, Sliman sent Sario a letter
informing him of the termination of his employment for his failure to comply with the standard
operating procedures/instructions; for his serious misconduct or willful disobedience of the lawful
orders of the company in connection with his work; and for his gross and habitual neglect of his
duties.

Sario, argued before the Labor Arbiter that he was a mere rank-and-file employee with no
discretion in the procurement of materials; his work was merely recommendatory as it was subject
to the approval of his supervisor and other company officers. He pointed out that the show cause
notice to him was the first and only communication from the company calling his attention to his
alleged infractions. He stressed that at any rate, he should have been meted a lighter penalty, such
as suspension, considering his length of service with the company, without a derogatory record.
LA found that Sario was illegally dismissed. Labor Arbiter Anni stressed that the 2002 and 2004
Procurement Manuals have no commensurate penalties for any breach of their provisions and that
Sario’s dismissal was neither due to fraud nor willful breach of the trust reposed on him by his
employer, likewise it was found that Sario’s dismissal was too harsh a penalty, considering his
almost eight years of service, without a derogatory record, with the company.

NLRC reversed the labor arbiter’s ruling and dismissed the complaint for lack of merit. It found that
Sario was dismissed on valid grounds and was afforded due process.

CA granted the petition. It set aside the NLRC rulings and reinstated the Labor Arbiter’s decision.
Like the Labor Arbiter, it found the penalty of dismissal meted on Sario too harsh.

Issue: WON dismissal was illegal.

Held:
Under the law, the burden of proving that the termination of a worker’s employment was for a valid
or authorized cause rests on the employer. In this case, the company was able to prove that Sario’s
dismissal was for a valid cause. Through his repeated violations of the company’s 2002 and 2004
Procurement Manuals, Sario committed a serious misconduct or willful disobedience of the lawful
directives or orders of his employer, constituting a just cause for termination of employment.

Sario was not an ordinary rank-and-file employee. He was a procurement officer. While he did not
occupy a high position in the company hierarchy, the nature of his work made him, as the company
avers, a vital cog in its procurement program. The effectiveness of the program depended in no
small measure on the people running it, i.e., from the lowliest employee to the highest official.
Sario was one of these people and he was occupying, not a lowly but, a middle position. This

91
position carries with it responsibilities which only he can, and should, answer for. As the records
show, Sario failed to faithfully discharge his duties as procurement officer. These duties placed him
at the early but critical stage of the company’s procurement process. The very first one in the list of
his duties at once suggests the heavy responsibility he had to bear and the sensitiveness of his
functions, considering that he had to “[p]erform the entire purchasing process of a Station’s set of
materials, parts, equipment, and/or project[.]” Flowing from this catch-all statement, Sario’s
activities consisted of (1) receiving purchase requisition form assignments; (2) identifying the
vendors/suppliers to be invited, setting bid periods and deadlines for bid submission, including the
RFQ process – coordinating critical issues with end-users and preparing the RFQ package, sending
RFQs to vendors and initiating RFQ confirmation status, and resolving commercial issues with
vendors; (3) receiving quotes/bids, reviewing tenders and performing tender analysis summary
when necessary; (4) securing and evaluating justification for single tender transactions, and
coordinating price, payment and delivery terms with vendors; (5) preparing purchase orders and
checking of approval of purchase orders in accordance with the limits of authority; and (6)
coordinating vendor performance evaluation, resolving disputes between end-users and vendors,
and recommending appropriate sanctions for infractions committed by the vendors.

Over a span of almost one-and-a-half years, from January 2004 to May 2005 (not two years as the
company claims), Sario committed violations of the 2002 and 2004 Procurement Manuals in critical
areas of the procurement process, in particular, non-compliance with the minimum bid/quotation
requirements, non-compliance with the single tender justification requirement, failure to provide
proof of approval of the purchase requisition form, failure to provide proof of authorized
recommendation of the purchase order, failure to award purchase order to the lowest bidder, and
no tender analysis summary.

We understand the company’s serious concerns over Sario’s repeated violations of the 2002 and
2004 Procurement Manuals. Indeed, these violations cannot but compromise the integrity of the
company’s procurement process. A prime concern is “Sario’s unabated practice of sending RFQs to
non-responding suppliers,” instead of “to other accredited suppliers who could respond to xxx said
request[.]”

It submits that in so doing, Sario did not comply with the minimum bid/quotation requirements for
the purchase orders, not to mention that he also favored certain suppliers over the others. In such a
case, it points out, the bidding process becomes a farce; it defeats the real purpose of bidding, which
is to secure the best possible price. Given the critical and sensitive role Sario played in the
company’s procurement program, we appreciate why the company has employed all legal means to
terminate his services. Sario’s continued employment has become inimical to its business interests
which rely critically on the effectiveness and integrity of its procurement procedure. We can,
therefore, also understand why it had to issue the 2002 and 2004 Procurement Manuals – to ensure
that the procedure is not compromised. To be sure, the company has the prerogative to issue the
2002 and 2004 Procurement Manuals.

92
As the NLRC aptly noted, “the issuance of the 2002 and 2004 Procurement Manuals was a
reasonable and valid exercise of management prerogative xxx to curb the rampant practice of some
unscrupulous employees to favor some suppliers over the others in the award of Purchase
Orders[.]” “Any employee may be dismissed for violation of a reasonable company rule or
regulation for the conduct of the latter’s business[.]

Sario has to account for his own actions. The circumstance that his recommendations were
approved by his superiors does not erase the fact that he repeatedly violated the 2002 and 2004
Procurement Manuals. He was well aware of his duties and their parameters, based on the 2002
and 2004 Procurement Manuals. He committed the violations for oneand-a-half years. These
repeated violations can only indicate a willful disobedience to reasonable company rules and
regulations.

We thus find no basis for the CA’s ruling which, in effect, condoned Sario’s grave infractions against
the company. To our mind, this is a reversible error.
Based on the facts, the law and jurisprudence, Sario deserves to be dismissed for willful
disobedience. In Gold City Integrated Port Services, Inc. v. NLRC, the Court stressed that willful
disobedience of an employee contemplates the concurrence of at least two requisites: the
employee’s assailed conduct must have been willful or intentional, the willfulness being
characterized by a “wrongful and perverse attitude”; and the order violated must have been
reasonable, lawful and made known to the employee, and must pertain to the duties which he had
been engaged to discharge. We find the two requisites present in this case.

Sario’s repeated violations of the company’s 2002 and 2004 Procurement Manuals – lawful orders
in themselves as they provide the dos and, necessarily, the don’ts of a procurement officer –
constitute willful disobedience. He committed the repeated violations because he knew or was
confident that he would not get caught since his actions were being approved, as he claims, by his
superiors, evidencing wrongful or perverse intent. While the Constitution urges the moderation of
the sanction that may be applied to an employee where a penalty less punitive would suffice, as the
Court pronounced in Marival Trading, Inc. v. NLRC, cited by the CA, we do not believe that such a
moderation is proper in this case. Sario has become unfit to remain in employment. A contrary view
would be oppressive to the employer. "The law, in protecting the rights of the laborer ,
authorizes neither oppression nor self-destruction of the employer."

[G.R. No. 193857. November 28, 2012.]

MA. MERCEDES L. BARBA, PETITIONER,


VS.
LICEO DE CAGAYAN UNIVERSITY, RESPONDENT.

FACTS:
Dr. Barba was the Dean of the College of Physical Therapy of Liceo de Cagayan University. She
started working for the university as a medical officer/school physician. In July 1994, she was

93
chosen by the university to be the recipient of a scholarship grant to pursue a three-year residency
training in Rehabilitation Medicine at the Veterans Memorial Medical Center (VMMC). The
Scholarship Contract provides:

5.That the SCHOLAR after the duration of her study and training shall serve the SCHOOL in
whatever position the SCHOOL desires related to the SCHOLAR's studies for a period of not less
than ten (10) years;

After completing her residency training with VMMC in June 1997, she returned to continue working
for the university. She was appointed as Acting Dean and then later as Dean of the College of
Physical Therapy.

Due to the declining number of enrollees beginning school year 2003-2004, the university decided
to freeze the operation of the College of Physical Therapy indefinitely beginning end of school year
2005. Dr. Barba was informed in a letter dated March 16, 2005 that her services as dean of the said
college will end at the close of the school year. Thereafter, the College of Physical Therapy ceased
operations on March 31, 2005, and Dr. Barba went on leave without pay starting on April 9, 2005.
Subsequently, the university sent Dr. Barba letter dated April 27, 2005 instructing her to return to
work on June 1, 2005 and report to the Acting Dean of the College of Nursing, to receive her
teaching load and assignment as a full-time faculty member in that department for the school
year 2005-2006.

In reply, Dr. Barba said that she had not committed to teach in the College of Nursing and her
employment is not dependent on any teaching load. She then requested for the processing of her
separation benefits in view of the closure of the College of Physical Therapy. She did not report to
the Acting Dean of the College of Nursing on June 1, 2005. Instead, she followed-up on her
separation pay and other benefits.
The university wrote her on two other occasions, June 21 and June 24, directing her to report to
work and to teach her assigned subjects, otherwise she will be dismissed from employment on the
ground of abandonment. She was still bound by the Scholarship contract to serve the university for
two more years. Dr. Barba refused reiterating that teaching in the College of Nursing is in no way
related to her scholarship and training in the field of rehabilitation medicine. Dr. Barba added that
coercing her to become a faculty member from her position as College Dean is a great demotion
which amounts to constructive dismissal.

On June 28, 2005, the university sent Dr. Barba a notice terminating her services on the
ground of abandonment.
Meanwhile, on June 22, 2005, prior to the termination of her services, Dr. Barba filed a complaint
before the Labor Arbiter for illegal dismissal, payment of separation pay and retirement
benefits against the university. She alleged that her transfer to the College of Nursing as a
faculty member is a demotion amounting to constructive dismissal.

ISSUES:

94
Was Dr. Barba an employee or a corporate officer of Liceo de Cagayan? Does the Labor Arbiter and
NLRC have jurisdiction over Barba’s complaint for constructive dismissal? Was Dr. Barba
constructively dismissed?

RULING:

Dr. Barba was an employee and not a corporate officer of the university; consequently, the Labor
Arbiter and NLRC had jurisdiction. Further, Dr. Barba was not constructively dismissed.
Employee and not corporate officer

Corporate officers are elected or appointed by the directors or stockholders, and are those who are
given that character either by the Corporation Code or by the corporation's by-laws. Section 25 of
the Corporation Code enumerates corporate officers as the president, the secretary, the treasurer
and such other officers as may be provided for in the by-laws. In Matling Industrial and Commercial
Corporation v. Coros, the phrase "such other officers as may be provided for in the by-laws" has
been clarified, thus:

Conformably with Section 25, a position must be expressly mentioned in the By-Laws in order
to be considered as a corporate office. Thus, the creation of an office pursuant to or under a By-
Law enabling provision is not enough to make a position a corporate office. Guerrea v. Lezama, the
first ruling on the matter, held that the only officers of a corporation were those given that
character either by the Corporation Code or by the By-Laws; the rest of the corporate officers
could be considered only as employees of subordinate officials. Thus, it was held in Easycall
Communications Phils., Inc. v. King:

An "office" is created by the charter of the corporation and the officer is elected by the directors
or stockholders. On the other hand, an employee occupies no office and generally is employed
not by the action of the directors or stockholders but by the managing officer of the corporation
who also determines the compensation to be paid to such employee. (Emphasis supplied)

In the university's by-laws, there are four officers specifically mentioned, namely, a president, a vice
president, a secretary and a treasurer. In addition, it is provided that there shall be other appointive
officials, a College Director and heads of departments whose appointments, compensations, powers
and duties shall be determined by the board of directors. It is worthy to note that a College Dean is
not among the corporate officers mentioned in the university's by-laws. Dr. Barba, being an
academic dean, also held an administrative post in the university but not a corporate office
as contemplated by law. Dr. Barba was not directly elected nor appointed by the board of
directors to any corporate office but her appointment was merely approved by the board together
with the other academic deans of the university university in accordance with the procedure
prescribed in the university's Administrative Manual. The act of the board of directors in
approving the appointment of Dr. Barba as Dean of the College of Therapy did not make her
a corporate officer of the corporation.

95
It bears stressing that the appointive officials mentioned in Article V of the university's by-laws are
not corporate officers under the contemplation of the law. Though the board of directors may
create appointive positions other than the positions of corporate officers, the persons occupying
such positions cannot be deemed as corporate officers as contemplated by Section 25 of the
Corporation Code. On this point, the SEC Opinion dated November 25, 1993 quoted in the case of
Matling Industrial and Commercial Corporation v. Coros, is instructive:

Thus, pursuant to the above provision (Section 25 of the Corporation Code), whoever are the
corporate officers enumerated in the by-laws are the exclusive Officers of the corporation and the
Board has no power to create other Offices without amending first the corporate By-laws.
However, the Board may create appointive positions other than the positions of corporate
Officers, but the persons occupying such positions are not considered as corporate officers
within the meaning of Section 25 of the Corporation Code and are not empowered to
exercise the functions of the corporate Officers, except those functions lawfully delegated to
them. Their functions and duties are to be determined by the Board of Directors/Trustees.

Undoubtedly, Dr. Barba is an employee of the university. Applying the four-fold test concerning (1)
the selection and engagement of the employee; (2) the payment of wages; (3) the power of
dismissal; (4) the employer's power to control the employee with respect to the means and
methods by which the work is to be accomplished, it is clear that there exists an employer-
employee relationship between Dr. Barba and the university. Records show that Dr. Barba was
appointed to her position as Dean by the university president and was paid a salary of P32,500 plus
transportation allowance. It was evident that the university had the power of control over Dr. Barba
as one of its deans. It was also the university president who informed Dr. Barba that her services as
Dean of the College of Physical Therapy was terminated effective March 31, 2005 and she was
subsequently directed to report to the Acting Dean of the College of Nursing for assignment of
teaching load.

Thus, Dr. Barba, being an employee of the university, her complaint for illegal/constructive
dismissal against the university was properly within the jurisdiction of the Labor Arbiter and the
NLRC.

No constructive dismissal
Dr. Barba's letter of appointment specifically appointed her as Dean of the College of Physical
Therapy and Doctor-in-Charge of the Rehabilitation Clinic "for a period of three years effective July 1,
2002 unless sooner revoked for valid cause or causes." Evidently, Dr. Barba's appointment as College
Dean was for a fixed term, subject to reappointment and revocation or termination for a valid cause.
When the university decided to close its College of Physical Therapy due to drastic decrease in
enrollees, Dr. Barba's appointment as its College Dean was validly revoked and her subsequent
assignment to teach in the College of Nursing was justified as it is still related to her scholarship
studies in Physical Therapy.

96
As we observed in Brent School, Inc. v. Zamora, also cited by the CA, it is common practice in
educational institutions to have fixed-term contracts in administrative positions, thus:

Some familiar examples may be cited of employment contracts which may be neither for seasonal
work nor for specific projects, but to which a fixed term is an essential and natural
appurtenance: overseas employment contracts, for one, to which, whatever the nature of the
engagement, the concept of regular employment with all that it implies does not appear ever to
have been applied, Article 280 of the Labor Code notwithstanding; also appointments to the
positions of dean, assistant dean, college secretary, principal, and other administrative
offices in educational institutions, which are by practice or tradition rotated among the
faculty members, and where fixed terms are a necessity without which no reasonable
rotation would be possible. . . . (Emphasis supplied)

In constructive dismissal cases, the employer has the burden of proving that its conduct and
action or the transfer of an employee are for valid and legitimate grounds such as genuine
business necessity. Particularly, for a transfer not to be considered a constructive dismissal,
the employer must be able to show that such transfer is not unreasonable, inconvenient, or
prejudicial to the employee.

In this case, Dr. Barba's transfer was not unreasonable, inconvenient or prejudicial to her. On the
contrary, the assignment of a teaching load in the College of Nursing was undertaken by the
university to accommodate Dr. Barba following the closure of the College of Physical Therapy. The
university further considered the fact that Dr. Barba still has two years to serve the university
under the Scholarship Contract.

Dr. Barba's subsequent transfer to another department or college is not tantamount to demotion as
it was a valid transfer. There is therefore no constructive dismissal to speak of. That Dr. Barba
ceased to enjoy the compensation, privileges and benefits as College Dean was but a logical
consequence of the valid revocation or termination of such fixed-term position. Indeed, it would be
absurd and unjust for the university to maintain a deanship position in a college or department that
has ceased to exist. Under the circumstances, giving Dr. Barba a teaching load in another
College/Department that is related to Physical Therapy — thus enabling her to serve and complete
her remaining two years under the Scholarship Contract — is a valid exercise of management
prerogative on the part of the university.

[G.R. No. 197384. January 30, 2013.]

SAMPAGUITA AUTO TRANSPORT CORPORATION, PETITIONER,


VS.
NATIONAL LABOR RELATIONS COMMISSION AND EFREN I. SAGAD, RESPONDENTS.

FACTS:

97
Efren I. Sagad charged Sampaguita Auto Transport Corporation with illegal dismissal and damages
plus attorney's fees.
Allegedly, an evaluator boarded Sagad's bus. The evaluator described Sagad's manner of driving as
"reckless driver, nakikipaggitgitan, nakikipaghabulan, nagsasakay sa gitna ng kalsada, sumusubsob
ang pasahero[.]"
The company further alleged that it conducted a thorough evaluation of Sagad's performance. The
conductors revealed that Sagad proposed that they cheat on the company by way of an unreported
early bus trip. The dispatcher likewise submitted a negative report and even recommended the
termination of Sagad's employment. The company also cited Sagad's involvement in a hit-and-run
accident along Commonwealth Avenue in Quezon City while on a trip. Allegedly, Sagad did not
report the accident to the company.
Upon conclusion of the evaluation, the company terminated Sagad's employment.

ISSUE:
Was Sagad illegally dismissed?

RULING:
Dismissal was for cause – serious misconduct; but twin notice requirement was not fulfilled.

Just cause for dismissal


The irregularities or infractions committed by Sagad in connection with his work as a bus driver
constitute a serious misconduct or, at the very least, conduct analogous to serious
misconduct, under the above-cited Article 282 of the Labor Code. To be sure, his tendency to speed
up during his trips, his reckless driving, his picking up passengers in the middle of the road, his
racing with other buses and his jostling for vantage positions do not speak well of him as a bus
driver. While he denies being informed, when he was hired, of the duties and responsibilities of a
driver — contained in a document submitted in evidence by the company — the requirement "3. to
obey traffic rules and regulations as well as the company policies. 4. to ensure the safety of the
riding public as well as the other vehicles and motorist (sic)" is so fundamental and so universal
that any bus driver is expected to satisfy the requirement whether or not he has been so informed.
Sagad tries to minimize the adverse effect of the evaluator's report about his conduct as a driver
with the argument that he had already been penalized with a five-day suspension for chasing
another bus at one time. The suspension is of no moment. He was penalized for one reckless driving
incident, but it does not erase all the other infractions he committed. The conductors' comments
and the dispatcher's evaluation, together with the earlier on-board evaluation, all paint a picture of
a reckless driver who endangers the safety of his passengers, other motorists and the general
public. With this record, it is not surprising that he figured in a hit-and-run accident.
Under the circumstances, Sagad has become a liability rather than an asset to his employer, more so
when we consider that he attempted to cheat on the company or could have, in fact, defrauded the
company during his brief tenure as a bus driver. This calls to mind the report on the low revenue of
Sagad's bus, an observation which is validated by the company's Daily Operation Reports.
All told, we find substantial evidence supporting Sagad's removal as a bus driver. Through his
reckless driving and his schemes to defraud the company, Sagad committed serious misconduct and

98
breach of the trust and confidence of his employer, which, without doubt, are just causes for his
separation from the service.
Non-compliance with the twin-notice requirement
Even as we find a just cause for Sagad's dismissal, we agree with the CA that the company failed to
comply with the two-notice rule. It failed to serve notice of: (1) the particular acts for which Sagad
was being dismissed on November 5, 2006 and (2) his actual dismissal. Consistent with our ruling
in Agabon v. NLRC, we hold that the violation of Sagad's right to procedural due process entitles him
to an indemnity in the form of nominal damages. Considering the circumstances in the present case,
we deem it appropriate to award Sagad P30,000.00.

G.R. No. 197384 Jan. 30, 2013

SAMPAGUITA AUTO TRANSPORT CORPORATION


VS.
NLRC

FACTS:

Sagad alleged that on May 14, 2006, the company hired him as a regular bus driver, not a
probationary employee as the company claimed. He disowned his signature on the contract of
employment submitted as evidence by the company by maintaining that his signature was forged.
He further alleged that on Nov 5, 2006, he was dismissed by the company for allegedly conniving
with conductor Vitola issuing tickets outside their assigned route.

The company countered that it employed Sagad as probationary bus driver from May 14-October
14, 2006. He was informed that during the probationary period, his attendance, performance and
work attitude shall be evaluated, therefore, as a matter of company policy, an evaluator was
deployed on a company bus (in the guise of a passenger) to observe the driver’s work performance
and attitude.

Allegedly, on Sept 21, 2006, an evaluator boarded Sagad’s bus. The evaluator described Sagad’s
manner of driving as “reckless driver, nakikipaggitgitan, nagsasakay sa gitna ng kalsa, sumusubsob
and pasahero.” Sagad claimed that he could not have been driving as reported because his pregnant
wife and his child were with him on the bus. He admitted thought that at one time, he chased an
“Everlasting” bus to serve warning on its driver not to block his bus when he was overtaking. He
also admitted that once in a while, he sped up to make up for lost time in making trips.

The company further alleged that on Oct 13, 2006, it conducted thorough evaluation of Sagad’s
performance. Conductors A. Hemoroz and Israel Lucero revealed that Sagad proposed that they
cheat on the company by way of an unreported early bus trip. Dispatcher E. Castillo likewise

99
submitted a negative report recommending Sagad’s termination. Also, the company cited Sagad’s
involvement in a hit and run accided which he allegedly did not report such accident to the
company.

On Oct 15, 2006, the company terminated Sagad’s employment for his failure to qualify as a regular
employee.

Labor Arbiiter Padolina dismissed complaint for lack of merit. NLRC declared Sagad had been
illegally dismissed. CA affirmed NLRC’s rulings in toto.

ISSUE:
Is Sagad’s dismissal illegal?

HELD:

YES. The Court disagrees with the finding that Sagad’s dismissal had no basis.

During his brief employment with the company, he exhibited the tendency to speed up when he
finds the need for it, very obviously in violation of traffic rules, regulations and company policy.
Instead of negating the evaluator’s observations, his admissions make them credible.

The CA misappreciated the law when it declared that the grounds relied upon by the company in
terminating Sagad are not among those enumerated under Art 282 of the Labor Code. The
irregularities or infractions committed by Sagad in connection with his work as a bus driver
constitute serious misconduct or, at the very least, conduct analagous to serious misconudct under
the Labor Code. To be sure, his tendency to speed up during his trips, his reckless driving, his
picking up passengers in the middle of the road, his racing with other buses and his jostling for
vantage positions do not speak well of his as a bus driver. While he denies being informed, when he
was hired, of the duties and responsibilities of a driver – contained in a document submitted in
evidence by the company – the requirement “3. To obey traffic rules and regulations as well as the
company policies. 4. To ensure the safety of the riding public as well as the other vehicles and
motorist (sic)” is so fundamental and so universal that any bus driver is expected to satisfy the
requirement whether or not he has been so informed.

The Court finds substantial evidence supporting Sagad’s removal as a bus driver. Through his
reckless driving and his schemes to defraud the company, Sagad committed serious misconduct and
breach of the trust and confidence of his employer, which, without doubt, are just causes for his
separation from the service.

TWIN – NOTICE REQUIREMENT


The company failed to comply with the two-notice rule. It failed to serve notice of: (1) particular
acts for which Sagad was being dismissed on Nov 5, 2006 and (2) his actual dismissal. Thus, the

100
violation of Sagad’s right to procedural due process entitles him to an indemnity in the form of
nominal damages of P30,000.

G.R. No. 181738 January 30, 2013

GENERAL MILLING CORPORATION


V.
VIAJAR

FACTS:

In October 2003, GMC terminated the services of 13 employees for redundancy, including herein
respondent Violeta Viajar. GMC alleged that it has been downsizing its Vismin Operations in Cebu.

Viajar filed a Complaint for Illegal Dismissal with damages against GMC et.al. She received a Letter-
Memorandum from GMC through Almocera, informing her that her services were no longer needed,
effective Nov 30, 2003 because her position as Purchasing Staff was deemed redundant.

When Viajar reported for work on Oct 31, 2003, a month before effectivity of her severance, the
guard barred her from entering GMC’s premises. She was also denied access to her office computer
and was restricted from punching her daily time record in the bundy clock.

On Nov 7, 2003, Viajar was invited to the HRD Cebu Office where she was asked to sign an
“Application for Retirement and Benefits.” The respondent refused to sign because she did not
apply for retirement and asserted that her services were terminated for alleged redundancy.
Almocera told her that her signature on the document was needed to process her separation pay.
The respondent also claimed that between July 4-Oct 13, 2003, GMC hired 15 new employees which
aroused her suspicion that her dismissal was not necessary.

ISSUE:
Is Viajar’s dismissal on the ground of redundancy done in good faith?

HELD:
No.

Art. 283 of the Labor Code provides that redundancy is one of the authorized causes for dismissal. It
is imperative from Art. 283 LC that the employer must comply with the requirements for a valid
implementation of the company’s redundancy program, to wit: (a) the employer must serve a
written notice to the affected employees and the DOLE at least one (1) month before the intended
date of retrenchment; (b) the employer must pay the employees a separation pay equivalent to at
least one month pay or at least one month pay for every year of service, whichever is higher; (c) the

101
employer must abolish the redundant positions in good faith; and (d) the employer must set fair
and reasonable criteria in ascertaining which positions are redundant and may be abolished.

While it is true that the “characterization of an employee’s services as superfluous or no longer


necessary and, therefore, properly terminable, is an exercise of business judgment on the part of
the employer,” the exercise of such judgment, however, must not be in violation of the law, and
must not be arbitrary or malicious. The Court has always stressed that a company cannot simply
declare redundancy without basis. To exhibit its good faith and that there was a fair and reasonable
criteria in ascertaining redundant positions, a company claiming to be over manned must produce
adequate proof of the same.

In the instant case, the Court agrees with the CA when it held that the petitioner failed to present
substantial proof to support GMC’s general allegations of redundancy. As shown from the records,
the petitioner simply presented as its evidence of good faith and compliance with the law the
notification letter to respondent Viajar; the “Establishment Termination Report” it submitted to the
DOLE Office; the two (2) checks issued in the respondent’s name amounting to P440,253.02 and
P21,211.35;41 and the list of terminated employees as of June 6, 2006. We agree with the CA that
these are not enough proof for the valid termination of Viajar’s employment on the ground of
redundancy.

The letter-memorandum which contains general allegations is not enough to convince this Court
that Viajar’s termination of employment due to redundancy was warranted under the
circumstances. There is no showing that GMC made an evaluation of the existing positions and their
effect to the company. Neither did GMC exert efforts to present tangible proof that it was
experiencing business slow down or over hiring.

While the petitioner had been harping that it was on a “reduction mode” of its employees, it has not
presented any evidence (such as new staffing pattern, feasibility studies or proposal, viability of
newly created positions, job description and the approval of the management of the restructuring,
audited financial documents like balance sheets, annual income tax returns and others)which could
readily show that the company’s declaration of redundant positions was justified. Such proofs, if
presented, would suffice to show the good faith on the part of the employer or that this business
prerogative was not whimsically exercised in terminating respondent’s employment on the ground
of redundancy. Unfortunately, these are wanting in the instant case.

On the other hand, the respondent presented proof that the petitioner had been hiring new
employees while it was firing the old ones, negating the claim of redundancy.

Furthermore, the Court cannot overlook the fact that Viajar was prohibited from entering the
company premises even before the effectivity date of termination; and was compelled to sign an
“Application for Retirement and Benefits.” These acts exhibit the petitioner’s bad faith since it
cannot be denied that the respondent was still entitled to report for work until November 30, 2003.
The demand for her to sign the “Application for Retirement and Benefits” also contravenes the fact

102
that she was terminated due to redundancy. Indeed, there is a difference between voluntary
retirement of an employee and forced termination due to authorized causes.

Clearly, the instant case is not about retirement since the term has its peculiar meaning and is
governed by Article 287 of the Labor Code. Rather, this is a case of termination due to redundancy
under Article 283 of the Labor Code. Thus, the demand of GMC for the respondent to sign an
“Application for Retirement and Benefits” is really suspect.

GR. No. 173189 Feb. 13, 2013

JONATHAN SANG-AN
VS
EQUATORN KNIGHTS DETECTIVE AND SECURITY AGENCY, INC.

Digested by: Angge

Facts: Jonathan was the Assistant OM of Equator. He was tasked, among others, of safekeeping
Equator’s firearms. Equator discovered that two firearms were missing from its inventory. The
investigation revealed that it was Jonathan who might have been responsible for the loss and thus
he was temporarily suspended from work pending further investigation.

While Jonathan was under suspension, a security guard from Equator was apprehended by
policemen for violating the COMELEC’s gun ban rule. The security guard stated in his Affidavit that
the unlicensed firearm had been issued to him by Jonathan.

Jonathan filed a complaint for illegal suspension with prayer for reinstatement. In his position
paper, however, he treated his case as one for illegal dismissal and alleged that he had been denied
due process when he was dismissed. Equator, on the other hand, argued that Jonathan’s dismissal
was not illegal but was instead for a just cause under Article 282 of the Labor Code.

LA rendered a decision declaring that no illegal dismissal took place as Jonathan’s services were
terminated pursuant to a just cause. The NLRC sustained the findings of the LA but found that
Jonathan had been denied his right to due process when he was dismissed and ordered
Equator to pay Jonathan backwages. The CA reversed the decision of the NLRC.

Issue: WON Jonathan was validly dismissed.

Held: In order to validly dismiss an employee, it is fundamental that the employer observe both
substantive and procedural due process – the termination of employment must be based on a just
or authorized cause and the dismissal can only be effected, after due notice and hearing.

This Court finds that Equator complied with the substantive requirements of due process when
Jonathan committed the two offenses.

103
Article 282(A) of the Labor Code provides that an employee may be dismissed on the ground
of serious misconduct or willful disobedience of the lawful orders of his employer or
representative in connection with his work. Misconduct is improper or wrongful conduct; it is the
transgression of some established and definite rule of action, a forbidden act, a dereliction of duty,
willful in character, and implies wrongful intent and not mere error of judgment. The misconduct,
to be serious within the meaning of the Labor Code, must be of such grave and aggravated character
and not merely trivial or unimportant. It is also important that the misconduct be in connection
with the employee's work to constitute just cause for his separation. By losing two firearms and
issuing an unlicensed firearm, Jonathan committed serious misconduct. He did not merely violate a
company policy; he violated the law itself (Presidential Decree No. 1866) and placed Equator and
its employees at risk of being made legally liable. Thus, Equator had a valid reason that warranted
Jonathan’s dismissal from employment as Assistant Operation Manager.

Equator, however, failed to observe the proper procedure in terminating Jonathan’s services.
Section 2, Rule XXIII, Book V of the Omnibus Rules require the employer to furnish the employee
with two written notices before termination of employment can be effected: a first written
notice that informs the employee of the particular acts or omissions for which his or her dismissal
is sought, and a second written notice which informs the employee of the employer's decision to
dismiss him. In considering whether the charge in the first notice is sufficient to warrant dismissal
under the second notice, the employer must afford the employee ample opportunity to be heard.

A review of the records shows that Jonathan was not furnished with any written notice that
informed him of the acts he committed justifying his dismissal from employment. The notice of
suspension given to Jonathan only pertained to the first offense, i.e., the loss of Equator’s firearms
under Jonathan’s watch. With respect to his second offense (i.e., the issuance of an unlicensed
firearm to Equator’s security guard – that became the basis for his dismissal), Jonathan was never
given any notice that allowed him to air his side and to avail of the guaranteed opportunity to be
heard. That Equator brought the second offense before the LA does not serve as notice because by
then, Jonathan had already been dismissed.

In order to validly dismiss an employee, the observance of both substantive and procedural due
process by the employer is a condition sine qua non. Procedural due process requires that the
employee be given a notice of the charge against him, an ample opportunity to be heard, and a
notice of termination. Since Jonathan had been dismissed in violation of his right to procedural due
process but for a just cause, Equator should pay him nominal damages of P30,000.00, in accordance
with Agabon v. NLR

104
G.R. No. 186344 February 20, 2013

LEOPARD SECURITY AND INVESTIGATION AGENCY


VS
TOMAS QUITOY, RAUL SABANG AND DIEGO MORALES

Digested by: Angge

Facts: Respondents were hired as security guards by petitioner Leopard Security and Investigation
Agency (LSIA). All being residents of Cebu City, respondents were assigned by LSIA to the different
branches of its only client in said locality, Union Bank of the Philippines (Union Bank). On 1 April
2005, it appears that Union Bank served a notice to LSIA, terminating the parties’ security service
contract effective at the end of business hours of 30 April 2005. Thru its representative,
Rogelio Morales, LSIA informed respondents on 29 April 2005 of the termination of its contract
with Union Bank which had decided to change its security provider.

On 3 May 2005, respondents filed a complaint for illegal dismissal, unpaid 13th month pay and
service incentive leave pay (SILP), moral and exemplary damages as well as attorney’s fees against
LSIA. Respondents asserted that, after introducing himself as a representative of LSIA on 29 April
2005, Morales belatedly informed them that their services would be terminated at the end of the
office hours on the same business day.

Informed by Union Bank on 1 April 2005 of the termination of their security service contract
effective 30 April 2005, LSIA claimed that it relieved respondents from their assignments by the
end of the business hours of the latter date and on 10 May 2005, it directed respondents to report
for work at its Mandaluyong City office. As respondents failed to do so, LSIA alleged that it issued
show cause letters on 21 June 2005, requiring the former to explain why they should not be
administratively sanctioned for their unexplained absences.

LA found LSIA liable for the illegal dismissal of respondents. Faulting LSIA for informing
respondents of the termination of their services only on 30 April 2005 despite Union Bank’s 1 April
2005 advice of the termination of its security service contract. LSIA was ordered to pay the former’s
claim for separation pay on the ground that reinstatement was no longer feasible under the
circumstances.

Applying the principle that security agencies like LSIA are allowed to put security guards on
temporary off-detail or floating status for a period not exceeding six months, the NLRC discounted
the factual and legal bases for the illegal dismissal determined by the Labor Arbiter as well as the
backwages awarded in favor of respondents. Even then, the NLRC upheld the Labor Arbiter’s award
of separation pay on the theory that reinstatement was no longer viable. The awards of
proportionate 13th month pay and SILP for which Union Bank and LSIA were held solidarily liable
were likewise sustained.

105
Applying the principle that respondents could not be considered illegally dismissed before the lapse
of six months from their being placed on floating status by LSIA, the CA justified the awards of
separation pay, proportionate 13th month pay and SILP.

Issue:

1. WON the respondents were illegally dismissed.


2. WON separation pay was proper.

Held: Applying Article 28624 of the Labor Code of the Philippines by analogy, this Court has
repeatedly recognized that security guards may be temporarily sidelined by their security agency
as their assignments primarily depend on the contracts entered into by the latter with third
parties. Temporary "off-detail" or "floating status" is the period of time when security guards are in
between assignments or when they are made to wait after being relieved from a previous post until
they are transferred to a new one. It takes place when, as here, the security agency’s clients decide
not to renew their contracts with the agency, resulting in a situation where the available posts
under its existing contracts are less than the number of guards in its roster. For as long as such
temporary inactivity does not continue for a period exceeding six months, it has been ruled that
placing an employee on temporary "off-detail" or "floating status" is not equivalent to
dismissal.

In the case at bench, respondents were informed on 29 April 2005 that they were going to be
relieved from duty as a consequence of the 30 April 2005 expiration of the security service contract
between Union Bank and LSIA. While respondents lost no time in immediately filing their complaint
on 3 May 2005, the record equally shows that they were directed by LSIA to report for work at its
Mandaluyong City office on 10 May 2005 or a mere ten days from the time the former were
effectively sidelined. Considering that a security guard is only considered illegally dismissed
from service when he is sidelined from duty for a period exceeding six months, respondents
were not illegally dismissed by LSIA.

Having correctly ruled out illegal dismissal of respondents, the CA reversibly erred, however, when
it sustained the NLRC’s award of separation pay on the ground that the parties’ relationship had
already been strained. For one, liability for the payment of separation pay is a legal
consequence of illegal dismissal where reinstatement is no longer viable or feasible. Under
Article 279 of the Labor Code, an illegally dismissed employee is entitled to the twin reliefs of full
backwages and reinstatement without loss of seniority rights. Separation pay is, however, granted
when reinstatement is no longer feasible because of strained relations between the employer and
the employee. In cases of illegal dismissal, the accepted doctrine is that separation pay is available
in lieu of reinstatement when the latter recourse is no longer practical or in the best interest of the
parties.

106
As a relief granted in lieu of reinstatement, however, it consequently goes without saying
that an award of separation pay is inconsistent with a finding that there was no illegal
dismissal. Standing alone, the doctrine of strained relations will not justify an award of separation
pay, a relief granted in instances where the common denominator is the fact that the employee was
dismissed by the employer.35

Even in cases of illegal dismissal, the doctrine of strained relations is not applied indiscriminately as
to bar reinstatement, especially when the employee has not indicated an aversion to returning to
work36 or does not occupy a position of trust and confidence in37 or has no say in the operation of
the employer’s business.38 Although litigation may also engender a certain degree of hostility, it has
likewise been ruled that the understandable strain in the parties’ relations would not necessarily
rule out reinstatement which would, otherwise, become the rule rather than the exception in illegal
dismissal cases. Absent illegal dismissal on the part of LSIA and abandonment of employment
on the part of respondents, we find that the latter’s reinstatement without backwages is,
instead, in order.

G.R. No. 192826 February 27, 2013

PHILIPPINE PLAZA HOLDINGS, INC., PETITIONER,


VS.
MA. FLORA M. EPISCOPE, RESPONDENT

FACTS:
Petitioner Philippine Plaza Holdings, Inc. (PPHI) is the owner and operator of the Westin Philippine
Plaza Hotel (Hotel). Respondent Ma. Flora M. Episcope (Episcope) was employed by PPHI since July
24, 1984 until she was terminated on November 4, 2004 for dishonesty, willful disobedience and
serious misconduct amounting to loss of trust and confidence.
Sycip, Gorres and Velayoauditors, who are independent auditors and/or professional shoppers to
check employee performance, dined at the Hotel’s Café Plaza on August 28, 2004. The amount
of P2,306.65 was billed to them. Episcope was one of those who attended to the auditors and was
the one who handed the check and received the payment of P2,400.00. She thereafter returned
Check No. 565938, which was stamp marked "paid," together with the change.
it was later discovered that the Hotel's copy of the receipt bore a discount of P906.45. Howevever,
the receipt issued by Episcope to the auditors reflected the undiscounted amount of P2,306.65
considering that none of the auditors had such discount card. In view of the foregoing, the amount
actually remitted to the Hotel was only P1,400.20thus, leaving a shortage of P906.45
On September 30, 2004, the Hotel issued a Show-Cause Memo. In her handwritten letter, Episcope
admitted that she was on duty on the date and time in question but alleged that she could no longer
recall if the concerned guests presented a Starwood Privilege Discount Card.

107
Episcope was placed on preventive suspension without pay. During the administrative hearing on,
Episcope, confirmed the fact that she was the one who presented the subject check and received the
corresponding payment from the guests. She, however, denied stamping the said check as "paid" or
that she gave any discount without a discount card, explaining that she could not have committed
such acts given that all receipts and discount applications were handled by the cashier. But when
asked why the discounted receipt was not given to the guests, she merely replied that she could no
longer remember. In a separate inquiry, the cashier of Café Plaza, however, maintained that a
Starwood Privilege Discount Card must have been presented during the said incident given that
there was a Discount Slip and a stamped receipt indicating such discounted payment.
Finding Episcope to have failed to sufficiently explain the questionable discount application on the
settlement bill of the auditors, her employment was terminated for committing acts of dishonesty,
which was classified as a Class D offense under the Hotel's Code of Discipline, as well as for willful
disobedience, serious misconduct and loss of trust and confidence.
On October 20, 2005, the Labor Arbiter (LA) rendered a Decision in favor of PPHI. On appeal, the
NLRC affirmed the LA's decision in the May 30, 2007 Resolution. Episcope's motion for
reconsiderationwas likewise denied in the November 14, 2007 Resolution.
However, on certiorari, the CA gave due course to the petition and reversed the NLRC's Decision

Dissatisfied, PPHI moved for reconsideration which was, however, denied in the assailed July 5,
2010 Resolution.
Hence, the instant petition anchored on the sole ground that:\

ISSUE:
THE HONORABLE COURT OF APPEALS SERIOUSLY ERRED AND RULED CONTRARY TO LAW
AND JURISPRUDENCE WHEN IT ACTED AS A TRIER OF FACTS AND ORDERED THE
REINSTATEMENT OF THE RESPONDENT AND PAYMENT OF BACKWAGES

RULING:
The petition is impressed with merit.

At the outset, it is settled that the jurisdiction of the Supreme Court in cases brought before it from
the CA via Rule 45 of the Rules of Court is generally limited to reviewing errors of law. The Court is
not the proper venue to consider a factual issue as it is not a trier of facts. The rule, however, is not
ironclad and a departure therefrom may be warranted where the findings of fact of the CA are
contrary to the findings and conclusions of the trial court or quasi-judicial agency, as in this case.
There is therefore a need to review the records to determine which of them should be preferred as
more conformable to evidentiary facts.

After a judicious review of the records, as well as the respective allegations and defenses of the
parties, the Court is constrained to reverse the findings and conclusion of the CA.
Article 293 (formerly Article 279) of the Labor Codeprovides that the employer shall not terminate
the services of an employee except only for a just or authorized cause

108
Among the just causes for termination is the employer’s loss of trust and confidence in its
employee. Article 296 (c) (formerly Article 282 [c]) of the Labor Code provides that an employer
may terminate the services of an employee for fraud or willful breach of the trust reposed in him.
But in order for the said cause to be properly invoked, certain requirements must be complied with
namely,(1) the employee concerned must be holding a position of trust and
confidence and (2) there must be an act that would justify the loss of trust and confidence.

There are two classes of positions of trust: (1), there are managerial employees whose primary duty
consists of the management of the establishment in which they are employed or of a department or
a subdivision thereof, and to other officers or members of the managerial staff; (2) there are
fiduciary rank-and-file employees, such as cashiers, auditors, property custodians, or those who, in
the normal exercise of their functions, regularly handle significant amounts of money or property.
These employees, though rank-and-file, are routinely charged with the care and custody of the
employer's money or property, and are thus classified as occupying positions of trust and
confidence. Episcope belongs to this latter class and therefore, occupies a position of trust and
confidence.

Anent the second requisite, records likewise reveal that Episcope committed an act which justified
her employer’s (PPHI’s) loss of trust and confidence in her.
Primarily, it is apt to point out that proof beyond reasonable doubt is not required in dismissing an
employee on the ground of loss of trust and confidence; it is sufficient that there lies some basis to
believe that the employee concerned is responsible for the misconduct and that the nature of the
employee's participation therein rendered him absolutely unworthy of trust and confidence
demanded by his position.

On this point, the Court, in the case of Bristol Myers Squibb (Phils.), Inc. v. Baban,28 citing Atlas
Fertilizer Corporation v. National Labor Relations Commission,29 ruled as follows:
As a general rule, employers are allowed a wider latitude of discretion in terminating the services of
employees who perform functions by which their nature require the employer's full trust and
confidence. Mere existence of basis for believing that the employee has breached the trust and
confidence of the employer is sufficient and does not require proof beyond reasonable doubt. Thus,
when an employee has been guilty of breach of trust or his employer has ample reason to distrust
him, a labor tribunal cannot deny the employer the authority to dismiss him.
The only substantial evidence is required in order to support a finding that an employer’s trust and
confidence accorded to its employee had been breached. As explained in the case of Lopez v. Alturas
Group of Companies:

it must be based on substantial evidence and not on the employer's whims or caprices or
suspicions otherwise, the employee would eternally remain at the mercy of the employer. Loss of
confidence must not be indiscriminately used as a shield by the employer against a claim that the
dismissal of an employee was arbitrary. And, in order to constitute a just cause for dismissal, the act
complained of must be work-related and shows that the employee concerned is unfit to continue
working for the employer. In addition, loss of confidence as a just cause for termination of

109
employment is premised on the fact that the employee concerned holds a position of responsibility,
trust and confidence or that the employee concerned is entrusted with confidence with respect to
delicate matters, such as the handling or care and protection of the property and assets of the
employer. The betrayal of this trust is the essence of the offense for which an employee is penalized.
(Emphasis supplied.)

In the present case, records would show that Episcope committed acts of dishonesty which resulted
to monetary loss on the part of PPHI and more significantly, led to the latter’s loss of trust and
confidence in her. Notwithstanding the impaired probative value of the unaudited and unsigned
auditor’s report, the totality of circumstances supports the foregoing findings:
First, it remains unrefuted that Episcope attended to the auditors when they dined at the Café Plaza
on the date and time in question

Second, it is likewise undisputed that the check receipt on file with the Hotel for the same
transaction reflected only the amount of P1,400.20 in view of the application of a certain Starwood
Privilege Discount Card registered in the name of one Peter Pamintuan, while the receipt given to
the auditors bore the undiscounted amount of P2,306.65 which thus, resulted to a P906.45
discrepancy. During the proceedings, both receipts were actually presented in evidence yet,
Episcope never interposed any objection on the authenticity of the same; and

Third, when asked to explain the said discrepancy, Episcope merely imputed culpability on the part
of the cashier, whom she claimed prepared all the receipts that were returned to the guests.

From the foregoing incidents, it is clear that Episcope was remiss in her duty to carefully account
for the money she received from the cafe's guests. It must be observed that though the receipts
were prepared by the cashier, Episcope; as a service attendant,. was the one who actually handled
the money tendered to her by the hotel clients. In this regard, prudence dictates that Episcope
should have at least known why there was a shortage in remittance. Yet when asked, Episcope
could not offer any plausible explanation but merely shifted the blame to the cashier. Irrefragably,
as an employee who was routinely charged with the care and custody of her employer's money,
Episcope was expected to have been more circumspect in the performance of her duties as a service
attendant. This she failed to observe in the case at bar which thus, justifies PPHI's loss of trust and
confidence in her as well as her consequent dismissal.

WHEREFORE, premises considered, the petition is GRANTED. The assailed March 26, 2010
Decision and July 5, 2010 Resolution of the Court of Appeals in CA-G.R. SP No. 102188
are REVERSED and SET ASIDE. The Decision of the Labor Arbiter, as affirmed by the NLRC,
dismissing respondent Ma. Flora M. Episcope's complaint for illegal dismissal and other monetary
claims is REINSTATED.

110
G.R. No. 184520 March 13, 2013

ROLANDO DS.TORRES, PETITIONER,


VS.
RURAL BANK OF SAN JUAN, INC., ANDRES CANO CHUA, JOBEL GO CHUA, JESUS CANO CHUA,
MEINRADO DALISAY, JOSE MANALANSAN III, OFELIA GINA BE AND NATY
ASTRERO, RESPONDENTS.

DECISION

FACTS:

This Petition for Review on Certiorari, under Rule 45 of the Rules of Court, seeks to reverse and set
aside the Decision dated February 21, 2008 of the Court of Appeals (CA) in CA-G.R. SP No. 94690
dismissing the complaint for illegal dismissal filed by petitioner Rolando OS. Torres (petitioner)
against respondents Rural Bank of San Juan, Inc. (RBSJT) and its officers.

The petitioner was initially hired by RBSJI as Personnel and Marketing Manager in 1991. After six
months RBSJI renewed his employment for the same post to a permanent/regular status. In June
1996, the petitioner was offered the position of Vice-President for RBSJI’s newly created
department, Allied Business Ventures. He accepted the offer and concomitantly relinquished his
post. On September 24, 1996, the petitioner was temporarily assigned as the manager of RBSJI’s N.
Domingo branch in view of the resignation of Jacinto Figueroa (Jacinto).

On September 27, 1996, Jacinto requested the petitioner to sign a standard employment clearance
pertaining to his accountabilities with RBSJI. When the petitioner declined his request, Jacinto
threw a fit and shouted foul invectives. To pacify him, the petitioner bargained to issue a clearance
but only for Jacinto’s paid cash advances and salary loan.

About seven months later or on April 17, 1997, respondent Jesus issued a memorandum to the
petitioner requiring him to explain why no administrative action should be imposed on him for his
unauthorized issuance of a clearance to Jacinto whose accountabilities were yet to be audited.
Jacinto was later found to have unliquidated cash advances and was responsible for a questionable
transaction involving P11 million for which RBSJI is being sued. The memorandum stressed that the
clearance petitioner issued effectively barred RBSJI from running after Jacinto.

The petitioner submitted his explanation on the same day clarifying that the clearance was limited
only to Jacinto’s paid cash advances and salary loan based on the receipts presented by Lily Aguilar
(Lily), the cashier of N. Domingo branch. He emphasized that he had no foreknowledge nor was he

111
forewarned of Jacinto’s unliquidated cash advances and questionable transactions and that the
clearance did not extend to those matters.7

After conducting an investigation, RBSJI’s Human Resources Department recommended the


petitioner’s termination from employment which RBSJI’s Board of Directors adopted, terminating
the petitioner from employment, the import of which was communicated to him in a Memorandum
dated May 30, 1997.

Feeling aggrieved, the petitioner filed the herein complaint for illegal dismissal, illegal deduction,
non-payment of service incentive, leave pay and retirement benefits. The petitioner averred that
the supposed loss of trust and confidence on him was a sham as it is in fact the calculated result of
the respondents’ dubious plot to conveniently oust him from RBSJI.

He claimed that he was deceived to accept a Vice-President position, which turned out to be a mere
clerical and menial work, so the respondents can install Jobel, the son of a major stockholder of
RBSJI, as Personnel and Marketing Manager. The plot to oust the petitioner allegedly began in 1996
when Jobel annexed the Personnel and Marketing Departments to the Business Development and
Corporate Planning Department thus usurping the functions of and displacing the petitioner, who
was put on a floating status and stripped of managerial privileges and allowances.

The petitioner further alleged that he was cunningly assigned at N. Domingo branch so he can be
implicated in the anomalous transaction perpetrated by Jacinto. He narrated that on September 27,
1996, the officers of RBSJI, namely: Jobel, Andres, Jose and Ofelia, were actually at the N. Domingo
branch but they all suspiciously left him to face the predicament caused by Jacinto.

He recounted that the next day he was assigned back at the Tarlac extension office and thereafter
repeatedly harassed and forced to resign. The petitioner requested for his transfer to the
operations or marketing department. His request was, however, not acted upon.

The petitioner claimed that on March 19, 1997, respondent Jesus verbally terminated him from
employment but he later on retracted the same and instead asked the petitioner to tender a
resignation letter. The petitioner refused. A month thereafter, the petitioner received the
memorandum asking him to explain why he cleared Jacinto of financial accountabilities and
thereafter another memorandum terminating him from employment.

For their part, the respondents maintained that the petitioner was validly dismissed for loss of trust
and confidence precipitated by his unauthorized issuance of a financial accountability clearance
sans audit to a resigned employee. They averred that a copy of the clearance mysteriously
disappeared from RBSJI’s records hence, the petitioner’s claim that it pertained only to Jacinto’s
paid cash advances and salary loan cannot stand for being uncorroborated.

ISSUE:

112
whether or not there was a loss of trust and confidence which gave respondents just cause to
terminate petitioner validly from employment.

RULING:
The petition is impressed with merit.
Settled is the rule that when supported by substantial evidence, the findings of fact of the CA are
conclusive and binding on the parties and are not reviewable by this Court. As such, only errors of
law are reviewed by the Court in petitions for review of CA decisions. By way of exception,
however, the Court will exercise its equity jurisdiction and re-evaluate, review and re-examine the
factual findings of the CA when, as in this case, the same are contradicting with the findings of the
labor tribunals.

The respondents failed to prove that the petitioner was dismissed for a just cause.
As provided in Article 282 of the Labor Code and as firmly entrenched in jurisprudence,33 an
employer has the right to dismiss an employee by reason of willful breach of the trust and
confidence reposed in him.

Proof beyond reasonable doubt is not necessary but the factual basis for the dismissal must be
clearly and convincingly established.
Further, the law mandates that before validity can be accorded to a dismissal premised on loss of
trust and confidence, two requisites must concur, viz: (1) the employee concerned must be holding
a position of trust; and (2) the loss of trust must be based on willful breach of trust founded on
clearly established facts.35

There is no arguing that the petitioner was part of the upper echelons of RBSJI’s management from
whom greater fidelity to trust is expected. At the time when he committed the act which allegedly
led to the loss of RBSJI’s trust and confidence in him, he was the Acting Manager of N. Domingo
branch. Prior thereto and ever since he was employed, he has occupied positions that entail the
power or prerogative to dictate management policies – as Personnel and Marketing Manager and
thereafter as Vice-President.

The presence of the first requisite is thus certain. Anent the second requisite, the Court finds that
the respondents failed to meet their burden of proving that the petitioner’s dismissal was for a just
cause.

The act alleged to have caused the loss of trust and confidence of the respondents in the petitioner
was his issuance, without prior authority and audit, of a clearance to Jacinto who turned out to be
still liable for unpaid cash advances and for an P11-million fraudulent transaction that exposed
RBSJI to suit. According to the respondents, the clearance barred RBSJI from running after Jacinto.
The records are, however, barren of any evidence in support of these claims.

The onus of submitting a copy of the clearance allegedly exonerating Jacinto from all his
accountabilities fell on the respondents. It was the single and absolute evidence of the petitioner’s

113
act that purportedly kindled the respondents’ loss of trust. Without it, the respondents’ allegation of
loss of trust and confidence has no leg to stand on and must thus be rejected. Moreover, one can
reasonably expect that a copy of the clearance, an essential personnel document, is with the
respondents. Their failure to present it and the lack of explanation for such failure or the
document’s unavailability props up the presumption that its contents are unfavorable to the
respondents’ assertions.

RBSJI also failed to substantiate its claim that the petitioner’s act estopped them from pursuing
Jacinto for his standing obligations. There is no proof that RBSJI attempted or at least considered to
demand from Jacinto the payment of his unpaid cash advances. Neither was RBSJI able to show that
it filed a civil or criminal suit against Jacinto to make him responsible for the alleged fraud. There is
thus no factual basis for RBSJI’s allegation that it incurred damages or was financially prejudiced by
the clearance issued by the petitioner.

More importantly, the complained act of the petitioner did not evince intentional breach of the
respondents’ trust and confidence. Neither was the petitioner grossly negligent or unjustified in
pursuing the course of action he took.

It cannot be concluded that the petitioner was in any way prompted by malicious motive in issuing
the clearance. He was also able to ensure that RBSJI’s interests are protected and that Jacinto is
pacified. He did what any person placed in a similar situation can prudently do. He was able to
competently evaluate and control Jacinto’s demands and thus prevent compromising RBSJI’s image,
employees and clients to an alarming scene.

The Court has repeatedly emphasized that the act that breached the trust must be willful such that
it was done intentionally, knowingly, and purposely, without justifiable excuse, as distinguished
from an act done carelessly, thoughtlessly, heedlessly or inadvertently. The conditions under which
the clearance was issued exclude any finding of deliberate or conscious effort on the part of the
petitioner to prejudice his employer.

Also, the petitioner did not commit an irregular or prohibited act. He did not falsify or misrepresent
any company record as it was officially confirmed by Lily that the items covered by the clearance
were truly settled by Jacinto.

The respondents cannot capitalize on the petitioner’s lack of authority to issue a clearance to
resigned employees. First, it remains but an unsubstantiated allegation despite the several
opportunities for them in the proceedings below to show, through bank documents, that the
petitioner is not among those officers so authorized. Second, it is the Court’s considered view that
by virtue of the petitioner’s stature in respondent bank, it was well-within his discretion to sign or
certify the truthfulness of facts as they appear in RBSJI’s records. Here, the records of RBSJI cashier
Lily clearly showed that Jacinto paid the cash advances and salary loan covered by the clearance
issued by the petitioner.

114
Lastly, the seven-month gap between the clearance incident and the April 17, 1997 memorandum
asking the petitioner to explain his action is too lengthy to be ignored. It likewise remains
uncontroverted that during such period, respondent Jesus verbally terminated the petitioner only
to recall the same and instead ask the latter to tender a resignation letter. When the petitioner
refused, he was sent the memorandum questioning his issuance of a clearance to Jacinto seven
months earlier. The confluence of these undisputed circumstances supports the inference that the
clearance incident was a mere afterthought used to gain ground for the petitioner’s dismissal.
Loss of trust and confidence as a ground for dismissal has never been intended to afford an
occasion for abuse because of its subjective nature. It should not be used as a subterfuge for causes
which are illegal, improper and unjustified. It must be genuine, not a mere afterthought intended to
justify an earlier action taken in bad faith.
.
WHEREFORE, the petition is GRANTED. The Decision dated February 21, 2008 and Resolution
dated June 3, 2008 of the Court of Appeals in CA-G.R. SP No. 94690 are REVERSED and SET ASIDE.
SO ORDERED.

(G.R. No. 178125, March 18, 2013).

THE ORCHARD GOLF AND COUNTRY CLUB


VS.
AMELIA R. FRANCISCO

Facts:
On February 17, 1997, respondent Amelia R. Francisco was employed by the petitioner, The
Orchard Golf and Country Club, as club accountant to head the club’s General Accounting Division
and the four divisions under it. She would report directly to the club’s financial comptroller, Jose
Ernilo P. Famy.

In a memorandum on June 29, 2000, Famy suspended Francisco without pay for 15 days for her
failure to comply with a directive and to explain her failure.
On July 20, 2000 or a day after Francisco’s period of suspension expired, Francisco was transferred
to the club’s Cost Accounting Section. On Aug. 11, 2000, Francisco filed a complaint for illegal
constructive dismissal against the club.

The petitioner claims that it did not commit any act which forced Francisco to quit; she continues to
be employed by the Club, and in fact continues to report for work.

Issue: Whether or not there was constructive dismissal.

Ruling: YES.
The Supreme Court agreed with the National Labor Relations Commission and the Court of Appeals
that Francisco’s transfer to the position of cost controller was without valid basis and that it
amounted to a demotion in rank. Hence, there was constructive dismissal.

115
Records show that when Francisco returned to work on July 20, 2000 fresh from her first
suspension, she was unceremoniously transferred by Famy, via his July 20, 2000 memorandum, to
the club’s Cost Accounting Section. x x x.

The fact that Francisco continued to report for work does not necessarily suggest that constructive
dismissal has not occurred, nor does it operate as a waiver. Constructive dismissal occurs not when
the employee ceases to report for work, but when the unwarranted acts of the employer are
committed to the end that the employee’s continued employment shall become so intolerable.
In these difficult times, an employee may be left with no choice but to continue with his
employment despite abuses committed against him by the employer, and even during the pendency
of a labor dispute between them.

This should not be taken against the employee. Instead, we must share the burden of his plight, ever
aware of the precept that necessitous men are not free men.

(G.R. No. 197353, April 1, 2013)

ALEXANDER B. BAÑARES
VS.
TABACO WOMEN’S TRANSPORT SERVICE COOPERATIVE (TAWTRASCO), ET. AL.

Facts:
Petitioner Alexander B. Bañares worked for some time as general manager of respondent Tabaco
Women’s Transport Service Cooperative (TAWTRASCO). He filed a complaint for illegal dismissal
and payment of monetary claims against TAWTRASCO. Among others, the Labor Arbiter ordered
TAWTRASCO to immediately reinstate petitioner to his former position.

In compliance with the decision, TAWTRASCO directed the petitioner to report at the company’s
Virac, Catanduanes terminal for 2 months then eventually to divide his time between the Virac
Terminal and the Araneta Center Bus Terminal. Petitioner asked for a lodging allowance, which he
used to enjoy in his previous assignment but was told to just stay at the Virac office.
He, however, found the Virac office very dilapidated and empty of an office table, chairs, filing
cabinet and other office supplies. He asked for expenses for renovation, which respondents turned
deaf ears to. Hence, he stopped reporting to work and filed a complaint for non-payment of salaries.

Issue: Whether or not there was genuine compliance with the order of reinstatement.

Ruling:
NO.

Under Article 223 of the Labor Code, an employee entitled to reinstatement “shall either be
admitted back to work under the same terms and conditions prevailing prior to his dismissal or
separation x x x.” An illegally dismissed employee is entitled to reinstatement without loss of

116
seniority rights and to other established employment privileges, and to his full back wages. The
boarding house privilege, being an established perk accorded to petitioner, ought to have been
granted him if a real and authentic reinstatement to his former position as general manager is to be
posited.

It cannot be stressed enough that TAWTRASCO withheld petitioner’s salaries for and after his
purported refusal to report for work at the Virac terminal. The reality, however, is that
TAWTRASCO directed petitioner to work under terms and conditions prejudicial to him, the most
hurtful cut being that he was required to work without a decent office, partly performing a
checker’s job. This embarrassing work arrangement is what doubtless triggered the refusal to work,
which under the premises appears justified.

Generally, employees have a demandable right over benefits voluntarily granted to them by their
employers. And if the grant or benefit is founded on an express policy or has, for a considerable
period, been given regularly and deliberately, then the grant ripens into a vested right that the
employer cannot unilaterally diminish, discontinue or eliminate. So it must be here with respect, at
the minimum, to the lodging accommodation which TAWTRASCO, as found by the National Labor
Relations Commission, appears to have regularly extended for free for some time to petitioner.

G.R. No. 193756 April 10, 2013

REYES, ET AL.
VS.
RP GUARDIANS SECURITY AGENCY, INC.

Facts
Petitioners Reyes et al. were hired by respondent RP Guardians Security Agency as security
guards and were deployed to the various branches of Banco Filipino. In September 2006, their
security contract with Banco Filipino was terminated. In separate letters, petitioners were informed
of the termination of the contract and were directed to turnover their duties and responsibilities to
the incoming security agency and were advised that they would be placed on floating status while
waiting for available posts. However, several months lapsed and they were not given new
assignments. Petitioners filed a complaint for constructive dismissal on April 10, 2007.
Respondent claimed that there was no dismissal as their termination was due to the
expiration of the service contract which was coterminous with their contract of employment.
The Labor Arbiter (LA) decided in favor of petitioners ordering respondent to pay
petitioners separation pay, backwages, refund of trust fund, moral and exemplary damages, and
attorney’s fees.
On appeal, the NLRC sustained the LA’s decision. Upon denial of its MR, respondent filed a
petition for certiorari before the Court of Appeals (CA). The CA affirmed the NLRC decision.
On motion for reconsideration (MR), the CA issued an Amended Decision. Citing Section 6.5
(4) of D.O. No. 14 of the DOLE, otherwise known as Guidelines Governing the Employment and

117
Working Conditions of Security Guards and Similar Personnel in the Private Security Industry, the
CA reduced the computation of the separation pay from one month pay per year of service to one-
half month pay for every year of service; reduced the refund of trust fund contribution from P60 to
P30; and deleted the award of backwages and attorney’s fees. Hence, the present petition.
Petitioners contend that the CA decided a question of substance contrary with law and
jurisprudence concerning their basic right to fair play, justice and due process, with more reason
that a conclusion of law cannot be made in the MR. The decision of the CA affirming the award of
both backwages and separation pay of one month pay for every year of service can only be set aside
upon proof of grave abuse of discretion, fraud or error of law. Finally, that they are entitled to
backwages for the period covered from the time the LA rendered the decision in their favor until it
was reversed by the CA in its Amended Decision.

Issues
Whether petitioners were constructively dismissed
Whether petitioners are entitled to backwages and separation pay of one month per year of service

Ruling

Petitioners were constructively dismissed when the floating status exceeded 6 months. No less than
the Constitution guarantees the right of workers to security of tenure, thus, employees can only be
dismissed for just or authorized causes and after they have been afforded the due process of law.
An employee who is unjustly dismissed from work shall be entitled to reinstatement without loss of
seniority rights and other privileges, and to his full backwages, inclusive of allowances and to his
other benefits or their monetary equivalent computed from the time his compensation was
withheld up to the time of actual reinstatement. If reinstatement is not possible, however, the
award of separation pay is proper.

Thus, an illegally dismissed employee is entitled to two reliefs: backwages and reinstatement. The
two reliefs provided are separate and distinct. In instances where reinstatement is no longer
feasible because of strained relations between the employee and the employer, separation pay is
granted. In effect, an illegally dismissed employee is entitled to either reinstatement, if viable, or
separation pay if reinstatement is no longer viable, and backwages.

Furthermore, the entitlement of the dismissed employee to separation pay of one month for every
year of service should not be confused with Section 6.5 (4) of DOLE D.O. No. 14 which grants a
separation pay of one-half month for every year of service. The said provision contemplates a
situation where a security guard is removed for authorized causes such as when the security
agency experiences a surplus of security guards brought about by lack of clients. In such a case, the
security agency has the option to resort to retrenchment upon compliance with the procedural
requirements of “two-notice rule” set forth in the Labor Code and to pay separation pay of one-half
month for every year of service.

118
G.R. No. 1895460, June 5, 2013.

CELDRAN
VS.
FORZA INTEGRATED SERVICES, ET AL.

Facts
Celdran was was hired by respondent City Service. City Service and Peerless Integrated are
affiliate companies of Forza Integrated. Celdran was paid a monthly salary of P150,000, P35,000
basic salary and P115,000 discretionary amount to be determined by the president and deposited
in his bank account on a monthly basis. Celdran averred that his compensation package included a
car benefit wherein City Service would assume the remaining balance of his car plan with his
former employer PhilamLife, with 50% participation by the company and 50% by Celdran himself.
City Service, on the other hand, avers that it was a car lease arrangement whereby an agreed
percentage of the amount paid to PhilamLife was to be set as residual value and the remaining
amount was to be spread over a certain period as monthly car rental with an option to buy the
vehicle as its residual value after then end of the lease period. With the monthly deductions in place
and the vehicle being already in the possession and use of Celdran, he was required to sign a motor
vehicle lease contract in order to formalize the arrangement. Celdran refused to sign as it was
contrary to what he allegedly agreed to.
Later, Celdran was accused of dishonesty for allegedly charging a personal lunch to the
company, and his resignation was demanded. Eight days later, Celdran received a termination
notice which led him to file a complaint for illegal dismissal. However, the case was settled as he
was reinstated to his position.
Upon his return, he was told to occupy the last open cubicle at the ground floor and not to
his former office. He was given a new copy of the motor vehicle lease contract for his signature
which he refused to sign. Later, he was relieved for no reason at all. He was subject to check and
inspection by the security guard and his transportation and cellular phone allowances were
subjected to new guidelines. Celdran was given the option to buy the vehicle at its residual value,
otherwise, the former would recover the vehicle from the latter.
Celdran filed complaint with the RAB of NLRC charging City Service with violation of the
terms of the car plan. Celdrain also filed an injunction case with a prayer for the issuance of a
temporary restraining order with the NLRC to prevent City Service from taking the car, which was
granted but refused to make it permanent in deference to the other proceedings before the NLRC.
Later, Celdran was placed under preventive suspension for 30 days due to his belligerent
attitude and required to explain why he should not be terminated. He was not asked to return to
work after his suspension for which reason he amended his complain to include charges of illegal
suspension, constructive dismissal and unpaid money claims.
Subsequently, City Service replaced the Visayas Regional Office with a small Liaison Office.
Celdran made a second amendment of his complaint to include charges of illegal lay-
off/downsizing.

The Executive Labor Arbiter’s ruling

119
The car arrangement was a long standing policy of City Service to offer its senior officials a
car lease program and not a car plan.
Celdran’s subsequent claim of illegal dismissal due to retrenchment had the effect of
conceding the allegation of constructive dismissal, since employment can only be terminated once.
There was nothing illegal in the preventive suspension because he was reinstated in the
payroll following the extension of the suspension. However, the suspension was still illegal because
it was not based on the valid ground under the Labor Code. City Service was ordered to pay him
salary corresponding to the time in which he was preventively suspended.
The downsizing was merely an exercise of its management prerogative and not to
circumvent the provisions of the Labor Code. It was done based on a report submitted to prevent
business losses and other employees similarly lost their jobs. The one month notice requirement
with the workers and DOLE was complied with.

The NLRC’s ruling


The NLRC affirmed the ELA’s judgment but gave credence to plaintiff’s claim that the he had
been extended a car plan benefit and not a car lease agreement.

The CA’s ruling


The CA affirmed the NLRC.

Issues
Whether Celdran was illegally dismissed
Whether the defendants are solidarily liable

Ruling
There was no constructive dismissal. Celdran never resigned and his complaint contained
no allegation concerning his inability to continue working for respondents due to an alleged ill
working environment. For there to be constructive dismissal, the employer must be shown to have
committed an act of clear discrimination, insensibility, or disdain, which had become so unbearable
on the part of the employee that it foreclosed any choice other than for the latter to forego
continued employment.
Celdran was not illegally dismissed when City Services implemented a downsizing program.
City Services was able to prove that their retrenchment program was justified and not implemented
in bad faith. They had shown to have experienced a downtrend since three years before the
downsizing. Furthermore, the notice requirement to the employees and the DOLE was complied
with.
The individual respondents may not be held personally liable because of the general rule
that corporate directors, trustees, or officers are not personally liable for their official acts, unless
they have exceeded the scope of their authority. There was no substantial evidence on record
proving bad faith in the termination of Celdran due to retrenchment.

120
G.R. No. 187722 June 10, 2013

SURIGAO DEL NORTE ELECTRIC COOPERATIVE, INC., AND/OR DANNY Z.


ESCALANTE, PETITIONERS, 

VS.

TEOFILO GONZAGA, RESPONDENT.

FACTS
On October 13, 1993, petitioner Surigao Del Norte Electric Cooperative, Inc. (SURNECO) hired
Gonzaga as its lineman. On February 15, 2000, he was assigned as Temporary Teller at SURNECO’s
sub-office in Gigaquit, Surigao Del Norte.7

On June 26, 2001, petitioner Danny Escalante (Escalante), General Manager of SURNECO, issued
Memorandum Order No. 34, series of 2001 (Memorandum 34-01), with attached report of
SURNECO’s Internal Auditor, Pedro Denolos (Collection Report) and two (2) sets of summaries of
collections and remittances (Summaries),8 seeking an explanation from Gonzaga regarding his
remittance shortages in the total amount of P314,252.23, covering the period from February 2000
to May 2001.9

On July 16, 2001, Gonzaga asked for an extension of three (3) weeks within which to submit his
explanation since he needed to go over the voluminous receipts of collections and remittances with
the assistance of an accountant. On the same day, he sent another letter, denying any unremitted
amount on his part and thereby, requesting that the charges against him be lifted.10
Attached to the same letter is an Audit Opinion11 prepared by one Leonides Laluna (Laluna), a
certified public accountant (CPA), stating that the Internal Auditor’s Report cannot accurately
establish any remittance shortage on Gonzaga’s part since the amount of collections stated in the
Summaries was not supported by any bills or official receipts.

In the meantime, SURNECO formed an Investigation Committee (Committee) to investigate


Gonzaga’s alleged remittance shortages. On July 30, 2001, the Committee sent Gonzaga an invitation
to attend the investigation proceedings, in which he participated.12 Pending investigation, Gonzaga
was placed under preventive suspension from July 31 to August 29, 2001.13

On August 9, 2001, the Committee tendered its report, finding Gonzaga guilty of
(a) gross and habitual neglect of duty under Section 5.2.15 of the Code of Ethics and Discipline for
Rural Electric Cooperative (REC) Employees (Code of Ethics);
(b) misappropriation of REC funds under Section 7.2.1 of the Code of Ethics; and
(c) failure to remit collections/monies under Section 7.2.2 of the Code of Ethics. Thereafter, a notice
of termination was served on Gonzaga on September 13, 2001.

Gonzaga sought reconsideration before SURNECO’s Board of Directors but the latter denied the
same after he presented his case.14 On October 25, 2001, another notice of termination (Final Notice
of Termination) was served on Gonzaga. Consequently, he was dismissed from the service on

121
November 26, 2001.15

In view of the foregoing incidents, Gonzaga filed a complaint with the NLRC Regional Arbitration
Branch No. XIII - Butuan City for illegal dismissal with payment of backwages including damages
and attorney’s fees, claiming that he was denied due process and dismissed without just cause.
LA’s Ruling: petitioners were unable to show that Gonzaga’s dismissal was just and valid and thus,
ordered that the latter be reinstated to his former position without loss of seniority rights and with
payment of full backwages, moral and exemplary damages, and attorney’s fees.24
NLRC’s: vacated the ruling of the LA, finding Gonzaga to have been dismissed for a just and valid
cause.

It observed that Gonzaga, by his admission, failed to subscribe to the company policy of remitting
cash collections daily, claiming that the distance and cost of doing so made it impractical. 31 With
respect to the imputed cash shortages, it did not give credence to Gonzaga’s position in view of his
general denial. In this light, the NLRC faulted Gonzaga for not demanding the production and
examination of the collection receipts during the investigation proceedings, noting that the said
omission meant that the collection receipts would confirm the shortage.32 Moreover, it ruled that
the procedure laid down in the Code of Ethics is not mandatory. It is sufficient that Gonzaga, with
the assistance of an accountant and a legal counsel, was given an ample opportunity to explain his
side and also participate in the investigation proceedings.33
CA: reinstated the LA’s decision

SC:

The petition is meritorious.


A. Cause of termination.
In termination cases, the burden of proof rests on the employer to show that the dismissal is for a
valid cause. Failing in which, the law considers the matter a case of illegal dismissal.45 In this
relation, the quantum of proof which the employer must discharge is substantial evidence which, as
defined in case law, means that amount of relevant evidence as a reasonable mind might accept as
adequate to support a conclusion, even if other minds, equally reasonable, might conceivably opine
otherwise.46

Applying the foregoing principles to this case, the Court finds that petitioners were able to prove, by
substantial evidence, that there lies a valid cause to terminate Gonzaga’s employment.

The Court concurs with the NLRC’s finding that petitioners’ evidence – which consists of the
Collection Report, the Summaries, and the September 15, 2003 Audit Report with attached Cash
Flow Summary – adequately supports the conclusion that Gonzaga misappropriated the funds of
the cooperative. The data indicated therein show gaping discrepancies between Gonzaga’s
collections and remittances, of which he was accountable for. In this accord, the burden of evidence
shifted to Gonzaga to prove that the reflected shortage was not attributable to him. However,
despite being allowed to peruse the bills and receipts on record together with the assistance of an

122
accountant and a counsel during the investigation proceedings, Gonzaga could not reconcile the
amounts of his collections and remittances and, instead, merely interposed bare and general
denials.

Neither does the lack of collection receipt numbers, as Gonzaga alleges, suffice to exculpate him
from the dismissal charges. This is because the said numbers had already been supplied by
petitioners through their eventual submission of the Cash Flow Summary which was attached to the
September 15, 2003 Audit Report. On this score, the Court observes that the CA should have
considered the foregoing documents as they corroborate the evidence presented by the petitioners
before the LA. Verily, labor tribunals, such as the NLRC, are not precluded from receiving evidence
submitted on appeal as technical rules are not binding in cases submitted before them. 48 In fact,
labor officials should use every and reasonable means to ascertain the facts in each case speedily
and objectively, without regard to technicalities of law or procedure, all in the interest of due
process.49

Also, it cannot be said that with the admission of the said evidence, Gonzaga would be denied due
process. Records show that he was furnished a copy of the Manifestation with the attached audit
report on September 23, 2003 and the NLRC only rendered a decision on August 31, 2004. This
interim period gave him ample time to rebut the same; however, he failed to do so.

Finally, the records are bereft of any showing that SURNECO’s internal auditor was ill-motivated
when he audited Gonzaga. Thus, there lies no reason for the Court not to afford full faith and credit
to his report.

All told, considering the totality of circumstances in this case, the Court finds the evidence
presented by the petitioners, as opposed to the bare denial of Gonzaga, sufficient to constitute
substantial evidence to prove that he committed serious misconduct and gross and habitual neglect
of duty to warrant his dismissal from employment. Such are just causes for termination which are
explicitly enumerated under Article 296 of the Labor Code.

At any rate, Gonzaga had admitted that he failed to remit his collections daily in violation of
SURNECO’s company policy, rendering such fact conclusive and binding upon him. Therefore, for
his equal violation of Section 7.2.2 of the Code of Ethics (failure to remit collections/monies), his
dismissal is justified altogether.

B. Termination procedure; statutory compliance.


Succinctly put, the foregoing procedure consists of (a) a first written notice stating the intended
grounds for termination; (b) a hearing or conference where the employee is given the opportunity
to explain his side; and (c) a second written notice informing the employee of his termination and
the grounds therefor. Records disclose that petitioners were able to prove that they sufficiently
complied with these procedural requirements:

First, petitioners have furnished Gonzaga a written first notice specifying the grounds on which his

123
termination was sought, in particular, Memorandum 34-01.

Not only was Gonzaga effectively notified of the charge of cash shortage against him, he was also
given an ample opportunity to answer the same through written explanation. Notably, attached to
Memorandum 34-01 are the Summaries which particularly detail the discrepancies in Gonzaga’s
collections vis-à-vis his remittances. As it turned out, Gonzaga submitted a letter to management on
July 16, 2001, attaching therewith an Audit Opinion prepared by Gonzaga’saccountant, Laluna, in
order to preliminarily answer the charges against him.

While the actual grounds of Gonzaga’s dismissal, i.e., gross and habitual neglect of duties and
responsibilities, misappropriation of REC funds and failure to remit collections/monies, were not
explicitly stated in Memorandum 34-01, these infractions are, however, implicit in the charge of
cash shortage. Due to the direct and logical relation between these grounds, Gonzaga could not have
been misled to proffer any mistaken defense or contrive any weakened position. Rather, precisely
because of the substantial identity of these grounds, any defense to the charge of cash shortage
equally constitutes an adequate defense to the charges of gross and habitual neglect of duties and
responsibilities, misappropriation of REC funds and failure to remit collections/monies. It stands to
reason that the core of all these infractions is similar – that is, the loss of money to which Gonzaga
was accountable – such that by reconciling the amounts purportedly missing, Gonzaga would have
been exculpated from all these charges. Therefore, based on these considerations, the Court finds
that the first notice requirement had been properly met.

Second, petitioners have conducted an informal inquiry in order to allow Gonzaga to explain his
side. To this end, SURNECO formed an investigation committee to investigate Gonzaga’s alleged
remittance shortages. After its formation, an invitation was sent to Gonzaga to attend the
investigation proceedings, in which he participated.53 Apropos to state, Gonzaga never denied his
participation during the said proceedings. Perforce, the second requirement had been equally
complied with.
Third, a second written notice was sent to Gonzaga informing him of the company’s decision to
relieve him from employment, as well as the grounds therefor.
Records indicate that the Committee tendered its report on August 9, 2001, finding Gonzaga guilty
of gross and habitual neglect of duties and responsibilities, misappropriation of REC funds and
failure to remit collections/monies. Subsequently, a notice of termination was served on Gonzaga
on September 13, 2001, stating the aforesaid grounds. Thereafter, Gonzaga tried to appeal his
dismissal before SURNECO’s Board of Directors which was, however, denied after again being given
an adequate opportunity to present his case.54 On October 25, 2001, a Final Notice of Termination
was served on Gonzaga which read as follows:

For violation of the Code of Ethics and Discipline for REC Employees, specifically Sections 5.2.15,
7.2.1 and 7.2.2 you are hereby notified of the termination of your employment with this cooperative
effective at the close of business hours on November 26, 2001.55

Based on the foregoing, it cannot be gainsaid that Gonzaga had been properly informed of the

124
company’s decision to dismiss him, as well as the grounds for the same. As such, the second notice
requirement had been finally observed.

At this juncture, it must be pointed out that while petitioners have complied with the procedure laid
down in the Omnibus Rules, they, however, failed to show that the established company policy in
investigating employees was adhered to. In this regard, SURNECO’s breach of its company
procedure necessitates the payment of nominal damages as will be discussed below.
C. Company procedure;
consequences of breach.

Jurisprudence dictates that it is not enough that the employee is given an "ample opportunity to be
heard" if company rules or practices require a formal hearing or conference. In such instance, the
requirement of a formal hearing and conference becomes mandatory. In Perez v. Philippine
Telegraph and Telephone Company,56 the Court laid down the following principles in dismissing
employees:
(a) "ample opportunity to be heard" means any meaningful opportunity (verbal or written) given to
the employee to answer the charges against him and submit evidence in support of his defense,
whether in a hearing, conference or some other fair, just and reasonable way.
(b) a formal hearing or conference becomes mandatory only when requested by the employee in
writing or substantial evidentiary disputes exists or a company rule or practice requires it, or when
similar circumstances justify it.
(c) the "ample opportunity to be heard" standard in the Labor Code prevails over the "hearing and
conference" requirement in the implementing rules and regulations. [emphases and underscoring
supplied]

The rationale behind this mandatory characterization is premised on the fact that company rules
and regulations which regulate the procedure and requirements for termination, are generally
binding on the employer. Thus, as pronounced in Suico v. NLRC, et al.:57
Company policies or practices are binding on the parties. Some can ripen into an obligation on the
part of the employer, such as those which confer benefits on employees or regulate the procedures
and requirements for their termination. [emphases supplied; citations omitted]
Records reveal that while Gonzaga was given an ample opportunity to be heard within the purview
of the foregoing principles, SURNECO, however, failed to show that it followed its own rules which
mandate that the employee who is sought to be terminated be afforded a formal hearing or
conference. As above-discussed, SURNECO remains bound by – and hence, must faithfully observe –
its company policy embodied in Section 16.5 of its own Code of Ethics which reads:

16.5. Investigation Proper. The conduct of investigation shall be open to the public.1âwphi1 If there
is no answer from the respondent, as prescribed, he shall be declared in default. Direct examination
of witnesses shall be dispensed with in the IAC. In lieu thereof, the IAC shall require the
complainant and his witnesses to submit their testimonies in affidavit form duly sworn to subject to
the right of the respondent or his counsel/s to cross-examine the complainant or his witnesses.
Cross examination shall be confined only to material and relevant matter. Prolonged argumentation
and other dilatory tactics shall not be entertained.

125
Accordingly, since only an informal inquiry58 was conducted in investigating Gonzaga’s alleged cash
shortages, SURNECO failed to comply with its own company policy, violating the proper
termination procedure altogether.

In this relation, case law states that an employer who terminates an employee for a valid cause but
does so through invalid procedure is liable to pay the latter nominal damages.
In Agabon v. NLRC (Agabon),59 the Court pronounced that where the dismissal is for a just cause,
the lack of statutory due process should not nullify the dismissal, or render it illegal, or ineffectual.
However, the employer should indemnify the employee for the violation of his statutory
rights.60 Thus, in Agabon, the employer was ordered to pay the employee nominal damages in the
amount of P30,000.00.61

By analogy, the Court finds that the same principle should apply to the case at bar for the reason
that an employer’s breach of its own company procedure is equally violative of the laborer’s rights,
albeit not statutory in source. Hence, although the dismissal stands, the Court deems it appropriate
to award Gonzaga nominal damages in the amount of P30,000.00.
WHEREFORE, the petition is GRANTED.

G.R. No. 182072 June 19, 2013

UNIVAC DEVELOPMENT, INC., PETITIONER, 



VS.

WILLIAM M. SORIANO, RESPONDENT.

FACTS:
The case stemmed from the Complaint5 for Illegal Dismissal filed by respondent against petitioner,
the company’s Chairperson Sadamu Watanabe (Watanabe), and the Head of the Engineering
Department Johnny Castro (Castro). Admittedly, respondent was hired on August 23, 2004 by
petitioner on probationary basis as legal assistant of the company with a monthly salary
of P15,000.00.6 Respondent claimed that on February 15, 2005, or eight (8) days prior to the
completion of his six months probationary period, Castro allegedly informed him that he was being
terminated from employment due to the company’s cost-cutting measures. He allegedly asked for a
thirty-day notice but his termination was ordered to be effective immediately.8 Thus, he was left
with no choice but to leave the company.9

Petitioner, on the other hand, denied the allegations of respondent and claimed instead that prior to
his employment, respondent was informed of the standards required for regularization. Petitioner
also supposedly informed him of his duties and obligations which included safekeeping of case
folders, proper coordination with the company’s lawyers, and monitoring of the status of the cases
filed by or against the company.10 Petitioner recalled that on January 5, 2005, a company meeting
was held where respondent allegedly expressed his intention to leave the company because he
wanted to review for the bar examinations. It was also in that meeting where he was informed of
his unsatisfactory performance in the company. Thus, when respondent did not report for work on

126
February 16, 2005, petitioner assumed that he pushed through with his plan to leave the
company.11 In other words, petitioner claimed that respondent was not illegally dismissed from
employment, rather, he in fact abandoned his job by his failure to report for work.

Labor Arbiter: respondent was informed of his unsatisfactory performance. As a law graduate and a
master’s degree holder, respondent was presumed to know that his probationary employment
would soon end. Considering, however, that respondent was dismissed from employment eight
days prior to the end of his probationary period, he was entitled to eight days backwages.
On appeal, the NLRC affirmed the LA decision in its entirety in its Resolution
CA via special civil action for certiorari: granted his petition

The CA gave more credence to respondent’s claim that he was illegally dismissed rather than
petitioner’s theory of abandonment. Contrary to the LA and NLRC conclusions, the appellate court
held that petitioner failed to apprise respondent of the standards required for regularization,
coupled with the fact that it failed to make an evaluation of his performance, making his dismissal
illegal. Petitioner’s employment of another person to replace respondent on the day of the alleged
abandonment was taken by the appellate court against petitioner as it negates the claim of
abandonment.

SC:
The petition is without merit.1âwphi1
It is undisputed that respondent was hired as a probationary employee.1âwphi1 As such, he did not
enjoy a permanent status. Nevertheless, he is accorded the constitutional protection of security of
tenure which means that he can only be dismissed from employment for a just cause or when he
fails to qualify as a regular employee in accordance with reasonable standards made known to him
by the employer at the time of his engagement.30

It is primordial that at the start of the probationary period, the standards for regularization be
made known to the probationary employee.31 In this case, as held by the CA, petitioner failed to
present adequate evidence to substantiate its claim that respondent was apprised of said standards.
It is evident from the LA and NLRC decisions that they merely relied on surmises and presumptions
in concluding that respondent should have known the standards considering his educational
background as a law graduate. Equally important is the requirement that in order to invoke "failure
to meet the probationary standards" as a justification for dismissal, the employer must show how
these standards have been applied to the subject employee. In this case, aside from its bare
allegation, it was not shown that a performance evaluation was conducted to prove that his
performance was indeed unsatisfactory.

Indeed, the power of the employer to terminate a probationary employee is subject to three
limitations, namely: (1) it must be exercised in accordance with the specific requirements of the
contract; (2) the dissatisfaction on the part of the employer must be real and in good faith, not
feigned so as to circumvent the contract or the law; and (3) there must be no unlawful
discrimination in the dismissal.32 In this case, not only did petitioner fail to show that respondent

127
was apprised of the standards for regularization but it was likewise not shown how these standards
had been applied in his case.

Pursuant to well-settled doctrine, petitioner’s failure to specify the reasonable standards by which
respondent’s alleged poor performance was evaluated as well as to prove that such standards were
made known to him at the start of his employment, makes respondent a regular employee. In other
words, because of this omission on the part of petitioner, respondent is deemed to have been hired
from day one as a regular employee.33

To justify the dismissal of an employee, the employer must, as a rule, prove that the dismissal was
for a just cause and that the employee was afforded due process prior to dismissal.34 We find no
reason to depart from the CA conclusion that respondent’s termination from employment is
without just and valid ground. Neither was due process observed, making his termination illegal. He
is, therefore, entitled to the twin relief of reinstatement and backwages granted under the Labor
Code.35 However, as aptly held by the CA, considering the strained relations between petitioner and
respondent, separation pay should be awarded in lieu of reinstatement. This Court has consistently
ruled that if reinstatement is no longer feasible, backwages shall be computed from the time of
illegal dismissal until the date the decision becomes final.36 Separation pay, on the other hand, is
equivalent to at least one month pay, or one month pay for every year of service, whichever is
higher (with a fraction of at least six months being considered as one whole year), 37 computed from
the time of employment or engagement up to the finality of the decision.38
WHEREFORE, premises considered, the petition is DENIED.

GR No. 201701, June 3, 2013

UNILEVER PHILS.
VS
RIVERA

FACTS:

This case involves a complaint for Illegal Dismissal and other monetary claims against Unilever filed
by Maria Ruby Rivera (Rivera). She was employed as its Area Activation Executive and was
primarily tasked with managing the sales, distribution, promotional activities and supervision of
Ventureslink International, Inc. In 2007, Unilever’s internal auditor conducted a random audit and
found out that there were fictitious billings and fabricated receipts supposedly from Ventureslink
but it was discovered that some funds were diverted from the original intended projects. Upon
further verification, Ventureslink reported that the fund deviations were upon the instruction of
Rivera.

128
Rivera was given a chance to explain the charges against her to wit: a) Conversion and
Misappropriation of Resources; b) Breach of Fiduciary Trust; c) Policy Breaches; and d) Integrity
Issues. Subsequently, Unilever found Rivera guilty of serious breach of the company’s Code of
Business Principles compelling it to sever their professional relations. Unilever denied Rivera’s
request to allow her to receive retirement benefits after having served the company for fourteen
(14) years already reasoning that the forfeiture of retirement benefits was a legal consequence of
her dismissal.

The Labor Arbiter dismissed Rivera’s complaint for lack of merit and denied her claim for
retirement benefits. NLRC partially granted Rivera's claim. Unilever was guilty of violating the twin
notice requirement in labor cases. Thus, Unilever was ordered to pay her P30,000.00 as nominal
damages, retirement benefits and separation pay.

ISSUES:

The pivotal issue in the case at bench is whether or not a validly dismissed employee, like Rivera, is
entitled to an award of separation pay.
Whether or not Unilver violated Rivera’s right to procedural due process before terminating her
employment and consequently in awarding nominal damages

RULING: PETITION IS PARTIALLY GRANTED

As a general rule, an employee who has been dismissed for any of the just causes enumerated under
Article 28215 of the Labor Code is not entitled to a separation pay.

Section 7, Rule I, Book VI of the Omnibus Rules Implementing the Labor Code provides:

Sec. 7. Termination of employment by employer. — The just causes for terminating the services of
an employee shall be those provided in Article 282 of the Code. The separation from work of an
employee for a just cause does not entitle him to the termination pay provided in the Code, without
prejudice, however, to whatever rights, benefits and privileges he may have under the applicable
individual or collective agreement with the employer or voluntary employer policy or practice.

In exceptional cases, however, the Court has granted separation pay to a legally dismissed
employee as an act of “social justice” or on “equitable grounds.” In both instances, it is required that
the dismissal (1) was not for serious misconduct; and (2) did not reflect on the moral character of
the employee.

In Reno Foods, Inc, v. Nagkakaisang Lakas ng Manggagawa (NLM)-Katipunan, the Court wrote that
“separation pay is only warranted when the cause for termination is not attributable to the
employee’s fault, such as those provided in Articles 283 and 284 of the Labor Code, as well as in
cases of illegal dismissal in which reinstatement is no longer feasible. It is not allowed when an
employee is dismissed for just cause.”

129
In this case, Rivera was dismissed from work because she intentionally circumvented a strict
company policy, manipulated another entity to carry out her instructions without the company’s
knowledge and approval, and directed the diversion of funds, which she even admitted doing
under the guise of shortening the laborious process of securing funds for promotional activities
from the head office. These transgressions were serious offenses that warranted her dismissal from
employment and proved that her termination from work was for a just cause. Hence, she is not
entitled to a separation pay

Secondly, Unilever questions the grant of nominal damages in favor of Rivera for its alleged non-
observance of the requirements of procedural due process. It insists that she was given ample
opportunity “to explain her side, interpose an intelligent defense and adduce evidence on her
behalf.

King of Kings Transport, detailed the steps on how procedural due process can be satisfactorily
complied with. Thus:
1) The first written notice to be served on the employees should contain the specific causes or
grounds for termination against them, and a directive that the employees are given the opportunity
to submit their written explanation within a reasonable period. “Reasonable opportunity “under the
Omnibus Rules means every kind of assistance that management must accord to the employees to
enable them to prepare adequately for their defense. This should be construed as a period of at
least five (5) calendar days from receipt of the notice to give the employees an opportunity to study
the accusation against them, consult a union official or lawyer, gather data and evidence, and decide
on the defenses they will raise against the complaint.

(2) After serving the first notice, the employers should schedule and conduct a hearing or
conference wherein the employees will be given the opportunity to: (1) explain and clarify their
defenses to the charge against them; (2) present evidence in support of their defenses; and (3)
rebut the evidence presented against them by the management. During the hearing or conference,
the employees are given the chance to defend themselves personally, with the assistance of a
representative or counsel of their choice.

(3) After determining that termination of employment is justified, the employers shall serve the
employees a written notice of termination indicating that: (1) all circumstances involving the
charge against the employees have been considered; and (2) grounds have been established to
justify the severance of their employment.

In this case, Unilever was not direct and specific in its first notice to Rivera. The words it used were
couched in general terms and were in no way informative of the charges against her that may result
in her dismissal from employment. Evidently, there was a violation of her right to statutory due
process warranting the payment of indemnity in the form of nominal damages.Hence, the Court
finds no compelling reason to reverse the award of nominal damages in her favor. The Court,
however, deems it proper to increase the award of nominal damages from 120,000.00 to

130
130,000.00, as initially awarded by the NLRC, in accordance with existing jurisprudence.

GR No. 162385, July 15, 2013

SAMAR-MED DISTRIBUTION
VS
NLRC

Facts:

This is a complaint for illegal dismissal money claims filed by Gutang against Samar-Med, a sole
proprietorship registered in the name of Danilo Roleda engaged in the sale and distribution of
intravenous fluids Gutang was tasked to supervise the company’s sales personnel and sales agents,
and of representing Samar-Med in transactions with the government in Region VIII. He explained
that Samar-Med had difficulty paying his compensation during his employment, resulting in his not
being paid salaries and allowances and that the company made illegual deductions. He also avers
that he had no knowledge of any infraction that had caused his dismissal
On the other hand, Roleda/Samar-Med denied liability for Gutang’s monetary claims, contending
that Gutang was not his employee but an employee of the City Council of Manila; that Gutang had
approached and asked him if he could assist in the operation of the business of Samar-Med in order
to have extra income; that Gutang was thus permitted to sell Samar-Med’s products in his own
hometown in Region VIII; that Gutang stopped selling and no longer returned to Manila after he
was tasked to conduct an investigation of the shortage in sales collections; that there was no
dismissal of Gutang, to speak of, but abandonment on his part; and that the complaint was a
harassment suit to retaliate for the criminal case he (Roleda) had meanwhile filed against Gutang
for misappropriating Samar-Med’s funds. The NLRC dismissed the complaint for illegal dismissal
but the CA reversed and set aside said decision declaring Gutang to have been illegally dismissed.

Issues;

Whether or not Gutang was validly dismissed


Whether or not Samar-Med is liable for nominal damages.
Ruling:
The onus of proving that an employee was not dismissed or, if dismissed, his dismissal was not
illegal fully rests on the employer, and the failure to discharge the onus would mean that the
dismissal was not justified
and was illegal.

In Gutang’s case, Roleda tendered no showing outside of his mere allegations to substantiate his
averment of abandonment by Gutang. Moreover, although Gutang had undoubtedly stopped
working for Samar Med, his doing so had been for a justifiable reason, consisting in the non-
payment of his salary since November 1995 and his being forced to stop working for Samar Med to

131
enable him to seek employment elsewhere, albeit temporarily in order to survive.

Thirdly, the CA concluded that Gutang’s termination from employment had been illegal. It
buttressed its conclusion on the absence of proof of abandonment of his employment by Gutang. It
observed that Gutang had not been shown to have had no desire to resume work; and that instead
Gutang had been forced to look for another gainful employment primarily because of Roleda’s non-
payment of his salary for several months, which actually proved that Gutang had no intention to
abandon his work.

The NLRC did not gravely abuse its discretion in concluding that Gutang’s dismissal had been
warranted. We note that Gutang was a managerial employee whom Roleda had vested with
confidence on delicate matters, Under Article 282(c) of the Labor Code, an employer may terminate
an employee’s employment on the ground of the latter’s fraud or wilful breach of the trust and
confidence reposed in him. For loss of trust and confidence to constitute a sufficient ground for
termination, the employer must have a reasonable ground to believe, if not to entertain the moral
conviction, that the employee was responsible for the misconduct, and that the nature of his
participation therein rendered him absolutely unworthy of the trust and confidence demanded by
his position.17 Those requirements were undeniably met in Gutang’s case.

As to the second issue, the Supreme Court finds that Gutang was not accorded due process
notwithstanding the finding of a just cause in dismissing him. Samar-Med did comply with the
obligation to send to him two written notices before finally dismissing him.

Article 277 of the Labor Code, as amended, enunciated this


requirement of two written notices, viz:

Article 277. Miscellaneous provisions. – x x x

(b) Subject to the constitutional right of workers to security of tenure and their right to be
protected against dismissal except for a just and authorized cause and without prejudice to the
requirement of notice under Article 283 of this Code, the employer shall furnish the worker whose
employment is sought to be terminated a written notice containing a statement of the causes for
termination and shall afford the latter ample opportunity to be heard and to defend himself with
the assistance of his representative if he so desires in accordance with company rules and
regulations promulgated pursuant to guidelines set by the Department of Labor and Employment.
Any decision taken by the employer shall be without prejudice to the right of the worker to contest
the validity or legality of his dismissal by filing a complaint with the regional branch of the National
Labor Relations Commission. The burden of proving that the termination was for a valid or
authorized cause shall rest on the employer. The Secretary of the Department of Labor and
Employment may suspend the effects of the termination pending resolution of the dispute in the
event of a prima facie finding by the appropriate official of the Department of Labor and
Employment before whom such dispute is pending that the termination may cause a serious

132
The requirement was also imposed in Section 219 and Section7,20 Rule I, Book VI of the
Implementing Rules of the Labor Code.

The first written notice would inform Gutang of the particular acts or omissions for which his
dismissal was being sought. The second written notice would notify him of the employer’s decision
to dismiss him. But the second written notice must not be made until after he was given a
reasonable period after receiving the first written notice within which to answer the charge, and
after he was given the ample opportunity to be heard and to defend himself with the assistance of
his representative, if he so desired. The requirement was mandatory.

Gutang’s receipt of the demand letter from Samar-Med to return the amount of P3,302,000.71 was
certainly not even a substantial compliance with the twin-notice requirement, because the purpose
of the demand letter was different from those defined for the sending of the required notices. Nor
was he thereby allowed a meaningful opportunity to be heard or to be notified of his impending
termination.

Conformably with the ruling in Agabon v. National Labor Relations Commission, the lack of
statutory due process would not nullify the dismissal or render it illegal or ineffectual when the
dismissal was for just cause. But the violation of Gutang’s right to statutory due process clearly
warranted the payment of indemnity in the form of nominal damages, whose amount is addressed
to the sound discretion of the Court taking into account the relevant circumstances. Accordingly,
the Court deems the amount of 30,000.00 as nominal damages sufficient vindication of Gutang's
right to due process under the circumstances. WHEREFORE, the Court PARTIALLY GRANTS the
petition for review; REVERSES AND SETS ASIDE

ALEX Q. NARANJO, DONNALYN DE GUZMAN, RONALD V. CRUZ, ROSEMARIE P. PIMENTEL, AND


ROWENA B. BARDAJE, PETITIONERS,
VS.
BIOMEDICA HEALTH CARE, INC. AND CARINA "KAREN" J. MOTOL, RESPONDENTS.

FACTS:
Petitioners were employees of Biomedica Health Care, Inc. On November 7, 2006, which happened
to be Motol's birthday, petitioners with two (2) other employees were all absent for various
personal reasons. Notably, these are the same employees who filed a letter-complaint dated
October 31, 2006 addressed to Director Lourdes M. Transmonte, National Director, National Capital
Region-Department of Labor and Employment (DOLE) against Biomedica for lack of salary
increases, failure to remit Social Security System and Pag-IBIG contributions, and violation of the
minimum wage law, among other grievances. Per available records, the complaint has not been

133
acted upon.
Later that day, petitioners reported for work after receiving text messages for them to proceed to
Biomedica. They were, however, refused entry and told to start looking for another workplace. The
next day, November 8, 2006, petitioners allegedly came in for work but were not allowed to enter
the premises. Motol purportedly informed petitioners, using foul language, to just find other
employment.

Correspondingly, on November 9, 2006, Biomedica issued a notice of preventive suspension and


notices to explain within 24 hours (Notices) to petitioners. In the Notices, Biomedica accused the
petitioners of having conducted an illegal strike and were accordingly directed to explain why they
should not be held guilty of and dismissed for violating the company policy against illegal strikes
under Article XI, Category Four, Sections 6, 8, 12, 18 and 25 of the Company Policy.
On November 15, 2006, petitioners were required to proceed to the Biomedica office where they
were each served their Notices. Only Angeles and Casimiro submitted their written explanation for
their absence wherein they alleged that petitioners forced them to go on a "mass leave" while
asking Biomedica for forgiveness for their actions.

On November 20, 2006, petitioners filed a Complaint with the NLRC for constructive dismissal and
nonpayment of salaries, overtime pay, 13th month pay as well as non-remittance of SSS, Pag-IBIG
and Philhealth contributions as well as loan payments. The case was docketed as Case No. 00-
09597-06.

Thereafter, Biomedica served Notices of Termination on petitioners. All dated November 29,
2006, the notices uniformly stated:

We regret to inform you that since you did not submit the written letter of explanation as requested
in your preventive suspension notice dated November 9, 2006, under Article XI, Category Four,
Sections 6, 8, 12, 18 and 25 you are hereby dismissed from service effective immediately.
On March 31, 2008, the Labor Arbiter issued a Decision and found that, indeed, petitioners engaged
in a mass leave akin to a strike. He added that, assuming that petitioners were not aware of the
company policies on illegal strikes, such mass leave can sufficiently be deemed as serious
misconduct under Art. 282 of the Labor Code. Thus, the Labor Arbiter concluded that petitioners
were validly dismissed.

Petitioners appealed the Labor Arbiter's Decision to the NLRC which rendered a modificatory
Decision dated November 21, 2008 and unlike the Labor Arbiter, the NLRC found and so declared
petitioners to have been illegally dismissed.

Thereafter, Biomedica moved but was denied reconsideration per the NLRC's Resolution dated
January 30, 2009. From the Decision and Resolution of the NLRC, Biomedica appealed the case to
the CA which reinstated the decision of the Labor Arbiter. The CA ruled that, indeed, petitioners
staged a mass leave in violation of company policy. This fact, coupled with their refusal to explain
their actions, constituted serious misconduct that would justify their dismissal.

134
Hence, the instant appeal.

The Issue
Whether or not petitioners were illegally dismissed.

The Court's Ruling


This petition is meritorious.
Petitioners were illegally dismissed
The fundamental law of the land guarantees security of tenure, thus:
Sec. 3.The State shall afford full protection to labor . . . .
. . . They shall be entitled to security of tenure, humane conditions of work and a living wage.
On the other hand, the Labor Code promotes the right of the worker to security of tenure protecting
them against illegal dismissal:
ARTICLE 279.Security of Tenure. — In cases of regular employment, the employer shall not
terminate the services of an employee except for a just cause or when authorized by this Title. An
Employee who is unjustly dismissed from work shall be entitled to reinstatement without loss of
seniority rights and other privileges and to his full backwages, inclusive of allowances, and to his
other benefits or their monetary equivalent computed from the time his compensation was
withheld from him up to the time of his actual reinstatement.
It bears pointing out that in the dismissal of an employee, the law requires that due process be
observed. Such due process requirement is two-fold, procedural and substantive, that is, "the
termination of employment must be based on a just or authorized cause of dismissal and the
dismissal must be effected after due notice and hearing." In the instant case, petitioners were
not afforded both procedural and substantive due process.
Petitioners were not afforded procedural due process
Art. 277 (b) of the Labor Code contains the procedural due process requirements in the dismissal of
an employee:

Art. 277.Miscellaneous Provisions. — . . .


(b) Subject to the constitutional right of workers to security of tenure and their right to be
protected against dismissal except for a just and authorized cause without prejudice to the
requirement of notice under Article 283 of this Code, the employer shall furnish the worker whose
employment is sought to be terminated a written notice containing a statement of the causes for
termination and shall afford the latter ample opportunity to be heard and to defend himself with
the assistance of his representative if he so desires in accordance with company rules and
regulations promulgated pursuant to guidelines set by the Department of Labor and Employment.
Any decision taken by the employer shall be without prejudice to the right of the worker to contest
the validity or legality of his dismissal by filing a complaint with the regional branch of the National
Labor Relations Commission. The burden of proving that the termination was for a valid or
authorized cause shall rest on the employer.
On the other hand, Rule XIII, Book V, Sec. 2 I (a) of the Implementing Rules and Regulations of the
Labor Code states: EAaHTI
SEC. 2.Standards of due process; requirements of notice. — In all cases of termination of employment,

135
the following standards of due process shall be substantially observed:
I.For termination of employment based on just causes as defined in Article 282 of the Code:
(a)A written notice served on the employee specifying the ground or grounds for
termination, and giving said employee reasonable opportunity within which to explain his
side.
Thus, the Court elaborated in King of Kings Transport, Inc. v. Mamac, that a mere general description
of the charges against an employee by the employer is insufficient to comply with the above
provisions of the law:
. . . Moreover, in order to enable the employees to intelligently prepare their explanation and
defenses, the notice should contain a detailed narration of the facts and circumstances that
will serve as basis for the charge against the employees. A general description of the charge
will not suffice. Lastly, the notice should specifically mention which company rules, if any, are
violated and/or which among the grounds under Art. 282 is being charged against the employees.
In the instant case, the notice specifying the grounds for termination dated November 9, 2006
states:
Effective upon receipt hereof, you are placed under preventive suspension for willfully organizing
and/or engaging in illegal strike on November 7, 2006. Your said illegal act-in conspiracy with
your other co-employees, paralyzed the company operation on that day and resulted to undue
damage and prejudice to the company and is direct violation of Article XI, Category Four
Sections 6, 8, 12, 18 & 25 of our Company Policy, which if found guilty, you will be meted a
penalty of dismissal.
Please explain in writing within 24 hours from receipt hereof why you should not be held guilty of
violating the company policy considering further that you committed and timed such act during the
birthday of our Company president.
Clearly, petitioners were charged with conducting an illegal strike, not a mass leave, without
specifying the exact acts that the company considers as constituting an illegal strike or violative of
company policies. Such allegation falls short of the requirement of "a detailed narration of the facts
and circumstances that will serve as basis for the charge against the employees." A bare mention of
an "illegal strike" will not suffice.
Further, while Biomedica cites the provisions of the company policy which petitioners purportedly
violated, it failed to quote said provisions in the notice so petitioners can be adequately informed of
the nature of the charges against them and intelligently file their explanation and defenses to said
accusations. The notice is bare of such description of the company policies. Moreover, it is
incumbent upon respondent company to show that petitioners were duly informed of said company
policies at the time of their employment and were given copies of these policies. No such proof was
presented by respondents. There was even no mention at all that such requirement was met.
Without a copy of the company policy being presented in the CA or the contents of the pertinent
policies being quoted in the pleadings, there is no way by which one can determine whether or not
there was, indeed, a violation of said company policies.
Moreover, the period of 24 hours allotted to petitioners to answer the notice was severely
insufficient and in violation of the implementing rules of the Labor Code. Under the implementing
rule of Art. 277, an employee should be given "reasonable opportunity" to file a response to the
notice. King of Kings Transport, Inc. elucidates in this wise: AHcaDC

136
To clarify, the following should be considered in terminating the services of employees:
xxxx "Reasonable opportunity" under the Omnibus Rules means every kind of assistance that
management must accord to the employees to enable them to prepare adequately for their
defense.This should be construed as a period of at least five (5) calendar days from receipt of
the notice to give the employees an opportunity to study the accusation against them,
consult a union official or lawyer, gather data and evidence, and decide on the defenses they
will raise against the complaint.
Following King of Kings Transport, Inc., the notice sent out by Biomedica in an attempt to comply
with the first notice of the due process requirements of the law was severely deficient. In addition,
Biomedica did not set the charges against petitioners for hearing or conference in accordance with
Sec. 2, Book V, Rule XIII of the Implementing Rules and Regulations of the Labor Code which states
that:
(2)After serving the first notice, the employers should schedule and conduct
a hearing or conference wherein the employees will be given the opportunity to: (1) explain and
clarify their defenses to the charge against them; (2) present evidence in support of their defenses;
and (3) rebut the evidence presented against them by the management. During the hearing or
conference, the employees are given the chance to defend themselves personally, with the
assistance of a representative or counsel of their choice. Moreover, this conference or hearing could
be used by the parties as an opportunity to come to an amicable settlement.
While petitioners did not submit any written explanation to the charges, it is incumbent for
Biomedica to set the matter for hearing or conference to hear the defenses and receive evidence of
the employees. More importantly, Biomedica is duty-bound to exert efforts, during said hearing or
conference, to hammer out a settlement of its differences with petitioners. These prescriptions
Biomedica failed to satisfy.
Lastly, Biomedica again deviated from the dictated contents of a written notice of termination as
laid down in Sec. 2, Book V, Rule XIII of the Implementing Rules that it should embody the facts and
circumstances to support the grounds justifying the termination. As amplified in King of Kings
Transport, Inc.:
(3)After determining that termination of employment is justified, the employers shall serve the
employees a written notice of termination indicating that: (1) all circumstances involving the
charge against the employees have been considered; and (2) grounds have been established to
justify the severance of their employment.
The November 26, 2006 Notice of Termination issued by Biomedica miserably failed to satisfy the
requisite contents of a valid notice of termination, as it simply mentioned the failure of petitioners
to submit their respective written explanations without discussing the facts and circumstances to
support the alleged violations of Secs. 6, 8, 12, 18 and 25 of Category Four, Art. XI of the alleged
company rules.
All told, Biomedica made mincemeat of the due process requirements under the Implementing
Rules and the King of Kings Transport, Inc. ruling by simply not following any of their dictates, to the
extreme prejudice of petitioners.
Petitioners were denied substantive due process
In any event, petitioners were also not afforded substantive due process, that is, they were illegally
dismissed.

137
The just causes for the dismissal of an employee are exclusively found in Art. 282 (a) of the Labor
Code, which states:
ARTICLE 282.Termination by employer. — An employer may terminate an employment for any of
the following causes:
Serious misconduct or willful disobedience by the employee of the lawful orders of his employer
or representative in connection with his work.
It was on this ground that the CA upheld the dismissal of petitioners from their employment.
Serious misconduct, as a justifying ground for the dismissal of an employee, has been explained
in Aliviado v. Procter & Gamble, Phils., Inc.:
Misconduct has been defined as improper or wrong conduct; the transgression of some
established and definite rule of action, a forbidden act, a dereliction of duty, unlawful in
character implying wrongful intent and not mere error of judgment. The misconduct to be
serious must be of such grave and aggravated character and not merely trivial and unimportant. To
be a just cause for dismissal, such misconduct (a) must be serious; (b) must relate to the
performance of the employee's duties; and (c) must show that the employee has become unfit to
continue working for the employer.
Clearly, to justify the dismissal of an employee on the ground of serious misconduct, the employer
must first establish that the employee is guilty of improper conduct, that the employee violated an
existing and valid company rule or regulation, or that the employee is guilty of a wrongdoing. In the
instant case, Biomedica failed to even establish that petitioners indeed violated company rules,
failing to even present a copy of the rules and to prove that petitioners were made aware of such
regulations. In fact, from the records of the case, Biomedica has failed to prove that petitioners are
guilty of a wrongdoing that is punishable with termination from employment. Art. 277 (b) of the
Labor Code states, "The burden of proving that the termination was for a valid or authorized cause
shall rest on the employer." In the instant case, Biomedica failed to overcome such burden. As will
be shown, petitioners' absence on November 7, 2006 cannot be considered a mass leave, much less
a strike and, thus, cannot justify their dismissal from employment.
Petitioners did not stage a mass leave
The accusation is for engaging in a mass leave tantamount to an illegal strike.
The term "Mass Leave" has been left undefined by the Labor Code. Plainly, the legislature intended
that the term's ordinary sense be used. "Mass" is defined as "participated in, attended by, or
affecting a large number of individuals; having a large-scale character." While the term "Leave" is
defined as "an authorized absence or vacation from duty or employment usually with pay." Thus,
the phrase "mass leave" may refer to a simultaneous availment of authorized leave benefits by a
large number of employees in a company.
It is undeniable that going on leave or absenting one's self from work for personal reasons when
they have leave benefits available is an employee's right. In Davao Integrated Port Stevedoring
Services v. Abarquez, the Court acknowledged sick leave benefits as a legitimate economic benefit of
an employee, carrying a purpose that is at once legal as it is practical:
Sick leave benefits, like other economic benefits stipulated in the CBA such as maternity leave and
vacation leave benefits, among others, are by their nature, intended to be replacements for regular
income which otherwise would not be earned because an employee is not working during the
period of said leaves. They are non-contributory in nature, in the sense that the employees

138
contribute nothing to the operation of the benefits. By their nature, upon agreement of the parties,
they are intended to alleviate the economic condition of the workers.
In addition to sick leave, the company, as a policy or practice or as agreed to in a CBA, grants
vacation leave to employees. Lastly, even the Labor Code grants a service incentive leave of 5 days
to employees. Moreover, the company or the CBA lays down the procedure in the availment of the
vacation leave, sick leave or service incentive leave.
In the factual milieu at bar, Biomedica did not submit a copy of the CBA or a company memorandum
or circular showing the authorized sick or vacation leaves which petitioners can avail of. Neither is
there any document to show the procedure by which such leaves can be enjoyed. Absent such
pertinent documentary evidence, the Court can only conclude that the availment of petitioners of
their respective leaves on November 7, 2006 was authorized, valid and in accordance with the
company or CBA rules on entitlement to and availment of such leaves. The contention of Biomedica
that the enjoyment of said leaves is in reality an illegal strike does not hold water in the absence of
strong controverting proof to overturn the presumption that "a person is innocent of . . .
wrong." Thus, the individual leaves of absence taken by the petitioners are not such absences that
can be regarded as an illegal mass action.
Moreover, a mass leave involves a large number of people or in this case, workers. Here, the five (5)
petitioners were absent on November 7, 2006. The records are bereft of any evidence to establish
how many workers are employed in Biomedica. There is no evidence on record that 5 employees
constitute a substantial number of employees of Biomedica. And, as earlier stated, it is incumbent
upon Biomedica to prove that petitioners were dismissed for just causes, this includes the duty to
prove that the leave was large-scale in character and unauthorized. This, Biomedica failed to prove.
Having failed to show that there was a mass leave, the Court concludes that there were only
individual availment of their leaves by petitioners and they cannot be held guilty of any
wrongdoing, much less anything to justify their dismissal from employment. On this ground alone,
the petition must be granted.
Petitioners did not go on strike
Granting for the sake of argument that the absence of the 5 petitioners on November 7, 2006 is
considered a mass leave, still, their actions cannot be considered a strike.
Art. 212 (o) of the Labor Code defines a strike as "any temporary stoppage of work by the
concerted action of employees as a result of any industrial or labor dispute." CHDAaS
"Concerted" is defined as "mutually contrived or planned" or "performed in unison." In the case at
bar, the 5 petitioners went on leave for various reasons. Petitioners were in different places on
November 7, 2006 to attend to their personal needs or affairs. They did not go to the company
premises to petition Biomedica for their grievance. To demonstrate their good faith in availing their
leaves, petitions reported for work and were at the company premises in the afternoon after they
received text messages asking them to do so. This shows that there was NO intent to go on strike.
Unfortunately, they were barred from entering the premises and were told to look for new jobs.
Surely the absence of petitioners in the morning of November 7, 2006 cannot in any way be
construed as a concerted action, as their absences are presumed to be for valid causes, in good faith,
and in the exercise of their right to avail themselves of CBA or company benefits. Moreover,
Biomedica did not prove that the individual absences can be considered as "temporary stoppage of
work." Biomedica's allegation that the mass leave "paralyzed the company operation on that day"

139
has remained unproved. It is erroneous, therefore, to liken the alleged mass leave to an illegal strike
much less to terminate petitioners' services for it.
Notably, the CA still ruled that petitioners went on strike as evidenced by the explanation letters of
Angeles and Casimiro sent by Biomedica. They stated in the letters that they, along with petitioners,
agreed to go on leave on the birthday on Motol to stress their demands against the company.
These statements do not deserve much weight and credit.
Dismissal is not the proper penalty
But setting aside from the nonce the facts established above, the most pivotal argument against the
dismissal of petitioners is that the penalty of dismissal from employment cannot be imposed even if
we assume that petitioners went on an illegal strike. It has not been shown that petitioners are
officers of the Union. On this issue, the NLRC correctly cited Gold City Integrated Port Service, Inc. v.
NLRC, wherein We ruled that: "An ordinary striking worker cannot be terminated for mere
participation in an illegal strike. There must be proof that he committed illegal acts during a strike."
In the instant case, Biomedica has not alleged, let alone, proved the commission by petitioners of
any illegal act during the alleged mass leave. There being none, the mere fact that petitioners
conducted an illegal strike cannot be a legal basis for their dismissal. DTISaH
Petitioners are entitled to separation pay in lieu of reinstatement, backwages and nominal
damages
Given the illegality of their dismissal, petitioners are entitled to reinstatement and backwages as
provided in Art. 279 of the Labor Code, which states:
An employee who is unjustly dismissed from work shall be entitled to reinstatement without loss of
seniority rights and other privileges and to his full backwages, inclusive of allowances, and to his
other benefits or their monetary equivalent computed from the time his compensation was
withheld from him up to the time of his actual reinstatement.
Thus, the Court ruled in Golden Ace Builders v. Talde, citing Macasero v. Southern Industrial Gases
Philippines:
Thus, an illegally dismissed employee is entitled to two reliefs: backwages and reinstatement. The
two reliefs provided are separate and distinct. In instances where reinstatement is no longer
feasible because of strained relations between the employee and the employer, separation pay is
granted. In effect, an illegally dismissed employee is entitled to either reinstatement, if viable, or
separation pay if reinstatement is no longer viable, and backwages.
The normal consequences of respondents' illegal dismissal, then, are reinstatement without
loss of seniority rights, and payment of backwages computed from the time compensation
was withheld up to the date of actual reinstatement. Where reinstatement is no longer viable
as an option, separation pay equivalent to one (1) month salary for every year of service
should be awarded as an alternative. The payment of separation pay is in addition to
payment of backwages. (Emphasis supplied.)
Petitioners were absent from work on Motol's birthday. Respondent Motol, in the course of denying
entry to them on November 8, 2006, uttered harsh, degrading and bad words. Petitioners were
terminated in swift fashion and in gross violation of their right to due process revealing that they
are no longer wanted in the company. The convergence of these facts coupled with the filing by
petitioners of their complaint with the DOLE shows a relationship governed by antipathy and
antagonism as to justify the award of separation pay in lieu of reinstatement. Thus, in addition to

140
backwages, owing to the strained relations between the parties, separation pay in lieu of
reinstatement would be proper. In Golden Ace Builders, We explained why:
Under the doctrine of strained relations, the payment of separation pay is considered an acceptable
alternative to reinstatement when the latter option is no longer desirable or viable. On one hand,
such payment liberates the employee from what could be a highly oppressive work environment.
On the other hand, it releases the employer from the grossly unpalatable obligation of maintaining
in its employ a worker it could no longer trust.
Strained relations must be demonstrated as a fact, however, to be adequately supported by
evidence — substantial evidence to show that the relationship between the employer and the
employee is indeed strained as a necessary consequence of the judicial controversy.
And in line with prevailing jurisprudence, petitioners are entitled to nominal damages in the
amount of PhP30,000 each for Biomedica's violation of procedural due process.

G.R. No. 160982. June 26, 2013

MANILA JOCKEY CLUB, INC., PETITIONER,


VS.
AIMEE O. TRAJANO, RESPONDENT.

FACTS:
MJCI had employed Trajano as a selling teller of betting tickets since November 1989. On April 25,
1998, she reported for work. At around 7:15 p.m., two regular bettors gave her their respective lists
of bets (rota) and money for the bets for Race 14. Although the bettors suddenly left her, she
entered their bets in the selling machine and segregated the tickets for pick up by the two bettors
upon their return. Before closing time, one of the bettors (requesting bettor) returned and asked
her to cancel one of his bets worth P2,000.00. Since she was also operating the negative machine on
that day, she obliged and immediately cancelled the bet as requested. She gave the remaining
tickets and the P2,000.00 to the requesting bettor, the money pertaining to the canceled bet. When
Race 14 was completed, she counted the bets received and the sold tickets. She found that the bets
and the tickets balanced. But then she saw in her drawer the receipt for the canceled ticket, but the
canceled ticket was not inside the drawer. Thinking she could have given the canceled ticket to the
requesting bettor, she immediately looked for him but could not find him. It was only then that she
remembered that there were two bettors who had earlier left their bets with her. Thus, she went to
look for the other bettor (second bettor) to ask if the canceled ticket was with him. When she
located the second bettor, she showed him the receipt of the canceled ticket to counter-check the
serial number with his tickets.

Thereafter, the second bettor returned to Trajano and told her that it was one of his bets that had
been canceled, instead of that of the requesting bettor. To complicate things, it was also the same
bet that had won Race 14. Considering that the bet was for a daily double, the second bettor only
needed to win Race 15 in order to claim dividends. At that point, she realized her mistake, and
explained to the second bettor that the cancellation of his ticket had not been intentional, but the
result of an honest mistake on her part. She offered to personally pay the dividends should the

141
second bettor win Race 15, which the latter accepted. When Race 15 was completed, the second
bettor lost. She was thus relieved of the obligation to pay any winnings to the second bettor.

To her surprise, the reliever-supervisor later approached Trajano and told her to submit a written
explanation about the ticket cancellation incident. The next day (April 26, 1998), she submitted the
handwritten explanation to Atty. Joey R. Galit, Assistant Racing Supervisor. She then resumed her
work as a selling teller, until later that day, when she received an inter-office correspondence
signed by Atty. Galit informing her that she was being placed under preventive suspension effective
April 28, 1998, for an unstated period of time. At the end of thirty days of her suspension, Trajano
reported for work. But she was no longer admitted. She then learned that she had been dismissed
when she read a copy of an inter-office correspondence about her termination posted in a selling
station of MJCI.

Trajano instituted a complaint for illegal dismissal against MJCI in the Department of Labor and
Employment (DOLE).

Decision of the Labor Arbiter


On April 23, 1999, the Labor Arbiter dismissed the complaint for illegal dismissal upon finding that
Trajano's gross negligence in the performance of her job warranted the termination of her
employment. The Labor Arbiter observed that the bet of P2,000.00 was "a huge amount that
necessarily requires extra care like [sic] its cancellation;" and that she had been given her chance to
dispute the charges made against her.

Decision of the NLRC


Aggrieved, Trajano appealed to the NLRC, arguing that she did not commit any gross dishonesty or
any serious misconduct or habitual neglect of duties, because what she committed was purely an
honest mistake that did not merit the imposition of the penalty of dismissal from the service. On
October 27, 1999, the NLRC rendered its decision reversing and setting aside the decision of the
Labor Arbiter and declaring Trajano to have been illegally dismissed by MJCI without just or
authorized cause and without due process of law.
The NLRC denied MJCI's motion for reconsideration on February 18, 2000.

Ruling of the CA
MJCI elevated the decision of the NLRC to the CA on certiorari, claiming that the NLRC thereby
gravely abused its discretion in reversing the Labor Arbiter's decision. On January 30, 2003,
however, the CA upheld the NLRC, pointing out that MJCI had not given the valid notice of
termination as required by law; that MJCI had not shown that the unauthorized cancellation of
tickets by Trajano had violated company policy; and that the cancellation of the ticket had been
only an honest mistake that did not amount to gross negligence as to warrant dismissal.
Aggrieved, MJCI filed a motion for reconsideration, but the CA denied its motion.
Hence, MJCI appealed to the Court

Issues

142
1.Whether or not there was just cause when Petitioner (MJCI) dismissed Respondent Aimee O.
Trajano from the service; and
2.Whether or not Petitioner MJCI complied with the due process requirement when it effected the
dismissal of Respondent Trajano.

Ruling of the Court


The appeal lacks merit.
MJCI posits that Trajano held a position of trust and confidence; that the act of canceling the ticket
was unauthorized because it was done without the consent of the bettor. We cannot sustain the
position of MJCI.
The valid termination of an employee may either be for just causes under Article 282 or for
authorized causes under Article 283 and Article 284, all of the Labor Code.
Specifically, loss of the employer's trust and confidence is a just cause under Article 282 (c), a
provision that ideally applies only to cases involving an employee occupying a position of trust and
confidence, or to a situation where the employee has been routinely charged with the care and
custody of the employer's money or property. But the loss of trust and confidence, to be a valid
ground for dismissal, must be based on a willful breach of trust and confidence founded on clearly
established facts.

"A breach is willful," according to AMA Computer College, Inc. v. Garay, "if it is done intentionally,
knowingly and purposely, without justifiable excuse, as distinguished from an act done carelessly,
thoughtlessly, heedlessly or inadvertently. It must rest on substantial grounds and not on the
employer's arbitrariness, whims, caprices or suspicion; otherwise, the employee would eternally
remain at the mercy of the employer." An ordinary breach is not enough.
Moreover, the loss of trust and confidence must be related to the employee's performance of duties.
As held in Gonzales v. National Labor Relations Commission:
Loss of confidence, as a just cause for termination of employment, is premised on the fact that the
employee concerned holds a position of responsibility, trust and confidence. He must be invested
with confidence on delicate matters such as the custody, handling, care and protection of the
employer's property and/or funds. But in order to constitute a just cause for dismissal, the act
complained of must be "work-related" such as would show the employee concerned to be unfit to
continue working for the employer.

As a selling teller, Trajano held a position of trust and confidence. The nature of her employment
required her to handle and keep in custody the tickets issued and the bets made in her assigned
selling station. The bets were funds belonging to her employer. Although the act complained of —
the unauthorized cancellation of the ticket (i.e., unauthorized because it was done without the
consent of the bettor) — was related to her work as a selling teller, MJCI did not establish that the
cancellation of the ticket was intentional, knowing and purposeful on her part in order for her to
have breached the trust and confidence reposed in her by MJCI, instead of being only out of an
honest mistake.
Still, to justify the supposed loss of its trust and confidence in Trajano, MJCI contends that the
unauthorized cancellation of the ticket could have greatly prejudiced MJCI for causing damage to

143
both its income and reputation. CASTDI

We consider the contention of MJCI unwarranted. As the records indicate, MJCI's prejudice
remained speculative and unrealized. To dismiss an employee based on speculation as to the
damage the employer could have suffered would be an injustice. The injustice in the case of Trajano
would be greater if the supposed just cause for her dismissal was not even sufficiently established.
While MJCI as the employer understandably had its own interests to protect, and could validly
terminate any employee for a just cause, its exercise of the power to dismiss should always be
tempered with compassion and imbued with understanding, avoiding its abuse.
In this regard, we have to stress that the loss of trust and confidence as a ground for the dismissal of
an employee must also be shown to be genuine, for, as the Court has aptly pointed out in Mabeza v.
National Labor Relations Commission: ". . . loss of confidence should not be simulated in order to
justify what would otherwise be, under the provisions of law, an illegal dismissal. It should not be
used as a subterfuge for causes which are illegal, improper and unjustified. It must be genuine, not a
mere afterthought to justify an earlier action taken in bad faith."

Anent compliance with due process, MJCI argues that Trajano's notification of her termination
through the posting in the selling stations should be deemed a substantial if not full compliance
with the due process requirement, considering that she herself even presented a copy of the posting
as evidence; that the rule on giving notice of termination to an employee did not expressly require
the personal service of the notice to the dismissed worker; and that what mattered was that she
was notified in writing of MJCI's decision to terminate her through the posting in its selling
stations.

The argument is bereft of worth and substance. DTEcSa


The procedure to be followed in the termination of employment based on just causes is laid down
in Section 2 (d), Rule I of the Implementing Rules of Book VI of the Labor Code, to wit:
Section 2.Security of Tenure. —
xxx xxx xxx
(d)In all cases of termination of employment, the following standards of due process shall be
substantially observed:
For termination of employment based on just causes as defined in Article 282 of the Labor Code:
(i)A written notice served on the employee specifying the ground or grounds for termination, and
giving said employee reasonable opportunity within which to explain his side.
(ii)A hearing or conference during which the employee concerned, with the assistance of counsel if
he so desires is given opportunity to respond to the charge, present his evidence, or rebut the
evidence presented against him.
(iii)A written notice of termination served on the employee, indicating that upon due consideration
of all the circumstances, grounds have been established to justify his termination. In case of
termination, the foregoing notices shall be served on the employee's last known address.
A review of the records warrants a finding that MJCI did not comply with the prescribed procedure.
In its October 27, 1999 decision, the NLRC declared that MJCI complied with the first notice
requirement by serving a copy of the first notice upon Trajano, who received the copy and affixed

144
her signature thereon on April 26, 1998. Such declaration seems to be supported by the
records. CIETDc
Yet, the NLRC concluded that the clarificatory meeting was not the hearing contemplated by law
because the supposed complainants were not there for Trajano to confront.
We disagree with the NLRC's conclusion, and instead find that there was a compliance with the
second requirement for a hearing or conference. It is undeniable that Trajano was accorded the real
opportunity to respond to the complaint against her, for she did submit her written explanation on
April 26, 1998 and was invited to the final clarificatory meeting set on June 5, 1998 in the presence
of the MJCI Raceday Union President.
Nor was it necessary at all for Trajano to be able to confront the complainant against her. In Muaje-
Tuazon v. Wenphil Corporation, the Court has clarified that the opportunity to confront a witness is
not demanded in company investigations of the administrative sins of an employee, holding thusly:
xxx xxx xxx
Petitioners must be reminded, however, that confrontation of witnesses is required only in
adversarial criminal prosecutions, and not in company investigations for the administrative liability
of the employee. Additionally, actual adversarial proceedings become necessary only for
clarification, or when there is a need to propound searching questions to witnesses who give vague
testimonies. This is not an inherent right, and in company investigations, summary proceedings
may be conducted.
As for the last procedural requirement of giving the second notice, the posting of the notice of
termination at MJCI's selling stations did not satisfy it, and the fact that Trajano was eventually
notified of her dismissal did not cure the infirmity. It is notable, indeed, that the NLRC explicitly
found in its October 27, 1999 decision that MJCI did not comply, to wit:
In this case, there is the first written notice required but none of the second notice that informs her
of the employer's or MJCI's decision to dismiss her. In fact, it was not even shown that the
investigator, Atty. Joey Galit, whose office is that of an assistant racing manager, has the company's
authority to dismiss the complainant, since that power is usually lodged with the head of the human
resource department or with the President, but unusual with an assistant manager. The
complainant asserts that she was never furnished a copy of her termination letter and what she had
submitted as evidence on record (Annex "A" for the complainant, Record, p. 25) was one of those
copies posted on all selling stations of MJCI. This accusation was not answered by the respondents
nor have they ever proved that they had furnished the complainant a written notice of the decision
of MJCI to terminate her services on the ground of serious violation of company policy
(dishonesty).
We uphold this finding of the NLRC, for the law on the matter has been clear. While personal service
of the notice of termination on the employee is not required, Section 2 (d), Rule I of
the Implementing Rules of Book VI of the Labor Codemandates that such notice be served on Trajano
at her last known address, viz.: aScIAC
xxx xxx xxx
(iii)A written notice of termination served on the employee, indicating that upon due consideration
of all the circumstances, grounds have been established to justify his termination. In case of
termination, the foregoing notices shall be served on the employee's last known
address. (Emphasis supplied)

145
xxx xxx xxx
It is worth repeating that in termination cases, the employer carries the burden of proving that its
dismissal of the employee was legal. The employer's failure discharged its burden will readily mean
that the dismissal has not been justified, and was, therefore, illegal. Accordingly, the failure of MJCI
to establish the just cause for terminating Trajano fully warranted the NLRC's finding that Trajano's
termination was illegal.
Considering the lapse of time between the rendition of the decision of the NLRC and this ultimate
resolution of the case, however, the Court holds that a review of the order of reinstatement and the
award of backwages is necessary and in order.
There is no question that an illegally dismissed employee is entitled to her reinstatement without
loss of seniority rights and other privileges, and to full backwages, inclusive of allowances and other
benefits or their monetary equivalent.
In case the reinstatement is no longer possible, however, an award of separation pay, in lieu of
reinstatement, will be justified. The Court has ruled that reinstatement is no longer
possible: (a) when the former position of the illegally dismissed employee no longer
exists; or (b) when the employer's business has closed down; or (c) when the employer-employee
relationship has already been strained as to render the reinstatement impossible. The Court
likewise considered reinstatement to be non-feasible because a "considerable time" has lapsed
between the dismissal and the resolution of the case. In that regard, a lag of eight years or ten years
is sufficient to justify an award of separation pay in lieu of reinstatement.
Applying the foregoing to this case, the Court concludes that the reinstatement of Trajano is no
longer feasible. More than 14 years have already passed since she initiated her complaint for illegal
dismissal in 1998, filing her position paper on September 3, 1998, before the Court could finally
resolve her case. The lapse of that long time has rendered her reinstatement an impractical, if not
an impossible, option for both her and MJCI. Consequently, an award of separation pay has become
the practical alternative, computed at one month pay for every year of service.
Anent backwages, Trajano is entitled to full backwages, inclusive of allowances and other benefits
or their monetary equivalent, computed from the time her actual compensation was withheld on
June 6, 1998 up to the finality of this decision (on account of her reinstatement having meanwhile
become non-feasible and impractical). This ruling is consistent with the legislative intent behind
Republic Act No. 6715.
WHEREFORE, the Court AFFIRMS the decision promulgated on January 30, 2003, subject to
the MODIFICATIONS that: (a) separation pay computed at one month pay for every year of service
be awarded in lieu of reinstatement, and (b)backwages, inclusive of allowances and other benefits
or their monetary equivalent, computed from June 6, 1998, the date of respondent's termination,
until the finality of this decision be paid to respondent.

[G.R. No. 163061. June 26, 2013.]

146
ALFONSO L. FIANZA, PETITIONER,
VS.
NATIONAL LABOR RELATIONS COMMISSION (SECOND DIVISION), BINGA HYDROELECTRIC
PLANT, INC., ANTHONY C. ESCOLAR, ROLAND M. LAUTCHANG, RESPONDENTS.

Facts:
Petitioner Fianza was hired by the private respondent Binga Hydroelectric Plant as a Social
Acceptance Officer. He was employed at the start as probationary employee for two months.
However, on the first month of work, he was not able to receive his pay for the first-half. He was
then instructed not to work in order to verify his status of work. When several inquiries were made
as to his status, he was told to report back. But his inclusion to the payroll was still under question.
Several months passed, there were no actions taken for his inclusion in the payroll. This prompted
him to file a complaint for illegal dismissal.
In the memoranda, the private respondent heavily argued on abandonment of work as the
cause for the termination of the petitioner. This line of reasoning became the basis for the NLRC to
conclude that the dismissal was valid.
Issue:
WON abandonment justified the dismissal of the petitioner
Ruling:
No. It is well-settled in our jurisprudence that "For abandonment to constitute a valid cause
for termination of employment, there must be a deliberate, unjustified refusal of the employee to
resume his employment. This refusal must be clearly shown. Mere absence is not sufficient, it must
be accompanied by overt acts unerringly pointing to the fact that the employee does not want to
work anymore" (Emphasis and italics supplied) 32
Abandonment as a fact and a defense can only be claimed as a ground for dismissal if the employer
follows the procedure set by law. 33 In line with the burden of proof set by law, the employer who
alleges abandonment "has the burden of proof to show a deliberate and unjustified refusal of the
employee to resume his employment without any intention of returning." 34 As this Court has
stated in Agabon v. National Labor Relations Commission: ECaTAI
For a valid finding of abandonment, these two factors should be present: (1) the failure to report for
work or absence without valid or justifiable reason; and (2) a clear intention to sever employer-
employee relationship, with the second as the more determinative factor which is manifested by
overt acts from which it may be deduced that the employees has no more intention to work. The
intent to discontinue the employment must be shown by clear proof that it was deliberate and
unjustified. 35
From the foregoing, it is clear that respondent company failed to prove the necessary elements of
abandonment. Additionally, the NLRC and the CA failed to take into account the strict requirements
set by jurisprudence when they determined the existence of abandonment on the basis of mere
allegations that were contradicted by the evidence shown.
The very act of filing the Complaint for illegal dismissal should have negated any intention on
petitioner's part to sever his employment. 36 In fact, it should already have been sufficient evidence
to declare that there was no abandonment of work. Moreover, petitioner went back to the company
several times to inquire about the status of his employment. 37The fact that his inquiries were not
answered does not prejudice this position.

147
Throughout the entire ordeal, petitioner was vigilant in protecting himself from any claim that he
had abandoned his work. The following circumstances evinced his intent to return to work:
1.His continuous inquiry with respondent about the status of his work. 38
2.His willingness to return to work at any time, subject to the approval of respondent, and his visits
to the plant to apply for work. 39
3.His filing of an illegal dismissal case. 40

[G.R. No. 192394. July 3, 2013.]

ROY D. PASOS, PETITIONER,


VS.
PHILIPPINE NATIONAL CONSTRUCTION CORPORATION,RESPONDENT.

Facts:
Petitioner was hired for the first time as a project employee by the respondent PNCC from
April 26 to July 25, 1996. Subsequently he was re-hired for another project with the same
construction company for three consecutive projects until October 19, 2000. Despite his
termination for the last project, his superior instructed him to report the following day. For
purposes of reemployment, he underwent medical examination for which it found that he had
suffered from Koch’s disease. For this reason, he was required to enjoy a 60-day paid sick leave.
When he had been issued a certificate of medical clearance that he was fit to work, the respondent
denied his reemployment on the ground that he was only a project employee for which his
employment status depends upon the completion of the project. Because of this, he filed a
complaint for illegal dismissal.
Issues:
1. whether or not petitioner is a project employee
2. whether or not his dismissal is valid

Ruling:
1. No.
In the instant case, the appointments issued to petitioner indicated that he was hired for
specific projects. This Court is convinced however that although he started as a project employee,
he eventually became a regular employee of PNCC.
Under Article 280 of the Labor Code, as amended, a project employee is one whose "employment
has been fixed for a specific project or undertaking the completion or termination of which has
been determined at the time of the engagement of the employee or where the work or services to
be performed is seasonal in nature and the employment is for the duration of the season." Thus, the
principal test used to determine whether employees are project employees is whether or not the
employees were assigned to carry out a specific project or undertaking, the duration or scope of
which was specified at the time the employees were engaged for that project. 33
In the case at bar, petitioner worked continuously for more than two years after the supposed
three-month duration of his project employment for the NAIA II Project. While his appointment for
said project allowed such extension since it specifically provided that in case his "services are still

148
needed beyond the validity of [the] contract, the Company shall extend [his] services," there was no
subsequent contract or appointment that specified a particular duration for the extension. His
services were just extended indefinitely until "Personnel Action Form — Project Employment"
dated July 7, 1998 was issued to him which provided that his employment will end a few weeks
later or on August 4, 1998. While for first three months, petitioner can be considered a project
employee of PNCC, his employment thereafter, when his services were extended without any
specification of as to the duration, made him a regular employee of PNCC. And his status as a
regular employee was not affected by the fact that he was assigned to several other projects and
there were intervals in between said projects since he enjoys security of tenure. CTHDcS
Failure of an employer to file
termination reports after every
project completion proves that
an
employee is not a project employee
In this case, records clearly show that PNCC did not report the termination of petitioner's supposed
project employment for the NAIA II Project to the DOLE. Department Order No. 19, or the
"Guidelines Governing the Employment of Workers in the Construction Industry," requires
employers to submit a report of an employee's termination to the nearest public employment office
every time an employee's employment is terminated due to a completion of a project. PNCC
submitted as evidence of its compliance with the requirement supposed photocopies of its
termination reports, each listing petitioner as among the employees affected. Unfortunately, none
of the reports submitted pertain to the NAIA II Project. Moreover, DOLE NCR verified that petitioner
is not included in the list of affected workers based on the termination reports filed by PNCC on
August 11, 17, 20 and 24, 1998 for petitioner's supposed dismissal from the NAIA II Project
effective August 4, 1998. This certification from DOLE was not refuted by PNCC. In Tomas Lao
Construction v. NLRC, 36 we emphasized the indispensability of the reportorial requirement:
Moreover, if private respondents were indeed employed as "project employees," petitioners should
have submitted a report of termination to the nearest public employment office every time their
employment was terminated due to completion of each construction project. The records show that
they did not. Policy Instruction No. 20 is explicit that employers of project employees are exempted
from the clearance requirement but not from the submission of termination report. We have
consistently held that failure of the employer to file termination reports after every project
completion proves that the employees are not project employees. Nowhere in the New Labor Code
is it provided that the reportorial requirement is dispensed with. The fact is that Department Order
No. 19 superseding Policy Instruction No. 20 expressly provides that the report of termination is
one of the indicators of project employment. 37 TAaCED
2. yes.
Petitioner's regular employment was terminated by PNCC due to contract expiration or project
completion, which are both not among the just or authorized causes provided in the Labor Code, as
amended, for dismissing a regular employee. Thus, petitioner was illegally dismissed.
Article 279 of the Labor Code, as amended, provides that an illegally dismissed employee is entitled
to reinstatement, full back wages, inclusive of allowances, and to his other benefits or their
monetary equivalent from the time his compensation was withheld from him up to the time of his
actual reinstatement.
We agree with petitioner that there was no basis for the Labor Arbiter's finding of strained
relations and order of separation pay in lieu of reinstatement. This was neither alleged nor proved.

149
Moreover, it has long been settled that the doctrine of strained relations should be strictly applied
so as not to deprive an illegally dismissed employee of his right to reinstatement. As held in Globe
Mackay Cable and Radio Corporation v. NLRC: 38
Obviously, the principle of "strained relations" cannot be applied indiscriminately. Otherwise,
reinstatement can never be possible simply because some hostility is invariably engendered
between the parties as a result of litigation. That is human nature.
Besides, no strained relations should arise from a valid and legal act of asserting one's right;
otherwise an employee who shall assert his right could be easily separated from the service, by
merely paying his separation pay on the pretext that his relationship with his employer had already
become strained. 39 IcDESA
As to the back wages due petitioner, there is likewise no basis in deducting therefrom back wages
equivalent to six months "representing the maximum period of confinement [PNCC] can require
him to undergo medical treatment." Besides, petitioner was not dismissed on the ground of disease
but expiration of term of project employment.
Regarding moral and exemplary damages, this Court rules that petitioner is not entitled to them.
Worth reiterating is the rule that moral damages are recoverable where the dismissal of the
employee was attended by bad faith or fraud or constituted an act oppressive to labor, or was done
in a manner contrary to morals, good customs, or public policy. Likewise, exemplary damages may
be awarded if the dismissal was effected in a wanton, oppressive or malevolent manner. 40 Apart
from his allegations, petitioner did not present any evidence to prove that his dismissal was
attended with bad faith or was done oppressively

G.R. No. 189686 July 10, 2013

UNIVERSAL ROBINA CORPORATION AND LANCE Y. GOKONGWEI, PETITIONERS, 



VS.

WILFREDO Z. CASTILLO, RESPONDENT.

Facts:
Respondent Wilfredo Z. Castillo (Castillo) was hired by petitioner Universal Robina Corporation
(URC) as a truck salesman on 23 March 1983 with a monthly salary of P4,000.00. He rose from the
ranks and became a Regional Sales Manager, until his dismissal on 12 January 2006.
As Regional Sales Manager, respondent was responsible for planning, monitoring, leading and
controlling all activities affecting smooth sales operation. He is particularly in charge of the
operational and administrative functions encompassing the formulation of sales forecast, selling
expense, budget preparation and control, sales analysis, formulation and review of policies and
procedures affecting the sales force and service provided to customers, including representation in
keeping and maintaining key accounts of the company. He is likewise tasked to transact, sign and
represent the company in all its dealings with key accounts or customers subject however to his
selling expense budget duly approved by URC Management. Consequently, he is obliged to give an
account of all his dealings or transactions with all his customers to URC. 3 His area of responsibility
covered some parts of Laguna, including Liana’s Supermart (Liana) in San Pablo City, Laguna.
On 19 August 2005, URC’s Credit and Collection Department (CCD) Analyst in Silangan, Laguna
Branch noted an outright deduction in the amount of P72,000.00 tagged as Gift Certificate (GC). The

150
CCD Analyst found the issuance of GCs as unusual. This finding prompted URC’s Corporate Internal
Audit (CIA) to conduct a routine audit of the unresolved accounts of Liana’s account receivables.
Based on its investigation, CIA came up with the following findings:
1. Per Ms. Prezy Manansala, Liana’s San Pablo Branch Manager, URC agreed to sponsor their "Back
to School Promo".xxxxxxxxxxxx 3. Liana’s issued GCs worth P72,000.00 to RSM Castillo. As claimed
by Ms. Manansala, this issuance of GC is part of the promo activity.x x x x 4. Ms. Manansala informed
us that the "Back to School Raffle Promo" was cancelled. x x x.xx 5. We showed her photocopies of
Charged Invoices Nos. 2189 and 2190 x x x. Ms. Manansala confirmed that RSM Castillo is the one
who signed on the received x x x portion of the documents we showed.
The CIA suspected that respondent might have committed an act of fraud against the company and
Liana’s for his personal gain.
On 14 November 2005, respondent was asked to explain in writing why the company should not
institute the appropriate disciplinary action against him for possible violation of Offenses Subject to
Disciplinary Action 2.04, to wit:
Directly or indirectly obtraining or accepting money or anything of value by entering into
unauthorized arrangement/s with supplier/s, client or other outsider/s.6
Respondent denied accepting any gift certificate. He repeatedly denied that he signed two (2) blank
Charge invoices intended for GCs. He also admitted that only two (2) cut-cases should have been
charged and he assumed liability for the undue payment of one (1) cut-case display.
On 9 January 2006, respondent was served a written notice of termination in the following tenor:
With deep regret, we hereby inform you that, after DUE PROCESS, you were found guilty of acts
inimical to the interest of the Company and for breach of trust & confidence.
On 30 May 2006, respondent filed a complaint for illegal dismissal against petitioners URC and its
President and Chief Operating Officer (COO) Lance Gokongwei. He alleged that the grounds for
which he was dismissed were totally different from the charges leveled against him during the
investigation.11
On 12 June 2007, the labor arbiter rendered a decision declaring respondent to have been illegally
dismissed and ordered the payment of backwages and separation pay.
The Labor Arbiter ruled that respondent was asked to explain on charges which are different from
the charges for which he was dismissed. The Labor Arbiter also held that URC failed to substantiate
the charges against respondent.
On appeal, the National Labor Relations Commission (NLRC) found the appeal meritorious and
reversed the decision of the labor arbiter. According to the NLRC, URC had more than sufficient
proof that respondent violated its trust. Respondent sought reconsideration of the reversal, but his
motion for reconsideration was denied.
URC raises the lone argument that respondent is not entitled to separation pay in accordance with
prevailing law and jurisprudence.14 Citing case law, URC contends that if an employee’s act or
violation of the company’s code constitutes serious misconduct or is reflective of lack of moral
character, then the employer is not required to give the dismissed employee financial assistance or
separation. URC maintains that respondent’s acts of signing blank Charge Invoices without any
authority and receiving P72,000.00 worth of GCs for his personal benefit clearly constitute serious
misconduct which preclude an award for separation pay.
In his Comment, respondent stresses that based on the tenor of the termination letter, he was never

151
dismissed on the ground of gross misconduct. Respondent concedes that at most, he may have
committed simple negligence. He reiterates that he did not commit any act constituting serious
misconduct nor does it reflect any deterioration in his moral character.
Issue:
Whether a validly dismissed employee is entitled to separation pay.
Held:
No.
The leading case of Philippine Long Distance Telephone Co. v. NLRC15 enunciated the ruling that
separation pay "as a measure of social justice" is allowed in those instances where the employee is
validly dismissed for causes other than serious misconduct or those reflecting on his moral
character.16 The case of Toyota Motor Phils. Corp. Workers Association (TMPCWA) v.
NLRC17 expanded the doctrine laid down in PLDT by adding dismissals other than those under Art.
282 of the Labor Code, like willful disobedience, gross and habitual neglect of duty, fraud or willful
breach of trust, and commission of a crime against the employer or his family which would preclude
award of separation pay.
As the rule now stands, the award of separation pay is authorized in the situations dealt with in
Article 283 and 284 of the Labor Code, but not in terminations of employment based on instances
enumerated in Article 282.18 Article 282 states that:
ART. 282. Termination by employer. – An employer may terminate an employment for any of the
following causes:
(a) Serious misconduct or willful disobedience by the employee of the lawful orders of his employer
or representative in connection with his work;
(b) Gross and habitual neglect by the employee of his duties;
(c) Fraud or willful breach by the employee of the trust reposed in him by his employer or duly
authorized representative;
(d) Commission of a crime or offense by the employee against the person of his employer or any
immediate member of his family or his duly authorized representatives; and
(e) Other causes analogous to the foregoing.
Central Philippines Bandag Retreaders, Inc. cautioned labor tribunals in indiscriminately awarding
separation pay as a measure of social justice, in this wise:
x x x Labor adjudicatory officials and the CA must demur the award of separation pay based on
social justice when an employee’s dismissal is based on serious misconduct or willful disobedience;
gross and habitual neglect of duty; fraud or willful breach of trust; or commission of a crime against
the person of the employer or his immediate family—grounds under Art. 282 of the Labor Code
that sanction dismissals of employees. They must be most judicious and circumspect in awarding
separation pay or financial assistance as the constitutional policy to provide full protection to labor
is not meant to be an instrument to oppress the employers. The commitment of the Court to the
cause of labor should not embarrass us from sustaining the employers when they are right, as here.
In fine, we should be more cautious in awarding financial assistance to the undeserving and those
who are unworthy of the liberality of the law.19
Indeed, respondent has committed acts constituting willful breach of trust and confidence reposed
on him by URC based on the following facts established by the Court of Appeals, thus:
x x x The principal charge against petitioner Castillo was hinged upon "unauthorized arrangements"

152
which he allegedly entered into. Petitioner Castillo’s unauthorized dealing with respect to the
changes in the Account Development Agreement is exactly the offending cause of the host of
infractions he committed, i.e., his neglect in signing the blank charge invoices and his improper
receipt of gift certificates for his personal gain. These acts taken together constitute a breach of the
trust and confidence reposed on petitioner Castillo by private respondent URC. x x x.
Indeed, petitioner Castillo’s acts of receiving the gift certificates and signing the blank invoices are
closely intertwined and inextricably connected with each other. In other words, petitioner Castillo’s
acquisition of the gift certificates could not have been facilitated without him signing the blank
invoices. Such signing was a ruse to cover up his receipt of the gift certificates. Oddly enough,
petitioner Castillo readily admitted to signing receipt on Charge Invoices Nos. 2189 and 2190
covering the gift certificates in the amounts of P60,000.00 and P12,000.00, respectively, but made
the qualification that the same were in blank when he signed on them. Such claim was obviously to
create the impression that he was really not aware of any gift certificates and that whatever
misstep he committed was merely brought about by his good faith.
Nonetheless, the evidence on record negates petitioner Castillo’s claim of good faith and furnishes
sufficient basis for the breach of trust and loss of confidence reposed on him by private respondent
URC. Petitioner Castillo’s receipt of the gift certificates is categorically confirmed by Peter Sy, the
Vice President of Marketing of Liana’s Supermarket. This piece of evidence, coming from a
disinterested party, speaks eloquently of petitioner Castillo’s perfidy. Such an affirmative statement
coupled with petitioner Castillo's signatures on the charge invoices convincingly established the
fact that he indeed received the P72,000.00 worth of gift certificates.
Assuming that he did not receive the gift certificates, petitioner Castillo’s ready admission that he
signed the charge invoices even if these were blank clearly shows his negligence and utter lack or
care in the interests of private respondent URC. As a Regional Sales Manager, petitioner Castillo
occupied a position or responsibility and as such, he should have known that he placed the interests
of the company at a disadvantage by signing the blank charge invoices. Because of such act, private
respondent URC was prejudiced by no less than P72,000.00. This alone is su1licicnt cause for
breach of trust and loss of confidence.20
In Bank of the Philippine Islands v. NLRC and Arambulo,21 we ruled that an employee who has been
dismissed for a just cause under Article 282 of the Labor Code is not entitled to separation pay. The
complainant therein was likewise dismissed on the ground of loss of trust and confidence. Applying
that rule to the instant case, we here hold that respondent is not entitled to separation pay.

G.R. No. 192306 July 15, 2013

JESSIE G. MARTINEZ, PETITIONER, 



VS.

CENTRAL PANGASINAN ELECTRIC COOPERATIVE, INC. (CENPELCO), RESPONDENT.

Facts:
In 1991, CENPELCO employed Martinez on a contractual basis and in 1993, was subsequently
regularized as a billing clerk at the former's main office in San Carlos City, Pangasinan. On January
7, 2002, CENPELCO gave Martinez the position of teller at Area VI in Malasiqui, Pangasinan.6

153
On April 26, 2002, CENPELCO’s Internal Audit Department (IAD) conducted a cash count audit at its
Area VI.7Josefina Mandapat (Mandapat), the IAD Officer-in-Charge, analyzed the audit results and
concluded that there was an error in the count of Benjamin Madriaga (Madriaga), cashier for Area
VI, regarding the breakdown of collection turned over by Martinez for April 23, 2002.8 Specifically,
Madriaga erroneously recorded that Martinez remitted 390 pieces of P500-bills, instead of the
correct number which was just 290, and issued a handwritten temporary receipt for P406,130.31
instead of P360,447.13. Upon noting that Madriaga issued Official Receipts Nos. 77365-77367 for
the amount of P360,447.139 with corresponding remittance stubs for Martinez’s April 23, 2002
collections, Mandapat concluded that Martinez’s overage for the same day in the amount
of P45,682.58 is questionable.10 Further, Mandapat noted that on April 25, 2002, Martinez
committed a shortage in the amount ofP44,846.77,11 considering that the latter’s total
accountability for the said date is in the amount of P212,258.56 but his actual cash count12 only
amounted to P167,411.79.
In view of such audit, Mandapat recommended that Madriaga and Martinez be made to explain why
no disciplinary action should be taken against them. Thus, on May 15, 2002, Martinez filed his
letter-explanation, explaining that he submitted his collections and remittance stubs to Madriaga
who was the one tasked to make the report thereon and who may have mishandled the proper
listing and tallying of the money collected vis-à-vis the collection stubs.13 He further admitted the
existence of such shortage and tried to offset the same with his alleged overage on April 23, 2002.14
On June 30 2002, the Company’s Grievance Committee, which was commissioned to investigate the
charges imputed to Martinez, submitted its report recommending Martinez’s termination from
employment as well as the filing of the appropriate case in court. On November 26, 2002, Martinez
was dismissed from service, prompting him to file a complaint15 for illegal dismissal with money
claims for 13th month pay, service incentive leave pay and allowances, as well as moral and
exemplary damages.16
Issue:
Whether Martinez’s dismissal on the ground of loss of trust and confidence is valid.
Held:
To validly dismiss an employee on the ground of loss of trust and confidence under Article 296(c)
(formerly Article 282[c]) of the Labor Code,26 the following guidelines must be observed: (1) the
employee concerned must be holding a position of trust and confidence; and (2) there must be an
act that would justify the loss of trust and confidence.27
Anent the first requisite, it is noteworthy to mention that there are two classes of positions of trust,
namely: (1) managerial employees whose primary duty consists of the management of the
establishment in which they are employed or of a department or a subdivision thereof, and to other
officers or members of the managerial staff; and (2) fiduciary rank-and-file employees such as
cashiers, auditors, property custodians, or those who, in the normal exercise of their functions,
regularly handle significant amounts of money or property. These employees, though rank-and-file,
are routinely charged with the care and custody of the employer’s money or property, and are thus
classified as occupying positions of trust and confidence.28 Being an employee tasked to collect
payments and remit the same to CENPELCO, Martinez belongs to the latter class and thus, occupies
a position of trust and confidence.
Anent the second requisite, the audit report conducted on Martinez's cash count revealed that he

154
had a shortage in the amount of P44,846. 77 in his remittance for April 25, 2002. When asked to
explain such shortage, Martinez not only admitted the same but even tried to exculpate himself
from liability by attempting to offset said shortage with his alleged overage on April 23, 2002 in the
amount of P45,682.58. The Court agrees with the CA that this practice should never be
countenanced because it would allow the employees to patch up inaccuracies or even their own
wrongdoings and thus, the true revenues or losses of the company will never be correctly
identified.29Verily, this irregular practice would be detrimental to the interests of the employer
whose bread and butter depends solely on realized profits.30 Perforce, Martinez's failure to
properly account for his shortage of such a significant amount is enough reason for CENPELCO to
lose trust and confidence in him.
On the contrary, the Court disagrees with Martinez's contention that CENPELCO had failed to prove
the validity of his dismissal. As correctly found by the CA and the NLRC, CENPELCO had properly
adduced evidence which substantially supports the conclusion that on April 25, 2002, Martinez had
a shortage in the amount of P44,846.77. As Martinez was accountable for the discrepancies in his
collections vis-a-vis his remittances, the burden of evidence shifted to him to prove that the
reflected shortage was not attributable to any form of negligence or infraction on his
part.1âwphi1 However, records disclose that instead of properly explaining the reason for such
shortage, Martinez merely admitted its existence. Worse, he even tried to offset such shortage with
his purported April 23, 2002 overage. In fine, CENPELCO had every right to dismiss Martinez on the
ground of loss of trust and confidence for the latter's inability to account for the shortages imputed
to him.

G.R. No.173587 July 15, 2013

ZUELLIG PHARMA CORPORATION, PETITIONER, 



VS.

ALICE M. SIBAL. ET. AL , RESPONDENT

Facts:
Petitioner Zuellig Pharma Corporation (Zuellig) is a domestic corporation engaged in the
manufacture and distribution of pharmaceutical products. It also distributes pharmaceutical
products manufactured by other companies like Syntex Pharmaceuticals (Syntex). Respondents (36
in all), on the other hand, were the employees of Zuellig at its Syntex Division.
In 1995, Roche Philippines, Inc. (Roche) purchased Syntex and took over from Zuellig the
distribution of Syntex products. Consequently, Zuellig closed its Syntex Division and terminated the
services of respondents due to redundancy. They were properly notified of their termination and
were paid their respective separation pay in accordance with Section 3(b), Article XIV of the March
21, 1995 Collective Bargaining Agreement (CBA) for which, respondents individually signed
Release and Quitclaim in full settlement of all claims arising from their employment with Zuellig.
Proceedings before the Labor Arbiter and the NLRC
Controversy arose when respondents filed before the Arbitration Branch of the NLRC separate
Complaints (which were later consolidated) for payment of retirement gratuity and monetary
equivalent of their unused sick leave on top of the separation pay already given them. Respondents

155
claimed that they are still entitled to retirement benefits and that their receipt of separation pay
and execution of Release and Quitclaim do not preclude pursuing such claim.
Labor Arbiter rendered a Decision denying respondents’ claims. He opined that only employees
whose separation from employment was brought about by sickness, death, compulsory or optional
retirement, or resignation are entitled to gratuity pay. However, employees whose separation from
employment was by reason of redundancy are not entitled to the monetary equivalent of their
unused sick leave if cessation from employment was caused by redundancy.
Upon respondents’ appeal, the NLRC rendered a Decision affirming the Decision of the Labor
Arbiter.
Twice rebuffed but still undeterred, the respondents filed a Petition for Certiorari with the CA.
CA granted respondents’ Petition and nullified the Decisions of both the Labor Arbiter and the
NLRC. Relying on the case of Aquino v. National Labor Relations Commission, the CA ruled that
since there is nothing in the CBA which expressly prohibits the grant of both benefits, those who
received separation pay are, therefore, still entitled to retirement gratuity. The CA also took note of
Section 5, Article V of Zuellig’s January 1, 1968 Retirement Gratuity Plan, which provides that an
employee who may be separated from the service for any cause not attributable to his or her own
fault or misconduct shall be entitled to full retirement benefits. Since the cause of respondents’
separation from work was redundancy, the CA ordered Zuellig to pay respondents retirement
gratuity and the monetary equivalent of their unused sick leave on top of the redundancy pay
previously granted to them.

Issue:
Whether or not under the terms and conditions of the CBA and the retirement and gratuity plan,
respondents could avail of both redundancy pay and retirement benefits

Ruling:
The CA’s ruling in effect put something into the CBA that is not written in it, contrary to the old and
familiar Latin maxim of expressio unius est exclusio alterius. The express mention of one person,
thing, act, or consequence excludes all others. Put differently, where the terms are expressly limited
to certain matters, it may not, by interpretation or construction, be extended to other matters. In
this case, Article VIII of the CBA covers only (1) an employee who is 60 years old and due for
compulsory retirement; (2) an employee who retires prior to attaining the compulsory retirement
age but has served at least 25 years; and, (3) an employee who retires before attaining compulsory
retirement age due to illness or disability. Necessarily, the enumeration cannot be extended to
include those who will be leaving the company due to redundancy, death, merger, installation of
labor cost-saving device, retrenchment, or closure of business as mistakenly ruled by the CA.
As the law between the parties, the CBA must be strictly complied with.
It is a familiar and fundamental doctrine in labor law that the CBA is the law between the parties
and they are obliged to comply with its provisions. In Honda Phils., Inc. v. Samahan ng Malayang
Manggagawa sa Hond this Court elucidated as follows:
A collective bargaining agreement [or CBA] refers to the negotiated contract between a legitimate
labor organization and the employer concerning wages, hours of work and all other terms and
conditions of employment in a bargaining unit. As in all contracts, the parties in a CBA may

156
establish such stipulations, clauses, terms and conditions as they may deem convenient provided
these are not contrary to law, morals, good customs, public order or public policy. Thus, where the
CBA is clear and unambiguous, it becomes the law between the parties and compliance therewith is
mandated by the express policy of the law.
Here, and as discussed above, the parties’ CBA provides in no uncertain terms that whatever
amount of money the employees will receive as retirement gratuity shall be chargeable against
separation pay. It is the unequivocal manifestation of their agreement that acceptance of retirement
gratuity forecloses receipt of separation pay and vice versa. The CBA likewise exclusively
enumerates departing employees who are entitled to the monetary equivalent of their unused sick
leave. These agreements must prevail and be given full effect.
The Release and Quitclaim executed by each of the respondents remains valid.
It is true that quitclaims executed by employees are often frowned upon as contrary to public
policy. But that is not to say that all waivers and quitclaims are invalid as against public policy.
Quitclaims will be upheld as valid if the following requisites are present: "(1) the employee
executes a deed of quitclaim voluntarily; (2) there is no fraud or deceit on the part of any of the
parties; (3) the consideration of the quitclaim is credible and reasonable; and, (4) the contract is not
contrary to law, public order, public policy, morals or good customs or prejudicial to a third person
with a right recognized by law."
In this case, there is no showing that Zuellig coerced or forced respondents to sign the Release and
Quitclaim. In fact, there is no allegation that Zuellig employed fraud or deceit in making
respondents sign the Release and Quitclaim. On the other hand, respondents declared that they had
received the separation pay in full settlement of all claims arising from their employment with
Zuellig. For which reason, they have remised, released and discharged Zuellig.
Notably, the Release and Quitclaim represents a reasonable and fair settlement of respondents’
claims. Under Article 283 of the Labor Code, the employers are required to pay employees
separated from employment by reason of redundancy at least one (1) month pay or at least one (1)
month pay for every year of service, whichever is higher. Here, respondents received 100% of their
one (1) month basic pay for every year of service, plus a premium ranging from 20% to 85% of such
basic pay for every year of service (depending on the number of years in service), as separation pay.
In Goodrich Manufacturing Corporation, v. Ativo, this Court declared that –
It is only where there is clear proof that the waiver was wangled from an unsuspecting or gullible
person, or the terms of settlement are unconscionable on its face, that the law will step in to annul
the questionable transaction. But where it is shown that the person making the waiver did so
voluntarily, with full understanding of what he was doing, and the consideration for the quitclaim is
credible and reasonable, the transaction must be recognized as a valid and binding undertaking.

G.R. No. 157900 July 22, 2013

ZUELLIG FREIGHT AND CARGO SYSTEMS, PETITIONER, 



VS. 

NATIONAL LABOR RELATIONS COMMISSION AND RONALDO V. SAN MIGUEL, RESPONDENTS.

157
Facts:
San Miguel brought a complaint for unfair labor practice, illegal dismissal, non-payment of salaries
and moral damages against petitioner, formerly known as Zeta Brokerage Corporation (Zeta). He
alleged that he had been a checker/customs representative of Zeta since December 16, 1985; that in
January 1994, he and other employees of Zeta were informed that Zeta would cease operations, and
that all affected employees, including him, would be separated; that by letter dated February 28,
1994, Zeta informed him of his termination effective March 31, 1994; that he reluctantly accepted
his separation pay subject to the standing offer to be hired to his former position by petitioner; and
that on April 15, 1994, he was summarily terminated, without any valid cause and due process.
San Miguel contended that the amendments of the articles of incorporation of Zeta were for the
purpose of changing the corporate name, broadening the primary functions, and increasing the
capital stock; and that such amendments could not mean that Zeta had been thereby dissolved.
On its part, petitioner countered that San Miguel’s termination from Zeta had been for a cause
authorized by the Labor Code; that its non-acceptance of him had not been by any means irregular
or discriminatory; that its predecessor-in-interest had complied with the requirements for
termination due to the cessation of business operations; that it had no obligation to employ San
Miguel in the exercise of its valid management prerogative; that all employees had been given
sufficient time to make their decision whether to accept its offer of employment or not, but he had
not responded to its offer within the time set; that because of his failure to meet the deadline, the
offer had expired; that he had nonetheless been hired on a temporary basis; and that when it
decided to hire another employee instead of San Miguel, such decision was not arbitrary because of
seniority considerations.
The NLRC ruled that there was merely a change of business name and primary purpose and
upgrading of stocks of the corporation. Zuellig and Zeta are therefore legally the same person and
entity. As such, the termination of complainant’s services allegedly due to cessation of business
operations of Zeta is deemed illegal. Notwithstanding his receipt of separation benefits from
respondents, complainant is not estopped from questioning the legality of his dismissal.

CA
affirmed NLRC decision.

Issue:
Whether or not the cessation of Zeta’s business, which resulted in the severance of San Miguel from
his employment, was valid.

Ruling:
It is worthy to point out that the Labor Arbiter, the NLRC, and the CA were united in concluding
that the cessation of business by Zeta was not a bona fide closure to be regarded as a valid ground
for the termination of employment of San Miguel within the ambit of Article 283 of the Labor Code.
The provision pertinently reads:
Article 283. Closure of establishment and reduction of personnel. — The employer may also
terminate the employment of any employee due to the installation of labor-saving devices,
redundancy, retrenchment to prevent losses or the closing or cessation of operation of the
establishment or undertaking unless the closing is for the purpose of circumventing the provisions
of this Title, by serving a written notice on the workers and the Department of Labor and

158
Employment at least one (1) month before the intended date thereof. x x x.
The unanimous conclusions of the CA, the NLRC and the Labor Arbiter, being in accord with law,
were not tainted with any abuse of discretion, least of all grave, on the part of the NLRC. Verily, the
amendments of the articles of incorporation of Zeta to change the corporate name to Zuellig Freight
and Cargo Systems, Inc. did not produce the dissolution of the former as a corporation. For sure, the
Corporation Code defined and delineated the different modes of dissolving a corporation, and
amendment of the articles of incorporation was not one of such modes. The effect of the change of
name was not a change of the corporate being, for, as well stated in Philippine First Insurance Co.,
Inc. v. Hartigan: "The changing of the name of a corporation is no more the creation of a corporation
than the changing of the name of a natural person is begetting of a natural person. The act, in both
cases, would seem to be what the language which we use to designate it imports – a change of
name, and not a change of being."
The mere change in the corporate name is not considered under the law as the creation of a new
corporation; hence, the renamed corporation remains liable for the illegal dismissal of its employee
separated under that guise.

[G.R. No. 192571. July 23, 2013.]

ABBOTT LABORATORIES, PHILS, ET. AL


VS.
PEARLIE ANN F. ALCARAZ

Topic: Termination of Employment (sufficient standards in terminating probationary


employee)
Facts:

On June 27, 2004, petitioner Abbott Laboratories, Philippines (Abbott) caused the publication in a
major broadsheet newspaper of its need for a Medical and Regulatory Affairs Manager (Regulatory
Affairs Manager) who would: (a) be responsible for drug safety surveillance operations, staffing,
and budget; (b) lead the development and implementation of standard operating
procedures/policies for drug safety surveillance and vigilance; and (c) act as the primary interface
with internal and external customers regarding safety operations and queries. Alcaraz — who was
then a Regulatory Affairs and Information Manager at Aventis Pasteur Philippines, Incorporated
(another pharmaceutical company like Abbott) — showed interest and submitted her application
on October 4, 2004.

On December 7, 2004, Abbott formally offered Alcaraz the above-mentioned position which was an
item under the company's Hospira Affiliate Local Surveillance Unit (ALSU) department. In Abbott's
offer sheet, it was stated that Alcaraz was to be employed on a probationary basis. Later that day,
she accepted the said offer and received an electronic mail (e-mail) from Abbott's Recruitment
Officer, petitioner Teresita C. Bernardo (Bernardo), confirming the same. Attached to Bernardo's e-

159
mail were Abbott's organizational chart and a job description of Alcaraz's work. A
On February 12, 2005, Alcaraz signed an employment contract which stated, that she was to be
placed on probation for a period of six (6) months beginning February 15, 2005 to August 14, 2005.
The said contract was also signed by Abbott's General Manager, petitioner Edwin Feist

On May 23, 2005, Alcaraz was handed a letter from the company stating that her services had been
terminated effective May 19, 2005. The letter detailed the reasons for Alcaraz's termination —
particularly, that Alcaraz: (a) did not manage her time effectively; (b) failed to gain the trust of her
staff and to build an effective rapport with them; (c) failed to train her staff effectively; and (d) was
not able to obtain the knowledge and ability to make sound judgments on case processing and
article review which were necessary for the proper performance of her duties. On May 27, 2005,
Alcaraz received another copy of the said termination letter via registered mail. CTSDa

Alcaraz felt that she was unjustly terminated from her employment and thus, filed a complaint for
illegal dismissal and damages against Abbott and its officers, namely, Misa, Bernardo, Almazar,
Walsh, Terrible, and Feist. She claimed that she should have already been considered as a regular
and not a probationary employee given Abbott's failure to inform her of the reasonable standards
for her regularization upon her engagement as required under Article 295 of the Labor Code. In
this relation, she contended that while her employment contract stated that she was to be engaged
on a probationary status, the same did not indicate the standards on which her regularization
would be based. She further averred that the individual petitioners maliciously connived to
illegally dismiss her when: (a) they threatened her with termination; (b) she was ordered not to
enter company premises even if she was still an employee thereof; and (c)they publicly announced
that she already resigned in order to humiliate her.

Issues :
Whether or not Alcaraz was sufficiently informed of the reasonable standards to qualify her as a
regular employee;
Whether or not Alcaraz was validly terminated from her employment; CTcSAE

Held:
A probationary employee, like a regular employee, enjoys security of tenure. However, in cases of
probationary employment, aside from just or authorized causes of termination, an additional
ground is provided under Article 295 of the Labor Code, i.e., the probationary employee may also be
terminated for failure to qualify as a regular employee in accordance with the reasonable standards
made known by the employer to the employee at the time of the engagement. Thus, the services of
an employee who has been engaged on probationary basis may be terminated for any of the
following: (a) a just or (b) an authorized cause; and (c) when he fails to qualify as a regular
employee in accordance with reasonable standards prescribed by the employer.
Corollary thereto, Section 6 (d), Rule I, Book VI of the Implementing Rules of the Labor Code
provides that if the employer fails to inform the probationary employee of the reasonable standards
upon which the regularization would be based on at the time of the engagement, then the said
employee shall be deemed a regular employee, viz.:

160
(d)In all cases of probationary employment, the employer shall make known to the employee the
standards under which he will qualify as a regular employee at the time of his engagement. Where
no standards are made known to the employee at that time, he shall be deemed a regular
employee. caEIDA

In other words, the employer is made to comply with two (2) requirements when dealing with a
probationary employee:first, the employer must communicate the regularization standards to the
probationary employee; and second, the employer must make such communication at the time of
the probationary employee's engagement. If the employer fails to comply with either, the employee
is deemed as a regular and not a probationary employee.
Keeping with these rules, an employer is deemed to have made known the standards that would
qualify a probationary employee to be a regular employee when it has exerted reasonable efforts to
apprise the employee of what he is expected to do or accomplish during the trial period of
probation. This goes without saying that the employee is sufficiently made aware of his
probationary status as well as the length of time of the probation.

The exception to the foregoing is when the job is self-descriptive in nature, for instance, in the case
of maids, cooks, drivers, or messengers. Also, in Aberdeen Court, Inc. v. Agustin, it has been held
that the rule on notifying a probationary employee of the standards of regularization should not be
used to exculpate an employee who acts in a manner contrary to basic knowledge and common
sense in regard to which there is no need to spell out a policy or standard to be met. In the same
light, an employee's failure to perform the duties and responsibilities which have been clearly made
known to him constitutes a justifiable basis for a probationary employee's non-regularization.
In this case, petitioners contend that Alcaraz was terminated because she failed to qualify as a
regular employee according to Abbott's standards which were made known to her at the time of her
engagement. Contrarily, Alcaraz claims that Abbott never apprised her of these standards and thus,
maintains that she is a regular and not a mere probationary employee.

The Court finds petitioners' assertions to be well-taken.


A punctilious examination of the records reveals that Abbott had indeed complied with the above-
stated requirements. This conclusion is largely impelled by the fact that Abbott clearly conveyed to
Alcaraz her duties and responsibilities as Regulatory Affairs Manager prior to, during the time of
her engagement, and the incipient stages of her employment. On this score, the Court finds it apt to
detail not only the incidents which point out to the efforts made by Abbott but also those
circumstances which would show that Alcaraz was well-apprised of her employer's expectations
that would, in turn, determine her regularization: DTCSHA
(a)On June 27, 2004, Abbott caused the publication in a major broadsheet newspaper of its need for
a Regulatory Affairs Manager, indicating therein the job description for as well as the duties and
responsibilities attendant to the aforesaid position; this prompted Alcaraz to submit her application
to Abbott on October 4, 2004;
(b)In Abbott's December 7, 2004 offer sheet, it was stated that Alcaraz was to be employed on a
probationary status;
(c)On February 12, 2005, Alcaraz signed an employment contract which specifically stated, inter

161
alia, that she was to be placed on probation for a period of six (6) months beginning February 15,
2005 to August 14, 2005;
(d)On the day Alcaraz accepted Abbott's employment offer, Bernardo sent her copies of Abbott's
organizational structure and her job description through e-mail;
(e)Alcaraz was made to undergo a pre-employment orientation where Almazar informed her that
she had to implement Abbott's Code of Conduct and office policies on human resources and finance
and that she would be reporting directly to Walsh;
(f)Alcaraz was also required to undergo a training program as part of her orientation;
(g)Alcaraz received copies of Abbott's Code of Conduct and Performance Modules from Misa who
explained to her the procedure for evaluating the performance of probationary employees; she was
further notified that Abbott had only one evaluation system for all of its employees; and
(h)Moreover, Alcaraz had previously worked for another pharmaceutical company and had
admitted to have an "extensive training and background" to acquire the necessary skills for her
job. DIcSHE
Considering the totality of the above-stated circumstances, it cannot, therefore, be doubted that
Alcaraz was well-aware that her regularization would depend on her ability and capacity to fulfill
the requirements of her position as Regulatory Affairs Manager and that her failure to perform such
would give Abbott a valid cause to terminate her probationary employment.
Verily, basic knowledge and common sense dictate that the adequate performance of one's duties is,
by and of itself, an inherent and implied standard for a probationary employee to be regularized;
such is a regularization standard which need not be literally spelled out or mapped into technical
indicators in every case. In this regard, it must be observed that the assessment of adequate duty
performance is in the nature of a management prerogative which when reasonably exercised — as
Abbott did in this case — should be respected. This is especially true of a managerial employee like
Alcaraz who was tasked with the vital responsibility of handling the personnel and important
matters of her department.
In fine, the Court rules that Alcaraz's status as a probationary employee and her consequent
dismissal must stand. Consequently, in holding that Alcaraz was illegally dismissed due to her
status as a regular and not a probationary employee, the Court finds that the NLRC committed a
grave abuse of discretion.
To elucidate, records show that the NLRC based its decision on the premise that Alcaraz's receipt of
her job description and Abbott's Code of Conduct and Performance Modules was not equivalent to
being actually informed of the performance standards upon which she should have been evaluated
on. It, however, overlooked the legal implication of the other attendant circumstances as detailed
herein which should have warranted a contrary finding that Alcaraz was indeed a probationary and
not a regular employee — more particularly the fact that she was well-aware of her duties and
responsibilities and that her failure to adequately perform the same would lead to her non-
regularization and eventually, her termination.
Accordingly, by affirming the NLRC's pronouncement which is tainted with grave abuse of
discretion, the CA committed a reversible error which, perforce, necessitates the reversal of its
decision.

Probationary employment;

162
termination procedure.
A different procedure is applied when terminating a probationary employee; the usual two-notice
rule does not govern. Section 2, Rule I, Book VI of the Implementing Rules of the Labor Code states
that "[i]f the termination is brought about by the . . . failure of an employee to meet the standards of
the employer in case of probationary employment, it shall be sufficient that a written notice is
served the employee, within a reasonable time from the effective date of termination." TIaCHA
As the records show, Alcaraz's dismissal was effected through a letter dated May 19, 2005 which
she received on May 23, 2005 and again on May 27, 2005. Stated therein were the reasons for her
termination, i.e., that after proper evaluation, Abbott determined that she failed to meet the
reasonable standards for her regularization considering her lack of time and people management
and decision-making skills, which are necessary in the performance of her functions as Regulatory
Affairs Manager. Undeniably, this written notice sufficiently meets the criteria set forth above,
thereby legitimizing the cause and manner of Alcaraz's dismissal as a probationary employee under
the parameters set by the Labor Code.

Employer's violation of
company policy and procedure.
Nonetheless, despite the existence of a sufficient ground to terminate Alcaraz's employment and
Abbott's compliance with the Labor Code termination procedure, it is readily apparent that Abbott
breached its contractual obligation to Alcaraz when it failed to abide by its own procedure in
evaluating the performance of a probationary employee.
Veritably, a company policy partakes of the nature of an implied contract between the employer
and employee. In Parts Depot, Inc. v. Beiswenger, it has been held that: EHaCTA
[E]mployer statements of policy . . . can give rise to contractual rights in employees without
evidence that the parties mutually agreed that the policy statements would create contractual
rights in the employee, and, hence, although the statement of policy is signed by neither party, can
be unilaterally amended by the employer without notice to the employee, and contains no reference
to a specific employee, his job description or compensation, and although no reference was made to
the policy statement in pre-employment interviews and the employee does not learn of its
existence until after his hiring. Toussaint, 292 N.W .2d at 892. The principle is akin to
estoppel. Once an employer establishes an express personnel policy and the employee
continues to work while the policy remains in effect, the policy is deemed an implied
contract for so long as it remains in effect. If the employer unilaterally changes the policy,
the terms of the implied contract are also thereby changed. (Emphasis and underscoring
supplied.)

Hence, given such nature, company personnel policies create an obligation on the part of both the
employee and the employer to abide by the same.
Records show that Abbott's PPSE procedure mandates, inter alia, that the job performance of a
probationary employee should be formally reviewed and discussed with the employee at least
twice: first on the third month and second on the fifth month from the date of employment. Abbott
is also required to come up with a Performance Improvement Plan during the third month review
to bridge the gap between the employee's performance and the standards set, if any. In addition, a

163
signed copy of the PPSE form should be submitted to Abbott's HRD as the same would serve as
basis for recommending the confirmation or termination of the probationary employment.
In this case, it is apparent that Abbott failed to follow the above-stated procedure in evaluating
Alcaraz. For one, there lies a hiatus of evidence that a signed copy of Alcaraz's PPSE form was
submitted to the HRD. It was not even shown that a PPSE form was completed to formally assess
her performance. Neither was the performance evaluation discussed with her during the third and
fifth months of her employment. Nor did Abbott come up with the necessary Performance
Improvement Plan to properly gauge Alcaraz's performance with the set company
standards. cHATSI

While it is Abbott's management prerogative to promulgate its own company rules and even
subsequently amend them, this right equally demands that when it does create its own policies and
thereafter notify its employee of the same, it accords upon itself the obligation to faithfully
implement them. Indeed, a contrary interpretation would entail a disharmonious relationship in the
work place for the laborer should never be mired by the uncertainty of flimsy rules in which the
latter's labor rights and duties would, to some extent, depend.
In this light, while there lies due cause to terminate Alcaraz's probationary employment for her
failure to meet the standards required for her regularization, and while it must be further pointed
out that Abbott had satisfied its statutory duty to serve a written notice of termination, the fact that
it violated its own company procedure renders the termination of Alcaraz's employment
procedurally infirm, warranting the payment of nominal damages. A further exposition is apropos.
Case law has settled that an employer who terminates an employee for a valid cause but does so
through invalid procedure is liable to pay the latter nominal damages.
In Agabon v. NLRC (Agabon), the Court pronounced that where the dismissal is for a just cause, the
lack of statutory due process should not nullify the dismissal, or render it illegal, or ineffectual.
However, the employer should indemnify the employee for the violation of his statutory
rights. Thus, in Agabon, the employer was ordered to pay the employee nominal damages in the
amount of P30,000.00.
Proceeding from the same ratio, the Court modified Agabon in the case of Jaka Food Processing
Corporation v. Pacot (Jaka) where it created a distinction between procedurally defective
dismissals due to a just cause, on one hand, and those due to an authorized cause, on the
other. ACcHIa
It was explained that if the dismissal is based on a just cause under Article 282 of the Labor Code
(now Article 296) but the employer failed to comply with the notice requirement, the sanction to be
imposed upon him should be temperedbecause the dismissal process was, in effect, initiated by an
act imputable to the employee; if the dismissal is based on an authorized cause under Article 283
(now Article 297) but the employer failed to comply with the notice requirement, the sanction
should be stiffer because the dismissal process was initiated by the employer's exercise of his
management prerogative. Hence, in Jaka, where the employee was dismissed for an authorized
cause of retrenchment — as contradistinguished from the employee in Agabon who was dismissed
for a just cause of neglect of duty — the Court ordered the employer to pay the employee nominal
damages at the higher amount of P50,000.00.
Evidently, the sanctions imposed in both Agabon and Jaka proceed from the necessity to deter

164
employers from future violations of the statutory due process rights of employees. In similar
regard, the Court deems it proper to apply the same principle to the case at bar for the reason that
an employer's contractual breach of its own company procedure — albeit not statutory in source —
has the parallel effect of violating the laborer's rights. Suffice it to state, the contract is the law
between the parties and thus, breaches of the same impel recompense to vindicate a right that has
been violated. Consequently, while the Court is wont to uphold the dismissal of Alcaraz because a
valid cause exists, the payment of nominal damages on account of Abbott's contractual breach is
warranted in accordance with Article 2221 of the Civil Code.
Anent the proper amount of damages to be awarded, the Court observes that Alcaraz's dismissal
proceeded from her failure to comply with the standards required for her regularization. As such, it
is undeniable that the dismissal process was, in effect, initiated by an act imputable to the
employee, akin to dismissals due to just causes under Article 296 of the Labor Code. Therefore, the
Court deems it appropriate to fix the amount of nominal damages at the amount of P30,000.00,
consistent with its rulings in both Agabon and Jaka. cD

[G.R. No. 172846. July 24, 2013.]

MANILA POLO CLUB EMPLOYEES' UNION (MPCEU) FUR-TUCP


VS.
MANILA POLO CLUB, INC.,

Summarizing the standards for a valid retrenchment and closure or cessation of business
operations.

Facts:
Petitioner Manila Polo Club Employees Union (MPCEU), which is affiliated with the Federation of
Unions of Rizal (FUR)-TUCP, is a legitimate labor organization duly registered with the Department
of Labor and Employment (DOLE), while respondent Manila Polo Club, Inc. is a non-profit and
proprietary membership organization which provides recreation and sports facilities to its
proprietary members, their dependents, and guests.
On December 13, 2001, the Board of Directors of respondent unanimously resolved to completely
terminate the entire operations of its Food and Beverage (F & B) outlets, except the Last Chukker,
and award its operations to a qualified restaurant operator or caterer. Cited as reasons were as
follows:
WHEREAS, the Food and Beverage (F & B) operations has resulted in yearly losses to the Club in six
(6) out of the last eight (8) years with FY 2001 suffering the largest loss at P10,647,981 and that
this loss is due mainly to the exceedingly high manpower cost and other management
inefficiencies; DHIETc
WHEREAS, due to the substantial losses incurred by the Club in both F&B operations and in its
recurring operations, the Board and management had instituted cost and loss-cutting measures;
WHEREAS, the Board recognized the non-viability of the operations of the Food and Beverage

165
Department and that its continued operations by the Club will result in substantial losses that will
seriously impair the Club's financial health and membership satisfaction;
WHEREAS, the Board recognized the urgent need to act and act decisively and eliminate factors
contributing to substantial losses in the operations of the Club, more particularly the food and
beverage operations. Thus, F & B operations are to cease wholly and totally, subject to observance
and requirements of the law and other rules.
Subsequently, on March 22, 2002, respondent's Board approved the implementation of the
retrenchment program of employees who are directly and indirectly involved with the operations
of the F & B outlets.

Issue: Whether or not the retrenchment of the 117 union members is legal.

It is apparent from the records that this case involves a closure of business undertaking, not
retrenchment. The legal requirements and consequences of these two authorized causes in the
termination of employment are discernible. We distinguished, in Alabang Country Club, Inc. v.
NLRC:
. . . While retrenchment and closure of a business establishment or undertaking are often used
interchangeably and are interrelated, they are actually two separate and independent authorized
causes for termination of employment.
Retrenchment is the reduction of personnel for the purpose of cutting down on costs of operations in
terms of salaries and wages resorted to by an employer because of losses in operation of a business
occasioned by lack of work and considerable reduction in the volume of business.
Closure of a business or undertaking due to business losses is the reversal of fortune of the employer
whereby there is a complete cessation of business operations to prevent further financial drain upon
an employer who cannot pay anymore his employees since business has already stopped.
One of the prerogatives of management is the decision to close the entire establishment or to close
or abolish a department or section thereof for economic reasons, such as to minimize expenses and
reduce capitalization.
While the Labor Code provides for the payment of separation package in case of retrenchment to
prevent losses, it does not obligate the employer for the payment thereof if there is closure of
business due to serious losses. cDCaTH
Likewise, the case of Eastridge Golf Club, Inc. v. Eastridge Golf Club, Inc., Labor-Union, Super stressed
the differences:
Retrenchment or lay-off is the termination of employment initiated by the employer, through no
fault of the employees and without prejudice to the latter, during periods of business recession,
industrial depression, or seasonal fluctuations, or during lulls occasioned by lack of orders,
shortage of materials, conversion of the plant for a new production program or the introduction of
new methods or more efficient machinery, or of automation. It is an exercise of management
prerogative which the Court upholds if compliant with certain substantive and procedural
requirements, namely:
1.That retrenchment is necessary to prevent losses and it is proven, by sufficient and convincing
evidence such as the employer's financial statements audited by an independent and credible
external auditor, that such losses are substantial and not merely flimsy and actual or reasonably

166
imminent; and that retrenchment is the only effective measure to prevent such imminent losses;
2.That written notice is served on to the employees and the DOLE at least one (1) month prior to
the intended date of retrenchment; and DaIAcC
3.That the retrenched employees receive separation pay equivalent to one (1) month pay or at least
one-half (1/2) month pay for every year of service, whichever is higher.
The employer must prove compliance with all the foregoing requirements. Failure to prove the first
requirement will render the retrenchment illegal and make the employer liable for the
reinstatement of its employees and payment of full backwages. However, were the retrenchment
undertaken by the employer isbona fide, the same will not be invalidated by the latter's failure to
serve prior notice on the employees and the DOLE; the employer will only be liable in nominal
damages, the reasonable rate of which the Court En Banc has set at P50,000.00 for each employee.
Closure or cessation of business is the complete or partial cessation of the operations and/or shut-
down of the establishment of the employer. It is carried out to either stave off the financial ruin or
promote the business interest of the employer.
Unlike retrenchment, closure or cessation of business, as an authorized cause of termination of
employment, need not depend for validity on evidence of actual or imminent reversal of the
employer's fortune. Article 283 authorizes termination of employment due to business closure,
regardless of the underlying reasons and motivations therefor, be it financial losses or not.
To be precise, closure or cessation of an employer's business operations, whether in whole or in
part, is governed by Article 283 of the Labor Code, as amended. It states:
Article 283.Closure of establishment and reduction of personnel. — The employer may also terminate
the employment of any employee due to the installation of labor-saving devices, redundancy,
retrenchment to prevent losses or the closing or cessation of operation of the establishment or
undertaking unless the closing is for the purpose of circumventing the provisions of this Title, by
serving a written notice on the workers and the Ministry of Labor and Employment at least one (1)
month before the intended date thereof. In case of termination due to the installation of labor-
saving devices or redundancy, the worker affected thereby shall be entitled to a separation pay
equivalent to at least his one (1) month pay or to at least one (1) month pay for every year of
service, whichever is higher. In case of retrenchment to prevent losses and in cases of closures or
cessation of operations of establishment or undertaking not due to serious business losses or
financial reverses, the separation pay shall be equivalent to one (1) month pay or at least one-half
(1/2) month pay for every year of service, whichever is higher. A fraction of at least six (6) months
shall be considered one (1) whole year. HC
In Industrial Timber Corporation v. Ababon, 27 the Court explained the above-quoted provision in
this wise:
A reading of the foregoing law shows that a partial or total closure or cessation of operations of
establishment or undertaking may either be due to serious business losses or financial reverses or
otherwise. Under the first kind, the employer must sufficiently and convincingly prove its allegation
of substantial losses, while under the second kind, the employer can lawfully close shop anytime as
long as cessation of or withdrawal from business operations was bona fide in character and not
impelled by a motive to defeat or circumvent the tenurial rights of employees, and as long as he
pays his employees their termination pay in the amount corresponding to their length of service.
Just as no law forces anyone to go into business, no law can compel anybody to continue the same.

167
It would be stretching the intent and spirit of the law if a court interferes with management's
prerogative to close or cease its business operations just because the business is not suffering from
any loss or because of the desire to provide the workers continued employment.
In sum, under Article 283 of the Labor Code, three requirements are necessary for a valid cessation
of business operations: (a) service of a written notice to the employees and to the DOLE at least one
month before the intended date thereof; (b) the cessation of business must be bona fide in
character; and (c) payment to the employees of termination pay amounting to one month pay or at
least one-half month pay for every year of service, whichever is higher.
Based on the above and cases of similar import, We summarize:
(Note: There are a lot of cases cited and discussed in relation to this case before the SC summarized
them)
1.Closure or cessation of operations of establishment or undertaking may either be partial or total.
2.Closure or cessation of operations of establishment or undertaking may or may not be due to
serious business losses or financial reverses. However, in both instances, proof must be shown that:
(1) it was done in good faith to advance the employer's interest and not for the purpose of defeating
or circumventing the rights of employees under the law or a valid agreement; and (2) a written
notice on the affected employees and the DOLE is served at least one month before the intended
date of termination of employment. EHCcIT
3.The employer can lawfully close shop even if not due to serious business losses or financial
reverses but separation pay, which is equivalent to at least one month pay as provided for by
Article 283 of the Labor Code, as amended, must be given to all the affected employees.
4.If the closure or cessation of operations of establishment or undertaking is due to serious
business losses or financial reverses, the employer must prove such allegation in order to avoid the
payment of separation pay. Otherwise, the affected employees are entitled to separation pay.
5.The burden of proving compliance with all the above-stated falls upon the employer.
Guided by the foregoing, the Court shall refuse to dwell on the issue of whether respondent was in
sound financial condition when it resolved to stop the operations of its F & B Department. As stated,
an employer can lawfully close shop anytime even if not due to serious business losses or financial
reverses. Furthermore, the issue would entail an inquiry into the factual veracity of the evidence
presented by the parties, the determination of which is not Our statutory function. Indeed,
petitioner is asking Us to sift through the evidence on record and pass upon whether respondent
had, in truth and in fact, suffered from serious business losses or financial reverses. That task,
however, would be contrary to the well-settled principle that this Court is not a trier of facts, and
cannot re-examine and re-evaluate the probative value of the evidence presented to the VA and the
CA, which formed the basis of the questioned decision. Cdpr
Respondent correctly asserted in its Memorandum that the instant case is similar to Alabang
Country Club, Inc. When it decided to cease operating its F & B Department and open the same to a
concessionaire, respondent did not reduce the number of personnel assigned thereat; instead, it
terminated the employment of all personnel assigned at the department and those who are directly
and indirectly involved in its operations. The closure of the F & B Department was due to legitimate
business considerations, a resolution which the Court has no business interfering with. We have
already resolved that the characterization of the employee's service as no longer necessary or
sustainable, and therefore, properly terminable, is an exercise of business judgment on the part of

168
the employer; the determination of the continuing necessity of a particular officer or position in a
business corporation is a management prerogative, and the courts will not interfere with the
exercise of such so long as no abuse of discretion or arbitrary or malicious action on the part of the
employer is shown. 32 As recognized by both the VA and the CA, evident proofs of respondent's
good faith to arrest the losses which the F & B Department had been incurring since 1994 are:
engagement of an independent consulting firm to conduct manpower audit/organizational
development; institution of cost-saving programs, termination of the services of probationary
employees, substantial reduction of a number of agency staff and personnel, and the retrenchment
of eight (8) managers. After the effective date of the termination of employment relation,
respondent even went on to aid the displaced employees in finding gainful employment by
soliciting the assistance of respondent's members, Makati Skyline, Human Resource Managers of
some companies, and the Association of Human Resource Managers. These were not refuted by
petitioner. Only that, it perceives them as inadequate and insists that the operational losses are
very well covered by the other income of respondent and that less drastic measures could have
been resorted to, like increasing the membership dues and the prices of food and beverage. Yet the
wisdom or soundness of the Management decision is not subject to discretionary review of the
Court for, even the VA admitted, it enjoys a pre-eminent role and is presumed to possess all
relevant and necessary information to guide its business decisions and actions.
Further, unlike in the case of Eastridge Golf Club, Inc., there is nothing on record to indicate that the
closure of respondent's F & B Department was made in bad faith. It was not motivated by any
specific and clearly determinable union activity of the employees; rather, it was truly dictated by
economic necessity. Despite petitioner's allegations, no convincing and credible proofs were
presented to establish the claim that such closure qualifies as an act of union-busting and ULP. No
evidence was shown that the closure is stirred not by a desire to avoid further losses but to
discourage the workers from organizing themselves into a union for more effective negotiations
with the management. Allegations are not proofs and it is incumbent upon petitioner to
substantiate the same. On the contrary, respondent continued to negotiate with petitioner even
after April 30, 2002. In fact, a Memorandum of Agreement was executed before the NCMB between
petitioner and respondent on June 10, 2002 whereby the parties agreed, among others, to maintain
the existing provisions of the CBA, except those pertaining to wage increases and signing
bonus. SCHTac
Finally, even if the members of petitioner are not considered as illegally dismissed, they are entitled
to separation pay pursuant to Article 283 of the Labor Code, as amended. Per respondent's
information, however, the separation packages of all 117 union members were already paid during
the pendency of the case. Petitioner did not oppose this representation; hence, We shall treat the
fact of receipt of separation pay as having been voluntarily entered into, with a full understanding
of its import, and the amount received as credible and reasonable settlement that should be
respected by the Court as the law between the parties are valid and binding between them.

G.R. No.179326 July 31, 2013

169
LUCIANO P. CANEDO,* PETITIONER, 

VS.

KAMPILAN SECURITY AND DETECTIVE AGENCY, INC. AND RAMONCITO L. ARQUIZA,
RESPONDENTS.

Facts:
Respondent agency hired petitioner as security guard on November 20, 1996 and assigned him at
the Naga Power Barge 102 of the National Power Corporation (NPC) at Sigpit Load Ends, Lutopan,
Toledo City.

For not wearing proper uniform while on duty as per report of Allan Alfafara (Alfafara) of the NPC,
petitioner was suspended for a month effective May 8, 2003.8
In a letter9 dated June 2, 2003, NPC informed respondent agency that it was no longer interested in
petitioner’s services and thus requested for his replacement.
On June 17, 2003, petitioner requested respondent Arquiza to issue a certification in connection
with his intended retirement effective that month.10 Thus, respondent Arquiza issued the
Certification11 dated June 25, 2003 (June 25, 2003 Certification):
CERTIFICATION
TO WHOM IT MAY CONCERN:
This is to certify that Mr. Luciano Paragoso Cañedo whose address is at Lower Bunga, Toledo City
was employed by this agency from November 20, 1996 up to May 7, 2003 as Security Guard
assigned at NPC, Sigpit Substation. He was terminated from his employment by this agency on May
7, 2003 as per client’s request.
Done this 25th day of June 2003 at Cebu City, Philippines.
(Signed)
RAMONCITO L. ARQUIZA
General Manager
KSDAI
Five days later, petitioner filed before the Labor Arbiter a Complaint for illegal dismissal, illegal
suspension and non-payment of monetary benefits against respondents.

Proceedings before the Labor Arbiter


Based on the June 25, 2003 Certification, the Labor Arbiter held that petitioner was illegally
dismissed from the service. He also found petitioner’s prior suspension illegal and granted him all
his monetary claims except for underpayment of wages

Proceedings before the National Labor Relations Commission


the NLRC opined that petitioner’s intention to retire as shown by his June 17, 2003 letter negated
his claim of termination. Nevertheless, it maintained that petitioner was suspended without being
notified of his infraction. Thus, he should be paid his salary during the period of his illegal
suspension.

Ruling of the Court of Appeals


The CA, in a Decision26 dated January 25, 2007, denied the Petition after it found no grave abuse of
discretion on the part of the NLRC.

170
In view of the above, the CA concluded that petitioner was merely placed on temporary "off-detail"
which is not equivalent to dismissal. However, like the NLRC, the CA found that petitioner was
deprived of due process when he was suspended and thus affirmed his entitlement to his salary
during the period of suspension. It also affirmed the awards for holiday pay and service incentive
leave pay as well as the deduction therefrom of P10,000.00 representing petitioner’s cash advance.

Issue:

whether petitioner was dismissed from service

Ruling:

Petitioner relies on the word "terminated" as used in the June 25, 2003 Certification issued him by
respondent Arquiza and argues that the same is a clear indication that he was dismissed from
service. We are, however, not persuaded. Petitioner cannot simply rely on this piece of document
since the fact of dismissal must be evidenced by positive and overt acts of an employer indicating
an intention to dismiss.36 Here, aside from this single document, petitioner proffered no other
evidence showing that he was dismissed from employment. While it is true that he was not allowed
to report for work after the period of his suspension expired, the same was due to NPC’s request for
his replacement as NPC was no longer interested in his services. And as correctly argued by
respondents, petitioner from that point onward is not considered dismissed but merely on a
floating status. "Such a ‘floating status’ is lawful and not unusual for security guards employed in
security agencies as their assignments primarily depend on the contracts entered into by the
agency with third parties."

A floating status can ripen into constructive dismissal only when it goes beyond the six-month
maximum period allowed by law. In this case, petitioner filed the Complaint for illegal dismissal
even before the lapse of the six-month period. Hence, his claim of illegal dismissal lacks basis.
Moreover and as aptly observed by the NLRC, it was in fact petitioner who intended to terminate
his relationship with respondents through his planned retirement. This is further bolstered by his
prayer in his Complaint where he sought for separation pay and not for reinstatement.

G.R. No. 185549 August 7, 2013

VICENTE ANG, PETITIONER,


VS.

CEFERINO SAN JOAQUIN, JR., AND DIOSDADO FERNANDEZ, RESPONDENTS.

Facts:

Petitioner Vicente Ang (Ang) is the proprietor of Virose Furniture and Glass Supply (Virose) in
Tayug, Pangasinan, a wholesaler/retailer of glass supplies, jalousies, aluminum windows, table

171
glass, and assorted furniture. Respondents Ceferino San Joaquin, Jr. (San Joaquin) and Diosdado
Fernandez (Fernandez) were regular employees of Virose: San Joaquin was hired in 1974 as helper,
while Fernandez was employed in 1982 as driver.5 Respondents have been continuously in Ang’s
employ without any derogatory record.6 Each received a daily salary of P166.00.7
Through the years, San Joaquin – who is Ang’s first cousin, their mothers being sisters – became a
pahinante or delivery helper, and later on an all-around worker of Virose.8
On August 24, 1999, respondents attended the court hearing relative to the 41 criminal cases filed
by former Virose employee Daniel Abrera (Abrera) against Ang for the latter’s non-remittance of
Social Security System (SSS) contributions.9 During that hearing, respondents testified against Ang;
it was the second time for San Joaquin to testify, while it was Fernandez’s first.

On August 30, 1999, San Joaquin returned to the store, only to find out that Ang had torn his DTR to
pieces that day while the DTR of Fernandez was torn to pieces by Ang immediately after the August
24, 1999 hearing in which the respondents testified.13 On the same day, Fernandez reported for
work and received a memorandum of even date issued by Ang informing him that he was placed on
a one-week suspension for insubordination.14 The memorandum did not specify the act of
insubordination.15
On August 31, 1999, respondents filed against Ang Complaints for illegal constructive dismissal
with claims for backwages and separation pay.

On September 21, 1999, Ang issued a memorandum terminating San Joaquin’s employment.20
Ruling of the Labor Arbiter
The Labor Arbiter held that respondents were unable to show how Ang discriminated against them.
He pointed out that respondents cited only two instances of alleged discrimination/reprisal
committed against them: the August 28, 1999 incident regarding the transfer of the monobloc
chairs and Fernandez’s failure to find his DTR when he reported for work following receipt of the
September 13, 1999 memorandum; but these were not acts of discrimination/ reprisal.

Ruling of the National Labor Relations Commission (NLRC)

The NLRC declared that there was no constructive dismissal. It held that respondents failed to
prove that they were constructively dismissed; nor do the facts of the case sufficiently show that
they were constructively dismissed from employment.

Ruling of the Court of Appeals


The CA held that the Labor Arbiter and the NLRC misappreciated the facts which thus led to the
erroneous conclusion that there was no constructive dismissal. It considered Ang’s act of tearing
the respondents DTRs or time cards as a categorical indication of their dismissal from
employment

Issue:
Whether the respondents were constructively dismissed from work

172
Ruling:
"Constructive dismissal exists where there is cessation of work because continued employment is
rendered impossible, unreasonable or unlikely, as an offer involving a demotion in rank and a
diminution in pay."51 It is a "dismissal in disguise or an act amounting to dismissal but made to
appear as if it were not."52 Constructive dismissal may likewise exist if an "act of clear
discrimination, insensibility, or disdain by an employer becomes so unbearable on the part of the
employee that it could foreclose any choice by him except to forego his continued employment."53
"Constructive dismissal exists when the employee involuntarily resigns due to the harsh, hostile,
and unfavorable conditions set by the employer."54 "The test of constructive dismissal is whether a
reasonable person in the employee’s position would have felt compelled to give up his position
under the circumstances."

The CA is correct in its pronouncement that respondents were constructively dismissed from work.
Moreover, by destroying respondents’ time cards, Ang discontinued and severed his relationship
with respondents. The purpose of a time record is to show an employee’s attendance in office for
work and to be paid accordingly, taking into account the policy of "no work, no pay". A daily time
record is primarily intended to prevent damage or loss to the employer, which could result in
instances where it pays an employee for no work done;56 it is a mandatory requirement for
inclusion in the payroll, and in the absence of an employment agreement, it constitutes evidence of
employment. Thus, when Ang tore the respondents’ time cards to pieces, he virtually removed them
from Virose’s payroll and erased all vestiges of respondents’ employment; respondents were
effectively dismissed from work. The act may be considered an outright – not only symbolic –
termination of the parties’ employment relationship; the "last straw that finally broke the camel’s
back", as respondents put it in their Position Paper.

For a termination of employment on the ground of abandonment to be valid, the employer "must
prove, by substantial evidence, the concurrence of [the employee’s] failure to report for work for no
valid reason and his categorical intention to discontinue employment."57 In the present case, it
appears that there is no intention to abandon employment; respondents’ repeated absence were
caused by Ang’s oppressive treatment and indifference which respondents simply grew tired of and
wanted a break from. Indeed, an employee cannot be expected to work efficiently in an atmosphere
where the employer’s hostility pervades; certainly, it is too stressful and depressing – the threat of
immediate termination from work, if not aggression, is a heavy burden carried on the employee’s
shoulder. Respondents may have stayed away from work to cool off, but not necessarily to abandon
their employment. The fact remains that respondents returned to work, but then their time cards
had been torn to pieces.

G.R. No. 187214, August 14, 2013

173
SANOH FULTON PHILS., INC. AND MR. EDDIE JOSE, PETITIONERS,
VS.
EMMANUEL BERNARDO AND SAMUEL TAGHOY, RESPONDENTS

Facts:
Sanoh is a domestic corporation engaged in the manufacture of automotive parts and wire
condensers for home appliances. Its Wire Condenser Department employed 61 employees.
Respondents belonged to this department.

In view of job order cancellations relating to the
manufacture of wire condensers by Matsushita, Sanyo and National Panasonic, Sanoh decided to
phase out the Wire Condenser Department. On 22 December 2003, the Human Resources Manager
of Sanoh informed the 17 employees, 16 of whom belonged to the Wire Condenser Department, of
retrenchment effective 22 January 2004. All 17 employees are union members.

A grievance
conference was held where the affected employees were informed of the following grounds for
retrenchment:
1) Lack of local market.

2) Competition from imported products.

3) Phasing out of Wire
Condenser Department.
Sanoh on its part, filed a petition for declaration of the partial closure of its Wire Condenser
Department and valid retrenchment of the 17 employees.
The complainants alleged that there was no valid cause for retrenchment and in effecting
retrenchment, there was a violation of the “first in-last out” and “last in-first out” (LIFO) policy
embodied in the Collective Bargaining Agreement.

Sanoh, on the other hand, asserted that
retrenchment was a valid exercise of management prerogative. Sanoh averred that some employees
who were hired much later were either assigned to other departments or were bound by the terms
of their job training agreement to stay with the company for 3 years.


Issue:
Whether the dismissal was with just cause and the retrenchment justified?

Ruling:
No. There was no Valid retrenchment nor was there a closure of business.
To justify retrenchment, Sanoh invokes as grounds serious business losses resulting in the closure
of the Wire Condenser Department, to which respondents belonged. In the same breadth, Sanoh
also contends that its decision to close the Wire Condenser Department is within its right even in
the absence of business losses as long as it is done in good faith.

Sanoh’s two-tiered argument
rests on the application of Article 283 of the Labor Code, which provides:
ART. 283. Closure of establishment and reduction of personnel. — The employer may also
terminate the employment of any employee due to the installation of labor saving devices,
redundancy, retrenchment to prevent losses or the closing or cessation of operation of the
establishment or undertaking unless the closing is for the purpose of circumventing the provisions
of this Title, by serving a written notice on the workers and the Department of Labor and
Employment at least one (1) month before the intended date thereof. In case of termination due to
the installation of labor saving devices or redundancy, the worker affected thereby shall be entitled
to a separation pay equivalent to at least his one (1) month pay or to at least one (1) month pay for
every year of service, whichever is higher. In case of retrenchment to prevent losses and in cases of

174
closures or cessation of operations of establishment or undertaking not due to serious business
losses or financial reverses, the separation pay shall be equivalent to one (1) month pay or to at
least one-half (½) month pay for every year of service, whichever is higher. A fraction of at least six
(6) months shall be considered one (1) whole year.
Retrenchment to prevent losses and closure not due to serious business losses are two separate
authorized causes for terminating the services of an employee. In J.A.T. General Services v.
NLRC,13the Court took the occasion to draw the distinction between retrenchment and closure, to
wit:
Closure of business, on one hand, is the reversal of fortune of the employer whereby there is a
complete cessation of business operations and/or an actual locking-up of the doors of
establishment, usually due to financial losses. Closure of business as an authorized cause for
termination of employment aims to prevent further financial drain upon an employer who cannot
pay anymore his employees since business has already stopped. On the other hand, retrenchment is
reduction of personnel usually due to poor financial returns so as to cut down on costs of
operations in terms of salaries and wages to prevent bankruptcy of the company. It is sometimes
also referred to as down-sizing. Retrenchment is an authorized cause for termination of
employment which the law accords an employer who is not making good in its operations in order
to cut back on expenses for salaries and wages by laying off some employees. The purpose of
retrenchment is to save a financially ailing business establishment from eventually collapsing.16
The respective requirements to sustain their validity are likewise different.

For retrenchment,
the three (3) basic requirements are: (a) proof that the retrenchment is necessary to prevent losses
or impending losses; (b) service of written notices to the employees and to the Department of Labor
and Employment at least one (1) month prior to the intended date of retrenchment; and (c)
payment of separation pay equivalent to one (1) month pay, or at least one-half (1/2) month pay
for every year of service, whichever is higher.14 In addition, jurisprudence has set the standards for
losses which may justify retrenchment, thus:
(1) the losses incurred are substantial and not de minimis; (2) the losses are actual or reasonably
imminent; (3) the retrenchment is reasonably necessary and is likely to be effective in preventing
the expected losses; and (4) the alleged losses, if already incurred, or the expected imminent losses
sought to be forestalled, are proven by sufficient and convincing evidence.15
Upon the other hand, in termination, the law authorizes termination of employment due to business
closure, regardless of the underlying reasons and motivations therefor, be it financial losses or not.
However, to put a stamp to its validity, the closure/cessation of business must be bona fide, i.e., its
purpose is to advance the interest of the employer and not to defeat or circumvent the rights of
employees under the law or a valid agreement.16

In termination cases either by retrenchment or
closure, the burden of proving that the termination of services is for a valid or authorized cause
rests upon the employer.17 Not every loss incurred or expected to be incurred by an employer can
justify retrenchment. The employer must prove, among others, that the losses are substantial and
that the retrenchment is reasonably necessary to avert such losses.18 And to repeat, in closures,
the bona fides of the employer must be proven.

In this case, there was no valid retrenchment. Nor
was there a closure of business

175
G.R. No. 199890, August 19, 2013

JEROME M. DAABAY, PETITIONER,


VS.
COCA-COLA BOTTLERS PHILS., INC., RESPONDENT.

Facts:
The case stems from a complaint for illegal dismissal, illegal suspension, unfair labor practice and
monetary claims filed by Daabay against respondent Coca-Cola Bottlers Phils., Inc. (Coca-Cola) and
three officers of the company.4 The records indicate that the employment of Daabay with Coca-Cola
as Sales Logistics Checker was terminated by the company in June 2005,5 following receipt of
information from one Cesar Sorin (Sorin) that Daabay was part of a conspiracy that allowed the
pilferage of company property.6cralaw virtualaw library

The allegations of Sorin were embodied
in an affidavit which he executed on April 16, 2005.7 The losses to the company were also
confirmed by an inventory and audit conducted by Coca-Cola’s Territory Finance Head, Silvia Ang.
Such losses comprised of cases of assorted softdrinks, empty bottles, missing shells and missing
pallets valued at P20,860,913.00.8cralaw virtualaw library

Coca-Cola then served upon Daabay a
Notice to Explain with Preventive Suspension, which required him to explain in writing his
participation in the scheme that was reported to involve logistics checkers and gate guards. In
compliance therewith, Daabay submitted an Explanation dated April 19, 2005 wherein he denied
any participation in the reported pilferage.9cralaw virtualaw library

A formal investigation on the matter ensued. Eventually, Coca-Cola served upon Daabay a Notice of
Termination that cited pilferage, serious misconduct and loss of trust and confidence as grounds. At
the time of his dismissal, Daabay had been a regular employee of Coca-Cola for eight years, and was
receiving a monthly pay of P20,861.00, exclusive of other benefits.10cralaw virtualarary



Issue:
Whether Daabay was validly dismissed?
Whether Daabay may still avail of retirement benefits?

Ruling:
Yes, Daabay was validly dismissed.
There was “reasonable and well-founded basis to dismiss [Daabay], not only for serious
misconduct, but also for breach of trust or loss of confidence arising from such company
losses.”15 Daabay’s participation in the conspiracy was sufficiently established. Several documents
such as checkers receipts and sales invoices that made the fraudulent scheme possible were signed
by Daabay.16 The NLRC also found fault in Daabay for his failure to detect the pilferage, considering
that the “timely recording and monitoring as security control for the outgoing [sic] of company
products are necessarily connected with the functions, duties and responsibilities reposed in him as
Sales Logistics Checker

No, He can no longer avail of retirement benefits. The award of retirement benefits by the NLRC

176
lacked basis considering that Daabay was dismissed for just cause.

We are not oblivious of the instances where the Court awarded financial assistance to dismissed
employees, even though they were terminated for just causes. Equity and social justice was the
vague justification. Quickly realizing the unjustness of these [s]o-called equitable awards, the
Supreme Court took the opportunity to curb and rationalize the grant of financial assistance to
legally dismissed employees.

However, In clear and unmistakable language, the Supreme Court also held that the award of
financial assistance should not be given to validly terminated employees, whose offenses
are iniquitous or reflective of some depravity in their moral character.

At the risk of stating the obvious, private respondent was not separated from petitioner’s
employ due to mandatory or optional retirement but, rather, by termination of employment
for a just cause.
Article 287 of the Labor Code does not operate nor can be made to operate for the benefit of private
respondent. Even private respondent’s assertion that, at the time of her lawful dismissal, she was
already qualified for retirement does not aid her case because the fact remains that private
respondent was already terminated for cause thereby rendering nugatory any entitlement
to mandatory or optional retirement pay that she might have previously possessed.

In ruling against the grant of the retirement benefits, we also take note of the NLRC’s lone
justification for the award, to wit:
Where from the facts obtaining, as in this case, there is a need to humanize the severe effects of
dismissal and where complainant’s entitlement to retirement benefits are even admitted in [Coca-
Cola’s] motion to reduce bond, [w]e can do no less but tilt the scales of justice in favor of labor
as a measure of equity and compassionate social justice, taking into consideration the
circumstances obtaining in this case.33 (Emphasis ours)
Being intended as a mere measure of equity and social justice, the NLRC’s award was then akin to a
financial assistance or separation pay that is granted to a dismissed employee notwithstanding the
legality of his dismissal. Jurisprudence on such financial assistance and separation pay then equally
apply to this case. The Court has ruled, time and again, that financial assistance, or whatever name
it is called, as a measure of social justice is allowed only in instances where the employee is validly
dismissed for causes other than serious misconduct or those reflecting on his moral character. 34 We
explained in Philippine Long Distance Telephone Company v. NLRC35:
[S]eparation pay shall be allowed as a measure of social justice only in those instances where the
employee is validly dismissed for causes other than serious misconduct or those reflecting on his
moral character. Where the reason for the valid dismissal is, for example, habitual intoxication or
an offense involving moral turpitude, like theft or illicit sexual relations with a fellow worker, the
employer may not be required to give the dismissed employee separation pay, or financial
assistance, or whatever other name it is called, on the ground of social justice.

A contrary
rule would, as the petitioner correctly argues, have the effect, of rewarding rather than punishing
the erring employee for his offense. And we do not agree that the punishment is his dismissal only

177
and that the separation pay has nothing to do with the wrong he has committed. Of course it has.
Indeed, if the employee who steals from the company is granted separation pay even as he is validly
dismissed, it is not unlikely that he will commit a similar offense in his next employment because he
thinks he can expect a like leniency if he is again found out. This kind of misplaced compassion is
not going to do labor in general any good as it will encourage the infiltration of its ranks by those
who do not deserve the protection and concern of the Constitution.36 (Emphasis ours)

Clearly, considering that Daabay was dismissed on the grounds of serious misconduct, breach of
trust and loss of confidence, the award based on equity was unwarranted.

(G.R. 179001, August 28, 2013)

MZR INDUSTRIES, MARILOU R. QUIROZ AND LEA TIMBAL


VS.
MAJEN COLAMBOT

Majen Colambot (Colambot) was a messenger in MZR Industries. Despite written warnings for
habitual tardiness, negligence, and violations of office policies and insubordination, Colambot failed
to mend his ways.

On October 25, 2004 Colambot was given a notice of suspension for insubordination and
negligence. He was suspended again from November 26, 2004 until December 6, 2004 thru a notice
of suspension dated November 25, 2004. Colambot did not report back to work on December 7,
2004. He filed a complaint for illegal dismissal.

Colambot claimed that he did not report back to work because after affixing his signature in the
notice of suspension, Quiroz (owner of MZR Industries) told him that his employment was already
terminated effective December 7, 2004.

In illegal dismissal cases, the employer bears the burden of proving that the termination was for a
valid or authorized cause. However, before the employer must bear the burden of proving that
the dismissal was legal, the employee must first establish by substantial evidence the fact of
his dismissal from service. If there is no dismissal, then there can be no question as to the legality
or illegality thereof.

Colambot did not present any evidence to show that he was indeed dismissed from work or was
prevented from returning to his work. His unsubstantiated allegation of having been verbally
terminated from his work is self serving and has no probative value. There was no dismissal. In the
absence of any showing of an overt or positive act proving that petitioners had dismissed
respondent, the latter's claim of illegal dismissal cannot be sustained.

178
The letter dated November 25, 2004 with subject “Suspension due to Insubordination” was a notice
of suspension, not a notice of termination. While the same appeared to contain a warning of
termination should Colambot fail to improve his behavior, there was also a specific instruction for
him to report back to work, on December 7, 2004, upon serving his suspension. There were no
wordings whatsoever implying actual or constructive dismissal.

In Abad v. Roselle Cinema, SC ruled that the substantial evidence proffered by the employer that it
had not terminated the employee should not be ignored on the pretext that the employee would not
have filed the complaint for illegal dismissal if he had not really been dismissed.

Colambot did not commit abandonment of work. Aside from mere absence or failure to report for
work without valid or justifiable reason, there must be a clear intention on to sever the employer-
employee relationship and some overt acts from which an employee may be deduced as having no
more intention to work. No evidence was presented to show Colambot’s intent to abandon his
work.

These circumstances, taken together, the lack of evidence of dismissal and the lack of intent on
the part of the respondent to abandon his work, the remedy is reinstatement but without
backwages. However, considering that reinstatement is no longer applicable due to the
strained relationship between the parties and that Colambot already found another
employment, each party must bear his or her own loss, thus, placing them on equal footing.

Verily, in a case where the employee's failure to work was occasioned neither by his
abandonment nor by a termination, the burden of economic loss is not rightfully shifted to
the employer; each party must bear his own loss.

(G.R. No. 200222, August 28, 2013)

INTEGRATED MICROELECTRONICS INC.,


VS.
ADONIS A. PIONILLA


Pionilla was a production worker at IMI. On May 5, 2005, Pionilla received a notice from IMI
requiring him to explain an incident where he was seen escorting a lady to ride the company shuttle
bus who was wearing a company ID even though she was just a job applicant.

Pionilla admitted that he lent his ID to the lady who is his relative to save on their transportation
expenses. 

A Conscience Committee (committee) was subsequently formed to investigate the
matter. During the committee hearing, Pionilla admitted that at the time of the incident, he had two
IDs in his name as he lost his original ID in November 2004 but was able to secure a temporary ID
later. Pionilla and his relative were about both holding separate IDs, both in his name.

179
Based on the foregoing, IMI found Pionilla guilty of violating Article 6.12 of the Company Rules and
Regulations (CRR) which prohibits the lending of one’s ID since the same is considered a breach of
its security rules and carries the penalty of dismissal. On August 17, 2005, Pionilla received a letter
dated August 16, 2005 informing him of his dismissal from service. Three days after, he filed a
complaint for illegal dismissal with damages against IMI. 


The Labor Arbiter (LA) rendered a Decision8 finding Pionilla to have been illegally dismissed by IMI
and, awarded reinstatement and full backwages. The NLRC, found Pionilla’s dismissal to be valid.


CA ruled that Pionilla was illegally dismissed. It found that while IMI’s regulations on
company IDs were reasonable, the penalty of dismissal was too harsh and not commensurate to the
misdeed committed, especially considering his nine years of unblemished service. It also stated that
the while the right of the employer to discipline is beyond question, it, nevertheless, remains
subject to reasonable regulation.

SC affirmed CA’s ruling including the award reinstatement and full backwages. IMI filed a motion
for recon contending that the award would not only be excessive and unfair, but would be contrary
to existing principles of law and jurisprudence.

Held:

As a general rule, an illegally dismissed employee is entitled to reinstatement (or
separation pay, if reinstatement is not viable) and payment of full backwages. However, the
Court has carved out an exception to the rule and ordered the reinstatement of the employee
without backwages on account of the following: (a) the fact that dismissal of the employee
would be too harsh of a penalty; and (b) that the employer was in good faith in terminating
the employee.

In the case of Cruz v. Minister of Labor and Employment24 the Court ruled as follows:
The Court is convinced that petitioner's guilt was substantially established. Nevertheless, we agree
with respondent Minister's order of reinstating petitioner without backwages instead of dismissal
which may be too drastic. Denial of backwages would sufficiently penalize her for her
infractions. The bank officials acted in good faith. They should be exempt from the burden of
paying backwages. The good faith of the employer, when clear under the circumstances, may
preclude or diminish recovery of backwages. Only employees discriminately dismissed are
entitled to backpay.

Likewise, in the case of Itogon-Suyoc Mines, Inc. v. National Labor Relations Commission, the Court
pronounced that “the ends of social and compassionate justice would therefore be served if private
respondent is reinstated but without backwages in view of petitioner's good faith.” 

In this case,
the Court observes that: (a) the penalty of dismissal was too harsh of a penalty to be imposed
against Pionilla for his infractions; and (b) IMI was in good faith when it dismissed Pionilla as his
dereliction of its policy on ID usage was honestly perceived to be a threat to the company’s security.
The Court found it proper to direct the deletion of the award of backwages notwithstanding the
illegality of his dismissal.

180
[G.R. Nos. 171594-96. September 18, 2013.]

ASIA BREWERY, INC., PETITIONER,


VS.
TUNAY NA PAGKAKAISA NG MGA MANGGAGAWA SA ASIA (TPMA), RESPONDENT.

FACTS:

This Petition for Review on Certiorari assails the Court of Appeal's (CA) October 6, 2005 Decision
and the February 17, 2006 Amended Decision which modified the January 19, 2004 Decision of the
Secretary of Labor in OS-AJ-0042-2003.

Respondent union TPMA has been negotiating CBA for the years 2003-2006. After about 18
sessions, the parties were still unable to reconcile their differences, particularly on wages and other
economic benefits. On October 21, 2003, the respondent union declared deadlock and then filed a
notice of strike with NCMB. Strike vote was conducted. 768 of 840 members voted in favor of
holding a strike.

Petitioner Asia Brewery Inc petitioned the Secretary of DOLE to assume jurisdiction over the labor
dispute. In the answer, respondent union opposed reasoning that the business of Asia Brewery in
no indispensable to the national interest. However, on December 19, 2003, the DOLE ordered to
assume jurisdiction over the dispute.

On January 19, 2004, [respondent union] filed another petition for certiorari with [the Court of
Appeals], docketed as CA-G.R. SP No. 81639, imputing bad faith and grave abuse of discretion to the
Secretary of Labor. [Respondent union] prayed therein for the nullification of the order of
assumption of jurisdiction and the declaration that [petitioner corporation] is not an industry
indispensable to the national interest.

In the meantime, the Secretary rendered a decision and granted arbitral awards: (1) Wage
Increases; (2) Health Care Premium to be shouldered by Asia Brewery, Inc. - Php1,300

In subsequent Petition for Certiorari before the CA, the latter affirmed the decision of the Secretary
with modifications: In modifying the arbitral award of the Secretary of Labor, the CA ruled that: (1)
The effectivity of the CBA should be August 1, 2003 because this is the date agreed upon by the
parties and not January 1, 2004 as decreed by the Secretary of Labor; (2) The computation of wage
increase should be remanded to the Secretary of Labor because the computation was based on
petitioner corporation's unaudited financial statements, which have no probative value pursuant to
the ruling in Restaurante Las Conchas v. Llego, and was done in contravention of DOLE Advisory No.

181
1, Series of 2004, which contained the guidelines in resolving bargaining deadlocks; and (3) The
health benefits should be P1,390.00 per covered employee because petitioner corporation had
already agreed to this amount and the same cannot be altered or reduced by the Secretary of Labor.

Issues:
II.Whether the CA erred when it remanded to the Secretary of Labor the issue on wage increase.

Held:

The remand of this case to the Secretary of Labor as to the issue of wage increase was proper. In cases
of compulsory arbitration before the Secretary of Labor pursuant to Article 263 (g) of the Labor
Code, the financial statements of the employer must be properly audited by an external and
independent auditor in order to be admissible in evidence for purposes of determining the proper
wage award.
[Termination of Employment, the SC cited the rule on proof necessary to establish losses in
cases of business closure or retrenchment; Same rule applies also to to the determination of the
proper level of wage award in cases where the Secretary of Labor assumes jurisdiction in a labor
dispute pursuant to Article 263 (g) of the Labor Code]
In Restaurante Las Conchas v. Llego, several employees filed a case for illegal dismissal after the
employer closed its restaurant business. The employer sought to justify the closure through
unaudited financial statements showing the alleged losses of the business. We ruled that such
financial statements are mere self-serving declarations and inadmissible in evidence even if the
employees did not object to their presentation before the Labor Arbiter. Similarly, in Uichico v.
National Labor Relations Commission, the services of several employees were terminated on the
ground of retrenchment due to alleged serious business losses suffered by the employer. We ruled
that by submitting unaudited financial statements, the employer failed to prove the alleged
business losses, viz.:
. . . It is true that administrative and quasi-judicial bodies like the NLRC are not bound by the
technical rules of procedure in the adjudication of cases. However, this procedural rule should not
be construed as a license to disregard certain fundamental evidentiary rules. While the rules of
evidence prevailing in the courts of law or equity are not controlling in proceedings before the
NLRC, the evidence presented before it must at least have a modicum of admissibility for it to be
given some probative value. The Statement of Profit and Losses submitted by Crispa, Inc. to prove
its alleged losses, without the accompanying signature of a certified public accountant or
audited by an independent auditor, are nothing but self-serving documents which ought to
be treated as a mere scrap of paper devoid of any probative value. For sure, this is not the kind
of sufficient and convincing evidence necessary to discharge the burden of proof required of
petitioners to establish the alleged losses suffered by Crispa, Inc. in the years immediately
preceding 1990 that would justify the retrenchment of respondent employees. . .
While the above-cited cases involve proof necessary to establish losses in cases of business closure
or retrenchment, we see no reason why this rule should not equally apply to the determination of
the proper level of wage award in cases where the Secretary of Labor assumes jurisdiction in a
labor dispute pursuant to Article 263 (g) of the Labor Code.

182
15 SUSPENSION OF BUSINESS OPERATION

[G.R. No. 174300. December 5, 2012.]

MINDANAO TERMINAL & BROKERAGE SERVICE ET. AL. , PETITIONERS


VS
NAGKAHIUSANG MAMUMUO SA MINTERBRO-SOURTHERN PHILS FED. OF LABOR
, RESPONDENTS

Facts:
Minterbro decided to rehabilitate the pier on August 1, 1997 and, on the same day, sent a letter to
the Department of Labor and Employment (DOLE) to inform DOLE of Minterbro's intention to
temporarily suspend arrastre and stevedoring operations. Minterbro alleged that, despite the
condition of the pier, it was able to service 16 vessels from January 1997 to April 13, 1997 and it
was ready and awaiting vessels to dock at the pier from April 14, 1997 to July 31, 1997 during
which Minterbro's office, motor pool, and field personnel continued operations.

On November 4, 1997, respondent Nagkahiusang Mamumuo sa Minterbro-Southern Philippines


Federation of Labor composed of respondents Manuel Abellana, et al., employees of Minterbro
working on a rotation basis and employed for arrastre and stevedoring work depending on the
actual requirements of the vessels serviced by Minterbro, filed a complaint for payment of
separation pay against Minterbro and De Castro in the Regional Arbitration Branch No. XI at Davao
City of the National Labor Relations Commission (NLRC)

On June 15, 1998, the Labor Arbiter rendered a Decision dismissing the complaint and declaring the
complainants to have abandoned their work.

However, the assailed Decision is Modified by NLRC. Respondents are ordered to pay complainants
their separation benefits.

Respondent filed Petition for Certiorari before the CA. In a Decision dated April 21, 2006, the Court
of Appeals dismissed the petition. It ruled that the seasonal nature of the services rendered by the
members of the union did not negate their status as regular employees and that the temporary
suspension of Minterbro's operations should be reckoned from April 14, 1997, the day no more
vessel was serviced at Minterbro's pier after MV Bosco Polar was serviced at the said pier on April
11 to 13, 1997. Thus, pursuant to Article 286 of the Labor Code and its application in Sebuguero v.
National Labor Relations Commission, 29 the NLRC correctly ordered Minterbro and De Castro to
pay the union members their separation benefits as their temporary lay-off exceeded six months.

183
Issue:

WON Court of Appeals erred in ruling that Sebuguero applies to this case.

Held: Petition is denied.

The Court of Appeals did not err in ruling that Sebuguero applies to this case, the consequences
arrived at in Sebuguero also apply. Lay-off is essentially retrenchment and under Article 283 of the
Labor Code a retrenched employee is entitled to separation pay equivalent to one (1) month salary
or one-half (1/2) month salary per year of service, whichever is higher.

When petitioners failed to make work available to the union members for a period of more than six
months starting April 14, 1997 by failing to call the attention of Del Monte on the latter's
obligations under the Contract of Use of Pier and to undertake a timely rehabilitation of the pier,
they are deemed to have constructively dismissed the union members. As this Court held in Valdez
v. National Labor Relations Commission:
Under Article 286 of the Labor Code, the bona fide suspension of the operation of a business or
undertaking for a period not exceeding six months shall not terminate employment. Consequently,
when the bona fide suspension of the operation of a business or undertaking exceeds six months,
then the employment of the employee shall be deemed terminated. By the same token and applying
said rule by analogy, if the employee was forced to remain without work or assignment for a
period exceeding six months, then he is in effect constructively dismissed. (Citation omitted.)
In Sebuguero, http://cdasiaonline.com/search/show_article/56499?search=gr%3A+(174300*)the
Court ruled on a case regarding layoff or temporary retrenchment, which subsequently resulted to
the separation from employment of the concerned employee as it lasted for more than six months,
as follows:
Article 283 of the Labor Code which covers retrenchment, reads as follows: ASTcEa
Art. 283.Closure of establishment and reduction of personnel. — The employer may also terminate
the employment of any employee due to the installation of labor saving devices, redundancy,
retrenchment to prevent losses or the closing or cessation of operation of the establishment or
undertaking unless the closing is for the purpose of circumventing the provisions of this Title, by
servicing a written notice on the workers and the Ministry of Labor and Employment at least one
(1) month before the intended date thereof. In case of termination due to the installation of labor
saving devices or redundancy, the worker affected thereby shall be entitled to a separation pay
equivalent to at least his one (1) month pay or to at least one (1) month pay for every year of
service, whichever is higher. In case of retrenchment to prevent losses and in cases of closure or
cessation of operations of establishment or undertaking not due to serious business losses or
financial reverses, the separation pay shall be equivalent to one (1) month pay or at least one-half
(1/2) month pay for every year of service, whichever is higher. A fraction of at least six (6) months
shall be considered one (1) whole year.
This provision, however, speaks of a permanent retrenchment as opposed to a temporary lay-off as
is the case here. There is no specific provision of law which treats of a temporary retrenchment or

184
lay-off and provides for the requisites in effecting it or a period or duration therefor. These
employees cannot forever be temporarily laid-off. To remedy this situation or fill the hiatus, Article
286 may be applied but only by analogy to set a specific period that employees may remain
temporarily laid-off or in floating status. 13 Six months is the period set by law that the operation of
a business or undertaking may be suspended thereby suspending the employment of the employees
concerned. The temporary lay-off wherein the employees likewise cease to work should also not
last longer than six months. After six months, the employees should either be recalled to work or
permanently retrenched following the requirements of the law, and that failing to comply with this
would be tantamount to dismissing the employees and the employer would thus be liable for such
dismissal.(Citation omitted.)

LEOPARD SECURITY AND INVESTIGATION AGENCY


VS
QUITOY

FACTS
RESPONDENTS Tomas Quitoy, Raul Sabang, and Diego Morales were hired as security guards by
petitioner Leopard Security and Investigation Agency (LSIA). They were assigned to the different
branches of its only client in Cebu City, Union Bank of the Philippines (Union Bank).
On April 1, 2005, Union Bank served a notice to LSIA, terminating the parties’ security service
contract effective at the end of the business hours of April 30, 2005. On April 29, 2005, LSIA
informed respondents of the termination. With respondents’ complaint already filed with the
Regional Arbitration Branch VII of the National Labor Relations Commission (NLRC) in Cebu City,
LSIA sent on May 10, 2005 a notice requiring respondents to report for work to its Mandaluyong
City office.
On May 3, 2005, respondents filed a complaint for illegal dismissal, money claims, damages and
attorney’s fees.

ISSUE
Was there illegal dismissal?

HELD
No.
Applying Article 286 of the Labor Code of the Philippines by analogy, the Supreme Court has
repeatedly recognized that security guards may be temporarily sidelined by their security agency
as their assignments primarily depend on the contracts entered into by the latter with third parties.
Temporary “off-detail” or “floating status” is the period of time when security guards are in
between assignments or when they are made to wait after being relieved from a previous post until
they are transferred to a new one. It takes place when, as here, the security agency’s clients decide
not to renew their contracts with the agency, resulting in a situation where the available posts

185
under its existing contracts are less than the number of guards in its roster.
For as long as such temporary inactivity does not continue for a period exceeding six months, it has
been ruled that placing an employee on temporary “off-detail” or “floating status” is not equivalent
to dismissal.
The respondents were informed on April 29, 2005 that they were going to be relieved from duty as
a consequence of the April 30, 2005 expiration of the security service contract between Union Bank
and LSIA. While respondents lost no time in immediately filing their complaint on May 3, 2005, the
record equally shows that they were directed by LSIA to report for work at its Mandaluyong City
office on May 10, 2005 or a mere 10 days from the time the former were effectively sidelined.
Considering that a security guard is only considered illegally dismissed from service when he is
sidelined from duty for a period exceeding six months, we find that the Court of Appeals (CA)
correctly upheld the National Labor Relations Commission’s (NLRC) ruling that respondents were
not illegally dismissed by LSIA. The ruling is binding on respondents who did not appeal either the
decision rendered by the NLRC or the CA in line with the entrenched procedural rule in this
jurisdiction that a party who did not appeal cannot assign such errors as are designed to have the
judgment modified.

16 DISEASE AS GROUND FOR TERMINATION

17 OTHER CAUSES OF SEVERANCE OF EMPLOYMENT RELATION

18 PRESCRIPTION OF CLAIMS

19JURISDICTION OF THE LABOR ARBITER

PORTILLO
VS
RUDOLF LIETZ

In a letter agreement, signed by Rudolf Lietz and conformed to by Portillo, the latter (Portillo) was
hired by the former under the conditions that Portillo “will not engage in any other gainful
employment by [her]self or with any other company either directly or indirectly without written
consent of [Lietz Inc.]” and “a breach of which will render [Portillo] liable to [Lietz Inc.] for
liquidated damages.”
On her tenth (10th) year of service with Lietz Inc., Portillo was promoted to Sales Representative. In

186
this regard, Portillo signed another letter agreement containing a "Goodwill Clause:" It remains
understood and you agreed that, on the termination of your employment by act of either you or
[Lietz Inc.], and for a period of three (3) years thereafter, you shall not engage directly or indirectly
as employee, manager, proprietor, or solicitor for yourself or others in a similar or competitive
business or the same character of work which you were employed by [Lietz Inc.] to do and perform.
Should you breach this good will clause of this Contract, you shall pay [Lietz Inc.] as liquidated
damages the amount of 100% of your gross compensation over the last 12 months, it being agreed
that this sum is reasonable and just.
Three (3) years thereafter Portillo resigned from Lietz Inc. During her exit interview, Portillo
declared that she intended to engage in business—a rice dealership, selling rice in wholesale. On 15
June 2005, Lietz Inc. accepted Portillo’s resignation and reminded her of the "Goodwill Clause" in
the last letter agreement she had signed. Subsequently, Lietz Inc. learned that Portillo had been
hired by Ed Keller Philippines, Limited to head its Pharma Raw Material Department. Ed Keller
Limited is purportedly a direct competitor of Lietz Inc. Meanwhile, Portillo’s demands from Lietz
Inc. for the payment of her remaining salaries and commissions went unheeded.
On 14 September 2005, Portillo filed a complaint with the NLRC for non-payment of 1½ months’
salary, two (2) months’ commission, 13th month pay, plus moral, exemplary and actual damages
and attorney’s fees. In its position paper, Lietz Inc. admitted liability for Portillo’s money claims in
the total amount of P110,662.16. However, Lietz Inc. raised the defense of legal compensation:
Portillo’s money claims should be offset against her liability to Lietz Inc. for liquidated damages in
the amount of ₱869,633.097 for Portillo’s alleged breach of the "Goodwill Clause" in the
employment contract when she became employed with Ed Keller Philippines, Limited.
On 25 May 2007, the Labor Arbiter granted Portillo’s complaint, ordering Lietz, Inc. to pay Portillo
the amount of Php110,662.16, representing her salary and commissions, including 13th month
pay.8 On appeal by respondents Lietz Inc., the NLRC affirmed the ruling of the Labor Arbiter. The
motion for reconsideration was denied by NLRC.
Lietz Inc. filed a petition for certiorari before the Court of Appeals, alleging grave abuse of
discretion in the labor tribunals’ rulings. The CA initially affirmed the labor tribunals, but on motion
for reconsideration, modified its previous decision. While upholding the monetary award in favor of
Portillo in the aggregate sum of ₱110,662.16, the CA allowed legal compensation or set-off of such
award of monetary claims by her liability to Lietz Inc. for liquidated damages arising from her
violation of the "Goodwill Clause" in her employment contract with them. Portillo’s motion for
reconsideration was denied.
Hence, this petition for certiorari before the SC.

ISSUE:

Whether Portillo’s money claims for unpaid salaries may be offset against Lietz Inc.’s claim for
liquidated damages.

RULING:
Paragraph 4 of Article 217 of the Labor Code appears to have caused the reliance by the CA on the
"causal connection between Portillo’s monetary claims against respondents and the latter’s claim

187
from liquidated damages against the former."
Art. 217. Jurisdiction of Labor Arbiters and the Commission. – (a) Except as otherwise provided
under this code, the Arbiters shall have original and exclusive jurisdiction to hear and decide,
within thirty (30) calendar days after the submission of the case by the parties for decision without
extension, even in the absence of stenographic notes, the following case involving all workers,
whether agricultural or nonagricultural:
xxxx
4. Claims for actual, moral, exemplary and other forms of damages arising from the employer-
employee relations; (Underscoring supplied)

Evidently, the CA is convinced that the claim for liquidated damages emanates from the "Goodwill
Clause of the employment contract and, therefore, is a claim for damages arising from the
employer-employee relations." As early as Singapore Airlines Limited v. Paño, we established that
not all disputes between an employer and his employee(s) fall within the jurisdiction of the labor
tribunals. We differentiated between abandonment per se and the manner and consequent effects
of such abandonment and ruled that the first, is a labor case, while the second, is a civil law case.

Stated differently, Lietz seeks protection under the civil laws and claims no benefits under the
Labor Code. The primary relief sought is for liquidated damages for breach of a contractual
obligation. The other items demanded are not labor benefits demanded by workers generally taken
cognizance of in labor disputes, such as payment of wages, overtime compensation or separation
pay. The items claimed are the natural consequences flowing from breach of an obligation,
intrinsically a civil dispute.
Subsequent rulings amplified the teaching in Singapore Airlines. The reasonable causal connection
rule was discussed. Thus, in San Miguel Corporation v. National Labor Relations Commission, we
held:
xxx The Court, therefore, believes and so holds that the "money claims of workers" referred to in
paragraph 3 of Article 217 embraces money claims which arise out of or in connection with the
employer-employee relationship, or some aspect or incident of such relationship. Put a little
differently, that money claims of workers which now fall within the original and exclusive
jurisdiction of Labor Arbiters are those money claims which have some reasonable causal
connection with the employer employee relationship.
We thereafter ruled that the "reasonable causal connection with the employer-employee
relationship" is a requirement not only in employees’ money claims against the employer but is,
likewise, a condition when the claimant is the employer. In Dai-Chi Electronics Manufacturing
Corporation v. Villarama, Jr., which reiterated the San Miguel ruling and allied jurisprudence, we
pronounced that a non-compete clause, as in the "Goodwill Clause" referred to in the present case,
with a stipulation that a violation thereof makes the employee liable to his former employer for
liquidated damages, refers to postemployment relations of the parties. We iterated that Article 217,
paragraph 4 does not automatically cover all disputes between an employer and its employee(s).
We noted that the cause of action was within the realm of Civil Law, thus, jurisdiction over the
controversy belongs to the regular courts. At bottom, we considered that the stipulation referred to
postemployment relations of the parties.

188
That the "Goodwill Clause" in this case is likewise a postemployment issue should brook no
argument. There is no dispute as to the cessation of Portillo’s employment with Lietz Inc. She
simply claims her unpaid salaries and commissions, which Lietz Inc. does not contest. At that
juncture, Portillo was no longer an employee of Lietz Inc. The "Goodwill Clause" or the "Non-
Compete Clause" is a contractual undertaking effective after the cessation of the employment
relationship between the parties. In accordance with jurisprudence, breach of the undertaking is a
civil law dispute, not a labor law case.
It is clear, therefore, that while Portillo’s claim for unpaid salaries is a money claim that arises out of
or in connection with an employer-employee relationship, Lietz Inc.’s claim against Portillo for
violation of the goodwill clause is a money claim based on an act done after the cessation of the
employment relationship. And, while the jurisdiction over Portillo’s claim is vested in the labor
arbiter, the jurisdiction over Lietz Inc.’s claim rests on the regular courts.
In the case at bar, the difference in the nature of the credits that one has against the other,
conversely, the nature of the debt one owes another, which difference in turn results in the
difference of the forum where the different credits can be enforced, prevents the application of
compensation. Simply, the labor tribunal in an employee’s claim for unpaid wages is without
authority to allow the compensation of such claims against the post-employment claim of the
former employer for breach of a post-employment condition. The labor tribunal does not have
jurisdiction over the civil case of breach of contract.
Indeed, the application of compensation in this case is effectively barred by Article 113 of the Labor
Code which prohibits wage deductions except in three circumstances: ART. 113. Wage Deduction. –
No employer, in his own behalf or in behalf of any person, shall make any deduction from wages of
his employees, except:
(a) In cases where the worker is insured with his consent by the employer, and the deduction is to
recompense the employer for the amount paid by him as premium on the insurance;
(b) For union dues, in cases where the right of the worker or his union to check-off has been
recognized by the employer or authorized in writing by the individual worker concerned; and
(c) In cases where the employer is authorized by law or regulations issued by the Secretary of
Labor.

[G.R. No. 197309. October 10, 2012.]

ACE NAVIGATION CO., INC., VELA INTERNATIONAL MARINE LTD., AND/OR RODOLFO
PAMINTUAN, PETITIONERS,
VS.
TEODORICO FERNANDEZ, ASSISTED BY GLENITA FERNANDEZ, RESPONDENT.

Facts:
On October 9, 2008, seaman Teodorico Fernandez filed with the National Labor Relations
Commission (NLRC) a complaint for disability benefits, with prayer for moral and exemplary
damages, plus attorney's fees, against Ace Navigation Co., Inc., Vela International Marine Ltd.,
and/or Rodolfo Pamintuan (petitioners).

189
The petitioners moved to dismiss the complaint, contending that the labor arbiter had no
jurisdiction over the dispute. They argued that exclusive original jurisdiction is with the voluntary
arbitrator or panel of voluntary arbitrators, pursuant to Section 29 of the POEA Standard
Employment Contract (POEA-SEC), since the parties are covered by the AMOSUP-TCC or AMOSUP-
VELA (as later cited by the petitioners) collective bargaining agreement. Under Section 14 of the
CBA, a dispute between a seafarer and the company shall be settled through the grievance
machinery and mandatory voluntary arbitration.
Fernandez opposed the motion. He argued that inasmuch as his complaint involves a money claim,
original and exclusive jurisdiction over the case is vested with the labor arbiter.

Issue:
Who has the original and exclusive jurisdiction over Fernandez's disability claim — the labor
arbiter under Section 10 of R.A. No. 8042, as amended, or the voluntary arbitration mechanism as
prescribed in the parties' CBA and the POEA-SEC?

Held:
The answer lies in the State's labor relations policy laid down in the Constitution and fleshed out in
the enabling statute, the Labor Code. Section 3, Article XIII (on Social Justice and Human Rights) of
the Constitution declares:
xxx xxx xxx

The State shall promote the principle of shared responsibility between workers and employers and
the preferential use of voluntary modes in settling disputes, including conciliation, and shall enforce
their mutual compliance therewith to foster industrial peace.

Article 260 of the Labor Code (Grievance machinery and voluntary arbitration) states:
The parties to a Collective Bargaining Agreement shall include therein provisions that will ensure
the mutual observance of its terms and conditions. They shall establish a machinery for the
adjustment and resolution of grievances arising from the interpretation or implementation of their
Collective Bargaining Agreement and those arising from the interpretation or enforcement of
company personnel policies.

Article 261 of the Labor Code (Jurisdiction of Voluntary Arbitrators or panel of Voluntary
Arbitrators), on the other hand, reads in part:
The Voluntary Arbitrator or panel of Voluntary Arbitrators shall have original and exclusive
jurisdiction to hear and decide all unresolved grievances arising from the interpretation or
implementation of the Collective Bargaining Agreement and those arising from the interpretation
or enforcement of company personnel policies[.]

Article 262 of the Labor Code (Jurisdiction over other labor disputes) declares:
The Voluntary Arbitrator or panel of Voluntary Arbitrators, upon agreement of the parties, shall
also hear and decide all other labor disputes including unfair labor practices and bargaining
deadlocks.

190
Further, the POEA-SEC, which governs the employment of Filipino seafarers, provides in its Section
29 on Dispute Settlement Procedures:

In cases of claims and disputes arising from this employment, the parties covered by a
collective bargaining agreement shall submit the claim or dispute to the original and
exclusive jurisdiction of the voluntary arbitrator or panel of voluntary arbitrators. If the
parties are not covered by a collective bargaining agreement, the parties may at their option submit
the claim or dispute to either the original and exclusive jurisdiction of the National Labor Relations
Commission (NLRC), pursuant to Republic Act (RA) 8042 otherwise known as the Migrant Workers
and Overseas Filipinos Act of 1995 or to the original and exclusive jurisdiction of the voluntary
arbitrator or panel of voluntary arbitrators. If there is no provision as to the voluntary arbitrators
to be appointed by the parties, the same shall be appointed from the accredited voluntary
arbitrators of the National Conciliation and Mediation Board of the Department of Labor and
Employment. [emphasis ours]

Under the above-quoted constitutional and legal provisions, the voluntary arbitrator or panel of
voluntary arbitrators has original and exclusive jurisdiction over Fernandez's disability claim.
There is no dispute that the claim arose out of Fernandez's employment with the petitioners and
that their relationship is covered by a CBA — the AMOSUP/TCC or the AMOSUP-VELA CBA. The
CBA provides for a grievance procedure for the resolution of grievances or disputes which occur
during the employment relationship and, like the grievance machinery created under Article 261 of
the Labor Code, it is a two-tiered mechanism, with voluntary arbitration as the last step.
Contrary to the CA's reading of the CBA's Article 14, there is unequivocal or unmistakable language
in the agreement which mandatorily requires the parties to submit to the grievance procedure any
dispute or cause of action they may have against each other. The relevant provisions of the CBA
state:

14.6 Any Dispute, grievance, or misunderstanding concerning any ruling, practice, wages or
working conditions in the COMPANY or any breach of the Contract of Employment, or any
dispute arising from the meaning or application of the provisions of this Agreement or a
claim of violation thereof or any complaint or cause of action that any such Seaman may
have against the COMPANY, as well as complaints which the COMPANY may have against
such Seaman shall be brought to the attention of the GRIEVANCE RESOLUTION COMMITTEE
before either party takes any action, legal or otherwise. Bringing such a dispute to the
Grievance Resolution Committee shall be unwaivable prerequisite or condition precedent
for bringing any action, legal or otherwise, in any forum and the failure to so refer the
dispute shall bar any and all legal or other actions.

14.7a) If by reason of the nature of the Dispute, the parties are unable to amicably settle the
dispute, either party may refer the case to a MANDATORY ARBITRATION COMMITTEE. The
MANDATORY ARBITRATION COMMITTEE shall consist of one representative to be designated by
the UNION, and one representative to be designated by the COMPANY and a third member who
shall act as Chairman and shall be nominated by mutual choice of the parties. . . .

191
h) Referral of all unresolved disputes from the Grievance Resolution Committee to the
Mandatory Arbitration Committee shall be unwaivable prerequisite or condition precedent
for bringing any action, claim, or cause of action, legal or otherwise, before any court,
tribunal, or panel in any jurisdiction. The failure by a party or seaman to so refer and avail
oneself to the dispute resolution mechanism contained in this action shall bar any legal or
other action. All parties expressly agree that the orderly resolution of all claims in the
prescribed manner served the interests of reaching settlements or claims in an orderly and
uniform manner, as well as preserving peaceful and harmonious labor relations between
seaman, the Union, and the Company. (emphases ours)
What might have caused the CA to miss the clear intent of the parties in prescribing a grievance
procedure in their CBA is, as the petitioners' have intimated, the use of the auxiliary verb "may" in
Article 14.7 (a) of the CBA which, to reiterate, provides that "[i]f by reason of the nature of the
Dispute, the parties are unable to amicably settle the dispute, either party may refer the case
to a MANDATORY ARBITRATION COMMITTEE."

We find this a strained interpretation of the CBA provision. The CA read the provision separately, or
in isolation of the other sections of Article 14, especially 14.7 (h), which, in clear, explicit language,
states that the "referral of all unresolved disputes from the Grievance Resolution Committee
to the Mandatory Arbitration Committee shall be unwaivable prerequisite or condition
precedent for bringing any action, claim, or cause of action, legal or otherwise, before any
court, tribunal, or panel in any jurisdiction" and that the failure by a party or seaman to so
refer the dispute to the prescribed dispute resolution mechanism shall bar any legal or other
action.

Consistent with this finding, Fernandez's contention — that his complaint for disability benefits is a
money claim that falls within the original and exclusive jurisdiction of the labor arbiter under
Section 10 of R.A. No. 8042 — is untenable. We likewise reject his argument that he never referred
his claim to the grievance machinery (so that no unresolved grievance exists as required under
Article 261 of the Labor Code), and that the parties to the case are not the union and the
employer. Needless to state, no such distinction exists in the parties' CBA and the POEA-SEC. I
It bears stressing at this point that we are upholding the jurisdiction of the voluntary arbitrator or
panel of voluntary arbitrators over the present dispute, not only because of the clear language of
the parties' CBA on the matter; more importantly, we so uphold the voluntary arbitrator's
jurisdiction, in recognition of the State's express preference for voluntary modes of dispute
settlement, such as conciliation and voluntary arbitration as expressed in the Constitution, the law
and the rules.

192
20 . 2011 NLRC RULES OF PROCEDURE

[G.R. No. 175251. May 30, 2011.]

RODOLFO LUNA, PETITIONER,


VS.
ALLADO CONSTRUCTION CO., INC., AND/OR RAMON ALLADO,RESPONDENT.

Facts:

Allado Construction Co., Inc. is a juridical entity engaged in the construction business; respondent
Ramon Allado is the President of the said corporation.
Petitioner filed a complaint before the Executive Labor Arbiter Arturo Gamolo, RAB Branch XI,
Davao City, alleging that he was an employee of herein respondents, having been a part of
respondents' construction pool of personnel. He had continuously rendered services as a
warehouseman and a timekeeper in every construction project undertaken by respondents.
Sometime, while at [respondents'] construction site in Maasim, Sarangani Province, he was given a
travel order to proceed to respondents' main office in Davao City for reassignment. Upon arrival at
the office, he was told by one MarilouMatilano, personnel manager of respondents, to sign several
sets of "Contract of Project Employment". He refused to sign the said contracts. Because of his
refusal, he was not given a reassignment or any other work. These incidents prompted him to file
the complaint.
Respondents, on the other hand, alleged that on November 29, 2001, petitioner applied for a leave
of absence until December 6, 2001, which was granted. Upon expiration of his leave, petitioner was
advised to report to the company's project in Kablacan, Sarangani Province. However, he refused to
report to his new assignment and claimed instead that he had been dismissed illegally.
Finding that petitioner should be deemed to have resigned, the Labor Arbiter dismissed petitioner's
complaint for illegal dismissal against respondents, but ordered the latter to pay the former the
amount of P18,000.00 by way of financial assistance.

Only respondents interposed an appeal with the National Labor Relations Commission (NLRC),
purely for the purpose of questioning the validity of the grant of financial assistance made by the
Labor Arbiter.

Issue:
Whether the award of financial assistance was proper

Held:

It was proper.
Section 4 (c), Rule VI of the 2002 Rules of Procedure of the NLRC, which was in effect at the time
respondents appealed the Labor Arbiter's decision, expressly provided that, on appeal, the NLRC
shall limit itself only to the specific issues that were elevated for review, to wit:
RULE VI

193
Appeals
Section 4.Requisites for Perfection of Appeal. . .
xxxxxxxxx

(c)Subject to the provisions of Article 218, once the appeal is perfected in accordance with these
Rules, the Commission shall limit itself to reviewing and deciding specific issues that were
elevated on appeal. (Emphasis supplied.)
As a testament to its effectivity and the NLRC's continued implementation of this procedural policy,
the same provision was retained as Section 4 (d), Rule VI of the 2005 Revised Rules of Procedure of
the NLRC.

In the case at bar, the NLRC evidently went against its own rules of procedure when it passed upon
the issue of illegal dismissal although the question raised by respondents in their appeal was
concerned solely with the legality of the labor arbiter's award of financial assistance despite the
finding that petitioner was lawfully terminated.

To reiterate, the clear import of the aforementioned procedural rule is that the NLRC shall, in cases
of perfected appeals, limit itself to reviewing those issues which are raised on appeal. As a
consequence thereof, any other issues which were not included in the appeal shall become final and
executory.
We are cognizant of the fact that Article 218(c) of the Labor Code grants the NLRC the authority to
"correct, amend or waive any error, defect or irregularity whether in substance or in form" in the
exercise of its appellate jurisdiction. However, a careful perusal of the body of jurisprudence
wherein we upheld the validity of the NLRC's invocation of that prerogative would reveal that the
said cases involved factual issues and circumstances materially dissimilar to the case at bar.

The Labor Code provision, read in its entirety, states that the NLRC's power to correct errors,
whether substantial or formal, may be exercised only in the determination of a question,
matter or controversy within its jurisdiction [Art. 218, Labor Code]. Therefore, by considering
the arguments and issues in the reply/opposition to appeal which were not properly raised by
timely appeal nor comprehended within the scope of the issue raised in petitioner's appeal, public
respondent committed grave abuse of discretion amounting to excess of jurisdiction.
The contention that the NLRC may nevertheless look into other issues although not raised on
appeal since it is not bound by technical rules of procedure, is likewise devoid of merit.

The law does not provide that the NLRC is totally free from "technical rules of procedure",
but only that the rules of evidence prevailing in courts of law or equity shall not be
controlling in proceedings before the NLRC [Art. 221, Labor Code]. This is hardly license for
the NLRC to disregard and violate the implementing rules it has itself promulgated. Having
done so, the NLRC committed grave abuse of discretion. (Emphases supplied.)
Petitioner argued in his Memorandum that, assuming without admitting that there was no illegal
dismissal, the award of financial assistance was in accordance with existing jurisprudence pursuant
to the principle of social justice. On this point, we agree with petitioner. In Eastern, the employer

194
likewise questioned the grant of financial assistance on the ground that the employee's refusal to
report back to work, despite being duly notified of the need for his service, is tantamount to
voluntary resignation. In that case, however, we ruled:
We are not unmindful of the rule that financial assistance is allowed only in instances where the
employee is validly dismissed for causes other than serious misconduct or those reflecting on his
moral character. Neither are we unmindful of this Court's pronouncements in Arc-Men Food
Industries Corporation v. NLRC, and Lemery Savings and Loan Bank v. NLRC, where the Court ruled
that when there is no dismissal to speak of, an award of financial assistance is not in order.

But we must stress that this Court did allow, in several instances, the grant of financial assistance.
In the words of Justice Sabino de Leon, Jr., now deceased, financial assistance may be allowed as
a measure of social justice and exceptional circumstances, and as an equitable concession.
The instant case equally calls for balancing the interests of the employer with those of the
worker, if only to approximate what Justice Laurel calls justice in its secular sense. (Emphases
supplied.)

There appears to be no reason why petitioner, who has served respondent corporation for more
than eight years without committing any infraction, cannot be extended the reasonable financial
assistance of P18,000.00 as awarded by the Labor Arbiter on equity considerations.

We see no merit in respondents' contention that petitioner was guilty of insubordination or


abandonment. Significantly, the Labor Arbiter made no finding that petitioner was guilty of
insubordination or abandonment. It would appear that a few days after the expiration of his applied
for leave, petitioner filed his complaint for illegal actual dismissal. Other than their self-serving
allegations, respondents offered no proof that upon the expiration of petitioner's leave they
directed petitioner to report to work but petitioner willfully failed to comply with said directive. On
the contrary, in their own position paper, respondents prayed, aside from the dismissal of the
complaint, that petitioner be directed by the Labor Arbiter to return to work and only when
petitioner fails to comply with such order did they pray that petitioner be considered to have
abandoned his work. The Labor Arbiter did not grant this particular relief prayed for by
respondents but instead awarded financial assistance to petitioner.

WHEREFORE, the petition is PARTLY GRANTED. The assailed Decision dated July 28, 2006 as well
as the Resolution dated September 28, 2006 of the Court of Appeals in CA-G.R. SP No. 81703
are AFFIRMED WITH THE MODIFICATION that the award of financial assistance
is REINSTATED. The Labor Arbiter's Decision dated June 26, 2002 is AFFIRMED in toto.

[G.R. No. 171673, May 30, 2011]

195
BANAHAW BROADCASTING CORPORATION
VS.
PACANA III, ET. AL.

Facts:
On August 29, 1995, the DXWG personnel (Pacana III et al.) filed with the Labor Arbiter a
complaint for illegal dismissal, unfair labor practice, reimbursement of unpaid Collective Bargaining
Agreement (CBA) benefits, and attorney’s fees against IBC and BBC.
On June 21, 1996, Labor Arbiter Abdullah L. Alug rendered his Decision awarding the DXWG
personnel a total ofP12,002,157.28 as unpaid CBA benefits consisting of unpaid wages and
increases, 13th month pay, longevity pay, sick leave cash conversion, rice and sugar subsidy,
retirement pay, loyalty reward and separation pay. The Labor Arbiter denied the other claims of the
DXWG personnel for Christmas bonus, educational assistance, medical check-up and optical
expenses. Both sets of parties appealed to the National Labor Relations Commission (NLRC).
The NLRC issued a resolution vacating the decision of the Labor Arbiter and remanded the case to
arbitration branch of origin. On October 15, 1998, Labor Arbiter Nicodemus G. Palangan rendered a
Decision adjudging BBC to be liable for the same amount in the vacated Decision of Labor Arbiter
Alug.

Both BBC and respondents appealed to the NLRC anew. In their appeal, the DXWG personnel
reasserted their claim for the remaining CBA benefits not awarded to them, and alleged error in the
reckoning date of the computation of the monetary award. BBC, in its own Memorandum of Appeal,
challenged the monetary award itself, claiming that such benefits were only due to IBC, not BBC,
employees. In the same Memorandum of Appeal, BBC incorporated a Motion for the Recomputation
of the Monetary Award (of the Labor Arbiter), in order that the appeal bond may be reduced.
The NLRC issued an Order denying the Motion for the Recomputation of the Monetary Award.
According to the NLRC, such recomputation would result in the premature resolution of the issue
raised on appeal. The NLRC ordered BBC to post the required bond within 10 days from receipt
of said Order, with a warning that noncompliance will cause the dismissal of the appeal for
non-perfection. Instead of complying with the Order to post the required bond, BBC filed a Motion
for Reconsideration, alleging this time that since it is wholly owned by the Republic of the
Philippines, it need not post an appeal bond.

Issue:
WHETHER OR NOT BANAHAW BROADCASTING CORPORATION (BBC), A GOVERNMENT OWNED
AND CONTROLLED CORPORATION IS EXEMPT FROM POSTING AN APPEAL BOND.

Held:
We can infer from the foregoing jurisprudential precedents that, as a general rule, the
government and all the attached agencies with no legal personality distinct from the former are
exempt from posting appeal bonds, whereas government-owned and controlled corporations
(GOCCs) are not similarly exempted. This distinction is brought about by the very reason of the
appeal bond itself: to protect the presumptive judgment creditor against the insolvency of the
presumptive judgment debtor. When the State litigates, it is not required to put up an appeal bond

196
because it is presumed to be always solvent. This exemption, however, does not, as a general rule,
apply to GOCCs for the reason that the latter has a personality distinct from its shareholders.
In the case at bar, BBC was organized as a private corporation, sequestered in the 1980’s and the
ownership of which was subsequently transferred to the government in a compromise agreement.
Further, it is stated in its Amended Articles of Incorporation that BBC has the following primary
function:

To engage in commercial radio and television broadcasting, and for this purpose, to establish, operate
and maintain such stations, both terrestrial and satellite or interplanetary, as may be necessary for
broadcasting on a network wide or international basis.
It is therefore crystal clear that BBC’s function is purely commercial or proprietary and not
governmental. As such, BBC cannot be deemed entitled to an exemption from the posting of an
appeal bond.

Consequently, the NLRC did not commit an error, and much less grave abuse of discretion, in
dismissing the appeal of BBC on account of non-perfection of the same.

[G.R. No. 167050 June 1, 2011]

SOCIAL SECURITY COMMISSION,


VS.
RIZAL POULTRY AND LIVESTOCK ASSOCIATION, INC., BSD AGRAO INDUSTRIAL
DEVELOPMENT CORPORATION AND BENJAMIN SAN DIEGO

Facts:
The instant case stemmed from a petition filed by Alberto Angeles (Angeles) before the Social
Security Commission (SSC) to compel respondents Rizal Poultry and Livestock Association, Inc.
(Rizal Poultry) or BSD Agro Industrial Development Corporation (BSD Agro) to remit to the Social
Security System (SSS) all contributions due for and in his behalf.
Angeles had earlier filed a complaint for illegal dismissal against BSD Agro and/or its
owner, Benjamin San Diego (San Diego). The Labor Arbiter initially found that Angeles was an
employee and that he was illegally dismissed. On appeal, however, the NLRC reversed the Labor
Arbiter’s Decision and held that no employer-employee relationship existed between Angeles and
respondents. The ruling was anchored on the finding that the duties performed by Angeles, such as
carpentry, plumbing, painting and electrical works, were not independent and integral steps in the
essential operations of the company, which is engaged in the poultry business. Angeles elevated the
case to the Court of Appeals via petition for certiorari. The appellate court affirmed the NLRC ruling
and upheld the absence of employer-employee relationship. Angeles moved for reconsideration but
it was denied by the Court of Appeals. No further appeal was undertaken, hence, an entry of
judgment was made on 26 May 2001.
At any rate, the SSC did not take into consideration the decision of the NLRC. It denied respondents’
motion to dismiss in an Order dated 19 February 2002. The SSC ratiocinated, thus:

197
Decisions of the NLRC and other tribunals on the issue of existence of employer-employee
relationship between parties are not binding on the Commission. At most, such finding has only a
persuasive effect and does not constitute res judicata as a ground for dismissal of an action pending
before Us. While it is true that the parties before the NLRC and in this case are the same, the issues
and subject matter are entirely different. The labor case is for illegal dismissal with demand for
backwages and other monetary claims, while the present action is for remittance of unpaid SS[S]
contributions. In other words, although in both suits the respondents invoke lack of employer-
employee relationship, the same does not proceed from identical causes of action as one is for
violation of the Labor Code while the instant case is for violation of the SS[S] Law.
Respondents sought recourse before the Court of Appeals by way of a petition for certiorari.
The Court of Appeals reversed the rulings of the SSC and held that there is a common issue between
the cases before the SSC and in the NLRC; and it is whether there existed an employer-employee
relationship between Angeles and respondents. Thus, the case falls squarely under the principle of
res judicata, particularly under the rule on conclusiveness of judgment, as enunciated in Smith Bell
and Co. v. Court of Appeals.

Issue:
WHETHER OR NOT THE DECISION OF THE NLRC AND THE COURT OF APPEALS, FINDING NO
EMPLOYER-EMPLOYEE RELATIONSHIP, CONSTITUTES RES JUDICATA AS A RULE ON
CONCLUSIVENESS OF JUDGMENT AS TO PRECLUDE THE RELITIGATION OF THE ISSUE OF
EMPLOYER-EMPLOYEE RELATIONSHIP IN A SUBSEQUENT CASE FILED BEFORE THE PETITIONER.

Held:
The elements of res judicata are: (1) the judgment sought to bar the new action must be final; (2)
the decision must have been rendered by a court having jurisdiction over the subject matter and the
parties; (3) the disposition of the case must be a judgment on the merits; and (4) there must be as
between the first and second action, identity of parties, subject matter, and causes of action. Should
identity of parties, subject matter, and causes of action be shown in the two cases, then res judicata
in its aspect as a "bar by prior judgment" would apply. If as between the two cases, only identity of
parties can be shown, but not identical causes of action, then res judicata as "conclusiveness of
judgment" applies.
Verily, the principle of res judicata in the mode of "conclusiveness of judgment" applies in this case.
The first element is present in this case. The NLRC ruling was affirmed by the Court of Appeals. It
was a judicial affirmation through a decision duly promulgated and rendered final and executory
when no appeal was undertaken within the reglementary period. The jurisdiction of the NLRC,
which is a quasi-judicial body, was undisputed. Neither can the jurisdiction of the Court of Appeals
over the NLRC decision be the subject of a dispute. The NLRC case was clearly decided on its merits;
likewise on the merits was the affirmance of the NLRC by the Court of Appeals.
With respect to the fourth element of identity of parties, we hold that there is substantial
compliance.
The parties in SSC and NLRC cases are not strictly identical. Rizal Poultry was impleaded as
additional respondent in the SSC case. Jurisprudence however does not dictate absolute identity but
only substantial identity. There is substantial identity of parties when there is a community of

198
interest between a party in the first case and a party in the second case, even if the latter was not
impleaded in the first case.

A case in point is Smith Bell and Co. v. Court of Appeals25 which, contrary to SSC, is apt and proper
reference. Smith Bell availed of the services of private respondents to transport cargoes from the
pier to the company's warehouse. Cases were filed against Smith Bell, one for illegal dismissal
before the NLRC and the other one with the SSC, to direct Smith Bell to report all private
respondents to the SSS for coverage. While the SSC case was pending before the Court of Appeals,
Smith Bell presented the resolution of the Supreme Court in G.R. No. L-44620, which affirmed the
NLRC, Secretary of Labor, and Court of Appeals’ finding that no employer-employee relationship
existed between the parties, to constitute as bar to the SSC case. We granted the petition of Smith
Bell and ordered the dismissal of the case. We held that the controversy is squarely covered by the
principle of res judicata, particularly under the rule on "conclusiveness of judgment." Therefore, the
judgment in G.R. No. L-44620 bars the SSC case, as the relief sought in the latter case is inextricably
related to the ruling in G.R. No. L-44620 to the effect that private respondents are not employees of
Smith Bell.

[G.R. No. 170416, June 22, 2011]

UNIVERSITY PLANS INC.


VS.
SOLANO

Facts:
Respondents Belinda P. Solano (Solano), Terry A. Lamug (Lamug), Glenda S. Belga (Belga), Melba S.
Alvarez (Alvarez), Welma R. Namata (Namata), Marietta D. Bacho (Bacho) and Manolo L. Cenido
(Cenido) filed before the Labor Arbiter complaints for illegal dismissal, illegal deductions,
overriding commissions, unfair labor practice, moral and exemplary damages, and actual damages
against petitioner University Plans Incorporated.

Ruling of the Labor Arbiter


In a Decision dated July 31, 2000, the Labor Arbiter found petitioner guilty of illegal dismissal and
ordered respondents’ reinstatement as well as the payment of their full backwages, proportionate
13th month pay, moral/exemplary damages, and attorney’s fees.

Ruling of the National Labor Relations Commission


Petitioner filed before the NLRC its Memorandum on Appeal as well as a Motion to Reduce
Bond. Simultaneous with the filing of said pleadings, it posted a cash bond in the amount of
P30,000.00.

In its Motion to Reduce Bond, petitioner alleged that it was under receivership and that it cannot
dispose of its assets at such a short notice. Because of this, it could not post the required
bond. Nevertheless, it has P30,000.00 available for immediate disposition and thus prayed that said
amount be deemed sufficient to satisfy the required bond for the perfection of its appeal.

199
Petitioner's motion to reduce bond was denied by the NLRC, stating that the amount of appeal bond
is fixed by law pursuant to Art. 223.

Petitioner filed a Motion for Reconsideration insisting that the NLRC has the discretion to reduce
the appeal bond upon motion of appellant and on meritorious grounds. It argued that the fact that
it was under receivership and could not dispose of any or all of its assets without prior court
approval are meritorious grounds justifying the reduction of the appeal bond.
The NLRC, however, denied petitioner’s motion for reconsideration, it ruled that while it has the
discretion to reduce the appeal bond, it is nevertheless not persuaded that petitioner was incapable
of posting the required bond. It noted that petitioner failed to submit any financial statement or
provide details anent its alleged receivership or its sources of income.

On appeal to the Court of Appeals, the said court likewise ruled against petitioner.

Issue:
Whether or not the Court of Appeals in affirming the decision of the NLRC gravely abused its
discretion when it refused to reduce the appeal bond of petitioner.

Held:
The Supreme Court found petitioner's argument meritorious.
Article 223 of the Labor Code provides in part:

Article 223. Appeal. – Decisions, awards, or orders of the Labor Arbiter are final and executory
unless appealed to the Commission by any or both parties within ten (10) calendar days from
receipt of such decisions, awards, or orders. x x x
xxxx

In case of a judgment involving a monetary award, an appeal by the employer may be


perfected only upon the posting of a cash or surety bond issued by a reputable bonding
company duly accredited by the Commission in the amount equivalent to the monetary award in
the judgment appealed from. (Emphasis supplied.)

x x x x.

While pertinent portions of Sections 4 and 6, Rule VI of the Revised Rules of Procedure of the NLRC
read:

SECTION 4. REQUISITES FOR PERFECTION OF APPEAL – a) The appeal shall be: 1) filed within the
reglementary period provided in Section 1 of this Rule; 2) verified by the appellant himself in
accordance with Section 4, Rule 7 of the Rules of Court, as amended; 3) in the form of a
memorandum of appeal which shall state the grounds relied upon and the arguments in support

200
thereof, the relief prayed for, and with a statement of the date the appellant received the appealed
decision, resolution or order; 4) in three (3) legibly typewritten or printed copies; and 5)
accompanied by i) proof of payment of the required appeal fee; ii) posting of a cash or surety
bond as provided in Section 6 of this Rule; iii) a certificate of non-forum shopping; and iv) proof
of service upon the other parties.

SECTION 6. BOND. – In case the decision of the Labor Arbiter or the Regional Director involves
a monetary award, an appeal by the employer may be perfected only upon the posting of a
bond, which shall either be in the form of cash deposit or surety bond equivalent in amount
to the monetary award, exclusive of damages and attorney’s fees.

No motion to reduce bond shall be entertained except on meritorious grounds, and only
upon the posting of a bond in a reasonable amount in relation to the monetary award.
When the amount of bond may be reduced.

Notably, however, under Section 6, Rule VI of the NLRC’s Revised Rules of Procedure, the bond may
be reduced albeit only on meritorious grounds and upon posting of a partial bond in a reasonable
amount in relation to the monetary award. Suffice it to state that while said Rules “allows the
Commission to reduce the amount of the bond, the exercise of the authority is not a matter of right
on the part of the movant, but lies within the sound discretion of the NLRC upon a showing of
meritorious grounds.”

In Nicol v. Footjoy Industrial Corporation, the Court reviewed the jurisprudence respecting the bond
requirement for perfecting appeal and summarized the guidelines under which the NLRC must
exercise its discretion in considering an appellant’s motion for reduction of bond, viz:

[T]he bond requirement on appeals involving monetary awards has been and may be relaxed in
meritorious cases. These cases include instances in which (1) there was substantial compliance
with the Rules, (2) surrounding facts and circumstances constitute meritorious grounds to reduce
the bond, (3) a liberal interpretation of the requirement of an appeal bond would serve the desired
objective of resolving controversies on the merits, or (4) the appellants, at the very least, exhibited
their willingness and/or good faith by posting a partial bond during the reglementary period.

The NLRC is not precluded from conducting a preliminary determination of the merit or lack of merit
of a motion to reduce bond.
The NLRC erred in not considering the merit or lack of merit of petitioner’s Motion to Reduce
Bond.
Petitioner attached to its Motion to Reduce Bond the SEC Orders dated August 23, 1999 and May 23,
2000. The Order of August 23, 1999 is a Cease and Desist Order which, among others, prohibited
the officers and agents of petitioner from withdrawing from its trust funds or from making any
disposition thereof and, ordered the freeze of all its assets and properties. On the other hand, the
May 23, 2000 Order reads in part that:

201
In view of the voluntary request for receivership of the University Plans, Inc. (UPI), after being
found to have a Trust Fund and Capital Deficiency, unable to pay the same despite its commitment
to pay, and pursuant to Presidential Decree No. 902-A, as amended, University Plans, Inc. is
therefore, placed under the management and control of a RECEIVER x x x (Emphasis supplied.)

From the said SEC Orders, it is unmistakable that petitioner was under receivership. And from the
tenor and contents of said Orders, it is possible that petitioner has no liquid asset which it could use
to post the required amount of bond. Also, it is quite understandable that because of petitioner’s
financial state, it cannot raise the amount of more than P3 million within a period of 10 days from
receipt of the Labor Arbiter’s judgment.

However, the NLRC ignored petitioner’s allegations and instead remained adamant that since the
amount of bond is fixed by law, petitioner must post an additional bond of more than P3
million. This, to us, is an utter disregard of the provision of the Labor Code and of the NLRC Revised
Rules of Procedure allowing the reduction of bond in meritorious cases. While the NLRC tried to
correct this error in its March 21, 2003 Resolution by further explaining that it was not persuaded
by petitioner’s alleged incapability of posting the required amount of bond for failure to submit
financial statement, list of sources of income and other details with respect to the alleged
receivership, we still find the hasty denial of the motion to reduce bond not proper.
Notwithstanding petitioner’s failure to submit its financial statement and list of sources of
income and to give more details relative to its receivership, it was nevertheless able to show
through the abovementioned SEC Orders that it was indeed under a state of
receivership. This should have been sufficient reason for the NLRC to not outrightly deny
petitioner’s motion. As to the lacking documents and details on the receivership, it is true
that they are needed by the NLRC in determining petitioner’s capacity to post the required
amount of bond. However, their absence should not lead to the outright denial of the motion
since as earlier discussed, the NLRC is not precluded from conducting a preliminary
determination on the merit or lack of merit of a motion to reduce bond.

[G.R. No. 178699, September 21, 2011]

BPI EMPLOYEES UNION-METRO MANILA


VS.
BANK OF THE PHIL. ISLANDS

Facts:
On December 14, 1995, Uy’s services as a bank teller in BPI’s Escolta Branch was terminated on
grounds of gross disrespect/discourtesy towards an officer, insubordination and absence without
leave. Uy, together with the Union, thus filed a case for illegal dismissal.

On December 31, 1997, the Voluntary Arbitrator rendered a Decision finding Uy's dismissal as
illegal and ordering BPI to immediately reinstate Uy and to pay her full back wages, including all her

202
other benefits under the Collective Bargaining Agreement (CBA) and attorney’s fees.

On October 28, 1998, the CA affirmed with modification the Decision of the Voluntary
Arbitrator. Instead of reinstatement, the CA ordered BPI to pay Uy her separation pay. Further,
instead of full back wages, the CA fixed Uy's back wages to three years.

The case eventually reached this Court when both parties separately filed petitions for review on
certiorari. While BPI’s petition which was docketed as G.R. No. 137856 was denied for failure to
comply with the requirements of a valid certification of non-forum shopping, Uy’s and the Union’s
petition which was docketed as G.R. No. 137863 was given due course.
On March 31, 2005, the Court rendered its Decision in G.R. No. 137863, the dispositive portion of
which reads:

WHEREFORE, the instant petition is GRANTED. The assailed 28 October 1998 Decision and 8
March 1999 Resolution of the Court of Appeals are hereby MODIFIED as follows: 1) respondent BPI
is DIRECTED to pay petitioner Uy backwages from the time of her illegal dismissal until her actual
reinstatement; and 2) respondent BPI is ORDERED to reinstate petitioner Uy to her former
position, or to a substantially equivalent one, without loss of seniority right and other benefits
attendant to the position.

SO ORDERED.

After the Decision in G.R. No. 137863 became final and executory, Uy and the Union filed with the
Office of the Voluntary Arbitrator a Motion for the Issuance of a Writ of Execution.

In Uy’s computation, she based the amount of her back wages on the current wage level and
included all the increases in wages and benefits under the CBA that were granted during the entire
period of her illegal dismissal. These include the following: Cost of Living Allowance (COLA),
Financial Assistance, Quarterly Bonus, CBA Signing Bonus, Uniform Allowance, Medicine Allowance,
Dental Care, Medical and Doctor’s Allowance, Teller’s Functional Allowance, Vacation Leave, Sick
Leave, Holiday Pay, Anniversary Bonus, Burial Assistance and Omega watch.

BPI disputed Uy's/Union’s computation arguing that it contains items which are not included in the
term “back wages” and that no proof was presented to show that Uy was receiving all the listed
items therein before her termination. It claimed that the basis for the computation of back wages
should be the employee’s wage rate at the time of dismissal.

The Voluntary Arbitrator agreed with Uy’s/Union’s contention that full back wages should include
all wage and benefit increases, including new benefits granted during the period of dismissal. The
Voluntary Arbitrator opined that this Court’s March 31, 2005 Decision in G.R. No. 137863 reinstated
his December 31, 1997 Decision which ordered the payment of full back wages computed from the
time of dismissal until actual reinstatement including all benefits under the CBA.

203
Imputing grave abuse of discretion BPI filed a petition for certiorari with the Court of Appeals and
averred that the Voluntary Arbitrator erred in computing back wages based on the current rate and
in including the wage increases or benefits given in the interim as well as attorney's fees.

In their Comment, Uy and the Union alleged that BPI’s remedy is not a certiorari petition under
Rule 65 of the Rules of Court but an appeal from judgments, final orders and resolutions of
voluntary arbitrators under Rule 43 of the Rules of Court. They also contended that BPI’s petition is
wanting in substance.

Meanwhile, the CA issued a TRO restraining the implementation of the December 6, 2005 Order of
the Voluntary Arbitrator and the corresponding Writ of Execution issued on December 12,
2005. Upon receipt of the TRO, Uy and the Union filed an Urgent Motion for Clarification on
whether the TRO encompasses even the implementation of the reinstatement aspect of the March
31, 2005 Decision of this Court in G.R. No. 137863.
BPI further argued that there was no basis for the award of teller’s functional allowance, cash
conversion of vacation and sick leaves and dental care allowance.
The CA found that the Voluntary Arbitrator erroneously computed Uy's back wages based on the
current rate. The CA also deleted the award of dental allowance since it was granted in 2002 or
more than six years after Uy's dismissal.

Both parties thereafter filed their respective motions for reconsideration.

In its Amended Decision, the CA upheld the propriety of BPI’s resort to certiorari. It also ruled that
this Court’s March 31, 2005 Decision in G.R. No. 137863 did not reinstate the December 31, 1997
Decision of the Voluntary Arbitrator awarding full back wages including CBA benefits. The CA ruled
that the computation of Uy’s full back wages, as defined under Republic Act No. 6715, should be
based on the basic salary at the time of her dismissal plus the regular allowances that she had been
receiving likewise at the time of her dismissal. It held that any increase in the basic salary occurring
after Uy’s dismissal as well as all benefits given after said dismissal should not be awarded to her in
consonance with settled jurisprudence on the matter. Accordingly, the CA pronounced that Uy’s
basic salary, which amounted to P10,895.00 at the time of her dismissal on December 14, 1995, is to
be used as the base figure in computing her back wages, exclusive of any increases and/or
modifications. As Uy’s entitlement to COLA, quarterly bonus and financial assistance are not
disputed, the CA retained their award provided that, again, the base figure for the computation of
these benefits should be the rate then prevailing at the time of Uy’s dismissal.

Finally, the CA likewise ruled that Uy’s reinstatement was effectively restrained by the TRO issued
by it.

Issues:
1. Whether or not in computing the amount of backwages of illegally dismissed employees inbclude
all wage and benefit increases, including new benefits granted during the period of dismissal.
2. WON BPI's resort to petition for Certiorari was proper.

204
3. WON the CA’s order of restraining Uy’s reinstatement despite the finality of this Court’s Decision
ordering such reinstatement. Was proper.

Held:
As to the first issue the SC ruled in the negative.
From the foregoing, it is clear that Uy’s and the Union’s contention that the March 31, 2005 Decision
of this Court in G.R. No. 137863 in effect reinstated the December 31, 1997 Decision of the
Voluntary Arbitrator awarding full back wages including the CBA benefits, is without basis. What is
clear is that the March 31, 2005 Decision modified the October 28, 1998 Decision of the CA by
awarding full back wages instead of limiting the award to a period of three years. This
interpretation is further bolstered by the Court’s discussion in the main body of March 31, 2005
Decision as to the meaning of “full back wages” in view of the passage of Republic Act No. 6715 on
March 21, 1989 which amended Article 279 of the Labor Code, as follows:

ART. 279. Security of Tenure. - In cases of regular employment, the employer shall not terminate the
services of an employee except for a just cause or when authorized by the Title. An employee who is
unjustly dismissed from work shall be entitled to reinstatement without loss of seniority rights and
other privileges and to his full backwages, inclusive of allowances, and to his other benefits or their
monetary equivalent computed from the time his compensation was withheld from him up to the
time of his actual reinstatement. (Italics supplied)

Jurisprudence dictates that such award of back wages is without qualifications and deductions, that
is, “unqualified by any wage increases or other benefits that may have been received by co-workers
who were not dismissed.” It is likewise settled that the base figure to be used in the computation of
back wages is pegged at the wage rate at the time of the employee’s dismissal unqualified by
deductions, increases and/or modifications.
With regards to the second issue the SC held that BPI's resort to certiorari under Rule 65 of the Rules
of Court is proper.
Section 1, Rule 41 of the Rules of Court explicitly provides that no appeal may be taken from an
order of execution, the remedy of an aggrieved party being an appropriate special civil action under
Rule 65 of the Rules of Court. Thus, BPI correctly availed of the remedy of certiorari under Rule 65
of the Rules of Court when it assailed the December 6, 2005 order of execution of the Voluntary
Arbitrator.

As to the third issue the SC agreed with Uy's/Union’s postulation that it was improper for the CA to
restrain the implementation of the reinstatement aspect of this Court’s final and executory Decision
considering that BPI’s appeal with the CA only questioned the propriety of the Voluntary
Arbitrator’s computation of back wages, suffice it to say that this particular issue has already been
rendered moot by Uy’s reinstatement. As manifested by BPI in its Comment, Uy, with her
acquiescence, was reinstated in BPI's payroll on August 1, 2006.

205
[G.R. No. 168317. November 21, 2011]

DUP SOUND PHILS. AND/OR MANUEL TAN, PETITIONERS,


VS.
COURT OF APPEALS AND CIRILO A. PIAL, RESPONDENTS.

Facts:

The instant petition arose from a complaint for illegal dismissal filed by herein private respondent
Cirilo A. Pial (Pial) on November 5, 2001 with the NLRC, Quezon City. In his Position Paper, Pial
alleged that he was an employee of herein petitioner DUP Sound Phils. (DUP), which is an entity
engaged in the business of recording cassette tapes for various recording companies; petitioner
Manuel Tan (Tan) is the owner and manager of DUP; Pial was first employed in May 1988 until
December 1988; on October 11, 1991, he was re-employed by DUP and was given the job of
"mastering tape"; his main function was to adjust the sound level and intensity of the music to be
recorded as well as arrange the sequence of the songs to be recorded in the cassette tapes; on
August 21, 2001, Pial got absent from work because he got sick; when he got well the following day
and was ready for work, he called up their office in accordance with his employer's policy that any
employee who gets absent shall first call their office before reporting back to work; to his surprise,
he was informed by the office secretary that the latter was instructed by Tan to tell him not to
report for work until such time that they will advise him to do so; after three weeks, without
receiving any notice, Pial again called up their office; this time the office secretary advised him to
look for another job because, per instruction of Tan, he is no longer allowed to work at DUP; Pial
asked the office secretary regarding the reason why he was not allowed to return to his job and
pleaded with her to accept him back, but the secretary simply reiterated Tan's order not to allow
him to go back to work. Pial prayed for the payment of his unpaid service incentive leave pay, full
backwages, separation pay, moral and exemplary damages as well as attorney's fees.
C
In their Position Paper, herein petitioners DUP and Tan denied the material allegations of Pial
contending that on or about January 1996 they hired Pial as a laborer; on August 21, 2001, the
latter failed to report for work following an altercation with his supervisor the previous day; on
September 12, 2001, Pial called up their office and informed the office secretary that he will be
going back to work on September 17, 2001; however, he failed to report for work on the said date;
petitioners were subsequently surprised when they learned that Pial filed a complaint for illegal
dismissal against them; Pial was never dismissed, instead, it was his unilateral decision not to work
at DUP anymore; Tan even offered him his old post during one of the hearings before the NLRC
hearing officer, but Pial refused such offer or any other offer of amicable settlement.

On July 25, 2002, the Labor Arbiter (LA) handling the case rendered a Decision declaring Pial to
have been illegally dismissed and ordering DUP and Tan to reinstate him to his former position and
pay him backwages, cost of living allowance, service incentive leave pay and attorney's fees.

206
On appeal, the NLRC, in its Decision promulgated on June 30, 2003, modified the Decision of the LA
by deleting the award of backwages and attorney's fees. The NLRC ruled that there was no illegal
dismissal on the part of DUP and Tan, but neither was there abandonment on the part of Pial.Pial
filed a Motion for Reconsideration, but the NLRC denied it in its Resolution.Pial then filed a special
civil action for certiorari with the CA. In which case, the CA reinstated the decision of the LA. DUP
and Tan filed a Motion for Reconsideration, but the same was denied by the CA. Hence, the instant
petition for review on certiorari.

Issue:
1. Whether or not the CA erred in finding that petitioners terminated private respondent's
employment,
2. Whether or not respondent was legally dismissed on grounds of abandonment.

Held:

The petition lacks merit.

At the outset, the Court finds it proper to reiterate the well-established rule that the jurisdiction of
this Court in cases brought before it via Rule 45 of the Rules of Court is limited to reviewing errors
of law.However, one of the admitted exceptions to this rule is where the findings of the NLRC
contradict those of the Labor Arbiter, the Court, in the exercise of its equity jurisdiction, may look
into the records of the case and reexamine the questioned findings. Which is what happened in the
case at bar.

After a review of the records at hand, the Court finds no cogent reason to depart from the
concurrent findings of the Labor Arbiter and the CA that private respondent was illegally dismissed.
Like the Labor Arbiter, the NLRC and the CA, this Court cannot give credence to petitioners' claim
that private respondent abandoned his job.

The settled rule in labor cases is that the employer has the burden of proving that the employee
was not dismissed, or, if dismissed, that the dismissal was not illegal, and failure to discharge the
same would mean that the dismissal is not justified and, therefore, illegal.Considering the hard
times in which we are in, it is incongruous for private respondent to simply give up his work
without any apparent reason at all. No employee would recklessly abandon his job knowing fully
well the acute unemployment problem and the difficulty of looking for a means of livelihood
nowadays. Certainly, no man in his right mind would do such thing.

The consistent rule is that the employer must affirmatively show rationally adequate evidence that
the dismissal was for a justifiable cause.In addition, the employer must also observe the
requirements of procedural due process. In the present case, petitioners failed to submit sufficient
evidence to show that private respondent's dismissal was for a justifiable cause and in accordance
with due process.

207
Neither may private respondent's refusal to report for work subsequent to the Labor Arbiter's
issuance of an order for his reinstatement be considered as another abandonment of his job. It is a
settled rule that failure to report for work after a notice to return to work has been served does not
necessarily constitute abandonment. As defined under established jurisprudence, abandonment is
the deliberate and unjustified refusal of an employee to resume his employment. It is a form of
neglect of duty, hence, a just cause for termination of employment by the employer.

For a valid finding of abandonment, these two factors should be present: (1) the failure to report for
work or absence without valid or justifiable reason; and (2) a clear intention to sever employer-
employee relationship, with the second as the more determinative factor which is manifested by
overt acts from which it may be deduced that the employee has no more intention to work. The
intent to discontinue the employment must be shown by clear proof that it was deliberate and
unjustified.

Under the existing law, an employee who is unjustly dismissed from work shall be entitled to
reinstatement without loss of seniority rights. Article 279 of the Labor Code clearly provides that an
employee who is dismissed without just cause and without due process is entitled to backwages
and reinstatement or payment of separation pay in lieu thereof.Article 223 of the same Code also
provides that an employee entitled to reinstatement shall either be admitted back to work under
the same terms and conditions prevailing prior to his dismissal or separation, or, at the option of
the employer, merely reinstated in the payroll. It is established in jurisprudence that reinstatement
means restoration to a state or condition from which one had been removed or separated.The
person reinstated assumes the position he had occupied prior to his dismissal.

Reinstatement presupposes that the previous position from which one had been removed still
exists, or that there is an unfilled position which is substantially equivalent or of similar nature as
the one previously occupied by the employee. It turned out, petitioners had other plans for private
respondent. Thus, private respondent's assignment to a different job, as well as transfer of work
assignment without any justification therefor, cannot be deemed as faithful compliance with the
reinstatement order.

With respect to private respondent's backwages, the same shall be reckoned from the date he was
illegally dismissed on August 22, 2001 until finality of this Decision, in accordance with prevailing
jurisprudence.

[G.R. No. 193484. January 18, 2012]

HYPTE R. AUJERO, PETITIONER,


VS.
PHILIPPINE COMMUNICATIONS SATELLITE CORPORATION, RESPONDENT.

Facts:

208
It was in 1967 that the petitioner started working for respondent Philippine Communications
Satellite Corporation (Philcomsat) as an accountant in the latter's Finance Department. On August
15, 2001 or after thirty-four (34) years of service, the petitioner applied for early retirement. His
application for retirement was approved, effective September 15, 2001, entitling him to receive
retirement benefits at a rate equivalent to one and a half of his monthly salary for every year of
service. At that time, the petitioner was Philcomsat's Senior Vice-President.

On September 12, 2001, the petitioner executed a Deed of Release and Quitclaimin Philcomsat's
favor, following his receipt from the latter of a check. Almost three (3) years thereafter, the
petitioner filed a complaint for unpaid retirement benefits, claiming that the actual amount of his
retirement less than the amount he received from Philcomsat as supposed settlement for all his
claims is unconscionable, which is more than enough reason to declare his quitclaim as null and
void. He said that had no choice but to accept a lesser amount as he was in dire need thereof and
was all set to return to his hometown and he signed the quitclaim despite the considerable
deficiency as no single centavo would be released to him if he did not execute a release and waiver
in Philcomsat's favor.

The petitioner claims that his right to receive the full amount of his retirement benefits, which is
equivalent to one and a half of his monthly salary for every year of service, is provided under the
Retirement Plan that Philcomsat created for the benefit of its employees.Petitioner he claims that
Philcomsat has no right to withhold any portion of his retirement benefits as the trust fund created
pursuant to the Retirement Plan is for the exclusive benefit of Philcomsat employees and
Philcomsat had expressly recognized that it has no right or claim over the trust fund even on the
portion pertaining to its contributions.

On May 31, 2006, Labor Arbiter Joel S. Lustria (LA Lustria) issued a Decision in the petitioner's
favor, directing Philcomsat to pay him the amount representing the balance of his retirement
benefits and salary for the period from August 15 to September 15, 2001, respectively. LA
Lustriaruled against the effectivity of the waiver and quitclaim, thus, complainant is entitled to the
balance of his retirement benefits.In its July 4, 2008 Resolution,the NLRC granted Philcomsat's
appeal and reversed and set aside LA Lustria's May 31, 2006 Decision.

Philcomsat's appeal to the NLRC from LA Lustria's May 31, 2006 Decision was filed and its surety
bond posted beyond the prescribed period of ten (10) days. On June 20, 2006, a copy of LA Lustria's
Decision was served on one of Philcomsat's executive assistants, as Philcomsat's counsel and the
executive assistant assigned to her were both out of the office. It was only the following day that
Querubin gave a copy of the said Decision to the executive assistant of Philcomsat's counsel, leading
the latter to believe that it was only then that the said Decision had been served. In turn, this led
Philcomsat's counsel to believe that it was on June 21, 2006 that the ten (10) day-period started to
run. Having in mind that the delay was only one (1) day and the explanation offered by Philcomsat's
counsel, the NLRC disregarded Philcomsat's procedural lapse and proceeded to decide the appeal
on its merits.

209
Be that as it may, the provisions of Section 10, Rule VII of the NLRC Rules of Procedure, states, that:
"SECTION 10.TECHNICAL RULES NOT BINDING. — The rules of procedure and evidence prevailing in
courts of law and equity shall not be controlling and the Commission shall use every and all reasonable
means to ascertain the facts in each case speedily and objectively, without regard to technicalities of
law or procedure, all in the interest of due process. . . ."

Issue:
Whether or not the NLRC acted with grave abuse of discretion in giving due course to the
respondent’s appeal.

Held:

Procedural rules may be relaxed to give way to the full determination of a case on its merits.
Confronted with the task of determining whether the CA erred in not finding grave abuse of
discretion in the NLRC's decision to give due course to Philcomsat's appeal despite its being
belatedly filed, this Court rules in Philcomsat's favor.Procedural rules may be waived or dispensed
with in absolutely meritorious cases. A review of the cases cited by the petitioner, Rubia v.
Government Service Insurance System and Videogram Regulatory Board v. Court of Appeals, where
this Court adhered to the strict implementation of the rules and considered them inviolable, shows
that the patent lack of merit of the appeals render liberal interpretation pointless and naught. The
contrary obtains in this case as Philcomsat's case is not entirely unmeritorious. Specifically,
Philcomsat alleged that the petitioner's execution of the subject quitclaim was voluntary and he
made no claim that he did so. Philcomsat likewise argued that the petitioner's educational
attainment and the position he occupied in Philcomsat's hierarchy militate against his claim that he
was pressured or coerced into signing the quitclaim.

The emerging trend in our jurisprudence is to afford every party-litigant the amplest opportunity
for the proper and just determination of his cause free from the constraints of technicalities.Far
from having gravely abused its discretion, the NLRC correctly prioritized substantial justice over
the rigid and stringent application of procedural rules. This, by all means, is not a case of grave
abuse of discretion calling for the issuance of a writ of certiorari.

The CA and the NLRC were unanimous in holding that the petitioner voluntarily executed the
subject quitclaim. The Supreme Court (SC) is not a trier of facts, and this doctrine applies with
greater force in labor cases. Factual questions are for the labor tribunals to resolve and whether the
petitioner voluntarily executed the subject quitclaim is a question of fact. In this case, the factual
issues have already been determined by the NLRC and its findings were affirmed by the CA. Judicial
review by this Court does not extend to a reevaluation of the sufficiency of the evidence upon which
the proper labor tribunal has based its determination.

Factual findings of labor officials who are deemed to have acquired expertise in matters within
their respective jurisdictions are generally accorded not only respect, but even finality, and are
binding on the SC. Verily, their conclusions are accorded great weight upon appeal, especially when

210
supported by substantial evidence. Consequently, the SC is not duty-bound to delve into the
accuracy of their factual findings, in the absence of a clear showing that the same were arbitrary
and bereft of any rational basis.

[G.R. No. 185280, JANUARY 18, 2012]

TIMOTEO H. SARONA
VS.
NATIONAL LABOR RELATIONS COMMISSION, ROYALE SECURITY AGENCY AND CESAR S. TAN

Facts:
Petitioner, a security guard in Sceptre since April 1976, was asked by Sceptre’s operations
manager on June 2003, to submit a resignation letter as a requirement for an application in Royale
and to fill up an employment application form for the said company. He was then assigned at
Highlight Metal Craft Inc. from July 29 to August 8, 2003 and was later transferred to Wide Wide
World Express Inc. On September 2003, he was informed that his assignment at WWWE Inc. was
withdrawn because Royale has been allegedly replaced by another security agency which he later
discovered to be untrue. Nevertheless, he was once again assigned at Highlight Metal sometime in
September 2003 and when he reported at Royale’s office on October 1, 2003, he was informed that
he would no longer be given any assignment as instructed by Sceptre’s general manager. He thus
filed a complaint for illegal dismissal. The LA ruled in petitioner’s favor as he found him illegally
dismissed and was not convinced by the respondent’s claim on petitioner’s abandonment.

Respondents were ordered to pay backwages computed from the day he was dismissed up to the
promulgation of his decision on May 11, 2005.The LA also ordered for the payment of separation
pay but refused to pierce Royale’s corporate veil.

Respondents appealed to the NLRC claiming that the LA acted with grave abuse of discretion upon
ruling on the illegal dismissal of petitioner. NLRC partially affirmed the LA’s decision with regard to
petitioner’s illegal dismissal and separation pay but modified the amount of backwages and limited
it to only 3 months of his last month salary reducing P95, 600 to P15, 600 since he worked for
Royale for only 1 month and 3 days.
Petitioner did not appeal to LA but raised the validity of LA’s findings on piercing Royale’s
corporate personality and computation of his separation pay and such petition was dismissed by
the NLRC.

Petitioner elevated NLRC’s decision to the CA on a petition for certiorari, and the CA disagreed with
the NLRC’s decision of not proceeding to review the evidence for determining if Royale is Sceptre’s
alter ego that would warrant the piercing of its corporate veil.

211
Issues:
Whether or not Royale’s corporate fiction should be pierced for the purpose of compelling it to
recognize the petitioner’s length of service with Sceptre and for holding it liable for the benefits that
have accrued to him arising from his employment with Sceptre; and
Whether or not petitioner’s backwages should be limited to his salary for 3 months

Held:
The doctrine of piercing the corporate veil is applicable on alter ego cases, where a corporation is
merely a farce since it is a mere alter ego or business conduit of a person, or where the corporation
is so organized and controlled and its affairs are so conducted as to make it merely an
instrumentality, agency, conduit or adjunct of another corporation. The way on how petitioner was
made to resign from Sceptre then later on made an employee of Royale, reflects the use of the legal
fiction of the separate corporate personality and is an implication of continued employment. Royale
is a continuation or successor or Sceptre since the employees of Sceptre and of Royale are the same
and said companies have the same principal place of business.

Because petitioner’s rights were violated and his employer has not changed, he is entitled to
separation pay which must be computed from the time he was hired until the finality of this
decision. Royale is also ordered to pay him backwages from his dismissal on October 1, 2003 until
the finality of this decision. However, the amount already received by petitioner from the
respondents shall be deducted. He is also awarded moral and exemplary damages amounting to P
25, 000.00 each for his dismissal which was tainted with bad faith and fraud. Petition is granted.
CA’s decision is reversed and set aside.

[G.R. No. 174941, FEBRUARY 1, 2012]

SALENGA, ET. AL.,


VS.
COURT OF APPEALS

Facts:

President/Chief Executive Officer (CEO) Rufo Colayco issued an Order informing petitioner that,
pursuant to the decision of the board of directors of respondent CDC, the position of head executive
assistant – the position held by petitioner – was declared redundant. Petitioner filed a Complaint
for illegal dismissal with a claim for reinstatement and payment of back wages, benefits, and moral
and exemplary damages against respondent CDC and Colayco. Respondents, represented by the
Office of the Government Corporate Counsel (OGCC), alleged that the NLRC had no jurisdiction to
entertain the case on the ground that petitioner was a corporate officer and, thus, his dismissal was
an intra-corporate matter falling properly within the jurisdiction of the Securities and Exchange

212
Commission (SEC). LA Darlucio rendered a Decision in favor of petitioner. From the decision, the
OCGCC filed an appeal with the National Labor Relations Commission (NLRC) via a Memorandum of
Appeal verified and certified by Hilana Timbol-Roman, the executive vice president of respondent
CDC. The petitioner opposed the appeal on the ground that the Memorandum of Appeal and Joint
Affidavit were not accompanied by a board resolution from respondent’s board of directors
authorizing either Timbol-Roman or Atty. Mallare, or both, to pursue the case or to file the appeal
on behalf of respondent.

Issue:
Whether or not the NLRC has jurisdiction to entertain the appeal.

Held:

NLRC has no jurisdiction to entertain the appeal. It is clear from the NLRC Rules of Procedure that
appeals must be verified and certified against forum-shopping by the parties-in-interest
themselves. A corporation can only exercise its powers and transact its business through its board
of directors and through its officers and agents when authorized by a board resolution or its
bylaws. Absent the requisite board resolution, neither Timbol-Roman nor Atty. Mallari, who signed
the Memorandum of Appeal and Joint Affidavit of Declaration allegedly on behalf of respondent
corporation, may be considered as the “appellant” and “employer” referred to by Rule VI, Sections 4
to 6 of the NLRC Rules of Procedure. The court cannot agree with the OGCC’s attempt to downplay
this procedural flaw by claiming that, as the statutorily assigned counsel for GOCCs, it does not need
such authorization. In Constantino-David v. Pangandaman-Gania, 456 Phil. 273, 294-298 (2003),
the court exhaustively explained why it was necessary for government agencies or
instrumentalities to execute the verification and the certification against forum-shopping through
their duly authorized representatives. The purpose of verification is to secure an assurance that the
allegations in the pleading are true and correct and have been filed in good faith. Unless equitable
circumstances which are manifest from the record of a case prevail, it therefore becomes necessary
for the concerned government agency or its authorized representatives to certify for non-forum
shopping if only to be sure that no other similar case or incident is pending before any other court.

Anent the corporation’s liability, the decision of the LA still stands. In the case at bar, respondents
failed to adduce any evidence showing that the position of Head Executive Assistant is superfluous.
There is no evidence on record to show that the position of Head Executive Assistant was abolished
by the Board of Directors in its meetings. Hence, the ground of redundancy is merely a device made
by respondent Colayco in order to ease out the complainant from the respondent corporation.
Moreover, the complainant was not accorded his right to due process prior to his termination. He
was not given the opportunity to be heard and defend himself. However, the court notes that with
regards to respondent Colayco’s solidary liability with the corporation, petitioner notably in the
case at hand, did not question the ruling made by NLRC in finding that respondent Colayco may not
be held solidarily responsible to him. As a result, it dropped him as a respondent. Based on the
foregoing, all other subsequent proceedings regarding the issue of petitioner’s dismissal are null
and void for having been conducted without jurisdiction.

213
[G.R. No. 185918 : April 18, 2012]

LOCKHEED DETECTIVE AND WATCHMAN AGENCY, INC., PETITIONER,


VS.
UNIVERSITY OF THE PHILIPPINES, RESPONDENT

Facts:

Petitioner Lockheed Detective and Watchman Agency, Inc. (Lockheed) entered into a contract for
security services with respondent University of the Philippines (UP).

In 1998, several security guards assigned to UP filed separate complaints against Lockheed and UP
for payment of underpaid wages, 25% overtime pay, premium pay for rest days and special
holidays, holiday pay, service incentive leave pay, night shift differentials, 13th month pay, refund of
cash bond, refund of deductions for the Mutual Benefits Aids System (MBAS), unpaid wages from
December 16-31, 1998, and attorney’s fees.

On February 16, 2000, the Labor Arbiter rendered a decision: respondents Lockheed Detective and
Watchman Agency, Inc. and UP as job contractor and principal, respectively, are hereby declared
to be solidarily liable to complainants for the following claims of the latter which are found
meritorious.

Both Lockheed and UP appealed the Labor Arbiter’s decision. By Decision dated April 12,
2002, the NLRC modified the Labor Arbiter’s decision: The respondent University of the Philippines
is still solidarily liable with Lockheed in the payment of the rest of the claims covering the period of
their service contract.

The complaining security guards and UP filed their respective motions for reconsideration. On
August 14, 2002, however, the NLRC denied said motions.

As the parties did not appeal the NLRC decision, the same became final and executory on
October 26, 2002. A writ of execution was then issued but later quashed by the Labor Arbiter
on November 23, 2003 on motion of UP due to disputes regarding the amount of the award. Later,
however, said order quashing the writ was reversed by the NLRC by Resolution..

UP moved to reconsider the NLRC resolution. On December 28, 2004, the NLRC upheld its
resolution but with modification that the satisfaction of the judgment award in favor of
Lockheed will be only against the funds of UP which are not identified as public funds.

The NLRC order and resolution having become final, Lockheed filed a motion for the issuance of
an alias writ of execution. The same was granted on May 23, 2005.

214
On July 25, 2005, a Notice of Garnishment was issued to Philippine National Bank (PNB) UP
Diliman Branch for the satisfaction of the award of P12,142,522.69 (inclusive of execution fee).

In a letter dated August 9, 2005, PNB informed UP that it has received an order of release. PNB
likewise reminded UP that the bank only has 10 working days from receipt of the order to deliver
the garnished funds and unless it receives a notice from UP or the NLRC before the expiry of the 10-
day period regarding the issuance of a court order or writ of injunction discharging or enjoining the
implementation and execution of the Notice of Garnishment and Writ of Execution, the bank shall
be constrained to cause the release of the garnished funds in favor of the NLRC.

On August 16, 2005, UP filed an Urgent Motion to Quash Garnishment on the ground that funds
being subjected to garnishment at PNB are government/public funds. UP argued that as public
funds, the subject PNB account cannot be disbursed except pursuant to an appropriation required
by law. The Labor Arbiter, however, dismissed the urgent motion for lack of merit.

On September 2, 2005, the amount of P12,062,398.71 was withdrawn by the sheriff from UP’s
PNB account.

On September 12, 2005, UP filed a petition for certiorari before the CA based on the following
grounds:

I.

The concept of “solidary liability” by an indirect employer notwithstanding, respondent NLRC


gravely abused its discretion in a manner amounting to lack or excess of jurisdiction by misusing
such concept to justify the garnishment by the executing Sheriff of public/government funds
belonging to UP.

II.

Respondents NLRC and Arbiter LORA acted without jurisdiction or gravely abused their discretion
in a manner amounting to lack or excess of jurisdiction when, by means of an Alias Writ of
Execution against petitioner UP, they authorized respondent Sheriff to garnish UP’s public funds.
Similarly, respondent LORA gravely abused her discretion when she resolved petitioner’s Motion to
Quash Notice of Garnishment addressed to, and intended for, the NLRC, and when she unilaterally
and arbitrarily disregarded an official Certification that the funds garnished are public/government
funds, and thereby allowed respondent Sheriff to withdraw the same from PNB.

III.

Respondents gravely abused their discretion in a manner amounting to lack or excess of

215
jurisdiction when they, despite prior knowledge, effected the execution that caused paralyzation
and dislocation to petitioner’s governmental functions.

On March 12, 2008, the CA rendered a decision dismissing UP’s petition for
certiorari. CitingRepublic v. COCOFED, which defines public funds as moneys belonging to the State
or to any political subdivisions of the State, more specifically taxes, customs, duties and moneys
raised by operation of law for the support of the government or the discharge of its obligations, the
appellate court ruled that the funds sought to be garnished do not seem to fall within the stated
definition.

On reconsideration, however, the CA issued the assailed Amended Decision. It held that
without departing from its findings that the funds covered in the savings account sought to be
garnished do not fall within the classification of public funds, it reconsiders the dismissal of the
petition in light of the ruling in the case of National Electrification Administration v.
Morales which mandates that all money claims against the government must first be filed
with the Commission on Audit (COA).

Lockheed moved to reconsider the amended decision but the same was denied in the assailed
CA Resolution dated December 23, 2008. The CA cited Manila International Airport Authority v.
Court of Appeals which held that UP ranks with MIAA, a government instrumentality exercising
corporate powers but not organized as a stock or non-stock corporation. While said corporations
are government instrumentalities, they are loosely called government corporate entities but not
government-owned and controlled corporations in the strict sense.

Hence this petition by Lockheed raising the following arguments:


1. RESPONDENT UP IS A GOVERNMENT ENTITY WITH A SEPARATE AND DISTINCT
PERSONALITY FROM THE NATIONAL GOVERNMENT AND HAS ITS OWN CHARTER GRANTING IT
THE RIGHT TO SUE AND BE SUED. IT THEREFORE CANNOT AVAIL OF THE IMMUNITY FROM SUIT
OF THE GOVERNMENT. NOT HAVING IMMUNITY FROM SUIT, RESPONDENT UP CAN BE HELD
LIABLE AND EXECUTION CAN THUS ENSUE.
2. MOREOVER, IF THE COURT LENDS IT ASSENT TO THE INVOCATION OF THE DOCTRINE OF
STATE IMMUNITY, THIS WILL RESULT [IN] GRAVE INJUSTICE.
3. FURTHERMORE, THE PROTESTATIONS OF THE RESPONDENT ARE TOO LATE IN THE DAY,
AS THE EXECUTION PROCEEDINGS HAVE ALREADY BEEN TERMINATED.

Lockheed’s contentions:
a. UP has its own separate and distinct juridical entity from the national government and has its
own charter. Thus, it can be sued and be held liable. Moreover, Executive Order No. 714 entitled
“Fiscal Control and Management of the Funds of UP” recognizes that “as an institution of higher
learning, UP has always granted full management and control of its affairs including its financial
affairs.” Therefore, it cannot shield itself from its private contractual liabilities by simply invoking
the public character of its funds. Lockheed also cites several cases wherein it was ruled that funds
of public corporations which can sue and be sued were not exempt from garnishment.

216
b. the rulings in the NEA and MIAA cases are inapplicable. It contends that UP is not similarly
situated with NEA because the jurisdiction of COA over the accounts of UP is only on a post-audit
basis. As to the MIAA case, the liability of MIAA pertains to the real estate taxes imposed by the City
of Paranaque while the obligation of UP in this case involves a private contractual
obligation. Lockheed also argues that the declaration in MIAA specifically citing UP was mere obiter
dictum.

c. UP cannot invoke state immunity to justify and perpetrate an injustice. UP itself admitted its
liability and thus it should not be allowed to renege on its contractual obligations. Lockheed
contends that this might create a ruinous precedent that would likely affect the relationship
between the public and private sectors.

d. UP cannot anymore seek the quashal of the writ of execution and notice of garnishment as they
are already fait accompli.

UP’s contentions:

a. It did not invoke the doctrine of state immunity from suit in the proceedings a quo and in fact, it
did not object to being sued before the labor department. It maintains, however, that suability does
not necessarily mean liability. UP argues that the CA correctly applied the NEA ruling when it held
that all money claims must be filed with the COA.

b. As to alleged injustice that may result for invocation of state immunity from suit, UP reiterates
that it consented to be sued and even participated in the proceedings below. Lockheed cannot now
claim that invocation of state immunity, which UP did not invoke in the first place, can result in
injustice.

c. On the fait accompli argument, UP argues that Lockheed cannot wash its hands from liability for
the consummated garnishment and execution of UP’s trust fund in the amount of
P12,062,398.71. UP cites that damage was done to UP and the beneficiaries of the fund when said
funds, which were earmarked for specific educational purposes, were misapplied, for instance, to
answer for the execution fee of P120,123.98 unilaterally stipulated by the sheriff. Lockheed, being
the party which procured the illegal garnishment, should be held primarily liable. The mere fact
that the CA set aside the writ of garnishment confirms the liability of Lockheed to reimburse and
indemnify in accordance with law.

RULING:

We agree with UP that there was no point for Lockheed in discussing the doctrine of state immunity
from suit as this was never an issue in this case. Clearly, UP consented to be sued when it
participated in the proceedings below. What UP questions is the hasty garnishment of its funds in
its PNB account.

217
This Court finds that the CA correctly applied the NEA case. Like NEA, UP is a juridical personality
separate and distinct from the government and has the capacity to sue and be sued. Thus, also like
NEA, it cannot evade execution, and its funds may be subject to garnishment or levy. However,
before execution may be had, a claim for payment of the judgment award must first be filed with the
COA. Under Commonwealth Act No. 327, as amended by Section 26 of P.D. No. 1445, it is the COA
which has primary jurisdiction to examine, audit and settle “all debts and claims of any sort” due
from or owing the Government or any of its subdivisions, agencies and instrumentalities, including
government-owned or controlled corporations and their subsidiaries. With respect to money
claims arising from the implementation of Republic Act No. 6758, their allowance or disallowance is
for COA to decide, subject only to the remedy of appeal by petition for certiorari to this Court.

We cannot subscribe to Lockheed’s argument that NEA is not similarly situated with UP because the
COA’s jurisdiction over the latter is only on post-audit basis. A reading of the pertinent
Commonwealth Act provision clearly shows that it does not make any distinction as to which of the
government subdivisions, agencies and instrumentalities, including government-owned or
controlled corporations and their subsidiaries whose debts should be filed before the COA.

As to the fait accompli argument of Lockheed, contrary to its claim that there is nothing that can be
done since the funds of UP had already been garnished, since the garnishment was erroneously
carried out and did not go through the proper procedure (the filing of a claim with the COA), UP is
entitled to reimbursement of the garnished funds plus interest of 6% per annum, to be computed
from the time of judicial demand to be reckoned from the time UP filed a petition for certiorari
before the CA which occurred right after the withdrawal of the garnished funds from PNB.

WHEREFORE, the petition for review on certiorari is DENIED for lack of merit. Petitioner
Lockheed Detective and Watchman Agency, Inc. is ordered to REIMBURSE respondent University
of the Philippines.

[G.R. No. 200653, June 13, 2012]

3RD ALERT SECURITY & DETECTIVE SERVICES INC.


VS.
NAVIA

Facts:

This case started from an illegal dismissal complaint filed by Romualdo Navia against 3rd Alert.

The labor arbiter issued a decision that Navia’s dismissal was illegal. 3rd Alert appealed to the
National Labor Relations Commission (NLRC) which affirmed the ruling of the labor arbiter.

218
3rd Alert’s motion for reconsideration of the NLRC decision was denied in a resolution dated
October 19, 2008.

From this ruling, 3rd Alert filed an appeal with the CA (docketed as CA-G.R. SP No. 106963) with a
prayer for the issuance of a temporary restraining order. The CA denied the appeal; 3rd Alert
moved for a motion for reconsideration but the motion was also denied.

The writ of execution (CA-G.R. SP No. 117361)

In the meantime, on January 29, 2009, the NLRC issued an Entry of Judgment certifying that the
NLRC resolution dated October 19, 2008 has become final and executory. Thus, Navia filed with the
labor arbiter an ex-parte motion for recomputation of back wages and an ex-parte motion for
execution based on the recomputed back wages.

On November 10, 2009, the labor arbiter issued a writ of execution to enforce the recomputed
monetary awards.

3rd Alert appealed the recomputed amount stated in the writ of execution to the NLRC.
3rd Alert also alleged that the writ was issued with grave abuse of discretion since there was
already a notice of reinstatement sent to Navia.

The NLRC dismissed the appeal, ruling that 3rd Alert is guilty of bad faith since there was no
earnest effort to reinstate Navia. The NLRC also ruled that there was no notice or
reinstatement sent to Navia’s counsel. A motion for reconsideration was filed, but it was
likewise denied.

3rd Alert filed a petition for certiorari with the CA which found the petition without merit
because Navia had not been reinstated either physically or in the payroll. The CA also denied
the motion for reconsideration filed by 3rd Alert; hence, this petition.

Issue:
Whether the CA erred in ruling that the NLRC did not commit any grave abuse of discretion.

Held:

NO GRAVE ABUSE OF DISCRETION

Time and again, we have held that this Court is not a trier of facts.[4] In the absence of any
attendant grave abuse of discretion, these findings are entitled not only to respect, but to our final
recognition in this appellate review. Since it was ruled that there had been no notice of
reinstatement sent to Navia or his counsel, as also affirmed by the CA, we cannot rule otherwise in
the absence of any compelling evidence.

219
Article 223 of the Labor Code provides that in case there is an order of reinstatement, the employer
must admit the dismissed employee under the same terms and conditions, or merely reinstate the
employee in the payroll. The order shall be immediately executory. Thus, 3rd Alert cannot escape
liability by simply invoking that Navia did not report for work. The law states that the employer
must still reinstate the employee in the payroll. Where reinstatement is no longer viable as an
option, separation pay equivalent to one (1) month salary for every year of service could be
awarded as an alternative.

Since the proceedings below indicate that 3rd Alert failed to adduce additional evidence to show
that it tried to reinstate Navia, either physically or in the payroll, we adopt as correct the finding
that there was no earnest effort to reinstate Navia. The CA was correct in affirming the judgment of
the NLRC in this regard.

We also take note that 3rd Alert resorted to legal tactics to frustrate the execution of the labor
arbiter’s order; for about four (4) years, it evaded the obligation to reinstate Navia. By so doing,
3rd Alert has made a mockery of justice. We thus find it proper, under the circumstances, to
impose treble costs against 3rd Alert for its utter disregard to comply with the writ of execution. To
reiterate, no indication exists showing that 3rd Alert exerted any efforts to reinstate Navia; worse,
3rd Alert’s lame excuse of having sent a notice of reinstatement to a certain “Biznar” only
compounded the intent to mislead the courts.

Also, the main issue of this case, finding Navia to have been illegally dismissed, has already attained
finality. Litigation must end and terminate sometime and somewhere, and it is essential for an
effective and efficient administration of justice that, once a judgment has become final, the winning
party be not deprived of the fruits of the verdict. The order is to reinstate Navia; sadly, the mere
execution of this judgment has to even reach the highest court of the land, thereby frustrating the
entire judicial process. This justifies the treble costs we now impose against 3rd Alert.

WHEREFORE, premises considered, we hereby DENY the petition outright.

[G.R. No. 172642, June 13, 2012]

ESTATE OF NELSON R. DULAY, REPRESENTED BY HIS WIFE MERRIDY JANE P. DULAY,


PETITIONER,
VS.
ABOITIZ JEBSEN MARITIME, INC. AND GENERAL CHARTERERS, INC., RESPONDENTS.

Facts:

Nelson R. Dulay (Nelson, for brevity) was employed by [herein respondent] General Charterers Inc.
(GCI), a subsidiary of co-petitioner [herein co-respondent] Aboitiz Jebsen Maritime Inc. since 1986.

220
He initially worked as an ordinary seaman and later as bosun on a contractual basis. From
September 3, 1999 up to July 19, 2000, Nelson was detailed in petitioners’ vessel, the MV Kickapoo
Belle.

On August 13, 2000, or 25 days after the completion of his employment contract, Nelson died due to
acute renal failure secondary to septicemia. At the time of his death, Nelson was a bona fide
member of the Associated Marine Officers and Seaman’s Union of the Philippines (AMOSUP), GCI’s
collective bargaining agent. Nelson’s widow, Merridy Jane, thereafter claimed for death benefits
through the grievance procedure of the Collective Bargaining Agreement (CBA) between AMOSUP
and GCI. However, on January 29, 2001, the grievance procedure was “declared deadlocked” as
petitioners refused to grant the benefits sought by the widow.

On March 5, 2001, Merridy Jane filed a complaint with the NLRC Sub-Regional Arbitration Board in
General Santos City against GCI for death and medical benefits and damages.

On March 8, 2001, Joven Mar, Nelson’s brother, received P20,000.00 from [respondents] pursuant
to article 20(A)2 of the CBA and signed a “Certification” acknowledging receipt of the amount and
releasing AMOSUP from further liability. Merridy Jane contended that she is entitled to the
aggregate sum of Ninety Thousand Dollars ($90,000.00) pursuant to [A]rticle 20 (A)1 of the CBA.

Merridy Jane averred that the P20,000.00 already received by Joven Mar should be considered
advance payment of the total claim of US$90,000.[00].

[Herein respondents], on the other hand, asserted that the NLRC had no jurisdiction over the action
on account of the absence of employer-employee relationship between GCI and Nelson at the time
of the latter’s death. Nelson also had no claims against petitioners for sick leave allowance/medical
benefit by reason of the completion of his contract with GCI. They further alleged that private
respondent is not entitled to death benefits because petitioners are only liable for such “in case of
death of the seafarer during the term of his contract pursuant to the POEA contract” and the cause
of his death is not work-related. Petitioners admitted liability only with respect to article 20(A)2 [of
the CBA].

The Labor Arbiter ruled in favor of private respondent. It took cognizance of the case by virtue of
Article 217 (a), paragraph 6 of the Labor Code and the existence of a reasonable causal connection
between the employer-employee relationship and the claim asserted. It ordered the petitioner to
pay P4,621,300.00, the equivalent of US$90,000.00 less P20,000.00.

On appeal, [the NLRC] affirmed the Labor Arbiter’s decision as to the grant of death benefits under
the CBA but reversed the latter’s ruling as to the proximate cause of Nelson’s death.1[3]

221
Herein respondents then filed a special civil action for certiorari with the CA contending that the
NLRC committed grave abuse of discretion in affirming the jurisdiction of the NLRC over the case; in
ruling that a different provision of the CBA covers the death claim; in reversing the findings of the
Labor Arbiter that the cause of death is not work-related; and, in setting aside the release and
quitclaim executed by the attorney-in-fact and not considering the P20,000.00 already received by
Merridy Jane through her attorney-in-fact. The CA ruled that while the suit filed by Merridy Jane is a
money claim, the same basically involves the interpretation and application of the provisions in the
subject CBA. As such, jurisdiction belongs to the voluntary arbitrator and not the labor arbiter.

Issue:

Whether or not the CA committed error in ruling that the Labor Arbiter has no jurisdiction over the
case.

Held:
The CA did not committed error in ruling that the Labor Arbiter has no jurisdiction over the
case.

According to the Supreme Court, it is true that R.A. 8042 is a special law governing overseas
Filipino workers. However, a careful reading of this special law would readily show that there is no
specific provision thereunder which provides for jurisdiction over disputes or unresolved
grievances regarding the interpretation or implementation of a CBA. Section 10 of R.A. 8042, which
is cited by petitioner, simply speaks, in general, of “claims arising out of an employer-employee
relationship or by virtue of any law or contract involving Filipino workers for overseas deployment
including claims for actual, moral, exemplary and other forms of damages.” On the other hand,
Articles 217(c) and 261 of the Labor Code are very specific in stating that voluntary arbitrators
have jurisdiction over cases arising from the interpretation or implementation of collective
bargaining agreements. Stated differently, the instant case involves a situation where the special
statute (R.A. 8042) refers to a subject in general, which the general statute (Labor Code) treats in
particular. In the present case, the basic issue raised by Merridy Jane in her complaint filed with the
NLRC is: which provision of the subject CBA applies insofar as death benefits due to the heirs of
Nelson are concerned. The Court agrees with the CA in holding that this issue clearly involves the
interpretation or implementation of the said CBA. Thus, the specific or special provisions of the
Labor Code govern.
In any case, the Court agrees with petitioner's contention that the CBA is the law or contract
between the parties. Article 13.1 of the CBA entered into by and between respondent GCI and
AMOSUP, the union to which petitioner belongs, provides as follows:
The Company and the Union agree that in case of dispute or conflict in the
interpretation or application of any of the provisions of this Agreement, or enforcement of
Company policies, the same shall be settled through negotiation, conciliation or voluntary
arbitration. The Company and the Union further agree that they will use their best endeavor to
ensure that any dispute will be discussed, resolved and settled amicably by the parties hereof

222
within ninety (90) days from the date of filing of the dispute or conflict and in case of failure to
settle thereof any of the parties retain their freedom to take appropriate action.
It is clear from the above that the interpretation of the DOLE, in consultation with their
counterparts in the respective committees of the Senate and the House of Representatives, as well
as the DFA and the POEA is that with respect to disputes involving claims of Filipino seafarers
wherein the parties are covered by a collective bargaining agreement, the dispute or claim should
be submitted to the jurisdiction of a voluntary arbitrator or panel of arbitrators. It is only in the
absence of a collective bargaining agreement that parties may opt to submit the dispute to either
the NLRC or to voluntary arbitration. It is elementary that rules and regulations issued by
administrative bodies to interpret the law which they are entrusted to enforce, have the force of
law, and are entitled to great respect.2[8] Such rules and regulations partake of the nature of a
statute and are just as binding as if they have been written in the statute itself.3[9] In the instant
case, the Court finds no cogent reason to depart from this rule.
The above interpretation of the DOLE, DFA and POEA is also in consonance with the policy of the
state to promote voluntary arbitration as a mode of settling labor disputes. No less than the
Philippine Constitution provides, under the third paragraph, Section 3, Article XIII, thereof that
“[t]he State shall promote the principle of shared responsibility between workers and employers
and the preferential use of voluntary modes in settling disputes, including conciliation, and shall
enforce their mutual compliance therewith to foster industrial peace.”
Consistent with this constitutional provision, Article 211 of the Labor Code provides the
declared policy of the State “[t]o promote and emphasize the primacy of free collective bargaining
and negotiations, including voluntary arbitration, mediation and conciliation, as modes of settling
labor or industrial disputes.”
On the basis of the foregoing, the Court finds no error in the ruling of the CA that the voluntary
arbitrator has jurisdiction over the instant case.

[G.R. No. 185335, June 13, 2012]

PRUDENTIAL GUARANTEE AND ASSURANCE EMPLOYEE LABOR UNION AND SANDY T.


VALLOTA, PETITIONERS,
VS.
NATIONAL LABOR RELATIONS COMMISSION, PRUDENTIAL GUARANTEE AND ASSURANCE
INC., AND/OR JOCELYN RETIZOS, RESPONDENTS

223
Facts:

Vallota commenced his employment with respondent Prudential Guarantee and Assurance, Inc.
(PGAI) on as a Junior Programmer assigned to the Electronic Data Processing (EDP) Department. He
reported directly to Gerald Dy Victory, then head of the EDP, until his replacement by respondent
Jocelyn Retizos (Retizos) sometime in 1997. In 2005, Vallota was elected to the Board of Directors of
the Union.

PGAI’s Human Resource Manager, Atty. Joaquin R. Rillo (Atty. Rillo), invited Union President, Mike
Apostol (Apostol) to his office. Atty. Rillo informed Apostol that PGAI was going to conduct an on-
the-spot security check in the Information and Technology (IT) Department. Atty. Rillo also
requested that Union representatives witness the inspection to which Apostol agreed.

The inspection team proceeded to the IT Department, and the EDP head, through PGAI network
administrator Angelo Gutierrez (Gutierrez), initiated the spot check of IT Department computers,
beginning with the one assigned to Vallota. After exploring the contents of all the folders and
subfolders in the "My Documents" folder, Gutierrez apparently did not find anything unusual with
Vallota’s computer but only a folder named "MAA." Gutierrez saved a copy of the contents of the
MAA folder in a floppy disk.

In the presence of Atty. Rillo, Vallota was informed that Retizos and Atty. Rillo would print the files
found in his computer under the folder "MAA." Vallota did not object. After the files were printed,
Vallota and the Union Secretary were asked to sign each page of the printout. Vallota, however, was
not given a copy of the printed file.

On November 14, 2005, Vallota received a memorandum directing him to explain within 72 hours
why highly confidential files were stored in his computer. The case was assigned Reference No. AC-
05-02. The same memorandum also informed him that he was being placed under preventive
suspension for 30 days effective upon receipt of the said notice. A second memorandum, also dated
November 14, 2005, notified Vallota of the extension of his preventive suspension for another 30
days, in view of the fact that the management needed more time to evaluate the administrative case
against him.
Vallota responded in writing on November 21, 2005. Three days later PGAI sent him another
memorandum requesting further details on some of the matters he raised in his response. In a letter
Vallota requested a conference, to be attended by a Union representative and counsel. In reply,
PGAI sent Vallota another memorandum which, among others, set a new deadline for Vallota to
submit his reply and evidence in his defense.In compliance with the deadline set, Vallota submitted
his reply-memorandum, outlining his response to the charges.

Meanwhile, the Union sent a letter to PGAI President Philip K. Rico (Rico) requesting that a
grievance committee be convened and that the contents of the computers of other IT personnel be
similarly produced. The request for the convening of a grievance committee was ignored. On
December 21, 2005, Vallota was given a notice of termination of his employment effective January

224
10, 2006 on the ground of loss of trust and confidence. The decision (AC-05-02) was embodied in a
memorandum dated December 21, 2005.

Thus, the petitioners filed a complaint for illegal dismissal with claims for full backwages, moral and
exemplary damages, and attorney’s fees.

Labor Arbiter Aliman D. Mangandog (LA) rendered a decision in favor of the petitioners, declaring
the dismissal of complainant Vallota illegal. The LA held that PGAI failed to meet its burden of
evidence, and the conflicting claims of the parties were resolved in favor of Vallota for failure of
PGAI to adduce substantial evidence to support its claim. The LA further held that the dismissal was
not commensurate to the misconduct complained of, especially considering that it was Vallota’s
first offense.

The respondents filed their Memorandum of Appeal docketed as NLRC NCR. National Labor
Relations Commission (NLRC) issued its Resolution dismissing the appeal on the ground that the
respondents failed to submit a certificate of non-forum shopping in accordance with the Rules of
Procedure of the NLRC.

The respondents filed their Motion for Reconsideration but the NLRC granted the respondents’
motion for reconsideration and reversed and set aside the decision of the LA.
The petitioners filed a petition for certiorari with the CA but the CA denied the petition for lack of
merit, and sustained the award of the NLRC.

Issue:

(1) Whether Vallota was validly dismissed on the ground of loss of trust and confidence; and

(2) Whether the requirements of procedural due process for termination were observed.

Held:

(1) Whether Vallota was validly dismissed on the ground of loss of trust and confidence

According to the Supreme Court, it is clear that Article 282(c) of the Labor Code allows an employer
to terminate the services of an employee for loss of trust and confidence. The right of employers to
dismiss employees by reason of loss of trust and confidence is well established in jurisprudence.

The first requisite for dismissal on the ground of loss of trust and confidence is that the
employee concerned must be one holding a position of trust and confidence. Verily, We must first
determine if respondent holds such a position.

225
There are two (2) classes of positions of trust. The first class consists of managerial employees.
They are defined as those vested with the powers or prerogatives to lay down management policies
and to hire, transfer suspend, lay-off, recall, discharge, assign or discipline employees or effectively
recommend such managerial actions. The second class consists of cashiers, auditors, property
custodians, etc. They are defined as those who in the normal and routine exercise of their functions,
regularly handle significant amounts of money or property.

The second requisite is that there must be an act that would justify the loss of trust and
confidence. Loss of trust and confidence to be a valid cause for dismissal must be based on a willful
breach of trust and founded on clearly established facts. The basis for the dismissal must be clearly
and convincingly established but proof beyond reasonable doubt is not necessary.
To be a valid ground for dismissal, loss of trust and confidence must be based on a willful breach of
trust and founded on clearly established facts. A breach is willful if it is done intentionally,
knowingly and purposely, without justifiable excuse, as distinguished from an act done carelessly,
thoughtlessly, heedlessly or inadvertently. It must rest on substantial grounds and not on the
employer’s arbitrariness, whims, caprices or suspicion; otherwise, the employee would remain
eternally at the mercy of the employer. Further, in order to constitute a just cause for dismissal, the
act complained of must be work-related and show that the employee concerned is unfit to continue
working for the employer. Such ground for dismissal has never been intended to afford an occasion
for abuse because of its subjective nature.
In this case, there was no other evidence presented to prove fraud in the manner of securing or
obtaining the files found in Vallota’s computer. In fact, aside from the presence of these files in
Vallota’s hard drive, there was no other evidence to prove any gross misconduct on his part. There
was no proof either that the presence of such files was part of an attempt to defraud his employer
or to use the files for a purpose other than that for which they were intended. If anything, the
presence of the files reveals some degree of carelessness or neglect in his failure to delete them, but
it is an extremely farfetched conclusion bordering on paranoia to state that it is part of a larger
conspiracy involving corporate espionage.
Moreover, contrary to the respondents’ allegations, the MAA files found in Vallota’s computer, the
prospectus and corporate profile, are not sensitive corporate documents. These are documents
routinely made available to the public, and serve as means to inform the public about the company
and to disseminate information about the products it sells or the services it provides, in order that
potential clients may make a sound and informed decision whether or not to purchase or avail of
such goods and services.
If anything, the presence of the files would merely merit the development of some suspicion on the
part of the employer, but should not amount to a loss of trust and confidence such as to justify the
termination of his employment. Such act is not of the same class, degree or gravity as the acts that
have been held to be of such character. While Vallota’s act or omission may have been done
carelessly, it falls short of the standard required for termination of employment. It does not
manifest either that the employee concerned is unfit to continue working for his employer.
Termination of employment is a drastic measure reserved for the most serious of offenses. When
the act complained of is not so grave as to result in a complete loss of trust and confidence, a lower
penalty such as censure, warning, or even suspension, would be more circumspect. This is of

226
particular significance here where during Vallota’s ten years of service to PGAI, not once was he
ever warned or reprimanded for such printing services.

(2) Whether the requirements of procedural due process for termination were observed.

This interpretation of Section 2(d), Rule I of the Implementing Rules of Book VI of the Labor Code
reasonably implements the "ample opportunity to be heard" standard under Article 277(b) of the
Labor Code without unduly restricting the language of the law or excessively burdening the
employer. This not only respects the power vested in the Secretary of Labor and Employment to
promulgate rules and regulations that will lay down the guidelines for the implementation of
Article 277(b). More importantly, this is faithful to the mandate of Article 4 of the Labor Code that
"[a]ll doubts in the implementation and interpretation of the provisions of [the Labor Code],
including its implementing rules and regulations shall be resolved in favor of labor."
In this case, the two-notice requirement was complied with. By the petitioners’ own admission,
PGAI issued to Vallota a written Notice of Charges & Preventive Suspension (Ref. No. AC-05-02) dated
November 14, 2005. After an exchange of memoranda, PGAI then informed Vallota of his dismissal
in its decision dated December 21, 2005.
Given, however, that the petitioners expressly requested a conference or a convening of a grievance
committee, following the Court’s ruling in the Perez case, which was later cited in the recent case of
Lopez v. Alturas Group of Companies, such formal hearing became mandatory. After PGAI failed to
affirmatively respond to such request, it follows that the hearing requirement was not complied
with and, therefore, Vallota was denied his right to procedural due process.
In light of the above discussion, Vallota is entitled to reinstatement and backwages, reckoned from
the date he was illegally dismissed until the finality of this decision in accordance with
jurisprudence.

[G.R. No. 187713, August 1, 2012]

RADIO PHILIPPINES NETWORK INC. ET AL.


VS.
YAP ET AL.

Facts:
Respondents were employees of RPN and former members of the RPN Employees Union (RPNEU)
whom the RPNEU's Grievance and Investigation Committee recommended for expulsion from the
union. Thereafter, RPN notified the respondents that their employment would be
terminated, whereupon the respondents filed with the LA a complaint for illegal dismissal and non-
payment of benefits.

LA rendered a decision ordering the reinstatement of the respondents with payment of backwages
and full benefits and without loss of seniority rights.

Petitioner submitted a Manifestation that it has complied with the reinstatement of the
complainants. RPN General Manager Linao informed respondents that they had been reinstated, but

227
only in the payroll. Four (4) days later, the respondents returned to RPN to collect their salaries, it
being a payday; but they were barred entry and the guards manhandled them, pulled them by the
hair and arms and pushed them back to the street.

LA then cited the petitioners for indirect contempt for "committing disobedience to lawful order."
When brought before the NLRC, the body dismissed the same and also denied the petitioners'
motion for reconsideration. It was appealed to the CA which also dismissed the petition as well as
the motion for reconsideration, for failure to submit the documents it enumerated in its Resolution.

Issue:
Whether or not the payroll reinstatement of the employees availed by RPN is proper.

Held:
Yes. The manner of reinstating a dismissed employee in the payroll generally involves an exercise
of management prerogative.

In the case of Pioneer Texturizing Corp. v. NLRC, it was held that an order reinstating a dismissed
employee is immediately self-executory without need of a writ of execution, in accordance with the
third paragraph of Article 223 of the Labor Code. The article states that the employee entitled to
reinstatement "shall either be admitted back to work under the same terms and conditions
prevailing prior to his dismissal or separation or, at the option of the employer, merely reinstated in
the payroll." Thus, even if the employee is able and raring to return to work, the option of payroll
reinstatement belongs to the employer.

The new NLRC Rules of Procedure, which took effect on January 7, 2006, now requires the
employer to submit a report of compliance within ten (10) calendar days from receipt of the LA's
decision, disobedience to which clearly denotes a refusal to reinstate. The employee need no longer
file a motion for issuance of a writ of execution, since the LA shall thereafter motu proprio issue the
writ. With the new rules, there will be no difficulty in determining the employer's intransigence in
immediately complying with the order.

It has been held that in case of strained relations or non-availability of positions, the employer is
given the option to reinstate the employee merely in the payroll, precisely in order to avoid the
intolerable presence in the workplace of the unwanted employee. The Court explained in Maranaw
Hotel Resort Corporation v. NLRC, thus:

This option [to reinstate a dismissed employee in the payroll] is based on practical considerations.
The employer may insist that the dismissal of the employee was for a just and valid cause and the
latter's presence within its premises is intolerable by any standard; or such presence would be
inimical to its interest or would demoralize the co-employees. Thus, while payroll reinstatement
would in fact be unacceptable because it sanctions the payment of salaries to one not rendering
service, it may still be the lesser evil compared to the intolerable presence in the workplace of an
unwanted employee.

228
The circumstances of the present case have more than amply shown that the physical restoration of
the respondents to their former positions would be impractical and would hardly promote the best
interest of both parties. Respondents have accused the petitioners of being directly complicit in the
plot to expel them from the union and to terminate their employment, while petitioners have
charged the respondents with trying to sabotage the peace of the workplace in "furthering their
dispute with the union." The resentment and enmity between the parties have so strained their
relationship and even provoked antipathy and antagonism, as amply borne out by the physical
clashes that had ensued every time the respondents attempted to enter the RPN compound, that
respondents' presence in the workplace will not only be distracting but even disruptive, to say the
least.

The proposal to pay the respondents' salaries through ATM cards, now a wide practice cannot be
said to be prejudicial or oppressive since it would not entail any unusual effort by the respondents
to collect their money. As to the respondents' demand to be paid their salaries on the 15th and 30th
of the month along with the other employees, instead of on the 5th and 20th days, petitioners
reason that the salaries must be staggered due to RPN's erratic cash flows. The law only requires
that the fortnightly intervals be observed.

[G.R. No. 197309, October 10, 2012]

ACE NAVIGATION CO. INC.


VS.
FERNANDEZ, ET AL.

Facts:
Seaman Teodorico Fernandez, assisted by his wife Glenita, filed with the NLRC a complaint for
disability benefits.

The petitioners moved to dismiss the complaint, contending that the labor arbiter had no
jurisdiction over the dispute. They argued that exclusive original jurisdiction is with the voluntary
arbitrator or panel of voluntary arbitrators, pursuant to Section 29 of the POEA Standard
Employment Contract, since the parties are covered by the AMOSUP-TCC or AMOSUP-VELA. Under
Section 14 of the CBA, a dispute between a seafarer and the company shall be settled through the
grievance machinery and mandatory voluntary arbitration.

The LA denied the motion to dismiss, holding that under Section 10 of RA 8042, the labor arbiter
has original and exclusive jurisdiction over money claims arising out of an employer-employee
relationship or by virtue of any law or contract, notwithstanding any provision of law to the
contrary. NLRC agreed with the LA that the case involves a money claim and is within the
jurisdiction of the labor arbiter.

229
CA also rejected the petitioners' submission and stressed that the jurisdiction of voluntary
arbitrators is limited to the seafarers' claims which do not fall within the labor arbiter's original and
exclusive jurisdiction or even in cases where the labor arbiter has jurisdiction, the parties have
agreed in unmistakable terms (through their CBA) to submit the case to voluntary arbitration.
Issue:
Who has the original and exclusive jurisdiction over Fernandez's disability claim — the labor
arbiter under Section 10 of R.A. No. 8042, as amended, or the voluntary arbitration mechanism as
prescribed in the parties' CBA and the POEA-SEC?

Held:
The answer lies in the State's labor relations policy laid down in the Constitution and fleshed out in
the enabling statute, the Labor Code. Section 3, Article XIII (on Social Justice and Human Rights) of
the Constitution declares:

The State shall promote the principle of shared responsibility between workers and employers and
the preferential use of voluntary modes in settling disputes, including conciliation, and shall enforce
their mutual compliance therewith to foster industrial peace.

Article 260 of the Labor Code (Grievance machinery and voluntary arbitration) states:
The parties to a Collective Bargaining Agreement shall include therein provisions that will ensure
the mutual observance of its terms and conditions. They shall establish a machinery for the
adjustment and resolution of grievances arising from the interpretation or implementation of their
Collective Bargaining Agreement and those arising from the interpretation or enforcement of
company personnel policies.
Article 261 of the Labor Code (Jurisdiction of Voluntary Arbitrators or panel of Voluntary
Arbitrators), on the other hand, reads in part:

The Voluntary Arbitrator or panel of Voluntary Arbitrators shall have original and exclusive
jurisdiction to hear and decide all unresolved grievances arising from the interpretation or
implementation of the Collective Bargaining Agreement and those arising from the interpretation
or enforcement of company personnel policies[.]

Article 262 of the Labor Code (Jurisdiction over other labor disputes) declares:
The Voluntary Arbitrator or panel of Voluntary Arbitrators, upon agreement of the parties, shall
also hear and decide all other labor disputes including unfair labor practices and bargaining
deadlocks.

Further, the POEA-SEC, which governs the employment of Filipino seafarers, provides in its Section
29 on Dispute Settlement Procedures:

In cases of claims and disputes arising from this employment, the parties covered by a
collective bargaining agreement shall submit the claim or dispute to the original and
exclusive jurisdiction of the voluntary arbitrator or panel of voluntary arbitrators. If the

230
parties are not covered by a collective bargaining agreement, the parties may at their option submit
the claim or dispute to either the original and exclusive jurisdiction of the National Labor Relations
Commission (NLRC), pursuant to Republic Act (RA) 8042 otherwise known as the Migrant Workers
and Overseas Filipinos Act of 1995 or to the original and exclusive jurisdiction of the voluntary
arbitrator or panel of voluntary arbitrators. If there is no provision as to the voluntary arbitrators
to be appointed by the parties, the same shall be appointed from the accredited voluntary
arbitrators of the National Conciliation and Mediation Board of the Department of Labor and
Employment. [emphasis ours]

Under the above-quoted constitutional and legal provisions, the voluntary arbitrator or panel of
voluntary arbitrators has original and exclusive jurisdiction over Fernandez's disability claim.
There is no dispute that the claim arose out of Fernandez's employment with the petitioners and
that their relationship is covered by a CBA — the AMOSUP/TCC or the AMOSUP-VELA CBA. The
CBA provides for a grievance procedure for the resolution of grievances or disputes which occur
during the employment relationship and, like the grievance machinery created under Article 261 of
the Labor Code, it is a two-tiered mechanism, with voluntary arbitration as the last step.

Contrary to the CA's reading of the CBA's Article 14, there is unequivocal or unmistakable language
in the agreement which mandatorily requires the parties to submit to the grievance procedure any
dispute or cause of action they may have against each other. The relevant provisions of the CBA
state:
14.6Any Dispute, grievance, or misunderstanding concerning any ruling, practice, wages or
working conditions in the COMPANY or any breach of the Contract of Employment, or any
dispute arising from the meaning or application of the provisions of this Agreement or a
claim of violation thereof or any complaint or cause of action that any such Seaman may
have against the COMPANY, as well as complaints which the COMPANY may have against
such Seaman shall be brought to the attention of the GRIEVANCE RESOLUTION COMMITTEE
before either party takes any action, legal or otherwise. Bringing such a dispute to the
Grievance Resolution Committee shall be unwaivable prerequisite or condition precedent
for bringing any action, legal or otherwise, in any forum and the failure to so refer the
dispute shall bar any and all legal or other actions.

14.7a)If by reason of the nature of the Dispute, the parties are unable to amicably settle the
dispute, either party may refer the case to a MANDATORY ARBITRATION COMMITTEE. The
MANDATORY ARBITRATION COMMITTEE shall consist of one representative to be designated by
the UNION, and one representative to be designated by the COMPANY and a third member who
shall act as Chairman and shall be nominated by mutual choice of the parties. . . .
h)Referral of all unresolved disputes from the Grievance Resolution Committee to the
Mandatory Arbitration Committee shall be unwaivable prerequisite or condition precedent
for bringing any action, claim, or cause of action, legal or otherwise, before any court,
tribunal, or panel in any jurisdiction. The failure by a party or seaman to so refer and avail
oneself to the dispute resolution mechanism contained in this action shall bar any legal or
other action. All parties expressly agree that the orderly resolution of all claims in the

231
prescribed manner served the interests of reaching settlements or claims in an orderly and
uniform manner, as well as preserving peaceful and harmonious labor relations between
seaman, the Union, and the Company.
What might have caused the CA to miss the clear intent of the parties in prescribing a grievance
procedure in their CBA is, as the petitioners' have intimated, the use of the auxiliary verb "may" in
Article 14.7 (a) of the CBA which, to reiterate, provides that "[i]f by reason of the nature of the
Dispute, the parties are unable to amicably settle the dispute, either party may refer the case
to a MANDATORY ARBITRATION COMMITTEE."

While the CA did not qualify its reading of the subject provision of the CBA, it is reasonable to
conclude that it viewed as optional the referral of a dispute to the mandatory arbitration committee
when the parties are unable to amicably settle the dispute.
We find this a strained interpretation of the CBA provision. The CA read the provision separately, or
in isolation of the other sections of Article 14, especially 14.7 (h), which, in clear, explicit language,
states that the "referral of all unresolved disputes from the Grievance Resolution Committee
to the Mandatory Arbitration Committee shall be unwaivable prerequisite or condition
precedent for bringing any action, claim, or cause of action, legal or otherwise, before any
court, tribunal, or panel in any jurisdiction" and that the failure by a party or seaman to so
refer the dispute to the prescribed dispute resolution mechanism shall bar any legal or other
action.

Read in its entirety, the CBA's Article 14 (Grievance Procedure) unmistakably reflects the parties'
agreement to submit any unresolved dispute at the grievance resolution stage to mandatory
voluntary arbitration under Article 14.7 (h) of the CBA. And, it should be added that, in compliance
with Section 29 of the POEA-SEC which requires that in cases of claims and disputes arising from a
seafarer's employment, the parties covered by a CBA shall submit the claim or dispute to the
original and exclusive jurisdiction of the voluntary arbitrator or panel of voluntary arbitrators.
Consistent with this finding, Fernandez's contention — that his complaint for disability benefits is a
money claim that falls within the original and exclusive jurisdiction of the labor arbiter under
Section 10 of R.A. No. 8042 — is untenable. We likewise reject his argument that he never referred
his claim to the grievance machinery (so that no unresolved grievance exists as required under
Article 261 of the Labor Code), and that the parties to the case are not the union and the
employer. Needless to state, no such distinction exists in the parties' CBA and the POEA-SEC.

It bears stressing at this point that we are upholding the jurisdiction of the voluntary arbitrator or
panel of voluntary arbitrators over the present dispute, not only because of the clear language of
the parties' CBA on the matter; more importantly, we so uphold the voluntary arbitrator's
jurisdiction, in recognition of the State's express preference for voluntary modes of dispute
settlement, such as conciliation and voluntary arbitration as expressed in the Constitution, the law
and the rules.

In closing, we quote with approval a most recent Court pronouncement on the same issue, thus —
It is settled that when the parties have validly agreed on a procedure for resolving

232
grievances and to submit a dispute to voluntary arbitration then that procedure should be
strictly observed.

[G.R. NO. 196539, October 10, 2012]

PORTILLO
VS.
RUDOLF LIETZ, INC.

Facts:
In a letter agreement, signed by individual respondent Rudolf Lietz and conformed to by
Portillo, the latter (Portillo) was hired by the former under the conditions that Portillo “will not
engage in any other gainful employment by [her]self or with any other company either directly or
indirectly without written consent of [Lietz Inc.]” and “a breach of which will render [Portillo] liable
to [Lietz Inc.] for liquidated damages.”
On her tenth (10th) year of service with Lietz Inc., Portillo was promoted to Sales
Representative. In this regard, Portillo signed another letter agreement containing a "Goodwill
Clause:"
It remains understood and you agreed that, on the termination of your employment by act of
either you or [Lietz Inc.], and for a period of three (3) years thereafter, you shall not engage directly
or indirectly as employee, manager, proprietor, or solicitor for yourself or others in a similar or
competitive business or the same character of work which you were employed by [Lietz Inc.] to do
and perform. Should you breach this good will clause of this Contract, you shall pay [Lietz Inc.] as
liquidated damages the amount of 100% of your gross compensation over the last 12 months, it
being agreed that this sum is reasonable and just.5
Three (3) years thereafter Portillo resigned from Lietz Inc. During her exit interview, Portillo
declared that she intended to engage in business—a rice dealership, selling rice in wholesale.
On 15 June 2005, Lietz Inc. accepted Portillo’s resignation and reminded her of the "Goodwill
Clause" in the last letter agreement she had signed.
Subsequently, Lietz Inc. learned that Portillo had been hired by Ed Keller Philippines, Limited
to head its Pharma Raw Material Department. Ed Keller Limited is purportedly a direct competitor
of Lietz Inc.
Meanwhile, Portillo’s demands from Lietz Inc. for the payment of her remaining salaries and
commissions went unheeded.
On 14 September 2005, Portillo filed a complaint with the NLRC for non-payment of 11⁄2
months’ salary, two (2) months’ commission, 13th month pay, plus moral, exemplary and actual
damages and attorney’s fees.
In its position paper, Lietz Inc. admitted liability for Portillo’s money claims in the total
amount of P110,662.16. However, Lietz Inc. raised the defense of legal compensation: Portillo’s
money claims should be offset against her liability to Lietz Inc. for liquidated damages in the
amount of ₱869,633.097 for Portillo’s alleged breach of the "Goodwill Clause" in the employment
contract when she became employed with Ed Keller Philippines, Limited.

233
On 25 May 2007, the Labor Arbiter granted Portillo’s complaint, ordering Lietz, Inc. to pay
Portillo the amount of Php110,662.16, representing her salary and commissions, including 13th
month pay.8 On appeal by respondents Lietz Inc., the NLRC affirmed the ruling of the Labor Arbiter.
The motion for reconsideration was denied by NLRC.
Lietz Inc. filed a petition for certiorari before the Court of Appeals, alleging grave abuse of
discretion in the labor tribunals’ rulings. The CA initially affirmed the labor tribunals, but on motion
for reconsideration, modified its previous decision. While upholding the monetary award in favor of
Portillo in the aggregate sum of ₱110,662.16, the CA allowed legal compensation or set-off of such
award of monetary claims by her liability to Lietz Inc. for liquidated damages arising from her
violation of the "Goodwill Clause" in her employment contract with them.10 Portillo’s motion for
reconsideration was denied. Hence, this petition for certiorari before the SC.

Issue:
Whether Portillo’s money claims for unpaid salaries may be offset against Lietz Inc.’s claim
for liquidated damages.

Held:
Paragraph 4 of Article 217 of the Labor Code appears to have caused the reliance by the CA on
the "causal connection between Portillo’s monetary claims against respondents and the latter’s
claim from liquidated damages against the former."
Art. 217. Jurisdiction of Labor Arbiters and the Commission. – (a) Except as otherwise
provided under this code, the Arbiters shall have original and exclusive jurisdiction to hear and
decide, within thirty (30) calendar days after the submission of the case by the parties for decision
without extension, even in the absence of stenographic notes, the following case involving all
workers, whether agricultural or nonagricultural:
xxxx 4. Claims for actual, moral, exemplary and other forms of damages arising from the
employer-employee relations; (Underscoring supplied)
Evidently, the CA is convinced that the claim for liquidated damages emanates from the
"Goodwill Clause of the employment contract and, therefore, is a claim for damages arising from the
employer-employee relations."
As early as Singapore Airlines Limited v. Paño,18 we established that not all disputes between an
employer and his employee(s) fall within the jurisdiction of the labor tribunals. We differentiated
between abandonment per se and the manner and consequent effects of such abandonment and
ruled that the first, is a labor case, while the second, is a civil law case.
Stated differently, petitioner seeks protection under the civil laws and claims no
benefits under the Labor Code. The primary relief sought is for liquidated damages for
breach of a contractual obligation. The other items demanded are not labor benefits
demanded by workers generally taken cognizance of in labor disputes, such as payment of
wages, overtime compensation or separation pay. The items claimed are the natural
consequences flowing from breach of an obligation, intrinsically a civil dispute.19 (Emphasis
supplied)
Subsequent rulings amplified the teaching in Singapore Airlines. The reasonable causal
connection rule was discussed. Thus, in San Miguel Corporation v. National Labor Relations

234
Commission,20 we held:
xxx The Court, therefore, believes and so holds that the "money claims of workers"
referred to in paragraph 3 of Article 217 embraces money claims which arise out of or in
connection with the employer-employee relationship, or some aspect or incident of such
relationship. Put a little differently, that money claims of workers which now fall within the
original and exclusive jurisdiction of Labor Arbiters are those money claims which have
some reasonable causal connection with the employer- employee relationship.21 (Emphasis
supplied)
We thereafter ruled that the "reasonable causal connection with the employer-employee
relationship" is a requirement not only in employees’ money claims against the employer but is,
likewise, a condition when the claimant is the employer.
In Dai-Chi Electronics Manufacturing Corporation v. Villarama, Jr.,22 which reiterated the San
Miguel ruling and allied jurisprudence, we pronounced that a non-compete clause, as in the
"Goodwill Clause" referred to in the present case, with a stipulation that a violation thereof makes
the employee liable to his former employer for liquidated damages, refers to post- employment
relations of the parties. We iterated that Article 217, paragraph 4 does not automatically cover all
disputes between an employer and its employee(s). We noted that the cause of action was within
the realm of Civil Law, thus, jurisdiction over the controversy belongs to the regular courts. At
bottom, we considered that the stipulation referred to post- employment relations of the parties.
That the "Goodwill Clause" in this case is likewise a postemployment issue should brook no
argument. There is no dispute as to the cessation of Portillo’s employment with Lietz Inc.23 She
simply claims her unpaid salaries and commissions, which Lietz Inc. does not contest. At that
juncture, Portillo was no longer an employee of Lietz Inc.24 The "Goodwill Clause" or the "Non-
Compete Clause" is a contractual undertaking effective after the cessation of the employment
relationship between the parties. In accordance with jurisprudence, breach of the undertaking is a
civil law dispute, not a labor law case.
It is clear, therefore, that while Portillo’s claim for unpaid salaries is a money claim that arises
out of or in connection with an employer-employee relationship, Lietz Inc.’s claim against Portillo
for violation of the goodwill clause is a money claim based on an act done after the cessation of the
employment relationship. And, while the jurisdiction over Portillo’s claim is vested in the labor
arbiter, the jurisdiction over Lietz Inc.’s claim rests on the regular courts.
In the case at bar, the difference in the nature of the credits that one has against the other,
conversely, the nature of the debt one owes another, which difference in turn results in the
difference of the forum where the different credits can be enforced, prevents the application of
compensation. Simply, the labor tribunal in an employee’s claim for unpaid wages is without
authority to allow the compensation of such claims against the post employment claim of the
former employer for breach of a post employment condition. The labor tribunal does not have
jurisdiction over the civil case of breach of contract.
Indeed, the application of compensation in this case is effectively barred by Article 113 of the
Labor Code which prohibits wage deductions except in three circumstances:
ART. 113. Wage Deduction. – No employer, in his own behalf or in behalf of any person, shall
make any deduction from wages of his employees, except: (a) In cases where the worker is insured
with his consent by the employer, and the deduction is to recompense the employer for the amount

235
paid by him as premium on the insurance;
(b) For union dues, in cases where the right of the worker or his union to check-off has been
recognized by the employer or authorized in writing by the individual worker concerned; and (c) In
cases where the employer is authorized by law or regulations issued by the Secretary of Labor.

[G.R. NO. 198423, October 23, 2012]

GONZALES
VS.
SOLID CEMENT CORPORATION AND ALLEN QUERUBIN

Facts:
The current petition arose from the execution of the final and executory judgment in the
parties’ illegal dismissal dispute (referred to as "original case," docketed in this Court as G.R. No.
165330 and entitled Solid Cement Corporation, et al. v. Leo Gonzales). The Labor Arbiter (LA)
resolved the case at his level on December 12, 2000. Since the LA found that an illegal dismissal
took place, the company reinstated petitioner Gonzales in the payroll on January 22, 2001.2
In the meanwhile, the parties continued to pursue the original case on the merits. The case
was appealed to the National Labor Relations Commission (NLRC) and from there to the Court of
Appeals (CA) on a petition for certiorari under Rule 65 of the Rules of Court. The LA’s ruling of
illegal dismissal was largely left undisturbed in these subsequent recourses. The original case
eventually came to this Court. In our Resolutions of March 9, 20053 and June 8, 2005,4 we denied
the petition of respondent Solid Cement Corporation (Solid Cement) for lack of merit. Our ruling
became final and entry of judgment took place on July 12, 2005.
Soon after its finality, the original case was remanded to the LA for execution. The LA decision
dated December 12, 2000 declared the respondents guilty of illegal dismissal and ordered the
reinstatement of Gonzales to his former position "with full backwages and without loss of seniority
rights and other benefits."5 Under this ruling, as modified by the NLRC ruling on appeal, Gonzales
was awarded the following:
(1) Backwages in the amount of P636,633.33; (2) Food and Transportation Allowance in the
amount of P18,080.00; (3) Moral damages in the amount of P100,000.00; (4) Exemplary damages in
the amount of P 50,000.00; and (5) Ten percent (10%) of all sums owing to the petitioner as
attorney’s fees.
Actual reinstatement and return to work for Gonzales (who had been on payroll
reinstatement since January 22, 2001) came on July 15, 2008.6
When Gonzales moved for the issuance of an alias writ of execution on August 4, 2008, he
included several items as components in computing the amount of his backwages. Acting on the
motion, the LA added P57,900.00 as rice allowance and P14,675.00 as medical reimbursement
(with the company’s apparent conformity). Under the LA’s execution order dated August 18, 2009,
Gonzales was entitled to a total of P965,014.15.7
The NLRC, in its decision8 dated February 19, 2010 and resolution dated May 18, 2010,
modified the LA’s execution order. The NLRC differed from the LA on the actual details of
implementation and modified the latter’s ruling by including:
Additional backwages from Dec. 13, 2000 to Jan. 21, 2001 P 50, 800.009

236
Salary differentials from year 2000 until August 2008 617,517.48
13th month pay differential 51,459.79
13th month pay for years 2000 and 2001 80,000.00
12% interest from July 12, 2005 878,183.42
This ruling increased Gonzales’ entitlement to P2,805,698.04 (from P965,014.15).
On a petition for certiorari under Rule 65 of the Rules of Court, the CA set aside the NLRC’s
decision and reinstated the LA’s order, prompting Gonzales to come to the Court via a petition for
review on certiorari (docketed as G.R. No. 198423) under Rule 45 of the Rules of Court. In our
Minute Resolutions, we denied Gonzales’ Rule 45 petition. At this point came the two motions now
under consideration.

Issue:
Whether the CA correctly determined the absence or presence of grave abuse of discretion by
the NLRC.

Held:

Re-computation of awards during execution of an illegal dismissal decision.


On the execution aspect of an illegal dismissal decision, the case of Session Delights Ice Cream
and Fast Foods v. Court of Appeals (Sixth Division),16 despite its lack of a complete factual
congruence with the present case, serves as a good guide on how to approach the execution of an
illegal dismissal decision that contains a monetary award. There the SC held the CA was not in error
in confirming that a re- computation is necessary as it essentially considered the labor arbiter’s
original decision in accordance with its basic component. The SC further ratiocinated that:
xxx under the terms of the decision under execution, no essential change is made by a re-
computation as this step is a necessary consequence that flows from the nature of the illegality of
dismissal declared in that decision. A re-computation (or an original computation, if no previous
computation has been made) is a part of the law – specifically, Article 279 of the Labor Code and the
established jurisprudence on this provision – that is read into the decision. By the nature of an
illegal dismissal case, the reliefs continue to add on until full satisfaction, as expressed under Article
279 of the Labor Code. The re-computation of the consequences of illegal dismissal upon execution
of the decision does not constitute an alteration or amendment of the final decision being
implemented. The illegal dismissal ruling stands; only the computation of monetary consequences
of this dismissal is affected and this is not a violation of the principle of immutability of final
judgments. xxx
The re-computation of the amounts still due took off from the LA’s decision that contained the
itemized and computed dispositive portion as of the time the LA rendered his judgment. It was
necessary because time transpired between the LA’s decision and the final termination of the case
on appeal, during which time the illegally dismissed employee should have been paid his salary and
benefit entitlements.
The current petition only generally involves a determination of the scope of the awards that
include the backwages.
The components of the backwages:

237
a. Salary and 13th month differential due after dismissal In the case of BPI Employees Union –
Metro Manila and Zenaida Uy v. Bank of the Philippine Islands and Bank of the Philippine Islands v.
BPI Employees Union – Metro Manila and Zenaida Uy,21 the Court ruled that in computing
backwages, salary increases from the time of dismissal until actual reinstatement, and benefits not
yet granted at the time of dismissal are excluded. Hence, we cannot fault the CA for finding that the
NLRC committed grave abuse of discretion in awarding the salary differential amounting to
P617,517.48 and the 13th month pay differentials amounting to P51,459.48 that accrued
subsequent to Gonzales’ dismissal.
b. Legal interest of 12% on total judgment
However, based on the same BPI case, Gonzales is entitled to 12% interest on the total unpaid
judgment amount, from the time the Court’s decision (on the merits in the original case) became
final. When the CA reversed the NLRC and reinstated the LA’s ruling (which did not order payment
of interest), the CA overstepped the due bounds of its jurisdiction under a certiorari petition as it
acted on the basis of wrong considerations and outside the contemplation of the law on the legal
interests that final orders and rulings on forbearance of money should bear.
c. Additional backwages and 13th month pay
We reach the same conclusion on the other deletions the CA made, particularly on the
deletion of the 13th month pay for 2000-2001, amounting to P80,000.00, and the additional
backwages for the period of December 13, 2000 to January 21, 2001, amounting to P50,800.00. We
note in this regard that the execution proceedings were conducted before the LA issued an Order
requiring the payment of P965,014.15 in Gonzales’ favor. An appeal of this computation to the
NLRC to question the LA’s determination of the amount due throws the LA’s determination wide
open for the NLRC’s review. In granting these monetary reliefs, the NLRC reasoned that –
Since there is no showing that complainant was paid his salaries from the time when he
should have been immediately reinstated until his payroll reinstatement, he is entitled thereto.25
(emphasis ours) SC found no reason to disturb the findings of respondent NLRC that the entire
amount of commissions was not paid, this by reason of the evident failure of herein petitioners to
present evidence that full payment thereof has been made. It is a basic rule in evidence that each
party must prove his affirmative allegations. These amounts are not excluded from the concept of
backwages as the salaries fell due after Gonzales should have been reinstated, while the 13th month
pay fell due for the same period by legal mandate. These are entitlements that cannot now be
glossed over if the final decision on the merits in this case were to be respected.

[G.R. No. 192650, October 24, 2012]

FELIX MARTOS, ET AL
VS.
NEW SAN JOSE BUILDERS, INC.

Facts:
New San Jose Builders, Inc (petitioner)is a domestic corporation duly organized and existing under
the laws of the Philippines and is engaged in the construction of road, bridges, buildings, and low
cost houses primarily for the government. One of the projects of Petitioner is the San Jose Plains

238
Project (hereafter SJPP), located in Montalban, Rizal. SJPP, which is also known as the "Erap City"
calls for the construction of low cost housing, which are being turned over to the National Housing
Authority to be awarded to deserving poor families.
Private respondents alleged that, on various dates, petitioner hired them on different positions.
Sometime in 2000, petitioner was constrained to slow down and suspend most of the works on the
SJPP project due to lack of funds of the National Housing Authority. Thus, the workers were
informed that many of them [would] be laid off and the rest would be reassigned to other projects.
Some were laid off while others were retained and were issued new appointment papers to their
respective assignments, indicating therein that they are project employees. However, these who
were retained refused to sign the appointment papers as project employees and subsequently
refused to continue to work.
On different dates, three (3) Complaints for Illegal Dismissal and for money claims were filed before
the NLRC against petitioner and Jose Acuzar, by private respondents who claimed to be the former
employees of petitioner.
Petitioner denies that private respondents were illegally dismissed, and alleged that they were
project employees, whose employments were automatically terminated upon completion of the
project for which they were hired. On the other hand, private respondents claim that petitioner
hired them as regular employees, continuously and without interruption, until their dismissal on
February 28, 2002.
Subsequently, the three Complaints were consolidated.

Ruling of the Labor Arbiter


As earlier stated, on May 23, 2003, the LA handed down a decision declaring, among others, that
petitioner Felix Martos (Martos) was illegally dismissed and entitled to separation pay, backwages
and other monetary benefits; and dismissing, without prejudice, the complaints/claims of the other
complainants (petitioners).

Ruling of The NLRC


Both parties appealed. On July 30, 2008, NLRC modified and partially granted the appeal of private
respondents (workers). Petitioner was are ordered to reinstate all the complainants to their former
positions, without loss of seniority rights and with full backwages, counted from the time their
compensation was withheld from them until actual reinstatement.
Petitioner was likewise ordered to pay complainants their salary differentials, service incentive
leave pay, and 13th month pay, using, as basis, the computation made on the claims of complainant
Felix Martos.

Ruling Of The CA
On July 31, 2009, the CA rendered a decision reversing the NLRC and reinstating the LA’s decision.

Issues:

1. Whether or not the CA was correct in dismissing the complaints filed by those complainants
(workers) who failed to verify their position papers; and

239
2. Whether or not Martos should be reinstated.

Held:

1. Yes, CA was correct in dismissing the complaints of the workers who failed to verify their
position papers.

Sections 4 and 5 of Rule 7 of the 1997 Rules of Civil Procedure provide:


SEC. 4.Verification. – Except when otherwise specifically required by law or rule, pleadings need not
be under oath, verified or accompanied by affidavit.
A pleading is verified by an affidavit that the affiant has read the pleadings and that the allegations
therein are true and correct of his personal knowledge or based on authentic records.
A pleading required to be verified which contains a verification based on "information and belief"
or upon "knowledge, information and belief" or lacks a proper verification, shall be treated as an
unsigned pleading.
SEC. 5.Certification against forum shopping. – The plaintiff or principal party shall certify under
oath in the complaint or other initiatory pleading asserting a claim for relief, or in a sworn
certification annexed thereto and simultaneously filed therewith:
(a) that he has not theretofore commenced any action or filed any claim involving the same issues
in any court, tribunal or quasi-judicial agency and, to the best of his knowledge, no such other
action or claim is pending therein; (b) if there is such other pending action or claim, a complete
statement of the present status thereof; and (c) if he should thereafter learn that the same or
similar action or claim has been filed or is pending, he shall report that fact within five (5) days
therefrom to the court wherein his aforesaid complaint or initiatory pleading has been filed.
Failure to comply with the foregoing requirements shall not be curable by mere amendment
of the complaint or other initiatory pleading but shall be cause for the dismissal of the case
without prejudice, unless otherwise provided, upon motion and after hearing. The
submission of a false certification or non-compliance with any of the undertakings therein shall
constitute indirect contempt of court, without prejudice to the corresponding administrative and
criminal actions. If the acts of the party or his counsel clearly constitute willful and deliberate forum
shopping, the same shall be ground for summary dismissal with prejudice and shall constitute
direct contempt, as well as a cause for administrative sanctions. x xx. [Emphases supplied]
The verification requirement is significant, as it is intended to secure an assurance that the
allegations in the pleading are true and correct and not the product of the imagination or a matter
of speculation, and that the pleading is filed in good faith.10 Verification is deemed substantially
complied with when, as in this case, one who has ample knowledge to swear to the truth of the
allegations in the complaint or petition signs the verification, and when matters alleged in the
petition have been made in good faith or are true and correct.11
The absence of a proper verification is cause to treat the pleading as unsigned and dismissible.12
The lone signature of Martos would have been sufficient if he was authorized by his co-petitioners
to sign for them. Unfortunately, petitioners failed to adduce proof that he was so authorized.

240
The liberal construction of the rules may be invoked in situations where there may be some
excusable formal deficiency or error in a pleading, provided that the same does not subvert the
essence of the proceeding and it at least connotes a reasonable attempt at compliance with the
rules. Besides, fundamental is the precept that rules of procedure are meant not to thwart but to
facilitate the attainment of justice; hence, their rigid application may, for deserving reasons, be
subordinated by the need for an apt dispensation of substantial justice in the normal course. They
ought to be relaxed when there is subsequent or even substantial compliance, consistent with the
policy of liberality espoused by Rule 1, Section 6.14 Not being inflexible, the rule on verification
allows for such liberality.15
Considering that the dismissal of the other complaints by the LA was without prejudice, the other
complainants should have taken the necessary steps to rectify their procedural mistake after the
decision of the LA was rendered. They should have corrected this procedural flaw by immediately
filing another complaint with the correct verification this time. Surprisingly, they did not even
attempt to correct this technical blunder. Worse, they committed the same procedural error when
they filed their appeal16 with the NLRC.
The petitioners were given a chance by the CA to comply with the Rules when they filed their
motion for reconsideration, but they refused to do so. Despite the opportunity given to them to
make all of them sign the verification and certification of non-forum shopping, they still failed to
comply. Thus, the CA was constrained to deny their motion and affirm the earlier resolution.
Under the circumstances, the Court agrees with the CA that the dismissal of the other complaints
were brought about by the own negligence and passive attitude of the complainants themselves

2. No, Martos should not be reinstated because of strained relationship. Separation pay should
be given in lieu thereof.

As to Martos, the Court agrees that the reinstatement being sought by him was no longer
practicable because of strained relation between the parties. Indeed, he can no longer question this
fact. This issue was never raised or taken up on appeal before the MLRC. It was only after he lost the
appeal in the CA that he raised it.
Thus, the Court deems it fair to award separation pay in lieu of reinstatement.1âwphi1 In addition
to his separation pay. Martos is also entitled to payment of full backwages, 13th month pay, service
incentive leave pay, and attorney’s fees.
The accepted doctrine is that separation pay may avail in lieu of reinstatement if reinstatement is
no longer practical or in the best interest of the parties. Separation pay in lieu of reinstatement may
likewise be awarded if the employee decides not to be reinstated.
Under the doctrine of stained relations, the payment of separation pay is considered an acceptable
alternative to reinstatement when the latter opinion is no longer desirable or viable. On one hand,
such payment liberates the employee from what could be highly oppressive work environment. On
the other hand, it release the employer from the grossly unpalatable obligation of maintaining in its
employ a worker it could no longer trust.24

241
[G.R. No. 198357, December 10, 2012]

BUILDING CARE CORPORATION


VS.
MACARAEG

Facts:
This resolves the Petition for Review on Certiorari under Rule 45 of the Rules of Court, praying that
the Decision of the Court of Appeals on March 24, 2011, and its Resolution dated August 19, 2011,
denying petitioner’s Motion for Reconsideration be reversed and set aside.

Petitioners are in the business of providing security services to their clients. They hired respondent
as a security guard beginning August 25, 1996, assigning her at Genato Building in Caloocan City.
On March 9, 2008, respondent was relieved of her post. She was re-assigned to Bayview Park Hotel
from March 9-13, 2008, but after said period, she was allegedly no longer given any assignment.
On September 9, 2008, respondent filed a complaint against petitioners for illegal dismissal,
underpayment of salaries, non-payment of separation pay and refund of cash bond.

Respondent claimed that petitioners failed to give her an assignment for more than nine months,
amounting to constructive dismissal, and compelled her to file the complaint for illegal dismissal.
Petitioners alleged that respondent was relieved from her post as requested by the client because
of her habitual tardiness, persistent borrowing of money from employees and tenants of the client,
and sleeping on the job. Petitioners allegedly directed respondent to explain why she committed
such infractions, but respondent failed to heed such order.

Respondent then filed an administrative complaint for illegal dismissal with the PNP-Security
Agencies and Guard Supervision Division on June 18, 2008, but she did not attend the conference
hearings for said case. On May 13, 2009, the Labor Arbiter rendered a Decision, dismissing the
charge of illegal dismissal as wanting in merit but, as explained above, ordering the Respondents
Leopard Security and Investigation Agency and Rupert Protacio to pay Complainant a financial
assistance in the amount of P5,000.00.

Respondent then filed a Notice of Appeal with the National Labor Relations Commission (NLRC),
but in a Decision dated October 23, 2009, the NLRC dismissed the appeal for having been filed out
of time. And thus, elevated the case to the CA via a petition for certiorari, and on March 24, 2011,
the CA promulgated its Decision, the petition for certiorari is GRANTED. The Decision dated
October 23, 2009 and Resolution dated March 2, 2010 rendered by public respondent in NLRC are
reversed and SET ASIDE, and in lieu thereof, a new judgment is entered declaring petitioner to have
been illegally dismissed and DIRECTING private respondents to reinstate petitioner without loss of
seniority rights, benefits and privileges; and to pay her back wages and other monetary benefits
during the period of her illegal dismissal up to actual reinstatement.

242
Public respondent NLRC is DIRECTED to conduct further proceedings, for the sole purpose of
determining the amount of private respondent’s monetary liabilities in accordance with this
decision.

Issue:
Whether the CA erred in liberally applying the rules of procedure and ruling that respondent’s
appeal should be allowed and resolved on the merits despite having been filed out of time.

Held:
The Court cannot sustain the CA’s Decision.
It should be emphasized that the resort to a liberal application, or suspension of the application of
procedural rules, must remain as the exception to the well-settled principle that rules must be
complied with for the orderly administration of justice.

While procedural rules may be relaxed in the interest of justice, it is well-settled that these are tools
designed to facilitate the adjudication of cases. The relaxation of procedural rules in the interest of
justice was never intended to be a license for erring litigants to violate the rules with impunity.
To be sure, the relaxation of procedural rules cannot be made without any valid reasons proffered
for or underpinning it. The desired leniency cannot be accorded absent valid and compelling
reasons for such a procedural lapse.

We must stress that the bare invocation of “the interest of substantial justice” line is not some
magic want that will automatically compel this Court to suspend procedural rules. Procedural rules
are not to be belittled, let alone dismissed simply because their non-observance may have resulted
in prejudice to a party’s substantial rights. Utter disregard of the rules cannot be just rationalized
by harping on the policy of liberal construction.
In this case, the justifications given by the CA for its liberality by choosing to overlook the belated
filing of the appeal are, the importance of the issue raised, i.e., whether respondent was illegally
dismissed; and the belief that respondent should be “afforded the amplest opportunity for the
proper and just determination of his cause, free from the constraints of technicalities.

In this case, there was no such deprivation of due process. Respondent was able to fully present and
argue her case before the Labor Arbiter. She was accorded the opportunity to be heard. Her failure
to appeal the Labor Arbiter’s Decision cannot therefore be deemed as a deprivation of her right to
due process.

It should also be borne in mind that the right of the winning party to enjoy the finality of the
resolution of the case is also an essential part of public policy and the orderly administration of
justice. Hence, such right is just as weighty or equally important as the right of the losing party to
appeal or seek reconsideration within the prescribed period.

When the Labor Arbiter’s Decision became final, petitioners attained a vested right to said
judgment. They had the right to fully rely on the immutability of said Decision.

243
The petition is GRANTED. The Decision of the Court of Appeals dated March 24, 2011, and its
Resolution dated August 19, 2011 are SET ASIDE, and the Decision of the NLRC in ruling that the
Decision of the Labor Arbiter has become final and executor, is REINSTATED.

[G.R. No. 175900, June 10, 2013]

KAPISANANG PANGKAUNLARAN NG KANANAIHANG POTRERO, INC.


VS.
BARRENO ET AL.

Facts:
Petitioner KapisanangPangkaunlaranngKababaihangPotrero, Inc. (KPKPI) is a non-stock, non-
profit, social service oriented corporation. Sometime in November 1997, the Technology and
Livelihood Resource Center (TLRC) tapped KPKPI to participate in its microlending program and
was granted a loan for microfinance or re-lending for the poor. As such, KPKPI hired respondents
for its KPKPI Mile Program.

Respondents filed a Complaint before the Department of Labor and Employment-National Capital
Region (DOLE-NCR) for underpayment of wages, non-payment of labor standard benefits.
Respondent Barreno filed another Complaintagainst petitioners, this time for illegal dismissal with
prayer for reinstatement and payment of their money claims before the NLRC.
Respondents Ametin, Nonay, Dionisio and Casio were also verbally informed by petitioner Reyes of
their termination. This prompted the filing of their Complaintwhich was subsequently consolidated
with Barreno’s Case.

Petitioners averred that respondents committed forum shopping when they filed the NLRC CASE
during the pendency of the DOLE CASE.Respondents denied having committed forum shopping,
explaining that the DOLE CASE referred only to money claims and that it had already been
withdrawn while the NLRC CASE involves the complaint for illegal dismissal with money claims.
Meanwhile, respondents filed a Motion to Withdraw Complaintwith regard to the DOLE CASE after
having instituted the NLRC CASE. Records, however, show that the said motion was left unresolved.
Labor Arbiter (LA) found no forum shopping, holding that the subsequent dismissal of the
respondents affected the jurisdiction of the DOLE-NCR since illegal dismissal cases are beyond the
latter’s jurisdiction. Necessarily therefore, the case for money claims pending before the DOLE-NCR
had to be consolidated with the illegal dismissal case before the NLRC.

NLRC set aside the LA’s ruling and dismissed respondents’ complaints. It found respondents guilty
of forum shopping in filing the same complaint against petitioners in two (2) fora, namely the DOLE
and the NLRC.

CA agreed with the NLRC that respondents committed forum shopping in seeking their money
claims before the DOLE and the NLRC. Nonetheless, it declared that the ends of justice would be

244
better served if respondents would be given the opportunity to be heard on their complaint for
illegal dismissal.

Issue:
Whether or not the CA erred in ordering the reinstatement and remand of the NLRC CASE to the
NLRC despite its finding of forum shopping.

Held:
The Court finds that contrary to the findings of both the NLRC and the CA, respondents are not
guilty of forum shopping.

Forum shopping exists "when one party repetitively avails of several judicial remedies in different
courts, simultaneously or successively, all substantially founded on the same transactions and the
same essential facts and circumstances, and all raising substantially the same issues either pending
in, or already resolved adversely, by some other court." What is truly important to consider in
determining whether it exists or not is the vexation caused the courts and parties-litigants by a
party who asks different courts and/or administrative agencies to rule on the same or related
causes and/or grant the same or substantially the same reliefs, in the process creating the
possibility of conflicting decisions being rendered by different fora upon the same issues.
respondents did not commit forum shopping. Clearly, there is no identity of causes of action
between the cases pending with the DOLE and the NLRC. The DOLE CASE involved violations of
labor standard provisions where an employer-employee relationship exists. On the other hand, the
NLRC CASE questioned the propriety of respondents’ dismissal. At the time the DOLE CASE was
initiated, respondents’ only cause of action was petitioners’ violation of labor standard laws which
falls within the jurisdiction of the DOLE. It was only after the same was filed that respondents were
dismissed from employment, prompting the filing of the NLRC CASE, which is within the mantle of
the NLRC’s jurisdiction. Respondents had no choice but to avail of different fora.
Thus, in cases where the complaint for violation of labor standard laws preceded the termination of
the employee and the filing of the illegal dismissal case, it would not be in consonance with justice
to charge the complainants with engaging in forum shopping when the remedy available to them at
the time their causes of action arose was to file separate cases before different fora.

[G.R. No. 202791, June 10, 2013]

PHILIPPINE TRANSMARINE CARRIERS, INC.


VS.
LEANDRO LEGASPI

This is a petition for review on certiorari under Rule 45 of the Rules of Court assailing the January 5,
2012 Resolution1 and July 20, 2012 Resolution2 of the Court of Appeals

Facts:

245
Respondent Leandro Legaspi (respondent) was employed as Utility Pastry on board the vessel
"Azamara Journey" under the employment of petitioner Philippine Transmarine Carriers, Inc.
(petitioner). Respondent’s employment was covered by a Collective Bargaining Agreement (CBA)
wherein it was agreed that the company shall pay a maximum disability compensation of up to
US$60,000.00 only.

While on board the vessel, respondent suffered "Cardiac Arrest S/P ICD Insertation." He was
checked by the ship’s doctor and was prescribed medications. On November 14, 2008, respondent
was repatriated to receive further medical treatment and examination. On May 23, 2009, the
company designated physician assessed his condition to be Disability Grade 2.
Not satisfied, respondent filed a complaint for full and permanent disability compensation against
petitioner before the Labor Arbiter (LA).

LA ruled in favor of the respondent and ordered to pay , the following:


1. US$80,000.00 or its peso equivalent at the time of payment as permanent disability
compensation;
2. US$1,320.00 or its peso equivalent as sick wages;
3. Attorney’s fees equivalent to 10% of the total award.

In its May 28, 2010 Decision, the NLRC affirmed the decision of the LA. Petitioner timely filed its
motion for reconsideration but it was denied by the NLRC in its July 30, 2010 Resolution. On
September 5, 2010, the NLRC issued the Entry of Judgment stating that its resolution affirming the
LA decision had become final and executory.

On October 22, 2010, during the hearing on the motion for execution before the NLRC, petitioner
agreed to pay respondent US$81,320.00. The terms and conditions of said payment were embodied
in the Receipt of Judgment Award with Undertaking,4 wherein respondent acknowledged receipt of
the said amount and undertook to return it to petitioner in the event the latter’s petition for
certiorari would be granted, without prejudice to respondent’s right to appeal

On November 8, 2010, petitioner timely filed a petition for certiorari with the CA.6
In the meantime, on March 2, 2011, the LA issued a writ of execution which noted petitioner’s
payment of the amount of US$81,320.00. On March 16, 2011, in compliance with the said writ,
petitioner tendered to the NLRC Cashier the additional amounts of US$8,132.00 as attorney’s fees
and P3,042.95 as execution fee. In its Order, dated March 31, 2011, the LA ordered the release of
the aforementioned amounts to respondent.

The CA’s Ruling


Unaware of a) the September 5, 2010 entry of judgment of the NLRC, b) the October 22, 2010
payment of US$81,320.00, and c) the writ of execution issued by the LA, the CA rendered its
Decision, dated June 29, 2011. The CA partially granted the petition for certiorari and modified the
assailed resolutions of the NLRC, awarding only US$60,000.00 pursuant to the CBA between
Celebrity Cruise Lines and Federazione Italianaa Transporti CISL.

246
Petitioner then filed its Manifestation with Motion to Amend the Dispositive Portion, submitting to
the CA the writ of execution issued by the LA in support of its motion. Petitioner contended that
since it had already paid the total amount of US$89,452.00, it was entitled to the return of the
excess payment in the amount of US$29,452.00

the CA denied the motion and ruled that the petition should have been dismissed for being moot
and academic not only because the assailed decision of the NLRC had become final and executory
on September 5, 2010, but also because the said judgment had been satisfied on October 22, 2010,
even before the filing of the petition for certiorari on November 8, 2010.

Issue:
I. WHETHER THE COURT OF APPEALS COMMITTED SERIOUS REVERSIBLE ERROR OF LAW IN
RULING THAT PETITIONER IS ESTOPPED IN COLLECTING THE EXCESS PAYMENT IT MADE TO
THE RESPONDENT NOTWITHSTANDING THE RECEIPT OF JUDGMENT AWARD SIGNED BY THE
RESPONDENT

Held:
Petition for Certiorari, Not Moot

Finality of NLRC’s decisions, resolutions and orders; Petition for Certiorari, Not Moot.Section 14, Rule
VII of the 2011 NLRC Rules of Procedure provides that decisions, resolutions or orders of the NLRC
shall become final and executory after ten (10) calendar days from receipt thereof by the parties,
and entry of judgment shall be made upon the expiration of the said period. In St. Martin Funeral
Home v. NLRC, however, it was ruled that judicial review of decisions of the NLRC may be sought
via a petition for certiorari before the CA under Rule 65 of the Rules of Court; and under Section 4
thereof, petitioners are allowed sixty (60) days from notice of the assailed order or resolution
within which to file the petition. Hence, in cases where a petition for certiorari is filed after the
expiration of the 10-day period under the 2011 NLRC Rules of Procedure but within the 60-day
period under Rule 65 of the Rules of Court, the CA can grant the petition and modify, nullify and
reverse a decision or resolution of the NLRC.

Accordingly, in this case, although the petition for certiorari was not filed within the 10-day period,
petitioner timely filed it before the CA within the 60-day reglementary period under Rule 65. It has,
thus, been held that the CA’s review of the decisions or resolutions of the NLRC under Rule 65,
particularly those which have already been executed, does not affect their statutory finality.
Xxx in Leonis Navigation, after the NLRC resolution awarding disability benefits became final and
executory, the employer paid the monetary award to the employee. The CA dismissed the
employer’s petition for certiorari, ruling that the final and executory decisions or resolutions of the
NLRC rendered appeals to superior courts moot and academic. This Court disagreed with the CA
and held that final and executed decisions of the NLRC did not prevent the CA from reviewing the
same under Rule 65 of the Rules of Court. It was further ruled that the employee was estopped from
claiming that the case was closed and terminated, considering that the employee’s

247
Acknowledgment Receipt stated that such was without prejudice to the final outcome of the
petition for certiorari pending before the CA.

[G.R. No. 192394, July 3, 2013]

PASOS
VS.
PHIL. NATIONAL CONSTRUCTION CORP.

Facts:
Petitioner Roy D. Pasos started working for respondent PNCC on April 26, 1996 to July 25, 1996.
The contract maybe terminated at anytime for cause as provided for by law and/or existing
Company Policy.

Petitioner’s employment, however, did not end on July 25, 1996 but was extended until August 4,
1998, or more than two years later, based on the "Personnel Action Form – Project Employment"
dated July 7, 1998.

Petitioner was rehired on even date as "Accounting Clerk (Reliever) and was stated therein that his
employment shall end on February 11, 1999 and may be terminated for cause or in accordance with
the provisions of Article 282 of the Labor Code, as amended. However, said employment did not
actually end on February 11, 1999 but was extended until February 19, 1999 based on the
"Personnel Action Form-Project Employment" dated February 17, 1999.8

On February 23, 1999, petitioner was again hired by PNCC as "Accounting Clerk" and was assigned
to the "SM-Project" based on the "Appointment for Project Employment" dated February 18,
1999.9 It did not specify the date when his employment will end but it was stated therein that it will
be "co-terminus with the completion of the project." Said employment supposedly ended on August
19, 1999 per "Personnel Action Form – Project Employment" dated August 18, 1999,10 where it was
stated, "termination of petitioner’s project employment due to completion of assigned phase/stage
of work or project effective at the close of office hours on 19 August 1999." However, it appears that
said employment was extended per "Appointment for Project employment" dated August 20,
199911 as petitioner was again appointed as "Accounting Clerk" for "SM Project (Package II)." It did
not state a specific date up to when his extended employment will be, but it provided that it will be
"co-terminus with the x x x project." In "Personnel Action Form – Project Employment" dated
October 17, 2000,12 it appears that such extension would eventually end on October 19, 2000.

Despite the termination of his employment on October 19, 2000, petitioner claims that his superior
instructed him to report for work the following day, intimating to him that he will again be
employed for the succeeding SM projects. For purposes of reemployment, he then underwent a
medical examination which allegedly revealed that he had pneumonitis. Petitioner was advised by
PNCC’s physician, Dr. Arthur C. Obena, to take a 14-day sick leave.

248
On November 27, 2000, after serving his sick leave, petitioner claims that he was again referred for
medical examination where it was revealed that he contracted Koch’s disease. He was then required
to take a 60-day leave of absence.13 The following day, he submitted his application for sick leave
but PNCC’s Project Personnel Officer, Mr. R.S. Sanchez, told him that he was not entitled to sick
leave because he was not a regular employee.

Petitioner still served a 60-day sick leave and underwent another medical examination on February
16, 2001. He was then given a clean bill of health and was given a medical clearance by Dr. Obena
that he was fit to work.

Petitioner claims that after he presented his medical clearance to the Project Personnel Officer on
even date, he was informed that his services were already terminated on October 19, 2000 and he
was already replaced due to expiration of his contract. This prompted petitioner on February 18,
2003 to file a complaint14 for illegal dismissal against PNCC with a prayer for reinstatement and
back wages. He argued that he is deemed a regular employee of PNCC due to his prolonged
employment as a project employee as well as the failure on the part of PNCC to report his
termination every time a project is completed. He further contended that his termination without
the benefit of an administrative investigation was tantamount to an illegal dismissal.

PNCC countered that petitioner was hired as a project employee in several projects with specific
dates of engagement and termination and had full knowledge and consent that his appointment
was only for the duration of each project. It further contended that it had sufficiently complied with
the reportorial requirements to the Department of Labor and Employment (DOLE). It submitted
photocopies of three Establishment Termination Reports it purportedly filed with the DOLE. They
were for: (1) the "PCSOQ.I. Project" for February 1999;15 (2) "SM Project" for August 1999;16 and
(3) "SM Project" for October 2000,17 all of which included petitioner as among the affected
employees. The submission of termination reports by PNCC was however disputed by petitioner
based on the verifications18 issued by the DOLE NCR office that he was not among the affected
employees listed in the reports filed by PNCC in August 1998, February 1999, August 1999 and
October 2000.

Labor Arbiter
The Labor Arbiter ruled that petitioner attained regular employment status with the repeated
hiring and rehiring of his services more so when the services he was made to render were usual
and necessary to PNCC’s business.
PNCC and petitioner appealed the Labor Arbiter’s decision. PNCC insisted that petitioner was just a
project employee and his termination was brought about by the completion of the contract and
therefore he was not illegally dismissed. Petitioner, on the other hand, argued that his
reinstatement should have been ordered by the Labor Arbiter since there was no proof that there
were strained relations between the parties. He also questioned the deduction of six months pay
from the back wages awarded to him and the failure of the Labor Arbiter to award him damages
and attorney’s fees. Petitioner likewise moved to dismiss PNCC’s appeal contending that the
supersedeas bond in the amount of P422,630.41 filed by the latter was insufficient considering that

249
the Labor Arbiter’s monetary award is P460,292.41. He also argued that the person who verified
the appeal, Felix M. Erece, Jr., Personnel Services Department Head of PNCC, has no authority to file
the same for and in behalf of PNCC.

Issues:
(1) Should an appeal be dismissed outright if the appeal bond filed is less than the adjudged
amount?
(2) Can the head of the personnel department sign the verification and certification on behalf of the
corporation sans any board resolution or secretary’s certificate authorizing such officer to do the
same? and
(3) Is petitioner a regular employee and not a mere project employee and thus can only be
dismissed for cause?

Held:

1) The perfection of an appeal within the reglementary period and in the manner prescribed
by law is jurisdictional, and noncompliance with such legal requirement is fatal and effectively
renders the judgment final and executory. As provided in Article 223 of the Labor Code, as
amended, in case of a judgment involving a monetary award, an appeal by the employer may be
perfected only upon the posting of a cash or surety bond issued by a reputable bonding company
duly accredited by the Commission in the amount equivalent to the monetary award in the
judgment appealed from.

However, not only in one case has this Court relaxed this requirement in order to bring about the
immediate and appropriate resolution of cases on the merits.24 In Quiambao v. National Labor
Relations Commission,25 this Court allowed the relaxation of the requirement when there is
substantial compliance with the rule. Likewise, in Ong v. Court of Appeals,26 the Court held that the
bond requirement on appeals may be relaxed when there is substantial compliance with the Rules
of Procedure of the NLRC or when the appellant shows willingness to post a partial bond. The Court
held that "while the bond requirement on appeals involving monetary awards has been relaxed in
certain cases, this can only be done where there was substantial compliance of the Rules or where
the appellants, at the very least, exhibited willingness to pay by posting a partial bond.

2) It has been the constant holding of this Court in cases instituted by corporations that an
individual corporate officer cannot exercise any corporate power pertaining to the corporation
without authority from the board of directors pursuant to Section 23, in relation to Section 25 of the
Corporation Code which clearly enunciates that all corporate powers are exercised, all business
conducted, and all properties controlled by the board of directors. However, we have in many cases
recognized the authority of some corporate officers to sign the verification and certification against
forum-shopping.

It has been the constant holding of this Court in cases instituted by corporations that an individual
corporate officer cannot exercise any corporate power pertaining to the corporation without
authority from the board of directors pursuant to Section 23, in relation to Section 25 of the

250
Corporation Code which clearly enunciates that all corporate powers are exercised, all business
conducted, and all properties controlled by the board of directors. However, we have in many cases
recognized the authority of some corporate officers to sign the verification and certification against
forum-shopping. Some of these cases were enumerated in Cagayan Valley Drug Corporation v.
Commissioner of Internal Revenue27 which was cited by the appellate court:

In Mactan-Cebu International Airport Authority v. CA, we recognized the authority of a general


manager or acting general manager to sign the verification and certificate against forum shopping;
in Pfizer v. Galan, we upheld the validity of a verification signed by an "employment specialist" who
had not even presented any proof of her authority to represent the company; in Novelty
Philippines, Inc. v. CA, we ruled that a personnel officer who signed the petition but did not attach
the authority from the company is authorized to sign the verification and non-forum shopping
certificate; and in Lepanto Consolidated Mining Company v. WMC Resources International Pty. Ltd.
(Lepanto), we ruled that the Chairperson of the Board and President of the Company can sign the
verification and certificate against non-forum shopping even without the submission of the board’s
authorization.

In sum, we have held that the following officials or employees of the company can sign the
verification and certification without need of a board resolution: (1) the Chairperson of the Board
of Directors, (2) the President of a corporation, (3) the General Manager or Acting General Manager,
(4) Personnel Officer, and (5) an Employment Specialist in a labor case.
While the above cases do not provide a complete listing of authorized signatories to the verification
and certification required by the rules, the determination of the sufficiency of the authority was
done on a case to case basis. The rationale applied in the foregoing cases is to justify the authority
of corporate officers or representatives of the corporation to sign the verification or certificate
against forum shopping, being "in a position to verify the truthfulness and correctness of the
allegations in the petition."

3) In the case at bar, petitioner worked continuously for more than two years after the
supposed three-month duration of his project employment for the NAIA II Project. While his
appointment for said project allowed such extension since it specifically provided that in case his
"services are still needed beyond the validity of the contract, the Company shall extend his
services," there was no subsequent contract or appointment that specified a particular duration for
the extension. His services were just extended indefinitely until "Personnel Action Form – Project
Employment" dated July 7, 1998 was issued to him which provided that his employment will end a
few weeks later or on August 4, 1998. While for first three months, petitioner can be considered a
project employee of PNCC, his employment thereafter, when his services were extended without
any specification of as to the duration, made him a regular employee of PNCC. And his status as a
regular employee was not affected by the fact that he was assigned to several other projects and
there were intervals in between said projects since he enjoys security of tenure.

Failure of an employer to file termination reports after every project completion proves that an
employee is not a project employee.

251
[G.R. No. 189871, August 13, 2013]

DARIO NACAR
VS.
GALLERY FRAMES AND/OR FELIPE BORDEY, JR.

Facts:
Petitioner Dario Nacar filed a complaint for constructive dismissal before the Arbitration Branch of
the NLRC against respondents Gallery Frames (GF) and/or Felipe Bordey, Jr.

On October 15, 1998, the Labor Arbiter rendered a decision (a.k.a. 1998 Decision) in favor of
petitioner and found that he was dismissed from employment without a valid or just cause. Thus,
petitioner was awarded backwages and separation pay in lieu of reinstatement in the amount
of P158,919.92.

Respondents appealed to the NLRC, but it was dismissed for lack of merit and sustained the
decision of the LA. Dissatisfied, respondents filed a Petition for Review on Certiorari before the CA.
The CA dismissed the petition. They then sought relief before the Supreme Court, but still it was
denied.

The 1998 Decision became final and executory and the case was referred back to the LA where a
pre-execution conference was scheduled, but respondents failed to appear.

Petitioner filed a Motion for Correct Computation, praying that his backwages be computed from
the date of his dismissal on January 24, 1997 up to the finality of the Resolution of the Supreme
Court on May 27, 2002. The NLRC arrived at an updated amount in the sum
of P471,320.31.Respondents argued that since the LA awardedbackwages and separation pay, no
more recomputation is required to be made because the same cannot be altered or amended after
the decision becomes final and executor. TheLA denied the Respondents’ motion.

Respondents again appealed before the NLRC, which was granted and a recomputation of the
judgment award was ordered. This Resolutionbecame final and executory and the judgment award
of petitioner was reassessed to be in the total amount of only P147,560.19.Petitioner then moved
that a writ of execution be issued on the 1998 Decision. The LA issued an Alias Writ of Execution to
satisfy the judgment award in the amount of P147,560.19, which petitioner eventually received.

Petitioner then requested for the recomputation to include the appropriate interests, which the LA
granted but only up to the amount ofP11,459.73. The LA reasoned that it is the 1998 Decision that
should be enforced considering that it was the one that became final and executory. Thus, since
petitioner already receivedP147,560.19, he is only entitled to the balance of P11,459.73.

252
Petitioner then appealed before the NLRC, which was denied. Petitioner then sought recourse
before the CA, but was again denied. The CA opined that since petitioner no longer appealed the
1998 Decision of the LA, which already became final and executory, a belated correction thereof is
no longer allowed. There is nothing left to be done except to enforce the said judgment.

Issue:
Whether or not the recomputation made the NLRC was proper.

Held:
The petition is meritorious.

A source of misunderstanding in implementing the final decision in this case proceeds from the way
the original labor arbiter framed his decision. The decision consists essentially of two parts.
The first is that part of the decision that cannot now be disputed because it has been confirmed
with finality. This is the finding of the illegality of the dismissal and the awards of separation pay in
lieu of reinstatement, backwages, attorney's fees, and legal interests.
The second part is the computation of the awards made. This part, being merely a computation of
what the first part of the decision established and declared, can, by its nature, be re-computed. This
is the part, too, that the petitioner now posits should no longer be re-computed because the
computation is already in the labor arbiter's decision that the CA had affirmed. The public and
private respondents, on the other hand, posit that a re-computation is necessary because the relief
in an illegal dismissal decision goes all the way up to reinstatement if reinstatement is to be made,
or up to the finality of the decision, if separation pay is to be given in lieu reinstatement.
That the labor arbiter's decision, at the same time that it found that an illegal dismissal had taken
place, also made a computation of the award, is understandable in light of Section 3, Rule VIII of the
then NLRC Rules of Procedure which requires that a computation be made. This Section in part
states:

[T]he Labor Arbiter of origin, in cases involving monetary awards and at all events, as far as
practicable, shall embody in any such decision or order the detailed and full amount awarded.
Clearly implied from this original computation is its currency up to the finality of the labor arbiter's
decision. As we noted above, this implication is apparent from the terms of the computation itself,
and no question would have arisen had the parties terminated the case and implemented the
decision at that point.

However, the petitioner disagreed with the labor arbiter's findings on all counts - i.e., on the finding
of illegality as well as on all the consequent awards made. Hence, the petitioner appealed the case to
the NLRC which, in turn, affirmed the labor arbiter's decision. By law, the NLRC decision is final,
reviewable only by the CA on jurisdictional grounds.The petitioner appropriately sought to nullify
the NLRC decision on jurisdictional grounds through a timely filed Rule 65 petition for certiorari.
Consequently, from the above disquisitions, under the terms of the decision which is sought to be
executed by the petitioner, no essential change is made by a recomputation as this step is a
necessary consequence that flows from the nature of the illegality of dismissal declared by the

253
Labor Arbiter in that decision. A recomputation (or an original computation, if no previous
computation has been made) is a part of the law – specifically, Article 279 of the Labor Code and the
established jurisprudence on this provision – that is read into the decision. By the nature of an
illegal dismissal case, the reliefs continue to add up until full satisfaction, as expressed under Article
279 of the Labor Code. The recomputation of the consequences of illegal dismissal upon execution
of the decision does not constitute an alteration or amendment of the final decision being
implemented. The illegal dismissal ruling stands; only the computation of monetary consequences
of this dismissal is affected, and this is not a violation of the principle of immutability of final
judgments.

That the amount respondents shall now pay has greatly increased is a consequence that it cannot
avoid as it is the risk that it ran when it continued to seek recourses against the Labor Arbiter's
decision. Article 279 provides for the consequences of illegal dismissal in no uncertain terms,
qualified only by jurisprudence in its interpretation of when separation pay in lieu of reinstatement
is allowed. When that happens, the finality of the illegal dismissal decision becomes the reckoning
point instead of the reinstatement that the law decrees. In allowing separation pay, the final
decision effectively declares that the employment relationship ended so that separation pay and
backwages are to be computed up to that point.

In sum, the Respondents are ordered to pay petitioner:


(1) backwages computed from the time petitioner was illegally dismissed on January 24, 1997 up to
May 27, 2002, when the Resolution of the Supreme Court became final and executory;
(2) separation pay computed from August 1990 up to May 27, 2002 at the rate of one month pay
per year of service; and
(3) interest of twelve percent (12%) per annum of the total monetary awards, computed from May
27, 2002 to June 30, 2013 and six percent (6%) per annum from July 1, 2013 until their full
satisfaction.

[G.R. No. 188514, August 28, 2013]

MARIA LOURDES D. CASTELLS AND SHALIMAR CENTI-MANDANAS


VS.
SAUDI ARABIAN AIRLINES

Facts:
Respondent Saudi Arabian Airlines (SAUDIA) issued a memo regarding the transfer of 10 flight
attendants, including Castells and Centi-Mandanas (petitioners), from Manila to Jeddahdue to
"operational requirements" (transfer order). Centi-Mandanas complied with the transfer order
while Castells did not.Centi-Mandanas alleged that upon her arrival in Jeddah, she was told that her
contract would no longer be renewed and that she was asked to sign a pre-typed resignation letter.
She averred that while she never wished to resign, SAUDIA left her with no other viable choice as it

254
would terminate her services anyway. Thus, she filled out the resignation form handed to
her.Castells alleged that upon her non-compliance with the transfer order, she prepared a
resignation letter stating that she felt she was being forced to resign. She then alleged that the
SAUDIA Manila Office Manager told her to amend the same to state that she was voluntarily
resigning; this she reluctantly followed.

Petitioners filed a complaint for illegal dismissal against SAUDIA, with prayer for reinstatement, full
backwages, moral and exemplary damages, and attorney’s fees.

For their defense, SAUDIA maintained that the resignations were intelligently and voluntarily made.
The Labor Arbiter (LA) held SAUDIA guilty of illegal dismissal and ordered it to pay each of
petitioners full backwages from the time of their illegal dismissal until finality of the decision and
separation pay of one month salary for every year of service, less the amount they already received,
including attorney’s fees.It found that petitioners did not voluntarily resign and that SAUDIA forced
them to do so only because of their "old" age.

Dissatisfied, SAUDIA appealed to NLRC. The NLRC reversed and set aside the LA’s ruling and
thereby dismissed the illegal dismissal complaint against SAUDIA. The NLRC held that the presence
of words of gratitude in the subject letters negates the claim that they were products of any form of
coercion or threat on SAUDIA’s part.

Aggrieved, the petitioners elevated the matter to the CA.On January 16, 2008, petitioners filed with
the CA a Motion for Extension to File a Petition for Certiorari, praying that they be given a period of
15 days from January 18, 2008, or until February 2, 2008, within which to file the subject petition.
The said motion was granted. Since February 2, 2008 was a Saturday, petitioners filed the subject
petition on the next working day, or on February 4, 2008, and the CA admitted the same.

On even date, SAUDIA filed a Motion for Reconsideration,primarily contending that A.M. No. 07-7-
12-SC,which took effect on December 27, 2007, no longer allowed the filing of an extension of time
to file a petition for certiorari; thus, the CA should not have admitted the subject petition. The CA
reconsidered its earlier resolution and granted SAUDIA’s motion. It deemed the subject petition not
admitted due to petitioners’ non-compliance with the reglementary period prescribed by Section 4,
Rule 65 of the Rules of Court, as amended by A.M. No. 07-7-12-SC. Hence, it considered the case
closed and terminated.

Issue:
Whether or not the CA correctly refused admission of the subject petition.

Held:
The petition is meritorious.

It is well-settled that procedural rules should be treated with utmost respect and due regard, since
they are designed to facilitate the adjudication of cases to remedy the worsening problem of delay

255
in the resolution of rival claims and in the administration of justice. From time to time, however, the
Court has recognized exceptions to the strict application of such rules, but only for the most
compelling reasons where stubborn obedience to the Rules would defeat rather than serve the ends
of justice.These exceptions, as enumerated in the case of Labao v. Flores,25 are as follows:
x xx (1) most persuasive and weighty reasons; (2) to relieve a litigant from an injustice not
commensurate with his failure to comply with the prescribed procedure; (3) good faith of the
defaulting party by immediately paying within a reasonable time from the time of the default; (4)
the existence of special or compelling circumstances; (5) the merits of the case; (6) a cause not
entirely attributable to the fault or negligence of the party favored by the suspension of the rules;
(7) a lack of any showing that the review sought is merely frivolous and dilatory; (8) the other party
will not be unjustly prejudiced thereby; (9) fraud, accident, mistake, or excusable negligence
without appellant’s fault; (10) peculiar legal and equitable circumstances attendant to each case;
(11) in the name of substantial justice and fair play; (12) importance of the issues involved; and
(13) exercise of sound discretion by the judge guided by all the attendant circumstances. x xx.
In view of the foregoing, despite the rigid wording of Section 4, Rule 65 of the Rules, as amended by
A.M. No. 07-7-12-SC– which now disallows an extension of the 60-day reglementary period to file a
petition for certiorari – courts may nevertheless extend the same, subject to its sound discretion.
In this case, the CA had already exercised its sound discretion in granting the extension to file the
subject petition thru a Resolution dated January 29, 2008. Consequently, it could not renege on
such grant by rendering another issuance almost seven months later, i.e., Resolution dated August
28, 2008, which resulted in the refusal to admit the same petition. Such course of action is clearly
antithetical to the tenets of fair play, not to mention the undue prejudice to petitioners' rights.
Verily, the more appropriate course of action would have been to admit the subject petition and
resolve the case on the merits. Thus, in order to rectify this lapse, the Court deems it prudent to
have the case remanded to the CA for its proper resolution.

[G.R. No. 163431, August 28, 2013]

DONGON
VS.
RAPID MOVERS AND FORWARDERS CO INC., ET AL.

Facts:
Rapid is engaged in the hauling and trucking business while private respondent Nathaniel T.
Dongon is a former truck helper leadman. On 23 April 2001, Dongon and his driver, Vicente
Villaruz, were in the vicinity of his area of assignment, the Tanduay warehouse as they tried to get
some goods to be distributed to their clients. Since the driver did not have an ID the security guard
called their attention and Dongon promised to settle the matter with the management. Instead, he
lent the driver his own ID and the security guard, upon knowing of the misrepresentation, reported
him. On 23 May 2001, after conducting an administrative investigation, Dongon was dismissed from
Rapid. On 01 June 2001, Dongon filed a Complaint for Illegal Dismissal.

The Labor Arbiter dismissed Dongon's complaint saying that Rapid rightly exercised its prerogative
to dismiss him. On appeal, the NLRC reversed the LA ruling that Rapid Movers had not discharged

256
its burden to prove the validity of Dongon's dismissal from his employment. It opined that Rapid
Movers did not suffer any pecuniary damage from his act; and that his dismissal was a penalty
disproportionate to the act of petitioner complained of. It awarded him backwages and separation
pay in lieu of reinstatement. However, the CA reinstated the LA's ruling and upheld Rapid's right to
discipline its workers holding that, ----while it can be conceded that there is no pecuniary damage
involved, the fact remains that the offense does not only constitute dishonesty but also willful
disobedience to the lawful order of the Company, e.g., to observe at all time the terms and
conditions of the Manual of Discipline. Article 282 of the Labor Code provides: “Termination by
Employer – An employer may terminate an employment for any of the following causes: (a) Serious
misconduct or willful disobedience by the employee of the lawful orders of his employer or
representative in connection with his work----
Undaunted, the petitioner is now on appeal.

Issues:
1. On the propriety of the petition
2. If the petition could prosper, was the dismissal of petitioner on the ground of willful disobedience
to the company regulation lawful

Held:
1. PETITION SHOULD NOT BE DISMISSED
In St. Martin Funeral Home v. National Labor Relations Commission,13 the Court has clarified that
parties seeking the review of decisions of the NLRC should file a petition for certiorari in the CA on
the ground of grave abuse of discretion amounting to lack or excess of jurisdiction on the part of the
NLRC. Thereafter, the remedy of the aggrieved party from the CA decision is an appeal via petition
for review on certiorari.

The petition filed here is self-styled as a petition for review on certiorari, but Rapid Movers points
out that the petition was really one for certiorari under Rule 65 of the Rules of Court due to its basis
being the commission by the CA of a grave abuse of its discretion and because the petition was filed
beyond the reglementary period of appeal under Rule 45. Hence, Rapid Movers insists that the
Court should dismiss the petition because certiorari under Rule 65 could not be a substitute of a
lost appeal under Rule 45.

Ordinarily, an original action for certiorari will not prosper if the remedy of appeal is available, for
an appeal by petition for review on certiorari under Rule 45 of the Rules of Court and an original
action for certiorari under Rule 65 of the Rules of Court are mutually exclusive, not alternative nor
successive, remedies.15 On several occasions, however, the Court has treated a petition for
certiorari as a petition for review on certiorari when: (a) the petition has been filed within the 15-
day reglementary period;16 (b) public welfare and the advancement of public policy dictate such
treatment; (c) the broader interests of justice require such treatment; (d) the writs issued were null
and void; or (e) the questioned decision or order amounts to an oppressive exercise of judicial
authority.

257
The Court deems it proper to allow due course to the petition as one for certiorari under Rule 65 in
the broader interest of substantial justice, particularly because the NLRC’s appellate adjudication
was set aside by the CA, and in order to put at rest the doubt that the CA, in so doing, exercised its
judicial authority oppressively. Given the spirit and intention underlying our labor laws of resolving
a doubtful situation in favor of the working man, we will have to review the judgment of the CA to
ascertain whether the NLRC had really committed grave abuse of its discretion. This will settle the
doubts on the propriety of terminating petitioner, and at the same time ensure that justice is served
to the parties.

2. PETITIONER WAS NOT GUILTY OF WILLFUL DISOBEDIENCE, HENCE HIS DISMISSAL ILLEGAL
For willful disobedience to be a ground, it is required that: (a) the conduct of the employee must be
willful or intentional; and (b) the order the employee violated must have been reasonable, lawful,
made known to the employee, and must pertain to the duties that he had been engaged to
discharge.20 Willfulness must be attended by a wrongful and perverse mental attitude rendering
the employee’s act inconsistent with proper subordination. In any case, the conduct of the
employee that is a valid ground for dismissal under the Labor Code constitutes harmful behavior
against the business interest or person of his employer. It is implied that in every act of willful
disobedience, the erring employee obtains undue advantage detrimental to the business interest of
the employer.

Under the foregoing standards, the disobedience attributed to petitioner could not be justly
characterized as willful within the contemplation of Article 296 of the Labor Code. He neither
benefitted from it, nor thereby prejudiced the business interest of Rapid Movers. His explanation
that his deed had been intended to benefit Rapid Movers was credible. There could be no wrong or
perversity on his part that warranted the termination of his employment based on willful
disobedience.

Although we recognize the inherent right of the employer to discipline its employees, we should
still ensure that the employer exercises the prerogative to discipline humanely and considerately,
and that the sanction imposed is commensurate to the offense involved and to the degree of the
infraction.

The NLRC’s reversal of the decision of the Labor Arbiter by holding that penalty too harsh and
disproportionate to the wrong attributed to him was legally and factually justified, not arbitrary or
whimsical. Consequently, for the CA to pronounce that the NLRC had thereby gravely abused its
discretion was not only erroneous but was itself a grave abuse of discretion amounting to lack of
jurisdiction for not being in conformity with the pertinent laws and jurisprudence.

[G.R. No. 155306, Aug. 28, 2013]

258
MALAYANG MANGGAGAWA NG STAYFAST PHILS., INC
VS.
NATIONAL RELATIONS LABOR COMMISSION, STAYFAST PHILIPPINES, INC./ MARIA ALMEIDA

Facts:
xxxxPetitioner and Nagkakaisang Lakas ng Manggagawa sa Stayfast (NLMS-Olalia) sought to be the
exclusive bargaining agent of the employees of respondent company, Stayfast Philippines, Inc. A
certification election was conducted on December 29, 1995. The NLMS-Olalia was certified by the
Med-Arbiter as the sole and exclusive bargaining agent of all rank and file employees of respondent
company. Petitioner appealed to the SOLE. The SOLE initially set aside the Order of the Med-
Arbiter and called for run-off election between petitioner and NLMS-Olalia. On motion of NLMS-
Olalia, however, the SOLE reconsidered his earlier decision and restored the Med-Arbiter’s Order.
Petitioner elevated the matter via petition for certiorari to this Court.Itwas dismissed in a
Resolution dated January 14, 1998.

Meanwhile, NLMS-Olalia demanded to collectively bargain with respondent company. The latter
rejected petitioner’s demand, insisting that it would negotiate a collective bargaining agreement
only with whichever union is finally certified as the sole and exclusive bargaining agent of the
workers. Nevertheless, NLMS-Olalia went on strike. Subsequently, on June 5, 1997, petitioner filed
its own notice of strike in the National Conciliation and Mediation Board (NCMB). Respondent
company opposed petitioner’s move and filed a motion to dismiss on the ground that petitioner was
not the certified bargaining agent and therefore lacked personality to file a notice of strike.
Thereafter, the parties were able to make concessions during the conciliation-mediation stage in
the NCMB which led petitioner to withdraw its notice of strike.

On July 21, 1997, however, petitioner’s members staged a “sit-down strike” as respondent company
allegedly continued to discriminate against them. Respondent company issued a memorandum
requiring the alleged participants in the “sit-down strike” to explain within 24 hours why they
should not be terminated or suspended. As no one complied,respondent company promptly
terminated the service of the participants in the “sit-down strike”. Consequently, on July 23, 1997,
petitioner staged a strike and filed a complaint for unfair labor practice, union busting and illegal
lockout against respondent company and its General Manager, Maria Almeida, in the NLRC.xxx

The Labor Arbiter dismissed the petition. Petitioner appealed but, in a Resolution dated January 31,
2000, the NLRC upheld the Labor Arbiter’s Decision. Petitioner filed a motion for reconsideration
but the NLRC denied it. Petitioner filed a petition for certiorari in the Court of Appeals.
The Court of Appeals found that petitioner was seeking a review of the findings of fact and
conclusion of the Labor Arbiter which was sustained by the NLRC.It applied the rule that findings of
fact made by the Labor Arbiter and affirmed by the NLRC are considered by the appellate court as
binding if supported by substantial evidence. It ruled that the NLRC Resolution was supported by
justifiable reason and cannot be faulted with grave abuse of discretion. Moreover, a petition for

259
certiorari is not used to correct a lower tribunal’s appreciation of evidence and findings of fact.
Thus, the Court of Appeals dismissed the petition.
Hence, this petition for certiorari under Rule 65 of the Rules of Court.

Held:
The petition fails for many reasons.
1. This petition for certiorari is a wrong remedy.
A petition for certiorari under Rule 65 of the Rules of Court is a special civil action that may be
resorted to only in the absence of appeal or any plain, speedy and adequate remedy in the ordinary
course of law.

Contrary to petitioner’s claim, that there was no appeal or any other plain, speedy and adequate
remedy, the right recourse was to appeal to this Court in the form of a petition for review on
certiorari under Rule 45 of the Rules of Court.

For purposes of appeal, the Decision of the Court of Appeals was a final judgment as it denied due
course to, and dismissed, the petition. Thus, the Decision disposed of the petition of petitioner in a
manner that left nothing more to be done by the Court of Appeals in respect to the said case. Thus,
petitioner should have filed an appeal by petition for review on certiorari under Rule 45, not a
petition for certiorari under Rule 65, in this Court.
The proper remedy to obtain a reversal of judgment on the merits, final order or resolution is
appeal. This holds true even if the error ascribed to the court rendering the judgment is its lack of
jurisdiction over the subject matter, or the exercise of power in excess thereof, or grave abuse of
discretion in the findings of fact or of law set out in the decision, order or resolution. The existence
and availability of the right of appeal prohibits the resort to certiorari because one of the
requirements for the latter remedy is that there should be no appeal.

Moreover, certiorari is not and cannot be made a substitute for an appeal where the latter remedy
is available but was lost through fault or negligence. In this case, petitioner received the Decision on
August 2, 2002 and, under the ruleshad until August 19, 2002 tofile an appeal by way of a petition
for review in this Court. Petitioner let this period lapse without filing an appeal and, instead, filed
this petition for certiorari on October 1, 2002.

2. Even assuming that a petition for certiorari is the correct remedy in this case, petitioner failed to
comply with the requirement of a prior motion for reconsideration. As a general rule, a motion for
reconsideration is a prerequisite for the availment of a petition for certiorari under Rule 65. The
filing of a motion for reconsideration before resort to certiorari will lie is intended to afford the
public respondent an opportunity to correct any actual or fancied error attributed to it by way of
re-examination of the legal and factual aspects of the case.

3. Petitioner was not able to establish its allegation of grave abuse of discretion on the part of the
Court of Appeals. In this case, nowhere in the petition did petitioner show that the issuance of the

260
Decision dated July 1, 2002 of the Court of Appeals was patent and gross that would warrant
striking it down through a petition for certiorari.

4. Petitioner essentially questioned the factual findings of the Labor Arbiter and the NLRC.
Petitioner cannot properly do that in a petition for certiorari. Only established or admitted facts can
be considered.

5. Considering that petitioner basically presented an issue of fact, its petition for certiorari
crumbles in view of the identical findings of the Labor Arbiter and the NLRC which were further
upheld by the Court of Appeals. This Court finds no compelling reason to rule otherwise.

6. Even on the merits, the case of petitioner has no leg to stand on. Petitioner’s case rests on the
alleged discriminatory acts of respondent company against petitioner’s officers and members.
However, both theLabor Arbiter and the NLRC held that there was no sufficient proof of respondent
company's alleged discriminatory acts.

[G.R. No. 167484, September 09, 2013]

HERNANDO BORRA, ET AL., PETITIONERS,


VS.
COURT OF APPEALS SECOND AND NINETEENTH DIVISIONS AND HAWAIIAN PHILIPPINE
COMPANY, RESPONDENTS

Facts:

On September 12, 1997, herein petitioners filed with the NLRC RAB No. VI in Bacolod City two
separate complaints. In RAB Case No. 06-09-10698-97, herein petitioners asked that they be
recognized and confirmed as regular employees of herein private respondent and further prayed
that they be awarded various benefits received by regular employees for three (3) years prior to
the filing of the complaint, while in RAB Case No. 06-09-10699-97, herein petitioners sought for
payment of unpaid wages, holiday pay, allowances, 13thmonth pay, service incentive leave pay,
moral and exemplary damages also during the three (3) years preceding the filing of the complaint.
Private respondent filed a Motion to Dismiss RAB Case No. 06-09-10698-97 on the ground of res
judicata. Private respondent cited an earlier decided case entitled “HumphreyPerez, et al. v.
Hawaiian Philippine Co. et al.” (Perez case) and docketed as RAB Case No. 06-04-10169-95, which
was an action for recovery of 13th month pay and service incentive leave pay, and it includes herein
petitioners among the complainants and herein private respondent and one Jose Castillon as
respondents. Private respondent contended that the Perez case, which has already become final and
executory, as no appeal was taken therefrom, serves as a bar to the litigation of RAB Case No. 06-
09-10698-97, because it was ruled therein that petitioners are not employees of private respondent
but of Castillon

261
The Labor Arbiter granted private respondent's Motion to Dismiss. Petitioners appealed to the
NLRC which set aside the Order of the Labor Arbiter, reinstated the complaint in RAB Case No. 06-
09-10698-97 and remanded the same for further proceedings.Private respondent appealed to the
CA. On January 12, 2001, the CA rendered judgment, affirming the Decision of the NLRC and denied
the subsequent motion for reconsideration

Aggrieved, private respondent filed a petition for review on certiorari before this Court. The case
was entitled as “Hawaiian Philippine Company v. Borra” and docketed as G.R. No. 151801. This Court
rendered its Decision denying the petition and affirming the Decision of the CA.

The factual milieu of the Perez case covered the period November 1987 to April 6, 1995 (date of
filing of the complaint), during which time private respondents, by their own admission, were
engaged by Castillon to work at petitioner’s warehouse. In contrast, the instant case was filed on
September 12, 1997, by which time, the contractor involved was Fela Contractor; and private
respondents’ prayer is for confirmation of their status as regular employees of petitioner.
Moreover, in Perez, the finding that no employer-employee relationship existed between petitioner
and private respondents was premised on absence of privity between Castillon and petitioner.
Consequently, Perez and the instant case involve different subject matters and causes of action.

In the meantime, the Labor Arbiter rendered a Decision in RAB Case No. 06-09-10699-97 holding
that there is no employer-employee relations between private respondent and petitioners.No
appeal was taken from the Decision, thus, the same became final and executory.

As a consequence of the finality of the Decision in RAB Case No. 06-09-10699-97, herein private
respondent again filed a Motion to Dismiss RAB Case No. 06-09-10698-97 on the ground, among
others, of res judicata. Private respondent contended that the final and executory Decision of the
Labor Arbiter in RAB Case No. 06-09-10699-97, which found no employer-employee relations
between private respondent and petitioners, serves as a bar to the further litigation of RAB Case No.
06-09-10698-97.

The Labor Arbiter handling RAB Case No. 06-09-10698-97 denied private respondent's Motion to
Dismiss. Private respondent then filed a petition for certiorari and prohibition with the CA which in
turned annulled and set aside the order of the Labor Arbiter. Petitioners filed a Motion for
Reconsideration, but the CA denied it in its Resolution; hence, the present petition.

Issue:
Whether or not the Court of Appeals CA has jurisdiction over private respondent's petition for
certiorari considering that this Court, in G.R. No. 151801, already lodged jurisdiction in the Labor
Arbiter by directing the remand of RAB Case No. 06-09-10698-97 thereto for further proceedings.\

Held:
The petition lacks merit.

262
It is settled that jurisdiction over the subject matter is conferred by law and it is not within the
courts, let alone the parties, to themselves determine or conveniently set aside.In this regard, it
should be reiterated that what has been filed by private respondent with the CA is a special civil
action for certiorari assailing the Labor Arbiter's Order which denied its motion to dismiss.

Section 3, Rule V of the NLRC Rules of Procedure, which was then prevailing at the time of the filing
of private respondent's petition for certiorari with the CA, clearly provides:

SECTION 3. MOTION TO DISMISS. - On or before the date set for the conference, the respondent may
file a motion to dismiss. Any motion to dismiss on the ground of lack of jurisdiction, improper
venue, or that the cause of action is barred by prior judgment, prescription or forum shopping, shall
be immediately resolved by the Labor Arbiter by a written order. An order denying the motion to
dismiss or suspending its resolution until the final determination of the case is not
appealable.

In the case of Metro Drug Distribution, Inc. v. Metro Drug Corporation Employees Association-
Federation of Free Workers, this Court held that:
x xx The NLRC rule proscribing appeal from a denial of a motion to dismiss is similar to the general
rule observed in civil procedure that an order denying a motion to dismiss is interlocutory and,
hence, not appealable until final judgment or order is rendered. The remedy of the aggrieved party
in case of denial of the motion to dismiss is to file an answer and interpose, as a defense or defenses,
the ground or grounds relied upon in the motion to dismiss, proceed to trial and, in case of adverse
judgment, to elevate the entire case by appeal in due course. In order to avail of the
extraordinary writ of certiorari, it is incumbent upon petitioner to establish that the denial
of the motion to dismiss was tainted with grave abuse of discretion.21

In this regard, Rule 41 of the Rules of Court, which is applied in a suppletory character to cases
covered by the NLRC Rules, provides that in all the instances enumerated under the said Rule,
where the judgment or final order is not appealable, the aggrieved party may file an appropriate
special civil action under Rule 65. Thus, this Court has held that when the denial of a motion to
dismiss is tainted with grave abuse of discretion, the grant of the extraordinary remedy
of certiorari may be justified. On the basis of the foregoing, it is clear that the CA has jurisdiction
over the special civil action for certiorari filed by private respondent as the latter was able to allege
and establish that the denial of its motion to dismiss was tainted with grave abuse of discretion.
Petitioners are wrong to argue that this Court's directive in G.R. No. 151801 to remand RAB Case No.
06-09-10698-97 to the Labor Arbiter for further proceedings deprives the CA of its jurisdiction
over private respondent's petition for certiorari. The essence of this Court's ruling in G.R. No.
151801 is simply to require resolution of the factual issue of whether or not Fela Contractor has
stepped into the shoes of Castillon and, thus, has taken petitioners in its employ. In other words,
this Court called for a prior determination as to who is the real employer of petitioners.

263
As correctly held by the CA, petitioners' prayer for regularization in RAB Case No. 06-09-10698-97
is essentially dependent on the existence of employer-employee relations between them and
private respondent, because one cannot be made a regular employee of one who is not his
employer. In the same vein, petitioners' prayer in RAB Case No. 06-09-10699-97 for the recovery of
backwages, 13th month pay, holiday pay and service incentive leave pay from private respondent
likewise rests on the determination of whether or not the former are, indeed, employees of the
latter.

As earlier mentioned, this issue has already been settled. In the already final and executory decision
of the Labor Arbiter in RAB Case No. 06-09-10699-97, it was ruled therein that no employer-
employee relationship exists between private respondent and petitioners because the latter's real
employer is Fela Contractor. Thus, insofar as the question of employer and employee relations
between private respondent and petitioners is concerned, the final judgment in RAB Case No. 06-
09-10699-97 has the effect and authority of res judicata by conclusiveness of judgment.

Stated differently, conclusiveness of judgment finds application when a fact or question has been
squarely put in issue, judicially passed upon, and adjudged in a former suit by a court of
competent jurisdiction. The fact or question settled by final judgment or order binds the parties
to that action (and persons in privity with them or their successors-in-interest), and continues to
bind them while the judgment or order remains standing and unreversed by proper authority on a
timely motion or petition; the conclusively-settled fact or question cannot again be litigated in any
future or other action between the same parties or their privies and successors-in-interest, in the
same or in any other court of concurrent jurisdiction, either for the same or for a different cause of
action. Thus, only the identities of parties and issues are required for the operation of the principle
of conclusiveness of judgment

Hence, there is no point in determining the main issue raised in RAB Case No. 06-09-10698-97, i.e.,
whether petitioners may be considered regular employees of private respondent, because, in the
first place, they are not even employees of the latter. As such, the CA correctly held that the Labor
Arbiter committed grave abuse of discretion in denying private respondent's motion to dismiss
RAB Case No. 06-09-10698-97.

21. RIGHT TO SELF-ORGANIZATION

[G.R. No.174287, August 12, 2013]

264
NATIONAL UNION OF BANK EMPLOYEES (NUBE), PETITIONER,
VS.
PHILNABANK EMPLOYEES ASSOCIATION (PEMA) AND PHILIPPINE NATIONAL
BANK, RESPONDENTS

Facts:

Respondent PNB used to be a government-owned and controlled banking institution. Its rank-and-
file employees, being government personnel, were represented for collective negotiation by the
Philnabank Employees Association (PEMA), a public sector union. In 1996, the SEC approved PNB’s
new Articles of Incorporation and By-laws and its changed status as a private corporation. PEMA
affiliated with petitioner National Union of Bank Employees (NUBE), which is a labor federation
composed of unions in the banking industry, adopting the name NUBE-PNB Employees Chapter
(NUBE-PEC). Later, NUBE-PEC was certified as the sole and exclusive bargaining agent of the PNB
rank-and-file employees. A collective bargaining agreement (CBA) was subsequently signed
between NUBE-PEC and PNB covering the period of January 1, 1997 to December 31, 2001.

Pursuant to Article V on Check-off and Agency Fees of the CBA, PNB shall deduct the monthly
membership fee and other assessments imposed by the union from the salary of each union
member, and agency fee (equivalent to the monthly membership dues) from the salary of the rank-
and-file employees within the bargaining unit who are not union members. Moreover, during the
effectivity of the CBA, NUBE, being the Federation union, agreed that PNB shall remit P15.00 of the
P65.00 union dues per month collected by PNB from every employee, and that PNB shall directly
credit the amount to NUBE’s current account with PNB.Following the expiration of the CBA, the
Philnabank Employees Association-FFW (PEMA-FFW) filed a petition for certification election
among the rank-and-file employees of PNB. The petition sought the conduct of a certification
election to be participated in by PEMA-FFW and NUBE-PEC.

While the petition for certification election was still pending, two significant events transpired – the
independent union registration of NUBE- PEC and its disaffiliation with NUBE.

With a legal personality derived only from a charter issued by NUBE, NUBE-PEC, under the
leadership of Mariano Soria, decided to apply for a separate registration with the DOLE. It was later
on registered as an independent labor organization under Registration Certificate No. NCR-UR-3-
3790-2002. Thereafter, the Board of Directors of NUBE-PEC adopted a Resolution disaffiliating
itself from NUBE. It further filed a Manifestation and Motion before the Med-Arbitration Unit of
DOLE, praying that, in view of its independent registration as a labor union and disaffiliation from
NUBE, its name as appearing in the official ballots of the certification election be changed to
“Philnabank Employees Association (PEMA)” or, in the alternative, both parties be allowed to use
the name “PEMA” but with PEMA-FFW and NUBE-PEC be denominated as “PEMA-Bustria Group”
and “PEMA-Serrana Group,” respectively.

265
On the same date, PEMA sent a letter to the PNB management informing its disaffiliation from
NUBE and requesting to stop, effective immediately, the check-off of the P15.00 due for
NUBE.Acting thereon, on July 4, 2003, PNB informed NUBE of PEMA’s letter and its decision to
continue the deduction of the P15.00 fees, but stop its remittance to NUBE effective July 2003. PNB
also notified NUBE that the amounts collected would be held in a trust account pending the
resolution of the issue on PEMA’s disaffiliation.

Alleging unfair labor practice (ULP) for non-implementation of the grievance machinery and
procedure, NUBE brought the matter to the National Conciliation and Mediation Board (NCMB) for
preventive mediation. In time, PNB and NUBE agreed to refer the case to the Office of the DOLE
Secretary for voluntary arbitration.

Meantime, the DOLE denied PEMA’s motion to change its name in the official ballots. The
certification election was finally held on October 17, 2003. The election yielded the following
results:

Number of eligible voters 3,742


Number of valid votes cast 2,993
Number of spoiled ballots 72
Total 3,065

Philnabank Employees Association-FFW 289


National Union of Bank Employees (NUBE)-Philippine National Bank (PNB) Chapter 2,683
No Union 21
2,993
Total

On April 28, 2004, PEMA filed before the voluntary arbitrator an Urgent Motion for
Intervention,15 alleging that it stands to be substantially affected by whatever judgment that may be
issued, because one of the issues for resolution is the validity of its disaffiliation from NUBE. It
further claimed that its presence is necessary so that a complete relief may be accorded to the
parties. Only NUBE opposed the motion, arguing that PEMA has no legal personality to intervene, as
it is not a party to the existing CBA; and that NUBE is the exclusive bargaining representative of the
PNB rank-and-file employees and, in dealing with a union other than NUBE, PNB is violating the
duty to bargain collectively, which is another form of ULP.

Barely a month after, DOLE Acting Secretary Manuel G. Imson denied PEMA’s motion for
intervention and ordered PNB to release all union dues withheld and to continue remitting the
same to NUBE.

Aggrieved, PEMA filed before the CA a petition under Rule 43 of the Rules on Civil Procedure with
prayer for the issuance of a temporary restraining order (TRO) or writ of preliminary injunction
(WPI). The CA denied the application for WPI. PEMA’s motion for reconsideration was also denied

266
on February 24, 2005, noting PNB’s manifestation that it would submit to the judgment of the CA as
to which party it should remit the funds collected from the employees.

On June 21, 2005, however, petitioner again filed an Urgent Motion for the Issuance of a TRO
against the June 10, 2005 Resolution of DOLE Acting Secretary Imson, which ordered PNB to
properly issue a check directly payable to the order of NUBE covering the withheld funds from the
trust account.20 Considering the different factual milieu, the CA resolved to grant the motion.

Subsequent to the parties’ submission of memoranda, the CA promulgated its May 22, 2006
Decision, declaring the validity of PEMA’s disaffiliation from NUBE and directing PNB to return to
the employees concerned the amounts deducted and held in trust for NUBE starting July 2003 and
to stop further deductions in favor of NUBE.

NUBE filed a motion for reconsideration, but it was denied; hence, this petition.

Issue:

Whether or not PEMA validly disaffiliated itself from NUBE, which in turn, affects the latter’s right
to collect the union dues held in trust by PNB.

Held:

We deny the petition.

The right of the local union to exercise the right to disaffiliate from its mother union is well settled
in this jurisdiction. In MSMG-UWP v. Hon. Ramos, We held:

A local union has the right to disaffiliate from its mother union or declare its autonomy. A local
union, being a separate and voluntary association, is free to serve the interests of all its members
including the freedom to disaffiliate or declare its autonomy from the federation which it belongs
when circumstances warrant, in accordance with the constitutional guarantee of freedom of
association.

The purpose of affiliation by a local union with a mother union [or]


a federation "x xx is to increase by collective action the bargaining power in respect of the terms
and conditions of labor. Yet the locals remained the basic units of association, free to serve their
own and the common interest of all, subject to the restraints imposed by the Constitution and By-
Laws of the Association, and free also to renounce the affiliation for mutual welfare upon the terms
laid down in the agreement which brought it into existence."

Thus, a local union which has affiliated itself with a federation is free to sever such affiliation
anytime and such disaffiliation cannot be considered disloyalty. In the absence of specific

267
provisions in the federation's constitution prohibiting disaffiliation or the declaration of autonomy
of a local union, a local may dissociate with its parent union.

The recent case of Cirtek Employees Labor Union-Federation of Free Workers v. Cirtek Electronics,
Inc35 ruled:

x xx [A] local union may disaffiliate at any time from its mother federation, absent any
showing that the same is prohibited under its constitution or rule. Such, however, does not
result in it losing its legal personality altogether.

Verily, Anglo-KMU v. Samahan Ng Mga Manggagawang Nagkakaisa Sa Manila Bar Spinning Mills At
J.P. Coatsenlightens:

A local labor union is a separate and distinct unit primarily designed to secure and maintain an
equality of bargaining power between the employer and their employee-members. A local union
does not owe its existence to the federation with which it is affiliated. It is a separate and
distinct voluntary association owing its creation to the will of its members. The mere act of
affiliation does not divest the local union of its own personality, neither does it give the
mother federation the license to act independently of the local union. It only gives rise to a
contract of agency where the former acts in representation of the latter.

These and many more have consistently reiterated the earlier view that the right of the local
members to withdraw from the federation and to form a new local union depends upon the
provisions of the union's constitution, by-laws and charter and, in the absence of enforceable
provisions in the federation's constitution preventing disaffiliation of a local union, a local may
sever its relationship with its parent. In the case at bar, there is nothing shown in the records nor is
it claimed by NUBE that PEMA was expressly forbidden to disaffiliate from the federation nor were
there any conditions imposed for a valid breakaway. This being so, PEMA is not precluded to
disaffiliate from NUBE after acquiring the status of an independent labor organization duly
registered before the DOLE.

Also, there is no merit on NUBE’s contention that PEMA’s disaffiliation is invalid for non-observance
of the procedure that union members should make such determination through secret ballot and
after due deliberation, conformably with Article 241 (d) of the Labor Code, as
amended.38 Conspicuously, other than citing the opinion of a “recognized labor law authority,”
NUBE failed to quote a specific provision of the law or rule mandating that a local union’s
disaffiliation from a federation must comply with Article 241 (d) in order to be valid and effective.

Granting, for argument’s sake, that Article 241 (d) is applicable, still, We uphold PEMA’s
disaffiliation from NUBE. First, non-compliance with the procedure on disaffiliation, being premised
on purely technical grounds cannot rise above the employees’ fundamental right to self-
organization and to form and join labor organizations of their own choosing for the purpose of
collective bargaining.39Second, the Article nonetheless provides that when the nature of the

268
organization renders such secret ballot impractical, the union officers may make the decision in
behalf of the general membership. In this case, NUBE did not even dare to contest PEMA’s
representation that “PNB employees, from where [PEMA] [derives] its membership, are scattered
from Aparri to Jolo, manning more than 300 branches in various towns and cities of the country,”
hence, “[to] gather the general membership of the union in a general membership to vote through
secret balloting is virtually impossible.”40 It is understandable, therefore, why PEMA’s board of
directors merely opted to submit for ratification of the majority their resolution to disaffiliate from
NUBE. Third, and most importantly, NUBE did not dispute the existence of the persons or their due
execution of the document showing their unequivocal support for the disaffiliation of PEMA from
NUBE. Note must be taken of the fact that the list of PEMA members (identifying themselves as
“PEMA-Serrana Group”41) who agreed with the board resolution was attached as Annex “H” of
PEMA’s petition before the CA and covered pages 115 to 440 of the CA rollo. While fully displaying
the employees’ printed name, identification number, branch, position, and signature, the list was
left unchallenged by NUBE. No evidence was presented that the union members’ ratification was
obtained by mistake or through fraud, force or intimidation. Surely, this is not a case where one or
two members of the local union decided to disaffiliate from the mother federation, but one where
more than a majority of the local union members decided to disaffiliate.

Consequently, by PEMA's valid disaffiliation from NUBE, the vinculum that previously bound the
two entities was completely severed. As NUBE was divested of any and all power to act in
representation of PEMA, any act performed by the former that affects the interests and affairs of the
latter, including the supposed expulsion of Serrana et al., is rendered without force and effect.

Also, in effect, NUBE loses it right to collect all union dues held in its trust by PNB. The moment that
PEMA separated from and left NUBE and exists as an independent labor organization with a
certificate of registration, the former is no longer obliged to pay dues and assessments to the latter;
naturally, there would be no longer any reason or occasion for PNB to continue making deductions.

On the other hand, it was entirely reasonable for PNB to enter into a CBA with PEMA as represented
by Serrana et al. Since PEMA had validly separated itself from NUBE, there would be no restrictions
which could validly hinder it from collectively bargaining with PNB.

[G.R. No. 175492, February 27, 2013]

CARLOS L. OCTAVIO, PETITIONER,


VS.
PHILIPPINE LONG DISTANCE TELEPHONE COMPANY, RESPONDENT.

Facts:
On May 28, 1999, PLDT and Gabay ng Unyon sa Telekominaksyon ng mga Superbisor (GUTS) entered
into a CBA covering the period January 1, 1999 to December 31, 2001 (CBA of 1999-2001). Article
VI, Section I thereof provides:

269
Section 1. The COMPANY agrees to grant the following across-theboard salary increase during the
three years covered by this Agreement to all employees covered by the bargaining unit as of the given
dates:
Effective January 1, 1999 – 10% of basic wage or P2,000.00 whichever is higher;
Effective January 1, 2000 – 11% of basic wage or P2,250.00 whichever is higher;
Effective January 1, 2001 – 12% of basic wage or P2,500.00 whichever is higher.8
On October 1, 2000, PLDT hired Octavio as Sales System Analyst I on a probationary status. He
became a member of GUTS. When Octavio was regularized on January 1, 2001, he was receiving a
monthly basic salary ofP10,000.00. On February 1, 2002, he was promoted to the position of Sales
System Analyst 2 and his salary was increased to P13,730.00.
On May 31, 2002, PLDT and GUTS entered into another CBA covering the period January 1, 2002 to
December 31, 2004 (CBA of 2002-2004) which provided for the following salary increases: 8% of
basic wage or P2,000.00 whichever is higher for the first year (2002); 10% of basic wage
or P2,700.00 whichever is higher for the second year (2003); and, 10% of basic wage or P2,400.00
whichever is higher for the third year (2004).9
Claiming that he was not given the salary increases of P2,500.00 effective January 1, 2001
and P2,000.00 effective January 1, 2002, Octavio wrote the President of GUTS, Adolfo Fajardo
(Fajardo).Accordingly, the Grievance Committee convened on October 7, 2002 consisting of
representatives from PLDT and GUTS. The Grievance Committee, however, failed to reach an
agreement. In effect, it denied Octavio’s demand for salary increases (Committee Resolution).
Aggrieved, Octavio filed before the Arbitration Branch of the NLRC a Complaint for payment of
said salary increases.

Issue:
Whether the Court of Appeals erred in upholding the decision of the NLRC in denying his
claim for salary increases against PLDT.

Held:
Under Article 26019 of the Labor Code, grievances arising from the interpretation or
implementation of the parties’ CBA should be resolved in accordance with the grievance procedure
embodied therein. It also provides that all unsettled grievances shall be automatically
referred for voluntary arbitration as prescribed in the CBA.
It is settled that "when parties have validly agreed on a procedure for resolving grievances
and to submit a dispute to voluntary arbitration then that procedure should be strictly
observed."22 Moreover, we have held time and again that "before a party is allowed to seek the
intervention of the court, it is a precondition that he should have availed of all the means of
administrative processes afforded him. Hence, if a remedy within the administrative machinery can
still be resorted to by giving the administrative officer concerned every opportunity to decide on a
matter that comes within his jurisdiction, then such remedy should be exhausted first before the
court’s judicial power can be sought. The premature invocation of the court’s judicial intervention is
fatal to one’s cause of action."23 "The underlying principle of the rule on exhaustion of
administrative remedies rests on the presumption that when the administrative body, or grievance
machinery, is afforded a chance to pass upon the matter, it will decide the same correctly."24

270
By failing to question the Committee Resolution through the proper procedure prescribed in
the CBA, that is, by raising the same before a Board of Arbitrators, Octavio is deemed to have
waived his right to question the same. Clearly, he departed from the grievance procedure
mandated in the CBA and denied the Board of Arbitrators the opportunity to pass upon a
matter over which it has jurisdiction. Hence, and as correctly held by the CA, Octavio’s failure
to assail the validity and enforceability of the Committee Resolution makes the same binding
upon him. On this score alone, Octavio’s recourse to the labor tribunals below, as well as to the
CA, and, finally, to this Court, must therefore fail.
At any rate, Octavio cannot claim that the Committee Resolution is not valid, binding and conclusive
as to him for being a modification of the CBA. It bears to stress that the said resolution is a product
of the grievance procedure outlined in the CBA itself. It was arrived at after the management and
the union through their respective representatives conducted negotiations in accordance
with the CBA. On the other hand, Octavio never assailed the competence of the grievance
committee to take cognizance of his case. Neither did he question the authority or credibility of the
union representatives; hence, the latter are deemed to have properly bargained on his behalf
since "unions are the agent of its members for the purpose of securing just and fair wages
and good working conditions."26 In fine, it cannot be gainsaid that the Committee Resolution is a
modification of the CBA. Rather, it only provides for the proper implementation of the CBA
provision respecting salary increases.
WHEREFORE, the petition is DENIED.

22. RIGHTS OF LEGITIMATE LABOR ORGANIZATION

[G.R. Nos. 184903, October 10, 2012]

DIGITAL TELECOMMUNICATIONS PHILIPPINES, INC., PETITIONER,


VS.
DIGITEL EMPLOYEES UNION (DEU), ARCELO RAFAEL A. ESPLANA, ALAN D. LICANDO,
FELICITO C. ROMERO, JR., ARNOLD D. GONZALES, REYNEL FRANCISCO B. GARCIA, ZOSIMO B.
PERALTA, REGINO T. UNIDAD AND JIM L. JAVIER, RESPONDENTS.

Facts:
By virtue of a certification election, Digitel Employees Union (Union) became the exclusive
bargaining agent of all rank and file employees of Digitel in 1994. The Union and Digitel then
commenced collective bargaining negotiations which resulted in a bargaining deadlock. The Union
threatened to go on strike, but then Acting Labor Secretary Bienvenido E. Laguesma assumed
jurisdiction over the dispute and eventually directed the parties to execute a CBA.2
However, no CBA was forged between Digitel and the Union. Some Union members abandoned
their employment with Digitel. The Union later became dormant.

271
Ten (10) years thereafter or on 28 September 2004, Digitel received from Arceo Rafael A. Esplana
(Esplana), who identified himself as President of the Union, a letter containing the list of officers,
CBA proposals and ground rules.3 The officers were respondents Esplana, Alan D. Licando (Vice-
President), Felicito C. Romero, Jr. (Secretary), Arnold D. Gonzales (Treasurer), Reynel Francisco B.
Garcia (Auditor), Zosimo B. Peralta (PRO), Regino T. Unidad (Sgt. at Arms), and Jim L. Javier (Sgt. at
Arms).
Digitel was reluctant to negotiate with the Union and demanded that the latter show compliance
with the provisions of the Union’s Constitution and By-laws on union membership and election of
officers.
On 4 November 2004, Esplana and his group filed a case for Preventive Mediation before the
National Conciliation and Mediation Board based on Digitel’s violation of the duty to bargain.
On 25 November 2004, Esplana filed a notice of strike.
On 10 March 2005, then Labor Secretary Patricia A. Sto. Tomas issued an Order4 assuming
jurisdiction over the labor dispute.
During the pendency of the controversy, Digitel Service, Inc. (Digiserv), a non-profit
enterprise engaged in call center servicing, filed with the Department of Labor and
Employment (DOLE) an Establishment Termination Report stating that it will cease its
business operation. The closure affected at least 100 employees, 42 of whom are members of
the herein respondent Union.
Alleging that the affected employees are its members and in reaction to Digiserv’s action, Esplana
and his group filed another Notice of Strike for union busting, illegal lock-out, and violation of the
assumption order.
On 23 May 2005, the Secretary of Labor ordered the second notice of strike subsumed by the
previous Assumption Order.5
Meanwhile, on 14 March 2005, Digitel filed a petition with the Bureau of Labor Relations
(BLR) seeking cancellation of the Union’s registration on the following grounds: 1) failure to
file the required reports from 1994-2004; 2) misrepresentation of its alleged officers; 3)
membership of the Union is composed of rank and file, supervisory and managerial
employees; and 4) substantial number of union members are not Digitel employees.6
In a Decision dated 11 May 2005, the Regional Director of the DOLE dismissed the petition for
cancellation of union registration for lack of merit. The appeal filed by Digitel with the BLR was
eventually dismissed for lack of merit in a Resolution dated 9 March 2007, thereby affirming the
decision of the Regional Director.

Issue:
Whether Digitel can be compelled to negotiate with a union for purposes of collective bargaining
when the very status of the same as the exclusive bargaining agent is in question.

Held:
The pendency of a petition for cancellation of union registration does not preclude collective
bargaining.
The 2005 case of Capitol Medical Center, Inc. v. Hon. Trajano13 is apropos. The respondent union
therein sent a letter to petitioner requesting a negotiation of their CBA. Petitioner refused to

272
bargain and instead filed a petition for cancellation of the union’s certificate of registration.
Petitioner’s refusal to bargain forced the union to file a notice of strike. They eventually staged a
strike. The Secretary of Labor assumed jurisdiction over the labor dispute and ordered all striking
workers to return to work. Petitioner challenged said order by contending that its petition for
cancellation of union’s certificate of registration involves a prejudicial question that should
first be settled before the Secretary of Labor could order the parties to bargain collectively.
When the case eventually reached this Court, we agreed with the Secretary of Labor that the
pendency of a petition for cancellation of union registration does not preclude collective
bargaining, thus:

That there is a pending cancellation proceeding against the respondent Union is not a bar to set in
motion the mechanics of collective bargaining. If a certification election may still be ordered despite
the pendency of a petition to cancel the union’s registration certificate (National Union of Bank
Employees vs. Minister of Labor, 110 SCRA 274), more so should the collective bargaining process
continue despite its pendency. We must emphasize that the majority status of the respondent
Union is not affected by the pendency of the Petition for Cancellation pending against it. Unless
its certificate of registration and its status as the certified bargaining agent are revoked, the
Hospital is, by express provision of the law, duty bound to collectively bargain with the Union.14
Trajano was reiterated in Legend International Resorts Limited v. KilusangManggagawangLegenda
(KML-Independent).15 Legend International Resorts reiterated the rationale for allowing the
continuation of either a CBA process or a certification election even during the pendency of
proceedings for the cancellation of the union’s certificate of registration. Citing the cases of
Association of Court of Appeals Employees v. Ferrer- Calleja16 and SamahanngManggagawasa
Pacific Plastic v. Hon. Laguesma,17 it was pointed out at the time of the filing of the petition for
certification election – or a CBA process as in the instant case – the union still had the personality to
file a petition for certification − or to ask for a CBA negotiation – as in the present case.
WHEREFORE, the Petition is DENIED.

[G.R. No. 160138, January 16, 2013]

AUTOMOTIVE ENGINE REBUILDERS


VS.
PROGRESIBONG UNYON NG MGA MANGGAGAWA SA AER
RESOLUTION ON THE MAIN DECISION OF JULY 13, 2011

Facts:
This labor controversy started when both parties filed charges against each other, blaming the
other party for violating labor laws. Thirty-two (32) employees filed and signed a complaint, dated
February 18, 1999, against Automotive Engine Rebuilders, Inc. (AER). The complaint prayed that
AER be declared guilty of Unfair Labor Practices, Illegal Dismissal, Illegal Suspension, and Run-away

273
shop; that the complainants be reinstated; and that they be paid "full backwages and without loss of
seniority rights and privileges, payment of wages during suspension, plus moral and exemplary
damages and attorney's fees.
Out of the 32, six (6) resigned and signed waivers and quitclaims.
On the other hand, AER filed a complaint against Unyon and eighteen (18) of its members for illegal
concerted activities.

AER likewise suspended seven (7) union members who tested positive for illegal drugs.
Out of the seven (7) suspended employees, only Edwin Fabian and NazarioMadala were allowed by
AER to report back to work. The other five (5) suspended employees were not admitted by AER
without first submitting the required medical certificate attesting to their fitness to work.
Labor Arbiter (LA) rendered a decision in favor of Unyon by directing AER to reinstate the
concerned employees but without backwages effective October 16, 2001
Both parties filed their respective appeals with the National Labor Relations Commission (NLRC).
The NLRC issued its Resolution modifying the LA decision by setting aside the order of
reinstatement. The NLRC likewise ruled that the concerned employees had no valid basis in
conducting a strike.

Unyon filed a motion for reconsideration insisting, among others, that AER was guilty of unfair
labor practice, illegal suspension and illegal dismissal. Unyon also argued that since AER charged
only 18 of the 32 employees with illegal strike, the employees who were not included in the said
charge should have been admitted back to work by AER.NLRC denied their motion for
reconsideration.

Unyon and the concerned employees filed a petition before the Court of Appeals (CA). Unyon
reiterated its argument that AER should admit back to work those excluded from its list of 18
employees charged with illegal strike.
CA rendered a decision ordering AER to reinstate the petitioners immediately but without
backwages, except those who were tested positive for illegal drugs and have failed to submit
their respective medical certificates.

The Union filed a motion for partial consideration.

CA ordered the immediate reinstatement of all the suspended employees without backwages.
Unsatisfied, both parties filed the present consolidated petitions. Unyon argued that the CA erred in
not awarding backwages to the suspended employees who were ordered reinstated. AER, on the
other hand, argued that the CA erred in ordering the reinstatement of the suspended employees.

On July 13, 2011, this Court rendered a decision:


WHEREFORE, the petitions are DENIED. Accordingly, the complaining employees should be
reinstated without backwages. If reinstatement is no longer feasible, the concerned employees
should be given separation pay up to the date set for their return in lieu of reinstatement

274
Unyon filed the subject Motion for Partial Reconsideration questioning the Court's July 13, 2011
Decision insofar as it failed to award backwages to fourteen (14) of its members.
Unyon argues that backwages should have been awarded to the 14 employees who were excluded
from the complaint filed by AER and that the latter should have reinstated them immediately
because they did not have any case at all.

Issues:

1. Whether the fourteen (14) excluded employees deserve to be reinstated.


2. Whether backwages is proper to be awarded to these fourteen (14) employees.

Held:
After going over the records again, the Court holds that only nine (9) of the fourteen (14) excluded
employees deserve to be reinstated immediately with backwages.
Records disclose that thirty-two (32) employees filed a complaint for illegal suspension and unfair
labor practice against AER. Out of these 32 workers, only eighteen (18) of them were charged by
AER with illegal strike leaving fourteen (14) of them excluded from its complaint.
Technically, as no charges for illegal strike were filed against these 14 employees, they cannot be
among those found guilty of illegal strike. They cannot be considered in pari delicto. They should be
reinstated and given their backwages.
Out of these 14 employees, however, five (5) failed to write their names and affix their signatures in
the Membership Resolution attached to the petition filed before the CA, authorizing Union
President Arnold Villota to represent them.
Because of their failure to affix their names and signatures in the Membership Resolution, Edwin
Mendoza, Tammy Punzalan, Edward Ferrancol, Menching Mariano, Jr. and Carlos Carolina cannot
be granted the relief that Unyon wanted for them in its Motion for Partial Reconsideration. Only the
following nine (9) employees who signed their names in the petition can be granted the relief
prayed for therein.
These excluded nine (9) workers, who signed their names in their petition before the CA, deserve to
be reinstated immediately and granted backwages. It is basic in jurisprudence that illegally
dismissed workers are entitled to reinstatement with backwages plus interest at the legal rate.
However the reinstatement shall be "without prejudice to the right of private respondent AER to
subject them for further medical check-up to determine if subject petitioners are drug dependents."
WHEREFORE, the Motion for Partial Reconsideration filed by ProgresibongUnyon Ng
MgaManggagawaSa AER is GRANTED only insofar as the nine (9) employees are concerned,
namely: Ruperto Mariano II, Arnold Rodriguez, FroilanMadamba, DaniloQuiboy, Roger Belatcha,
Roberto Caldeo, CrisantoLumbao, Jr., Arnold Villota, and Renato Sarabuno.
Accordingly, the July 13, 2011 Decision is hereby MODIFIED in that the aforementioned nine (9)
workers are entitled to be reinstated and granted backwages with interest at the rate of six percent
(6%) per annum which shall be increased to twelve percent (12%) after the finality of this
judgment.

275
[G.R. No. 179146, July 23, 2013]

HOLY CHILD CATHOLIC SCHOOL


VS.
HON. STO. TOMAS, ET AL.

Facts:
On May 31, 2002, a petition for certification election was filed by private respondent HCCS-TELU-
PIGLA. PIGLAS is a legitimate labor organization duly registered with the Department of Labor and
Employment (DOLE) representing HCCS-TELU-PIGLAS.

Petitioner HCCS is a private educational institution duly registered and operating under Philippine
laws. HCCS is unorganized, there is no collective bargaining agreement or a duly certified
bargaining agent or a labor organization certified as the sole and exclusive bargaining agent of the
proposed bargaining unit within one year prior to the filing of the petition.

Petitioner raised that members of private respondent do not belong to the same class; it is not only
a mixture of managerial, supervisory, and rank-and-file employees — as three (3) are vice-
principals, one (1) is a department head/supervisor, and eleven (11) are coordinators — but also a
combination of teaching and non-teaching personnel — as twenty-seven (27) are non-teaching
personnel. It insisted that, for not being in accord with Article 245 of the Labor Code, private
respondent is an illegitimate labor organization lacking in personality to file a petition for
certification election, as held in Toyota Motor Philippines Corporation v. Toyota Motor Philippines
Corporation Labor Union; and an inappropriate bargaining unit for want of community or mutuality
of interest, as ruled in Dunlop Slazenger (Phils.), Inc. v. Secretary of Labor and Employment and De La
Salle University Medical Center and College of Medicine v. Laguesma. THAID

rivate respondent cites Sections 9 and 12 of Republic Act (R.A.) No. 9481 to buttress its contention
that petitioner has no standing to oppose the petition for certification election. On the basis of the
statutory provisions, it reasons that an employer is not a party-in-interest in a certification election;
thus, petitioner does not have the requisite right to protect even by way of restraining order or
injunction.

The Med-Arbiter denied the petition for certification election. SOLE ruled against the dismissal of
the petition and directed the conduct of two separate certification elections for the teaching and the
non-teaching personnel.Petitioner filed before the CA a Petition for Certiorari with Prayer for
Temporary Restraining Order and Preliminary Injunction. The CA eventually dismissed the petition.

Issues:

1. Whether a petition for certification election is dismissible on the ground that the labor
organization's membership allegedly consists of supervisory and rank-and-file employees.

276
2. Whether private respondent is not qualified to file a petition for certification election for its
failure to qualify as a legitimate labor organization due to the absence of mutuality of interest
among its members in view of the improper mixture of teaching and non-teaching personnel.

Held:
First off, We cannot agree with private respondent's invocation of R.A. No. 9481. Said law took
effect only on June 14, 2007; hence, its applicability is limited to labor representation cases filed on
or after said date. Instead, the law and rules in force at the time private respondent filed its petition
for certification election on May 31, 2002 are R.A. No. 6715, which amended Book V of Presidential
Decree (P.D.) No. 442 (the Labor Code), as amended, and the Rules and Regulations Implementing
R.A. No. 6715, as amended by D.O. No. 9, which was dated May 1, 1997 but took effect on June 21,
1997.
However, note must be taken that even without the express provision of Section 12 of RA No. 9481,
the "Bystander Rule" is already well entrenched in this jurisdiction. It has been consistently held in
a number of cases that a certification election is the sole concern of the workers, except when the
employer itself has to file the petition pursuant to Article 259 of the Labor Code, as amended, but
even after such filing its role in the certification process ceases and becomes merely a bystander.
The employer clearly lacks the personality to dispute the election and has no right to interfere at all
therein. This is so since any uncalled-for concern on the part of the employer may give rise to the
suspicion that it is batting for a company union. Indeed, the demand of the law and policy for an
employer to take a strict, hands-off stance in certification elections is based on the rationale that the
employees' bargaining representative should be chosen free from any extraneous influence of the
management; that, to be effective, the bargaining representative must owe its loyalty to the
employees alone and to no other.

As to the 1st issue:


It should be emphasized that the petitions for certification election involved in Toyota and Dunlop
were filed on November 26, 1992 and September 15, 1995, respectively; hence, the 1989 Rules was
applied in both cases.
But then, on June 21, 1997, the 1989 Amended Omnibus Rules was further amended by Department
Order No. 9, series of 1997 (1997 Amended Omnibus Rules). Specifically, the requirement under
Sec. 2(c) of the 1989 Amended Omnibus Rules — that the petition for certification election indicate
that the bargaining unit of rank-and-file employees has not been mingled with supervisory
employees — was removed.
In Pagpalain Haulers, Inc. v. Trajano, the Court had occasion to uphold the validity of the 1997
Amended Omnibus Rules.
Then cameTagaytay Highlands Int'l. Golf Club, Inc. v. Tagaytay Highlands Employees Union-PTGWO in
which the core issue was whether mingling affects the legitimacy of a labor organization and its
right to file a petition for certification election. This time, given the altered legal milieu, the Court
abandoned the view in Toyota and Dunlop and reverted to its pronouncement in Lopez that while
there is a prohibition against the mingling of supervisory and rank-and-file employees in one labor
organization, the Labor Code does not provide for the effects thereof. Thus, the Court held that after

277
a labor organization has been registered, it may exercise all the rights and privileges of a legitimate
labor organization. Any mingling between supervisory and rank-and-file employees in its
membership cannot affect its legitimacy for that is not among the grounds for cancellation of its
registration, unless such mingling was brought about by misrepresentation, false statement or
fraud under Article 239 of the Labor Code.
In San Miguel Corp. (Mandaue Packaging Products Plants) v. Mandaue Packing Products Plants-San
Miguel Packaging Products-San Miguel Corp. Monthlies Rank-and-File Union-FFW, the Court
explained that since the 1997 Amended Omnibus Rules does not require a local or chapter to
provide a list of its members, it would be improper for the DOLE to deny recognition to said local or
chapter on account of any question pertaining to its individual members.
More to the point is Air Philippines Corporation v. Bureau of Labor Relations, which involved a
petition for cancellation of union registration filed by the employer in 1999 against a rank-and-file
labor organization on the ground of mixed membership: the Court therein reiterated its ruling in
Tagaytay Highlands that the inclusion in a union of disqualified employees is not among the
grounds for cancellation, unless such inclusion is due to misrepresentation, false statement or fraud
under the circumstances enumerated in Sections (a) and (c) of Article 239 of the Labor Code.
All said, while the latest issuance is R.A. No. 9481, the 1997 Amended Omnibus Rules, as interpreted
by the Court in Tagaytay Highlands, San Miguel and Air Philippines, had already set the tone for it.
Toyota and Dunlop no longer hold sway in the present altered state of the law and the rules.
When a similar issue confronted this Court close to three years later, the above ruling was
substantially quoted in SamahangManggagawasa Charter Chemical Solidarity of Unions in the
Philippines for Empowerment and Reforms (SMCC-Super) v. Charter Chemical and Coating
Corporation. In unequivocal terms, We reiterated that the alleged inclusion of supervisory
employees in a labor organization seeking to represent the bargaining unit of rank-and-file
employees does not divest it of its status as a legitimate labor organization.
Indeed, Toyota and Dunlop no longer hold true under the law and rules governing the instant case.
The petitions for certification election involved in Toyota and Dunlop were filed on November 26,
1992 and September 15, 1995, respectively; hence, the 1989 Rules and Regulations Implementing
R.A. No. 6715 (1989 Amended Omnibus Rules) was applied. In contrast, D.O. No. 9 is applicable in
the petition for certification election of private respondent as it was filed on May 31, 2002.
Following the doctrine laid down in Kawashima and SMCC-Super, it must be stressed that petitioner
cannot collaterally attack the legitimacy of private respondent by praying for the dismissal of the
petition for certification election.

As to the 2nd issue:


The concepts of a union and of a legitimate labor organization are different from, but related to, the
concept of a bargaining unit:
Article 212(g) of the Labor Code defines a labor organization as "any union or association of
employees which exists in whole or in part for the purpose of collective bargaining or of dealing
with employers concerning terms and conditions of employment." Upon compliance with all the
documentary requirements, the Regional Office or Bureau shall issue in favor of the applicant labor
organization a certificate indicating that it is included in the roster of legitimate labor organizations.
Any applicant labor organization shall acquire legal personality and shall be entitled to the rights

278
and privileges granted by law to legitimate labor organizations upon issuance of the certificate of
registration.
In case of alleged inclusion of disqualified employees in a union, the proper procedure for an
employer like petitioner is to directly file a petition for cancellation of the union's certificate of
registration due to misrepresentation, false statement or fraud under the circumstances
enumerated in Article 239 of the Labor Code, as amended. To reiterate, private respondent, having
been validly issued a certificate of registration, should be considered as having acquired juridical
personality which may not be attacked collaterally.
On the other hand, a bargaining unit has been defined as a "group of employees of a given employer,
comprised of all or less than all of the entire body of employees, which the collective interests of all
the employees, consistent with equity to the employer, indicated to be best suited to serve
reciprocal rights and duties of the parties under the collective bargaining provisions of the law."
As the SOLE correctly observed, petitioner failed to comprehend the full import of Our ruling in U.P.
“xxxxA union certified as an exclusive bargaining agent represents not only its members but also
other employees who are not union members.xxx”
Indeed, the purpose of a certification election is precisely to ascertain the majority of the
employees' choice of an appropriate bargaining unit — to be or not to be represented by a labor
organization and, if in the affirmative case, by which one.
WHEREFORE, the petition is DENIED. The April 18, 2007 Decision and July 31, 2007, Resolution of
the Court of Appeals in CA-G.R. SP No. 76175, which affirmed the December 27, 2002 Decision of
the Secretary of the Department of Labor and Employment that set aside the August 10, 2002
Decision of the Med-Arbiter denying private respondent's petition for certification election are
hereby AFFIRMED.

23. REVISED GUIDELINES OF THE NCMB FOR THE CONDUCT OF


VOLUNTARY ARBITRATION PROCEEDINGS

7K CORPORATION
VS.
ALBARICO

Respondent Eddie Albarico (Albarico) was a regular employee of petitioner 7K Corporation, a


company selling water purifiers. He started working for the company in 1990 as a salesman.
Because of his good performance, his employment was regularized. He was also promoted several
times: from salesman, he was promoted to senior sales representative and then to acting team field
supervisor. In 1992, he was awarded the President’s Trophy for being one of the company’s top
water purifier specialist distributors.

In April of 1993, the chief operating officer of petitioner 7K Corporation terminated Albarico’s
employment allegedly for his poor sales performance. Respondent had to stop reporting for work,

279
and he subsequently submitted his money claims against petitioner for arbitration before the
National Conciliation and Mediation Board (NCMB). The issue for voluntary arbitration before the
NCMB, according to the parties’ Submission Agreement dated 19 April 1993, was whether
respondent Albarico was entitled to the payment of separation pay and the sales commission
reserved for him by the corporation.

On 17 September 1997, petitioner corporation filed its Position Paper in the NCMB arbitration case.
It denied that respondent was terminated from work, much less illegally dismissed. The
corporation claimed that he had voluntarily stopped reporting for work after receiving a verbal
reprimand for his sales performance; hence, it was he who was guilty of abandonment of
employment. Respondent made an oral manifestation that he was adopting the position paper he
submitted to the labor arbiter, a position paper in which the former claimed that he had been
illegally dismissed.

Respondent manifested that he was willing to settle the case amicably with petitioner based on the
decision of the labor arbiter ordering the payment of separation pay in lieu of reinstatement,
backwages and attorney’s fees. On its part, petitioner made a counter-manifestation that it was
likewise amenable to settling the dispute. However, it was willing to pay only the separation pay
and the sales commission according to the Submission Agreement dated 19 April 1993.

On 18 November 2005, the NCMB voluntary arbitrator rendered a Decision finding


petitioner corporation liable for illegal dismissal. The termination of respondent Albarico, by
reason of alleged poor performance, was found invalid. The arbitrator explained that the
promotions, increases in salary, and awards received by respondent belied the claim that the latter
was performing poorly. It was also found that Albarico could not have abandoned his job, as the
abandonment should have been clearly shown. Mere absence was not sufficient, according to the
arbitrator, but must have been accompanied by overt acts pointing to the fact that the employee did
not want to work anymore. It was noted that, in the present case, the immediate filing of a
complaint for illegal dismissal against the employer, with a prayer for reinstatement, showed that
the employee was not abandoning his work. The voluntary arbitrator also found that Albarico was
dismissed from his work without due process.

However, it was found that reinstatement was no longer possible because of the strained
relationship of the parties. Thus, in lieu of reinstatement, the voluntary arbitrator ordered the
corporation to pay separation pay for two years at P4,456 for each year, or a total amount of
P8,912. Additionally, in view of the finding that Albarico had been illegally dismissed, the voluntary
arbitrator also ruled that the former was entitled to backwages in the amount of P90,804.19 Finally,
the arbitrator awarded attorney’s fees in respondent’s favor, because he had been compelled to file
an action for illegal dismissal.

280
The CA affirmed the Decision of the voluntary arbitrator, but eliminated the award of attorney’s
fees for having been made without factual, legal or equitable justification. Petitioner’s Motion for
Partial Reconsideration was denied as well.

Issue:

The issue before the Court is whether the CA committed reversible error in finding that the
voluntary arbitrator properly assumed jurisdiction to decide the issue of the legality of the
dismissal of respondent as well as the latter’s entitlement to backwages, even if neither the legality
nor the entitlement was expressedly claimed in the Submission Agreement of the parties.

The Petition is denied for being devoid of merit.

Preliminarily, we address petitioner’s claim that under Article 217 of the Labor Code, original and
exclusive jurisdiction over termination disputes, such as the present case, is lodged only with the
labor arbiter of the NLRC.

Petitioner overlooks the proviso in the said article, thus:

Art. 217. Jurisdiction of the Labor Arbiters and the Commission.


a. Except as otherwise provided under this Code, the Labor Arbiters shall have original and
exclusive jurisdiction to hear and decide, within thirty (30) calendar days after the submission of
the case by the parties for decision without extension, even in the absence of stenographic notes,
the following cases involving all workers, whether agricultural or nonagricultural:
xxxx
2. Termination disputes;
xxxx
6. Except claims for Employees Compensation, Social Security, Medicare and maternity benefits, all
other claims arising from employer-employee relations, including those of persons in domestic or
household service, involving an amount exceeding five thousand pesos (P5,000.00) regardless of
whether accompanied with a claim for reinstatement. (Emphases supplied)

Thus, although the general rule under the Labor Code gives the labor arbiter exclusive and
original jurisdiction over termination disputes, it also recognizes exceptions. One of the
exceptions is provided in Article 262 of the Labor Code. In San Jose v. NLRC, we said:

The phrase "Except as otherwise provided under this Code" refers to the following
exceptions:
A. Art. 217. Jurisdiction of Labor Arbiters . . .
xxxx
(c) Cases arising from the interpretation or implementation of collective bargaining agreement and
those arising from the interpretation or enforcement of company procedure/policies shall be

281
disposed of by the Labor Arbiter by referring the same to the grievance machinery and voluntary
arbitrator as may be provided in said agreement.

B. Art. 262. Jurisdiction over other labor disputes. The Voluntary Arbitrator or panel of Voluntary
Arbitrators, upon agreement of the parties, shall also hear and decide all other labor disputes
including unfair labor practices and bargaining deadlocks. (Emphasis supplied)

Petitioner argues that, assuming that the voluntary arbitrator has jurisdiction over the present
termination dispute, the latter should have limited his decision to the issue contained in the
Submission Agreement of the parties – the issue of whether respondent Albarico was entitled to
separation pay and to the sales commission the latter earned before being terminated. Petitioner
asserts that under Article 262 of the Labor Code, the jurisdiction of a voluntary arbitrator is strictly
limited to the issues that the parties agree to submit. Thus, it contends that the voluntary arbitrator
exceeded his jurisdiction when he resolved the issues of the legality of the dismissal of respondent
and the latter’s entitlement to backwages on the basis of a finding of illegal dismissal.

According to petitioner, the CA wrongly concluded that the issue of respondent’s entitlement to
separation pay was necessarily based on his allegation of illegal dismissal, thereby making the issue
of the legality of his dismissal implicitly submitted to the voluntary arbitrator for resolution.
Petitioner argues that this was an erroneous conclusion, because separation pay may in fact be
awarded even in circumstances in which there is no illegal dismissal.

The circumstances of the instant case lead to no other conclusion than that the claim of
respondent Albarico for separation pay was premised on his allegation of illegal dismissal.
Thus, the voluntary arbitrator properly assumed jurisdiction over the issue of the legality of
his dismissal.

Hence, the voluntary arbitrator correctly assumed that the core issue behind the issue of separation
pay is the legality of the dismissal of respondent. Moreover, we have ruled in Sime Darby Pilipinas,
Inc. v. Deputy Administrator Magsalin that a voluntary arbitrator has plenary jurisdiction and
authority to interpret an agreement to arbitrate and to determine the scope of his own authority
when the said agreement is vague — subject only, in a proper case, to the certiorari jurisdiction of
this Court.

Having established that the issue of the legality of dismissal of Albarico was in fact necessarily –
albeit not explicitly – included in the Submission Agreement signed by the parties, this Court rules
that the voluntary arbitrator rightly assumed jurisdiction to decide the said issue.

Consequently, we also rule that the voluntary arbitrator may award backwages upon a finding of
illegal dismissal, even though the issue of entitlement thereto is not explicitly claimed in the

282
Submission Agreement. Backwages, in general, are awarded on the ground of equity as a form of
relief that restores the income lost by the terminated employee by reason of his illegal dismissal.

In the present case, there is no indication that the issue of illegal dismissal should be treated
as a two-tiered issue whereupon entitlement to backwages must be determined separately.
Besides, "since arbitration is a final resort for the adjudication of disputes," the voluntary
arbitrator in the present case can assume that he has the necessary power to make a final
settlement. Thus, we rule that the voluntary arbitrator correctly assumed jurisdiction over
the issue of entitlement of respondent Albarico to backwages on the basis of the former's
finding of illegal dismissal.

GR No. 157086

LEPANTO CONSOLIDATED MINING COMPANY


VS.
LEPANTO

Capatazes are not rank-and-file employees because they perform supervisory functions for the
management; hence, they may form their own union that is separate and distinct from the labor
organization of rank-and-file employees.

As a domestic corporation authorized to engage in large-scale mining, Lepanto operated several


mining claims in Mankayan, Benguet. On May 27, 1998, respondent Lepanto Capataz Union (Union),
a labor organization duly registered with DOLE, filed a petition for consent election with the
Industrial Relations Division of the Cordillera Regional Office (CAR) of DOLE, thereby proposing to
represent 139 capatazes of Lepanto.
In due course, Lepanto opposed the petition, contending that the Union was in reality seeking a
certification election, not a consent election, and would be thereby competing with the Lepanto
Employees Union (LEU), the current collective bargaining agent. Lepanto pointed out that the
capatazes were already members of LEU, the exclusive representative of all rank-and-file
employees of its Mine Division.
On May 2, 2000, Med-Arbiter Michaela A. Lontoc of DOLE-CAR issued a ruling to the effect that the
capatazes could form a separate bargaining unit due to their not being rank-and-file employees, viz:
xxxx
We agree with petitioner that its members perform a function totally different from the rank-and-
file employees. The word capataz is defined in Webster’s Third International Dictionary, 1986 as "a
boss", "foreman" and "an overseer". The employer did not dispute during the hearing that the
capatazes indeed take charge of the implementation of the job orders by supervising and
instructing the miners, mackers and other rank-and-file workers under them, assess and
evaluate their performance, make regular reports and recommends (sic) new systems and
procedure of work, as well as guidelines for the discipline of employees. As testified to by

283
petitioner’s president, the capatazes are neither rank-and-file nor supervisory and, more or
less, fall in the middle of their rank. In this respect, we can see that indeed the capatazes
differ from the rank-and-file and can by themselves constitute a separate bargaining unit.
Lepanto appealed to the DOLE Secretary.
On July 12, 2000, then DOLE Undersecretary Rosalinda Dimapilis- Baldoz (Baldoz), acting by
authority of the DOLE Secretary, affirmed the ruling of Med-Arbiter Lontoc.
On the day of the certification election, however, Lepanto presented an opposition/protest. Hence,
on February 8, 2001, a hearing was held on Lepanto’s opposition/protest. Although the parties
were required in that hearing to submit their respective position papers, Lepanto later opted not to
submit its position paper, and contended that the issues identified during the hearing did not pose
any legal issue to be addressed in a position paper.
Appellant accused Med-Arbiter Ulep of grave abuse of discretion amounting to lack of jurisdiction
based on her failure to resolve appellant’s motion to modify order to submit position papers and on
rendering judgment on the basis only of appellee’s position paper.
We deny.
Section 5, Rule XXV of Department Order No. 9, otherwise known as the New Rules Implementing
Book V of the Labor Code, states that "in all proceedings at all levels, incidental motions shall not be
given due course, but shall remain as part of the records for whatever they may be worth when the
case is decided on the merits".

Section 9, Rule XI of Department Order No. 9, which is applied with equal force in the disposition of
protests on the conduct of election, states that "the Med-Arbiter shall in the same hearing direct all
concerned parties, including the employer, to simultaneously submit their respective position
papers within a non-extendible period of ten days". The issues as recorded in the minutes of 28
February 2001 hearing before the Med- Arbiter are clear. The parties, including appellant company
were required to submit their respective positions on whether there was proper challenge of the
voters, whether LEU failed to participate in the proceedings, if so, whether it should be allowed to
participate at this belated stage and whether the arguments raised during the pre-election
conferences and in the protests are valid. The parties, including appellant company were apprised
of these issues and they agreed thereto. The minutes of the hearing even contained the statement
that "no order will issue" and that "the parties are informed accordingly". If there is any matter that
had to be clarified, appellant should have clarified the same during the said hearing and refused to
file its position paper simultaneously with LCU and LEU. It appears that appellant did not do so and
acquiesced to the filing of its position paper within fifteen days from the date of said hearing.
Still dissatisfied with the result, but without first filing a motion for reconsideration, Lepanto
challenged in the CA the foregoing decision of the DOLE Secretary through a petition for certiorari.

Considering that the petitioner failed to file a prior motion for reconsideration of the Decision of the
public respondent before instituting the present petition as mandated by Section 1 of Rule 65 of the
1997 Rules of Civil Procedure, as amended, the instant "Petition for Certiorari Under Rule 65 with
Prayer for Temporary Restraining Order and Injunction" is hereby DISMISSED.
Well-settled is the rule that the "filing of a petition for certiorari under Rule 65 without first moving
for reconsideration of the assailed resolution generally warrants the petition’s outright dismissal. As

284
we consistently held in numerous cases, a motion for reconsideration by a concerned party is
indispensable for it affords the NLRC an opportunity to rectify errors or mistakes it might have
committed before resort to the courts can be had.
Issues:
I
THE COURT OF APPEALS ERRED IN SUMMARILY DISMISSING THE PETITION FOR CERTIORARI ON
THE GROUND THAT NO PRIOR MOTION FOR RECONSIDERATION WAS FILED. THE DECISION OF
THE SECRETARY BEING FINAL AND EXECUTORY, A MOTION FOR RECONSIDERATION WAS NOT
AN AVAILABLE REMEDY FOR PETITIONER.
II
ON THE MERITS, THE SECRETARY OF LABOR ACTED WITHOUT OR IN EXCESS OF JURISDICTION,
[O]R WITH GRAVE ABUSE OF DISCRETION AMOUNTING TO LACK OR EXCESS OF JURISDICTION IN
ISSUNG THE DECISION DATED SEPTEMBER 17, 2002, WHEN SHE DELIBERATELY IGNORED THE
FACTS AND RULED IN FAVOR OF THE RESPONDENT UNION, DESPITE HER OWN FINDING THAT
THERE HAD BEEN A PREMATURE CANVASS OF VOTES.

The petition for review has no merit.


I. The filing of the motion for reconsideration is a pre-requisite to the filing of a petition for
certiorari to assail the decision of the DOLE Secretary
To start with, the requirement of the timely filing of a motion for reconsideration as a precondition
to the filing of a petition for certiorari accords with the principle of exhausting administrative
remedies as a means to afford every opportunity to the respondent agency to resolve the matter
and correct itself if need be.
And, secondly, the ruling in National Federation of Labor v. Laguesma reiterates St. Martin’s Funeral
Home v. National Labor Relations Commission, where the Court has pronounced that the special civil
action of certiorari is the appropriate remedy from the decision of the National Labor Relations
Commission (NLRC) in view of the lack of any appellate remedy provided by the Labor Code to a
party aggrieved by the decision of the NLRC. Accordingly, any decision, resolution or ruling of the
DOLE Secretary from which the Labor Code affords no remedy to the aggrieved party may be
reviewed through a petition for certiorari initiated only in the CA in deference to the principle of the
hierarchy of courts.
Yet, it is also significant to note that National Federation of Labor v. Laguesma also reaffirmed the
dictum issued in St. Martin’s Funeral Homes v. National Labor Relations Commission to the effect that
"the remedy of the aggrieved party is to timely file a motion for reconsideration as a precondition
for any further or subsequent remedy, and then seasonably avail of the special civil action of
certiorari under Rule 65 x x x."
Under the circumstances, Lepanto’s failure to timely file a motion for reconsideration prior to filing
its petition for certiorari in the CA rendered the September 17, 2002 resolution of the DOLE
Secretary beyond challenge.
II. Capatazes are not rank-and-file employees; hence, they could form their own union

285
Anent the second issue, we note that Med-Arbiter Lontoc found in her Decision issued on May 2,
2000 that the capatazes were performing functions totally different from those performed by the
rank-and-file employees, and that the capatazes were "supervising and instructing the miners,
mackers and other rank-and-file workers under them, assess[ing] and evaluat[ing] their
performance, mak[ing] regular reports and recommend[ing] new systems and procedure of work,
as well as guidelines for the discipline of employees." Hence, Med-Arbiter Lontoc concluded, the
capatazes "differ[ed] from the rank-and-file and [could] by themselves constitute a separate
bargaining unit."
The bargaining unit sought to be represented by the appellee are the capataz employees of the
appellant. There is no other labor organization of capatazes within the employer unit except herein
appellant. Thus, appellant is an unorganized establishment in so far as the bargaining unit of
capatazes is concerned. In accordance with the last paragraph of Section 11, Rule XI, Department
Order No. 9 which provides that "in a petition filed by a legitimate labor organization involving an
unorganized establishment, the Med-Arbiter shall, pursuant to Article 257 of the Code,
automatically order the conduct of certification election after determining that the petition has
complied with all requirements under Section 1, 2 and 4 of the same rules and that none of the
grounds for dismissal thereof exists", the order for the conduct of a certification election is proper.
In any event, we affirm that capatazes or foremen are not rank-andfile employees because they are
an extension of the management, and as such they may influence the rank-and-file workers under
them to engage in slowdowns or similar activities detrimental to the policies, interests or business
objectives of the employers.

24. UNFAIR LABOR PRACTICE

GR No. 170054, January 21, 2013

GOYA INC.
VS.
GOYA INC. EMPLOYEES UNION-FFW

FACTS:

Sometime in January 2004, petitioner Goya, Inc. (Company), a domestic corporation engaged in the
manufacture, importation, and wholesale of top quality food products, hired contractual employees
from PESO Resources Development Corporation (PESO) to perform temporary and occasional
services in its factory in Parang, Marikina City. This prompted respondent Goya, Inc. Employees
Union–FFW (Union) to request for a grievance conference on the ground that the contractual
workers do not belong to the categories of employees stipulated in the existing Collective
Bargaining Agreement (CBA). When the matter remained unresolved, the grievance was referred to
the National Conciliation and Mediation Board (NCMB) for voluntary arbitration.

286
During the hearing on July 1, 2004, the Company and the Union manifested before Voluntary
Arbitrator (VA) Bienvenido E. Laguesma that amicable settlement was no longer possible; hence,
they agreed to submit for resolution the solitary issue of "[w]hether or not the Company is guilty of
unfair labor acts in engaging the services of PESO, a third party service provider, under the existing
CBA, laws, and jurisprudence."
Respondent Union’s contention
The Union asserted that the hiring of contractual employees from PESO is not a management
prerogative and in gross violation of the CBA tantamount to unfair labor practice (ULP). It noted
that the contractual workers engaged have been assigned to work in positions previously handled
by regular workers and Union members, in effect violating Section 4, Article I of the CBA, which
provides for three categories of employees in the Company, to wit:
Section 4. Categories of Employees.– The parties agree on the following categories of employees:
(a) Probationary Employee. – One hired to occupy a regular rank-and-file position in the Company
and is serving a probationary period. If the probationary employee is hired or comes from outside
the Company (non-Goya, Inc. employee), he shall be required to undergo a probationary period of
six (6) months, which period, in the sole judgment of management, may be shortened if the
employee has already acquired the knowledge or skills required of the job. If the employee is hired
from the casual pool and has worked in the same position at any time during the past two (2) years,
the probationary period shall be three (3) months.
(b) Regular Employee. – An employee who has satisfactorily completed his probationary period and
automatically granted regular employment status in the Company.
(c) Casual Employee, – One hired by the Company to perform occasional or seasonal work directly
connected with the regular operations of the Company, or one hired for specific projects of limited
duration not connected directly with the regular operations of the Company.
It was averred that the categories of employees had been a part of the CBA since the 1970s and that
due to this provision, a pool of casual employees had been maintained by the Company from which
it hired workers who then became regular workers when urgently necessary to employ them for
more than a year. Likewise, the Company sometimes hired probationary employees who also later
became regular workers after passing the probationary period. With the hiring of contractual
employees, the Union contended that it would no longer have probationary and casual employees
from which it could obtain additional Union members; thus, rendering inutile Section 1, Article III
(Union Security) of the CBA, which states:
Section 1. Condition of Employment. – As a condition of continued employment in the Company, all
regular rank-and-file employees shall remain members of the Union in good standing and that new
employees covered by the appropriate bargaining unit shall automatically become regular
employees of the Company and shall remain members of the Union in good standing as a condition
of continued employment.
The Union moreover advanced that sustaining the Company’s position would easily weaken and
ultimately destroy the former with the latter’s resort to retrenchment and/or retirement of
employees and not filling up the vacant regular positions through the hiring of contractual workers
from PESO, and that a possible scenario could also be created by the Company wherein it could
"import" workers from PESO during an actual strike.
Petitioner’s contention

287
In countering the Union’s allegations, the Company argued that: (a) the law expressly allows
contracting and subcontracting arrangements through Department of Labor and Employment
(DOLE) Order No. 18-02; (b) the engagement of contractual employees did not, in any way,
prejudice the Union, since not a single employee was terminated and neither did it result in a
reduction of working hours nor a reduction or splitting of the bargaining unit; and (c) Section 4,
Article I of the CBA merely provides for the definition of the categories of employees and does not
put a limitation on the Company’s right to engage the services of job contractors or its management
prerogative to address temporary/occasional needs in its operation. The company contends that it
is within its management prerogative.
VA’s ruling
On October 26, 2004, VA Laguesma dismissed the Union’s charge of ULP for being purely
speculative and for lacking in factual basis, but the Company was directed to observe and comply
with its commitment under the CBA. The VA opined:
“We examined the CBA provision Section 4, Article I of the CBA allegedly violated by the Company
and indeed the agreement prescribes three (3) categories of employees in the Company and
provides for the definition, functions and duties of each. Material to the case at hand is the
definition as regards the functions of a casual employee.
xxx
While the foregoing agreement between the parties did eliminate management’s prerogative of
outsourcing parts of its operations, it serves as a limitation on such prerogative particularly if it
involves functions or duties specified under the aforequoted agreement. It is clear that the parties
agreed that in the event that the Company needs to engage the services of additional workers who
will perform "occasional or seasonal work directly connected with the regular operations of the
COMPANY," or "specific projects of limited duration not connected directly with the regular
operations of the COMPANY", the Company can hire casual employees which is akin to contractual
employees. If we note the Company’s own declaration that PESO was engaged to perform
"temporary or occasional services" (See the Company’s Position Paper, at p. 1), then it should have
directly hired the services of casual employees rather than do it through PESO.
It is evident, therefore, that the engagement of PESO is not in keeping with the intent and spirit of
the CBA provision in question. It must, however, be stressed that the right of management to
outsource parts of its operations is not totally eliminated but is merely limited by the CBA. Given
the foregoing, the Company’s engagement of PESO for the given purpose is indubitably a violation
of the CBA.”
Aggrieved, the Company immediately filed a petition for review9 before the Court of Appeals (CA)
under Rule 43 of the Revised Rules of Civil Procedure to set aside the directive to observe and
comply with the CBA commitment pertaining to the hiring of casual employees when necessitated
by business circumstances.
CA’s ruling
The CA dismissed the petition of the petitioner. The CA declared:
“A careful reading of the above-enumerated categories of employees reveals that the PESO
contractual employees do not fall within the enumerated categories of employees stated in the CBA
of the parties. Following the said categories, the Company should have observed and complied with
the provision of their CBA. Since the Company had admitted that it engaged the services of PESO to

288
perform temporary or occasional services which is akin to those performed by casual employees,
the Company should have tapped the services of casual employees instead of engaging PESO.
In justifying its act, the Company posits that its engagement of PESO was a management
prerogative. The Company’s engagement of PESO was indeed a management prerogative. This is in
consonance with the pronouncement of the Supreme Court in the case of Manila Electric Company
vs. Quisumbing where it ruled that contracting out of services is an exercise of business judgment
or management prerogative.
This management prerogative of contracting out services, however, is not without limitation. In
contracting out services, the management must be motivated by good faith and the contracting out
should not be resorted to circumvent the law or must not have been the result of malicious
arbitrary actions. In the case at bench, the CBA of the parties has already provided for the
categories of the employees in the Company’s establishment. These categories of employees
particularly with respect to casual employees serve as limitation to the Company’s prerogative to
outsource parts of its operations especially when hiring contractual employees. As stated earlier,
the work to be performed by PESO was similar to that of the casual employees. With the provision
on casual employees, the hiring of PESO contractual employees, therefore, is not in keeping with the
spirit and intent of their CBA.”
Aggrieved yet again, the Company raised the issue to the Supreme Court.

ISSUE:
Whether or not the Company is guilty of unfair labor acts in engaging the services of PESO.

RULING:
The Company is guilty of unfair labor acts in engaging the services of PESO, which is in violation of
the existing CBA in the company in accordance with Article 248 of the Labor Code. The CBA is the
law between the parties and they are obliged to comply with its provisions. The company in this
case violated such CBA.
What the VA and the CA correctly ruled was that the Company’s act of contracting out/outsourcing
is within the purview of management prerogative. Both did not say, however, that such act is a valid
exercise thereof. Obviously, this is due to the recognition that the CBA provisions agreed upon by
the Company and the Union delimit the free exercise of management prerogative pertaining to the
hiring of contractual employees. Indeed, the VA opined that "the right of the management to
outsource parts of its operations is not totally eliminated but is merely limited by the CBA," while
the CA held that "this management prerogative of contracting out services, however, is not without
limitation. x x x These categories of employees particularly with respect to casual employees serve
as limitation to the Company’s prerogative to outsource parts of its operations especially when
hiring contractual employees."
A collective bargaining agreement is the law between the parties. It is familiar and fundamental
doctrine in labor law that the CBA is the law between the parties and they are obliged to comply
with its provisions.

GR No. 171118, September 10 2012

289
PARK HOTEL, ET AL.
VS.
SORIANO ET AL.,

FACTS:
Petitioner Park Hotel is a corporation engaged in the hotel business. Petitioners Gregg Harbutt
(Harbutt) and Bill Percy (Percy) are the General Manager and owner, respectively, of Park Hotel.
Percy, Harbutt and Atty. Roberto Enriquez are also the officers and stockholders of Burgos
Corporation (Burgos), a sister company of Park Hotel.

Respondent Manolo Soriano (Soriano) was hired by Park Hotel in July 1990 as Maintenance
Electrician, and then transferred to Burgos in 1992. Respondent Lester Gonzales (Gonzales) was
employed by Burgos as Doorman, and later promoted as Supervisor. Respondent Yolanda Badilla
(Badilla) was a bartender of J's Playhouse operated by Burgos.
In October of 1997, Soriano, Gonzales and Badilla were dismissed from work for allegedly stealing
company properties. As a result, respondents filed complaints for illegal dismissal, unfair labor
practice, and payment of moral and exemplary damages and attorney's fees, before the Labor
Arbiter (LA). In their complaints, respondents alleged that the real reason for their dismissal was
that they were organizing a union for the company's employees.
On the other hand, petitioners alleged that aside from the charge of theft, Soriano and Gonzales
have violated various company rules and regulations contained in several memoranda issued to
them. After dismissing respondents, Burgos filed a case for qualified theft against Soriano and
Gonzales before the Makati City Prosecutor's Office, but the case was dismissed for insufficiency of
evidence.
In his Affidavit, Soriano claimed that on October 4, 1997, he was barred from entering the company
premises and that the following day, Harbutt shouted at him for having participated in the
formation of a union. He was later dismissed from work. For his part, Gonzales averred that he was
coerced to resign by Percy and Harbutt in the presence of their goons. Badilla claimed that she was
also forced by Percy and Harbutt to sign a resignation letter, but she refused to do so because she
was innocent of the charges against her. She was nevertheless dismissed from service.

The three (3) respondents averred that they never received the memoranda containing their
alleged violation of company rules and they argued that these memoranda were fabricated to give a
semblance of cause to their termination. Soriano and Gonzales further claimed that the complaint
filed against them was only an afterthought as the same was filed after petitioners learned that a
complaint for illegal dismissal was already instituted against them.

LABOR ARTIBER’S RULING


The LA rendered a Decision finding that respondents were illegally dismissed because the alleged
violations they were charged with were not reduced in writing and were not made known to them,
thus, denying them due process. The LA found that respondents did not actually receive the

290
memoranda allegedly issued by petitioners, and that the same were mere afterthought to conceal
the illegal dismissal. The LA declared the petitioners guilty of unfair labor practice for terminating
complainants due to their union activities, which is union-busting.
DISCONTENTED WITH THE LA’S DECISION, THE PETITONERS APPEALED THE CASE TO THE
NLRC. THE NLRC affirmed the LA's decision and dismissed the appeal for lack of merit.

CA’s ruling
Later it was appealed to the CA but was again dismissed. The CA ruled that petitioners failed to
observe the mandatory requirements provided by law in the conduct of terminating respondents,
i.e., lack of due process and just cause. The CA also found that petitioners' primary objective in
terminating respondents' employment was to suppress their right to self-organization.

Hence, this petition.

ISSUES:

1. Whether or not the respondents were validly dismissed.


2. WHETHER OR NOT THE PETITIONERS ARE GUILY OF UNFAIR LABOR PRACTICE.

RULING:

1. The respondents were illegally dismissed.

To begin with, it is significant to note that the LA, the NLRC and the CA were unanimous in their
findings that respondents were dismissed without just cause and due process. They were also in
agreement that unfair labor practice was committed against respondents. We reiterate the rule that
findings of fact of the Court of Appeals, particularly where it is in absolute agreement with that of
the NLRC and the LA, as in this case, are accorded not only respect but even finality and are deemed
binding upon this Court so long as they are supported by substantial evidence. The function of this
Court is limited to the review of the appellate court’s alleged errors of law. It is not required to
weigh all over again the factual evidence already considered in the proceedings below. In any event,
we found no compelling reason to disturb the unanimous findings and conclusions of the CA, the
NLRC and the LA with respect to the finding of illegal dismissal.

The requisites for a valid dismissal are: (a) the employee must be afforded due process, i.e., he must
be given an opportunity to be heard and defend himself; and (b) the dismissal must be for a valid
cause as provided in Article 282 of the Labor Code, or for any of the authorized causes under
Articles 283 and 284 of the same Code. In the case before us, both elements are completely lacking.
Respondents were dismissed without any just or authorized cause and without being given the
opportunity to be heard and defend themselves. The law mandates that the burden of proving the
validity of the termination of employment rests with the employer. Failure to discharge this
evidentiary burden would necessarily mean that the dismissal was not justified and, therefore,

291
illegal. Unsubstantiated suspicions, accusations, and conclusions of employers do not provide for
legal justification for dismissing employees. In case of doubt, such cases should be resolved in favor
of labor, pursuant to the social justice policy of labor laws and the Constitution.

2. The petitioners are guilty of unfair labor practice.

Anent the unfair labor practice, Article 248 (a) of the Labor Code considers it an unfair labor
practice when an employer interferes, restrains or coerces employees in the exercise of their right
to self-organization or the right to form an association. In order to show that the employer
committed unfair labor practice under the Labor Code, substantial evidence is required to support
the claim. Substantial evidence has been defined as such relevant evidence as a reasonable mind
might accept as adequate to support a conclusion. In the case at bar, respondents were indeed
unceremoniously dismissed from work by reason of their intent to form and organize a union. As
found by the LA:

The immediate impulse of respondents (petitioners herein), as in the case at bar, was to terminate
the organizers. Respondents (petitioners herein) have to cripple the union at sight, to frustrate
attempts of employees from joining or supporting it, preventing them, at all cost and to frustrate
the employees’ bid to exercise their right to self-organization. x x x

G.R. No. 194709 July 31, 2013

MINETTE BAPTISTA, BANNIE EDSEL SAN MIGUEL, AND MA. FEDAYON, PETITIONERS,
VS.
ROSARIO VILLANUEVA, JANETTE ROLDAN, DANILO OLAYVAR, ONOFRE ESTRELLA, CATALINO
LEDDA, MANOLO GUBANGCO, GILBERT ORIBIANA, CONSTANCIO SANTIAGO, RUTH BAYQUEN,
RUBY CASTANEDA, ALFRED LANDAS, JR., ROSELYN GARCES, EUGENE CRUZ, MENANDRO
SAMSON, FEDERICO MUNOZ AND SALVADOR DIWA, RESPONDENTS.

The Facts
Petitioners were former union members of Radio Philippines Network Employees Union (RPNEU),
a legitimate labor organization and the sole and exclusive bargaining agent of the rank and file
employees of Radio Philippines Network (RPN), a government-sequestered corporation involved in
commercial radio and television broadcasting affairs, while the respondents were the union’s
elected officers and members.

On April 26, 2005, on suspicion of union mismanagement, petitioners, together with some other
union members, filed a complaint for impeachment of their union president, Reynato Siozon, before
the executive board of RPN, which was eventually abandoned. They later re-lodged the
impeachment complaint, this time, against all the union officers and members of RPNEU before the

292
Department of Labor and Employment (DOLE). They likewise filed various petitions for audit
covering the period from 2000 to 2004.5
Thereafter, two (2) written complaints, dated May 26, 2005 and May 27, 2005, were filed against
petitioners and several others for alleged violation of the union’s Constitution and By-
Laws.6 Months later, on September 19, 2005, a different group of union members filed a third
complaint against petitioners and 12 others,7 before the Chairman of RPNEU’s Committee on
Grievance and Investigation (the Committee) citing as grounds the "commission of an act which
violates RPNEU Constitution and By-Laws, specifically, Article IX, Section 2.2 for joining or forming
a union outside the sixty (60) days period and Article IX, Section 2.5 for urging or advocating that a
member start an action in any court of justice or external investigative body against the Union or its
officer without first exhausting all internal remedies open to him or available in accordance with
the CBL."8 These complaints were, later on, consolidated.

Through a Memorandum,13 dated December 27, 2005, petitioners were served an expulsion notice
from the union, which was set to take effect on December 29, 2005. On January 2, 2006, petitioners
with the 12 others wrote to RPNEU’s President and Board of Directors that their expulsion from the
union was an ultra vires act because the Committee failed to observe the basic elements of due
process because they were not given the chance to physically confront and examine their
complainants.

In a letter, dated January 24, 2006, RPNEU’s officers informed their company of the expulsion of
petitioners and the 12 others from the union and requested the management to serve them notices
of termination from employment in compliance with their CBA’s union security clause. 15 On
February 17, 2006, RPN HRD Manager, Lourdes Angeles, informed petitioners and the 12 others of
the termination of their employment effective March 20, 2006, enforcing Article II, Section 216 also
known as the union security clause of their current CBA.17

Aggrieved, petitioners filed three (3) separate complaints for ULP against the respondents, which
were later consolidated,18 questioning legality of their expulsion from the union and their
subsequent termination from employment.

In a decision,19 dated April 30, 2007, the Labor Arbiter (LA) ruled in favor of the petitioners and
adjudged the respondents guilty of ULP pursuant to Article 249 (a) and (b) of the Labor Code. The
LA clarified that only the union officers of RPNEU could be held responsible for ULP, so they
exonerated six (6) of the original defendants who were mere union members. The LA also ordered
the reinstatement of petitioners as bonafide members of RPNEU.
Undaunted, the respondents appealed the LA decision to the NLRC.

In its Decision,21 dated March 31, 2008, the NLRC vacated and set aside the LA decision and
dismissed the complaint for ULP for lack of merit. The NLRC found that petitioners filed a suit
calling for the impeachment of the officers and members of the Executive Board of RPNEU without
first resorting to internal remedies available under its own Constitution and By-Laws. The NLRC
likewise decreed that the LA’s order of reinstatement was improper because the legality of the

293
membership expulsion was not raised in the proceedings and, hence, beyond the jurisdiction of the
LA.

Petitioners filed for a motion for reconsideration, but the NLRC denied it in its Resolution,24 dated
May 30, 2008.

The CA, in its March 9, 2010 Decision, sustained the NLRC decision. The CA stated that the
termination of employment by virtue of a union security clause was recognized in our jurisdiction.
It explained that the said practice fortified the union and averted disunity in the bargaining unit
within the duration of the CBA. The CA declared that petitioners were accorded due process before
they were removed from office. In fact, petitioners were given the opportunity to explain their case
and they actually availed of said opportunity by submitting letters containing their arguments.2

Anent petitioners’ charge of ULP against respondents, the records are barren of proof to sustain
such charge. What remains apparent is that petitioners were expelled from the union due to their
violation of Section 2.5 of Article IX of the CBL which punishes the act of "urging or advocating that
a member start an action in any court of justice or external investigative body against the Union or
any of its officer, without first exhausting all internal remedies open to him or available in
accordance with the Constitution and By-Laws of Union." As petitioners’ expulsion was pursuant to
the union’s CBL, We absolve respondents of the charges of ULP absent any substantial evidence to
sustain it.

The importance of a union’s constitution and bylaws cannot be overemphasized. They embody a
covenant between a union and its members and constitute the fundamental law governing the
member’s rights and obligations. As such, the union’s constitution and bylaws should be upheld, as
long as they are not contrary to law, good morals or public policy

Petitioners submit that the respondents committed ULP under Article 289 (a) and (b) of the Labor
Code.29 They insist that they were denied substantive and procedural due process of law when they
were expelled from the RPNEU.

Ruling:
The petition is bereft of merit.
Article 247. Concept of unfair labor practice and procedure for prosecution thereof.––Unfair labor
practices violate the constitutional right of workers and employees to self-organization, are inimical
to the legitimate interests of both labor and management, including their right to bargain
collectively and otherwise deal with each other in an atmosphere of freedom and mutual respect,
disrupt industrial peace and hinder the promotion of healthy and stable labor-management
relations.

In essence, ULP relates to the commission of acts that transgress the workers’ right to organize. As
specified in Articles 248 and 249 of the Labor Code, the prohibited acts must necessarily relate to

294
the workers' right to self-organization and to the observance of a CBA.30 Absent the said vital
elements, the acts complained, although seemingly unjust, would not constitute ULP.31
In the case at bench, petitioners claim that the respondents, as union officers, are guilty of ULP for
violating paragraphs (a) and (b) of Article 249 of the Labor Code, to wit:

ART. 249. UNFAIR LABOR PRACTICES OF LABOR ORGANIZATIONS.- It shall be unfair labor practice
for a labor organization, its officers, agents or representatives:
(a) To restrain or coerce employees in the exercise of their rights to self-organization. However, a
labor organization shall have the right to prescribe its own rules with respect to the acquisition or
retention of membership:
(b) To cause or attempt to cause an employer to discriminate against an employee, including
discrimination against an employee with respect to whom membership in such organization has
been denied or to terminate an employee on any ground other than the usual terms and conditions
under which membership or continuation of membership is made available to other members;
Petitioners posit that the procedure that should have been followed by the respondents in resolving
the charges against them was Article XVII, Settlement of Internal Disputes of their Constitution and
By-Laws, specifically, Section 232 thereof, requiring members to put their grievance in writing to be
submitted to their union president, who shall strive to have the parties settle their differences
amicably. Petitioners maintain that any form of grievance would be referred only to the committee
upon failure of the parties to settle amicably.33

The Court is not persuaded.


Based on RPNEU’s Constitution and By-Laws, the charges against petitioners were not mere
internal squabbles, but violations that demand proper investigation because, if proven, would
constitute grounds for their expulsion from the union. As such, Article X, Investigation Procedures
and Appeal Process of RPNEU’s Constitution and By-Laws, which reads –

SECTION 1. Charge against any member or officer of the Union shall be submitted to the Board of
Directors (BOD) in writing, which shall refer the same, if necessary, to the committee on Grievance
and Investigation. The Committee shall hear any charge and subsequently, forward its finding and
recommendation to the BOD. The BOD has the power to approve or nullify the recommendation of
the Committee on Grievance and Investigation based on the merit of the appeal.
was correctly applied under the circumstances.

Besides, any supposed procedural flaw in the proceedings before the Committee was deemed cured
when petitioners were given the opportunity to be heard. Due process, as a constitutional precept,
is satisfied when a person was notified of the charge against him and was given an opportunity to
explain or defend himself. In administrative proceedings, the filing of charges and giving reasonable
opportunity for the person so charged to answer the accusations against him constitute the
minimum requirements of due process.34 The essence of due process is simply to be heard, or as
applied to administrative proceedings, an opportunity to explain one’s side, or an opportunity to
seek a reconsideration of the action or ruling complained of.35 It cannot be denied that petitioners

295
were properly notified of the charges filed against them and were equally afforded the opportunity
to present their side.

Next, petitioners point out that they were not given the opportunity to personally face and confront
their accusers, which were violative of their right to examine the complainants and the supposed
charges against them.36

Petitioners’ contention is without merit. Mere absence of a one-onone confrontation between the
petitioners and their complainants does not automatically affect the validity of the proceedings
before the Committee. Not all cases necessitate a trial-type hearing.37 As in this case, what is
indispensable is that a party be given the right to explain one’s side, which was adequately afforded
to the petitioners.

It is well-settled that workers’ and employers’ organizations shall have the right to draw up their
constitutions and rules to elect their representatives in full freedom, to organize their
administration and activities and to formulate their programs.38 In this case, RPNEU’s Constitution
and By-Laws expressly mandate that before a party is allowed to seek the intervention of the court,
it is a pre-condition that he should have availed of all the internal remedies within the organization.
Petitioners were found to have violated the provisions of the union’s Constitution and By-Laws
when they filed petitions for impeachment against their union officers and for audit before the
DOLE without first exhausting all internal remedies available within their organization. This act is a
ground for expulsion from union membership. Thus, petitioners’ expulsion from the union was not
a deliberate attempt to curtail or restrict their right to organize, but was triggered by the
commission of an act, expressly sanctioned by Section 2.5 of Article IX of the union’s Constitution
and By-Laws.1âwphi1

For a charge of ULP against a labor organization to prosper, the onus probandi rests upon the party
alleging it to prove or substantiate such claims by the requisite quantum of evidence.39 In labor
cases, as in other administrative proceedings, substantial evidence or such relevant evidence as a
reasonable mind might accept as sufficient to support a conclusion is required.40 Moreover, it is
indubitable that all the prohibited acts constituting unfair labor practice should materially relate to
the workers' right to self-organization.41

Unfortunately, petitioners failed to discharge the burden required to prove the charge of ULP
against the respondents. Aside from their self-serving allegations, petitioners were not able to
establish how they were restrained or coerced by their union in a way that curtailed their right to
self-organization. The records likewise failed to sufficiently show that the respondents unduly
persuaded management into discriminating against petitioners. other than to bring to its attention
their expulsion from the union, which in turn, resulted in the implementation of their CBA' s union
security clause. As earlier stated, petitioners had the burden of adducing substantial evidence to
support its allegations of ULP,42 which burden they failed to discharge. In fact, both the NLRC and
the CA found that petitioners were unable to prove their charge of ULP against the respondents.

296
25. OTHER IMPORTANT LABOR PROVISIONS

A. CONTRACTING ARRANGEMENT

G.R. No. 182915 December 12, 2011

MARIALY O. SY, ET AL
VS.
FAIRLAND KNITCRAFT CO., INC., RESPONDENT.

G.R. NO. 189658


SUSAN T. DE LEON, PETITIONER,
VS.
FAIRLAND KNITCRAFT CO., INC., ET AL, RESPONDENTS.

Fairland is a domestic corporation engaged in garments business, while Susan de Leon (Susan) is
the owner/proprietress of Weesan Garments (Weesan).
On the other hand, the complaining workers (the workers) are sewers, trimmers, helpers, a guard
and a secretary who were hired by Weesan
On December 23, 2002, workers Marialy O. Sy, Vivencia Penullar, Aurora Aguinaldo, Gina Aniano,
Gemma dela Peña and Efremia Matias filed with the Arbitration Branch of the NLRC a
Complaint9 for underpayment and/or non-payment of wages, overtime pay, premium pay for
holidays, 13th month pay and other monetary benefits against Susan/Weesan. In January 2003, the
rest of the aforementioned workers also filed similar complaints. Eventually all the cases were
consolidated as they involved the same causes of action.
On February 5, 2003, Weesan filed before the Department of Labor and Employment-National
Capital Region (DOLE-NCR) a report on its temporary closure for a period of not less than six
months. As the workers were not anymore allowed to work on that same day, they filed on
February 18, 2003 an Amended Complaint,10 and on March 13, 2003, another pleading entitled
Amended Complaints and Position Paper for Complainants,11 to include the charge of illegal
dismissal and impleaded Fairland and its manager, Debbie Manduabas (Debbie), as additional
respondents.

Ruling of the Labor Arbiter


Dismissing the complaint for lack of merit; and ordering the respondents to pay each
complainant P5,000.00 by way of financial assistance.

Ruling of the National Labor Relations Commission


The workers filed their appeal which was granted by the NLRC.
Ruling of the Court of Appeals in CA-G.R. SP No. 93204

297
On July 25, 2007, the CA’s First Division denied Fairland’s petition.28 It affirmed the NLRC’s ruling
that the workers were illegally dismissed and that Weesan and Fairland are solidarily liable to them
as labor-only contractor and principal, respectively.
On May 9, 2008, the CA’s Special Ninth Division reversed33 the First Division’s ruling. It held that the
labor tribunals did not acquire jurisdiction over the person of Fairland, and even assuming they did,
Fairland is not liable to the workers since Weesan is not a mere labor-only contractor but a bona
fide independent contractor. The Special Ninth Division thus annulled and set aside the assailed
NLRC Decision and Resolution insofar as Fairland is concerned and excluded the latter therefrom.
Aggrieved, the workers filed before us their Petition for Review on Certiorari docketed as G.R. No.
182915.

Ruling of the Court of Appeals in CA-G.R. SP No. 93860


With regard to Susan’s petition, the CA Special Ninth Division issued on May 11, 2006 a
Resolution35 temporarily restraining the NLRC from enforcing its assailed November 30, 2004
Decision and thereafter the CA Special Eighth Division issued a writ of preliminary prohibitory
injunction.36 On July 20, 2009, the Special Former Special Eighth Division of the CA resolved the
case through a Decision.
Hence, she filed before this Court a Petition for Review on Certiorari docketed as G.R. No. 189658
which was denied in this Court’s December 16, 2009 Resolution41 on technicality and for failure to
sufficiently show any reversible error in the assailed judgment.
Susan and Fairland filed their respective Motions for Reconsideration.42 But before said motions
could be resolved, the Court ordered the consolidation of Susan’s petition with that of the workers.

Issues
In G.R. No. 189658, Susan imputes upon the CA the following errors:
I. The Court of Appeals erred in finding that petitioner is a labor-only contractor acting as an agent
of respondent Fairland.
II. The Court of Appeals erred in finding that the individual private respondents were illegally
dismissed.
III. The Court of Appeals erred in not resolving the issue raised by petitioner in her reply DATED
JULY 8, 2006 regarding the propriety of the appeal taken by private respondent Richon Cainoy
Aparre who was already dead prior to the filing of the memorandum of appeal before the NLRC.48

Susan’s Arguments
Susan insists that the CA erred in ruling that Weesan is a labor-only contractor based on the finding
that its workplace is owned by Fairland. She maintains that the place is owned by De Luxe Shirt
Factory, Inc. (De Luxe) and not by Fairland as shown by the Contracts of Lease between Weesan
and De Luxe.

Susan also avers that the CA erred in ruling that Weesan was guilty of illegal dismissal. She
maintains that the termination of the workers was due to financial losses suffered by Weesan as
shown by various documents submitted by the latter to the tribunals below. In fact, Weesan

298
submitted its Establishment Termination Report with the DOLE-NCR and same was duly received
by the latter.

Lastly, Susan argues that the appeal of one of the workers, Richon Cainoy Aparre (Richon), should
not have been given due course because in the Notice of Appeal with Appeal Memorandum filed
with the NLRC, a certain Luzvilla A. Rayon (Luzvilla), whose identity was never established, signed
for and on his behalf. However, there is no information submitted before the NLRC that Richon is
already dead, and in any event, no proper substitution was ever made.

The Workers’ Arguments


The workers claim that Weesan is a labor-only contractor because it does not have substantial
capital or investment in the form of tools, equipment, machineries, and work premises, among
others, and that the workers it recruited are performing activities which are directly related to the
garments business of Fairland. Hence, Weesan should be considered as a mere agent of Fairland,
who shall be responsible to the workers as if they were directly employed by it (Fairland).49
The workers also allege that the temporary suspension of operations of Weesan was motivated not
by a desire to prevent further losses, but to discourage the workers from ventilating their claims for
non-payment/underpayment of wages and benefits. The fact that Weesan was experiencing serious
business losses was not sufficiently established and therefore the termination of the workers due to
alleged business losses is invalid.50

Fairland’s Arguments
Fairland maintains that it was never served with summons to appear in the proceedings before the
Labor Arbiter nor furnished copies of the Labor Arbiter’s Decision and Resolution on the workers’
complaints for illegal dismissal; that it never voluntarily appeared before the labor tribunals
through Atty. Geronimo;51 that it is a separate and distinct business entity from Weesan; that
Weesan is a legitimate job contractor, hence, the workers were actually its (Weesan’s) employees;
and that, consequently, the workers have no cause of action against Fairland.52
At any rate, assuming that the workers have a cause of action against Fairland, their claims are
already barred by prescription. Of the 34 individual complainants (the workers), only six were
employees of Weesan during the period of its contractual relationship with Fairland in 1996 and
1997. They were Marialy Sy, Olivia Abuan, Amelia Pescadero, Regina Relox, Hermina Hernandez
and Trinidad Relox. These workers filed their complaints in December 2002 and January 2003 or
more than four years from the expiration of Weesan’s contractual arrangement with Fairland in
1997. Article 291 of the Labor Code provides that all money claims arising from employer-
employee relationship shall be filed within three years from the time the cause of action accrued;
otherwise, they shall be forever barred. Illegal dismissal prescribes in four years and damages due
to separation from employment for alleged unjustifiable causes injuring a plaintiff’s right must
likewise be brought within four years under the Civil Code. Clearly, the claims of said six employees
are already barred by prescription.53

In G.R. No. 182915, the workers advance the following issues:


I.

299
Whether x x x the National Labor Relations Commission acquired jurisdiction over the [person of
the] respondent[;]
II.
Whether x x x the decision of the National Labor Relations Commission became final and
executory[; and]
III.
Whether x x x respondent is solidarily liable with WEESAN GARMENT/ SUSAN DE LEON[.]54

The Workers’ Arguments


The workers contend that the Labor Arbiter and the NLRC properly acquired jurisdiction over the
person of Fairland because the latter voluntarily appeared and actively participated in the
proceedings below when Atty. Geronimo submitted on its behalf a Position Paper verified by its
manager, Debbie. As manager, Debbie knew of all the material and significant events which
transpired in Fairland since she had constant contact with the people in the day-to-day operations
of the company. Thus, the workers maintain that the Labor Arbiter and the NLRC acquired
jurisdiction over the person of Fairland and the Decisions rendered by the said tribunals are valid
and binding upon it.
Lastly, the workers aver that Fairland is solidarily liable with Susan/ Weesan because it was shown
that the latter was indeed the sewing arm of the former and is a mere "labor-only contractor".

Fairland’s Arguments
In gist, Fairland contests the labor tribunals’ acquisition of jurisdiction over its person either
through service of summons or voluntary appearance. It denies that it engaged the services of Atty.
Geronimo and asserts that it has its own legal counsel, Atty. Tecson, who would have represented it
had it known of the pendency of the complaints against Fairland.
Fairland likewise emphasizes that when it filed its Motion for Reconsideration with the NLRC, it
made an express reservation that the same was without prejudice to its right to question the
jurisdiction over its person and the binding effect of the assailed decision. In the absence, therefore,
of a valid service of summons or voluntary appearance, the proceedings conducted and the
judgment rendered by the labor tribunals are null and void as against it. Hence, Fairland cannot be
held solidarily liable with Susan/Weesan.

Our Ruling
We grant the workers’ petition (G.R. No. 182915) but deny the petition of Susan (G.R. No. 189658).
G.R. No. 189658
Susan/Weesan is a mere labor-only contractor.
"There is labor-only contracting when the contractor or subcontractor merely recruits, supplies or
places workers to perform a job, work or service for a principal. In labor-only contracting, the
following elements are present:
(a) The person supplying workers to an employer does not have substantial capital or investment
in the form of tools, equipment, machineries, work premises, among others; and
(b) The workers recruited and placed by such person are performing activities which are directly
related to the principal business of the employer."55

300
Here, there is no question that the workers, majority of whom are sewers, were recruited by
Susan/Weesan and that they performed activities which are directly related to Fairland’s principal
business of garments. What must be determined is whether Susan/Weesan has substantial capital
or investment in the form of tools, equipment, machineries, work premises, among others.
We have examined the records but found nothing therein to show that Weesan has investment in
the form of tools, equipment or machineries. The records show that Fairland has to furnish Weesan
with sewing machines for it to be able to provide the sewing needs of the former. 56 Also, save for
the Balance Sheets57 purportedly submitted by Weesan to the Bureau of Internal Revenue (BIR)
indicating its fixed assets (factory equipment) in the amount ofP243,000.00, Weesan was unable to
show that apart from the borrowed sewing machines, it owned and possessed any other tools,
equipment, and machineries necessary to its being a contractor or sub-contractor for garments.
Neither was Weesan able to prove that it has substantial capital for its business.
Court finds that Susan’s effort to negate Fairland’s ownership of the work premises is futile. The
logical conclusion now is that Weesan does not have its own workplace and is only utilizing the
workplace of Fairland to whom it supplied workers for its garment business.
Suffice it to say that "[t]he presumption is that a contractor is a labor-only contractor unless such
contractor overcomes the burden of proving that it has substantial capital, investment, tools and
the like."69 As Susan/Weesan was not able to adduce evidence that Weesan had any substantial
capital, investment or assets to perform the work contracted for, the presumption that Weesan is a
labor-only contractor stands.70
The National Labor Relations Commission and the Court of Appeals did not err in their findings of
illegal dismissal.
To negate illegal dismissal, Susan relies on the due closure of Weesan pursuant to the
Establishment Termination Report it submitted to the DOLE-NCR.
Indeed, Article 28371 of the Labor Code allows as a mode of termination of employment the closure
or termination of business. "Closure or cessation of business is the complete or partial cessation of
the operations and/or shut-down of the establishment of the employer. It is carried out to either
stave off the financial ruin or promote the business interest of the employer." 72 "The decision to
close business [or to temporarily suspend operation] is a management prerogative exclusive to the
employer, the exercise of which no court or tribunal can meddle with, except only when the
employer fails to prove compliance with the requirements of Art. 283, to wit: a) that the
closure/cessation of business is bona fide, i.e., its purpose is to advance the interest of the employer
and not to defeat or circumvent the rights of employees under the law or a valid agreement; b) that
written notice was served on the employees and the DOLE at least one month before the intended
date of closure or cessation of business; and c) in case of closure/cessation of business not due to
financial losses, that the employees affected have been given separation pay equivalent to ½ month
pay for every year of service or one month pay, whichever is higher."73
Here, Weesan filed its Establishment Termination Report74 allegedly due to serious business losses
and other economic reasons. However, we are mindful of the doubtful character of Weesan’s
application for closure given the circumstances surrounding the same.
First, workers Marialy Sy, Vivencia Penullar, Aurora Aguinaldo, Gina Aniano, Gemma Dela Peña and
Efremia Matias filed before the Labor Arbiter their complaint for underpayment of salary, non-
payment of benefits, damages and attorney’s fees against Weesan on December 23,

301
2002.75 Summons76 was accordingly issued and same was received by Susan on January 15,
2003.77 Meanwhile, other workers followed suit and filed their respective complaints on January 2,
6, 17 and 28, 2003.78 Shortly thereafter or merely eight days after the filing of the last complaint,
Weesan filed with the DOLE-NCR its Establishment Termination Report.
Second, the Income Tax Returns79 for the years 2000, 2001 and 2002 attached to the Establishment
Termination Report, although bearing the stamped receipt of the Revenue District Office where
they were purportedly filed, contain no signature or initials of the receiving officer. The same holds
true with Weesan’s audited financial statements.80 This engenders doubt as to whether these
documents were indeed filed with the proper authorities.
Third, there was no showing that Weesan served upon the workers written notice at least one
month before the intended date of closure of business, as required under Art. 283 of the Labor
Code. In fact, the workers alleged that when Weesan filed its Establishment Termination Report on
February 5, 2003, it already closed the work premises and did not anymore allow them to report
for work. This is the reason why the workers on February 18, 2003 amended their complaint to
include the charge of illegal dismissal.81
It bears stressing that "[t]he burden of proving that x x x a temporary suspension is bona fide falls
upon the employer."82 Clearly here, Susan/Weesan was not able to discharge this burden. The
documents Weesan submitted to support its claim of severe business losses cannot be considered
as proof of financial crisis to justify the temporary suspension of its operations since they clearly
appear to have not been duly filed with the BIR. Weesan failed to satisfactorily explain why the
Income Tax Returns and financial statements it submitted do not bear the signature of the receiving
officers. Also hard to ignore is the absence of the mandatory 30-day prior notice to the workers.
Hence, the Court finds that Susan failed to prove that the suspension of operations of Weesan
was bona fide and that it complied with the mandatory requirement of notice under the law. Susan
likewise failed to discharge her burden of proving that the termination of the workers was for a
lawful cause. Therefore, the NLRC and the CA, in CA-G.R. SP No. 93860, did not err in their findings
that the workers were illegally dismissed by Susan/Weesan.
Fairland’s claim of prescription deserves scant consideration.
Fairland asserts that assuming that the workers have valid claims against it, same only pertain to
six out of the 34 workers-complainants. According to Fairland, these six workers were the only
ones who were in the employ of Weesan at the time Fairland and Weesan had existing contractual
relationship in 1996 to 1997. But then, Fairland contends that the claims of these six workers have
already been barred by prescription as they filed their complaint more than four years from the
expiration of the alleged contractual relationship in 1997. However, the Court notes that the
records are bereft of anything that provides for such alleged contractual relationship and the period
covered by it. Absent anything to support Fairland’s claim, same deserves scant consideration.
Interestingly, we noticed Fairland’s letter86 dated January 31, 2003 informing Weesan that it would
temporarily not be availing of the latter’s sewing services and at the same time requesting for the
return of the sewing machines it lent to Weesan. Assuming said letter to be true, why was Fairland
terminating Weesan’s services only on January 31, 2003 when it is now claiming that its contractual
relationship with the latter only lasted until 1997? Thus, we find the contentions rather abstruse.

G.R. No. 182915

302
"It is basic that the Labor Arbiter cannot acquire jurisdiction over the person of the respondent
without the latter being served with summons."87 However, "if there is no valid service of summons,
the court can still acquire jurisdiction over the person of the defendant by virtue of the latter’s
voluntary appearance."88
Although not served with summons, jurisdiction over Fairland and Debbie was acquired through their
voluntary appearance.
It can be recalled that the workers’ original complaints for non-payment/ underpayment of wages
and benefits were only against Susan/Weesan. For these complaints, the Labor Arbiter issued
summons89 to Susan/Weesan which was received by the latter on January 15, 2003. 90 The workers
thereafter amended their then already consolidated complaints to include illegal dismissal as an
additional cause of action as well as Fairland and Debbie as additional respondents. We have,
however, scanned the records but found nothing to indicate that summons with respect to the said
amended complaints was ever served upon Weesan, Susan, or Fairland. True to their claim,
Fairland and Debbie were indeed never summoned by the Labor Arbiter.
The crucial question now is: Did Fairland and Debbie voluntarily appear before the Labor Arbiter as
to submit themselves to its jurisdiction?
Fairland argued before the CA that it did not engage Atty. Geronimo as its counsel.
From the records, it appears that Atty. Geronimo first entered his appearance on behalf of
Susan/Weesan in the hearing held on April 3, 2003.93 Being then newly hired, he requested for an
extension of time within which to file a position paper for said respondents. On the next scheduled
hearing on April 28, 2003, Atty. Geronimo again asked for another extension to file a position paper
for all the respondents considering that he likewise entered his appearance for
Fairland.94 Thereafter, said counsel filed pleadings such as Respondents’ Position Paper95 and
Respondents’ Consolidated Reply96 on behalf of all the respondents namely, Susan/Weesan,
Fairland and Debbie. The fact that Atty. Geronimo entered his appearance for Fairland and Debbie
and that he actively defended them before the Labor Arbiter raised the presumption that he is
authorized to appear for them. As held in Santos, it is unlikely that Atty. Geronimo would have been
so irresponsible as to represent Fairland and Debbie if he were not in fact authorized. As an officer
of the Court, Atty. Geronimo is presumed to have acted with due propriety. Moreover, "[i]t strains
credulity that a counsel who has no personal interest in the case would fight for and defend a case
with persistence and vigor if he has not been authorized or employed by the party concerned."97
The presumption of authority of counsel to appear on behalf of a client is found both in the Rules of
Court and in the New Rules of Procedure of the NLRC.98
Sec. 21, Rule 138 of the Rules of Court provides:
Sec. 21. Authority of attorney to appear – An attorney is presumed to be properly authorized to
represent any cause in which he appears, and no written power of attorney is required to authorize
him to appear in court for his client, but the presiding judge may, on motion of either party and
reasonable grounds therefor being shown, require any attorney who assumes the right to appear in
a case to produce or prove the authority under which he appears, and to disclose whenever
pertinent to any issue, the name of the person who employed him, and may thereupon make such
order as justice requires. An attorney willfully appearing in court for a person without being
employed, unless by leave of the court, may be punished for contempt as an officer of the court who
has misbehaved in his official transactions.

303
On the other hand, Sec. 8, Rule III of the New Rules of Procedure of the NLRC, 99 which is the rules
prevailing at that time, states in part:
SECTION 8. APPEARANCES. - An attorney appearing for a party is presumed to be properly
authorized for that purpose. However, he shall be required to indicate in his pleadings his PTR and
IBP numbers for the current year.
Between the two provisions providing for such authority of counsel to appear, the Labor Arbiter is
primarily bound by the latter one, the NLRC Rules of Procedure being specifically applicable to
labor cases. As Atty. Geronimo consistently indicated his PTR and IBP numbers in the pleadings he
filed, there is no reason for the Labor Arbiter not to extend to Atty. Geronimo the presumption that
he is authorized to represent Fairland.
Even if we are to apply Sec. 21, Rule 138 of the Rules of Court, the Labor Arbiter cannot be expected
to require Atty. Geronimo to prove his authority under said provision since there was no motion to
that effect from either party showing reasonable grounds therefor. Moreover, the fact that Debbie
signed the verification attached to the position paper filed by Atty. Geronimo, without a secretary’s
certificate or board resolution attached thereto, is not sufficient reason for the Labor Arbiter to be
on his guard and require Atty. Geronimo to prove his authority. Debbie, as General Manager of
Fairland is one of the officials of the company who can sign the verification without need of a board
resolution because as such, she is in a position to verify the truthfulness and correctness of the
allegations in the petition.100
Although we note that Fairland filed a disbarment case against Atty. Geronimo due to the former’s
claim of unauthorized appearance, we hold that same is not sufficient to overcome the presumption
of authority. Such mere filing is not proof of Atty. Geronimo’s alleged unauthorized appearance.
Suffice it to say that an attorney’s presumption of authority is a strong one.101 "A mere denial by a
party that he authorized an attorney to appear for him, in the absence of a compelling reason, is
insufficient to overcome the presumption, especially when the denial comes after the rendition of
an adverse judgment,"102 such as in the present case.

Fairland is Weesan’s principal.


In addition to our discussion in G.R. No. 189658 with respect to the finding that Susan/Weesan is a
mere labor-only contractor which we find to be likewise significant here, a careful examination of
the records reveals other telling facts that Fairland is Susan/Weesan’s principal, to wit: (1) aside
from sewing machines, Fairland also lent Weesan other equipment such as fire extinguishers, office
tables and chairs, and plastic chairs;109 (2) no proof evidencing the contractual arrangement
between Weesan and Fairland was ever submitted by Fairland; (3) while both Weesan and Fairland
assert that the former had other clients aside from the latter, no proof of Weesan’s contractual
relationship with its other alleged client is extant on the records; and (4) there is no showing that
any of the workers were assigned to other clients aside from Fairland. Moreover, as found by the
NLRC and affirmed by both the Special Former Special Eighth Division in CA-G.R. SP No. 93860 and
the First Division in CA-G.R. SP No. 93204, the activities, the manner of work and the movement of
the workers were subject to Fairland’s control. It bears emphasizing that "factual findings of quasi-
judicial agencies like the NLRC, when affirmed by the Court of Appeals, as in the present case, are
conclusive upon the parties and binding on this Court."110

304
Viewed in its entirety, we thus declare that Fairland is the principal of the labor-only contractor,
Weesan.
Fairland, therefore, as the principal employer, is solidarily liable with Susan/Weesan, the labor-only
contractor, for the rightful claims of the employees. Under this set-up, Susan/Weesan, as the "labor-
only" contractor, is deemed an agent of the principal, Fairland, and the law makes the principal
responsible to the employees of the "labor-only" contractor as if the principal itself directly hired or
employed the employees.111

POLYFOAM-RGC INTERNATIONAL, CORPORATION AND PRECILLA A. GRAMAJE,


VS
EDGARDO CONCEPCION,

FACTS:
On February 8, 2000, respondent filed a Complaint for illegal dismissal, non-payment of
wages, premium pay for rest day, separation pay, service incentive leave pay, 13th month pay,
damages, and attorney’s fees against Polyfoam and Ms. Natividad Cheng (Cheng). Respondent
alleged that he was hired by Polyfoam as an “all-around” factory worker and served as such for
almost six years. On January 14, 2000, he allegedly discovered that his time card was not in the rack
and was later informed by the security guard that he could no longer punch his time card. When he
protested to his supervisor, the latter allegedly told him that the management decided to dismiss
him due to an infraction of a company rule. Cheng, the company’s manager, also refused to face
him. Respondent’s counsel later wrote a letterto Polyfoam’s manager requesting that respondent
be re-admitted to work, but the request remained unheeded prompting the latter to file the
complaint for illegal dismissal.
On April 28, 2000, Gramaje Employement Services filed a Motion for Intervention claiming to
be the real employer of respondent. On the other hand, Polyfoam and Cheng filed a Motion to
Dismiss on the grounds that the NLRC has no jurisdiction over the case, because of the absence of
employer-employee relationship between Polyfoam and respondent and that the money claims had
already prescribed.
Labor Arbiter ruled that respondent was illegally dismissed but NLRC exonerated Polyfoam
abd ordered Gramaje to pay the separation pay , damages and and backwages.

ISSUES: 1. Wherther or Not Gramaje is a Labor-Only Contractor


2. WON there is EE-ER relationship between Polyfoam and Edgardo
Concepcion
3. WON Edgardo was Illegally Dismissed

SC HELD:
1. Gramaje is not an independent job contractor, but a “labor-only” contractor.
First, Gramaje has no substantial capital or investment. The presumption is that a contractor is a
labor-only contractor unless he overcomes the burden of proving that it has substantial capital,
investment, tools, and the like. The employee should not be expected to prove the negative fact that
the contractor does not have substantial capital, investment and tools to engage in job-contracting

305
Second, Gramaje did not carry on an independent business or undertake the performance of its
service contract according to its own manner and method, free from the control and supervision of
its principal, Polyfoam, its apparent role having been merely to recruit persons to work for
Polyfoam. It is undisputed that respondent had performed his task of packing Polyfoam’s foam
products in Polyfoam’s premises. Edgardo was even He was even furnished a copy of Polyfoam’s
“Mga Alituntunin at Karampatang Parusa,” which embodied Polyfoam’s rules on attendance, the
manner of performing the employee’s duties, ethical standards, cleanliness, health, safety, peace
and order. These rules carried with them the corresponding penalties in case of violation.
2. A finding that a contractor is a “labor-only” contractor, as opposed to permissible job
contracting, is equivalent to declaring that there is an employer-employee relationship between the
principal and the employees of the supposed contractor, and the “labor-only” contractor is
considered as a mere agent of the principal, the real employer. In this case, Polyfoam is the
principal employer and Gramaje is the labor-only contractor. Polyfoam and Gramaje are,
therefore, solidarily liable for the rightful claims of respondent

3. Respondent was illegaly dismissed. Petitioners failed to show any valid or authorized cause
under the Labor Code which allowed it to terminate the services of respondent. Neither was it
shown that respondent was given ample opportunity to contest the legality of his dismissal. No
notice of termination was given to him. Clearly, respondent was not afforded due process. Having
failed to establish compliance with the requirements of termination of employment under the
Labor Code, the dismissal of respondent was tainted with illegality.

GR. NO. 178809 OCTOBER 10, 2012

SUPERIOR PACKAGING CORP.


VS
BALAGSAY

FACTS;
The petitioner engaged the services of Lancer to provide reliever services to its business, which
involves the manufacture and sale of commercial and industrial corrugated boxes. According to
petitioner, the respondents were engaged for four (4) months from February to June 1998 and their
tasks included loading, unloading and segregation of corrugated boxes.
Pursuant to a complaint filed by the respondents against the petitioner and its President, Cesar Luz
(Luz), for underpayment of wages, non-payment of premium pay for worked rest, overtime pay and
non-payment of salary, the Department of Labor and Employment (DOLE) conducted an inspection
of the petitioners premises and found several violations, to wit:
(1) non-presentation of payrolls and daily time records;
(2) non-submission of annual report of safety organization;
(3) medical and accident/illness reports;
(4) non-registration of establishment under Rule 1020 of Occupational and Health Standards;
and
(5) no trained first aide

306
Due to the petitioners failure to appear in the summary investigations conducted by the DOLE, an
Order2was issued finding in favor of the respondents and adopting the computation of the claims
submitted. Petitioner and Luz were ordered, among others, to pay respondents their total claims in
the amount of (P 840,463.38)./

They filed a motion for reconsideration on the ground that respondents are not its employees but of
Lancer and that they pay Lancer in lump sum for the services rendered.

ISSUE:
The main issue in this case is whether Superior Packaging Corporation (petitioner) may be held
solidarily liable with Lancer Staffing & Services Network, Inc. (Lancer) for respondents unpaid money
claims.

Seecondary Issues: WON DOLE has power to determine EE-ER RElationship

SC HELD:
It was the consistent conclusion of the DOLE and the CA that Lancer was not an independent
contractor but was engaged in "labor-only contracting"; hence, the petitioner was considered an
indirect employer of respondents and liable to the latter for their unpaid money claims.

At the time of the respondents employment in 1998, the applicable regulation was DOLE
Department Order No. 10, Series of 1997.Under said Department Order, labor-only contracting was
defined as follows:

Sec. 9. Labor-only contracting. (a) Any person who undertakes to supply workers to an employer
shall be deemed to be engaged in labor-only contracting where such person:

(1) Does not have substantial capital or investment in the form of tools, equipment,
machineries, work premises and other materials; and

(2) The workers recruited and placed by such persons are performing activities which are
directly related to the principal business or operations of the employer in which workers are
habitually employed.

Labor-only contracting is prohibited and the person acting as contractor shall be considered merely
as an agent or intermediary of the employer who shall be responsible to the workers in the same
manner and extent as if the latter were directly employed by him.

According to the CA, the totality of the facts and surrounding circumstances of this case point to
such conclusion. The Court agrees.

307
A finding that a contractor is a "labor-only" contractor is equivalent to declaring that there is an
employer-employee relationship between the principal and the employees of the supposed
contractor, and the "labor only" contractor is considered as a mere agent of the principal, the real
employer.The former becomes solidarily liable for all the rightful claims of the employees.

The petitioner therefore, being the principal employer and Lancer, being the labor-only
contractor, are solidarily liable for respondents unpaid money claims.

RE: EE-ER Relationship


The existence of an employer-employee relationship is ultimately a question of fact. The determination
made in this case by the DOLE, albeit provisional, and as affirmed by the Secretary of DOLE and the CA
is beyond the ambit of a petition for review on certiorari.
In Peoples Broadcasting (Bombo Radyo Phils., Inc.) v. Secretary of the Department of Labor and
Employment, the Court stated that it can be assumed that the DOLE in the exercise of its visitorial and
enforcement power somehow has to make a determination of the existence of an employer-employee
relationship. Such determination, however, is merely preliminary, incidental and collateral to the
DOLEs primary function of enforcing labor standards provisions.

[GR Nos. 184903-04, October 10, 2012]

DIGITAL TELECOMMUNICATIONS PHILIPPINES, INC., PETITIONER,


VS.
DIGITEL EMPLOYEES UNION (DEU), ARCEO RAFAEL A. ESPLANA, ALAN D. LICANDO, FELICITO
C. ROMERO, JR., ARNOLD D. GONZALES, REYNEL FRANCISCO B. GARCIA, ZOSIMO B. PERALTA,
REGINO T. UNIDAD AND JIM L. JAVIER, RESPONDENTS.

Facts:
Digitel Service, Inc. (Digiserv), a non-profit enterprise engaged in call center servicing, filed
with the Department of Labor and Employment (DOLE) an Establishment Termination Report
stating that it will cease its business operation. The closure affected at least 100 employees, 42 of
whom are members of the herein respondent Union. The respondent union is the collective
bargaining unit of the company with Digitel whom Digiserve provided with manpower for its
customer service.

Alleging that the affected employees are its members and in reaction to Digiserv's action,
Esplana and his group filed another Notice of Strike for union busting, illegal lock-out, and violation
of the assumption order.
On 23 May 2005, the Secretary of Labor ordered the second notice of strike subsumed by the
previous Assumption Order. 5
Meanwhile, on 14 March 2005, Digitel filed a petition with the Bureau of Labor Relations (BLR)
seeking cancellation of the Union's registration on the following grounds: 1) failure to file the
required reports from 1994-2004; 2) misrepresentation of its alleged officers; 3) membership of

308
the Union is composed of rank and file, supervisory and managerial employees; and 4) substantial
number of union members are not Digitel employees. 6
In a Decision dated 11 May 2005, the Regional Director of the DOLE dismissed the petition for
cancellation of union registration for lack of merit. The Regional Director ruled that it does not have
jurisdiction over the issue of non-compliance with the reportorial requirements. He also held that
Digitel failed to adduce substantial evidence to prove misrepresentation and the mixing of non-
Digitel employees with the Union. Finally, he declared that the inclusion of supervisory and
managerial employees with the rank and file employees is no longer a ground for cancellation of the
Union's certificate of registration.

Issue:
WON Digiserve is a labor-only contractor

Ruling:
YES. Labor-only contracting is expressly prohibited by our labor laws. Article 106 of the
Labor Code defines labor-only contracting as "supplying workers to an employer [who] does not
have substantial capital or investment in the form of tools, equipment, machineries, work premises,
among others, and the workers recruited and placed by such person are performing activities
which are directly related to the principal business of such employer."
Section 5, Rule VIII-A, Book III of the Omnibus Rules Implementing the Labor Code (Implementing
Rules), as amended by Department Order No. 18-02, expounds on the prohibition against labor-
only contracting, thus:
Section 5.Prohibition against labor-only contracting. — Labor-only contracting is hereby declared
prohibited. For this purpose, labor-only contracting shall refer to an arrangement where the
contractor or subcontractor merely recruits, supplies or places workers to perform a job, work or
service for a principal, and any of the following elements are present: TcEaAS
i)The contractor or subcontractor does not have substantial capital or investment which relates to
the job, work or service to be performed and the employees recruited, supplied or placed by such
contractor or subcontractor are performing activities which are directly related to the main
business of the principal; or
ii)The contractor does not exercise the right to control over the performance of the work of the
contractual employee.
The foregoing provisions shall be without prejudice to the application of Article 248 (c) of the Labor
Code, as amended.
xxx xxx xxx
The "right to control" shall refer to the right reserved to the person for whom, the services of the
contractual workers are performed, to determine not only the end to be achieved, but also the
manner and means to be used in reaching that end.
The law and its implementing rules allow contracting arrangements for the performance of specific
jobs, works or services. Indeed, it is management prerogative to farm out any of its activities,
regardless of whether such activity is peripheral or core in nature. However, in order for such
outsourcing to be valid, it must be made to an independent contractor because the current labor
rules expressly prohibit labor-only contracting. 18

309
After an exhaustive review of the records, there is no showing that first, Digiserv has substantial
investment in the form of capital, equipment or tools. Under the Implementing Rules, substantial
capital or investment refers to "capital stocks and subscribed capitalization in the case of
corporations, tools, equipment, implements, machineries and work premises, actually and directly
used by the contractor or subcontractor in the performance or completion of the job, work or
service contracted out." The NLRC, as echoed by the Court of Appeals, did not find substantial
Digiserv's authorized capital stock of One Million Pesos (P1,000,000.00). It pointed out that only
Two Hundred Fifty Thousand Pesos (P250,000.00) of the authorized capital stock had been
subscribed and only Sixty-Two Thousand Five Hundred Pesos (P62,500.00) had been paid up.
There was no increase in capitalization for the last ten (10) years. 19
Moreover, in the Amended Articles of Incorporation, as well as in the General Information Sheets
for the years 1994, 2001 and 2005, the primary purpose of Digiserv is to provide manpower
services. In PCI Automation Center, Inc. v. National Labor Relations Commission, 20 the Court made
the following distinction: "the legitimate job contractor provides services while the labor-only
contractor provides only manpower. The legitimate job contractor undertakes to perform a specific
job for the principal employer while the labor-only contractor merely provides the personnel to
work for the principal employer." The services provided by employees of Digiserv are directly
related to the business of Digitel, as rationalized by the NLRC in this wise: ASTcaE
It is undisputed that as early as March 1994, the affected employees, except for two, were already
performing their job as Traffic Operator which was later renamed as Customer Service
Representative (CSR). It is equally undisputed that all throughout their employment, their function
as CSR remains the same until they were terminated effective May 30, 2005. Their long period of
employment as such is an indication that their job is directly related to the main business of
DIGITEL which is telecommunication[s]. Because, if it was not, DIGITEL would not have allowed
them to render services as Customer Service Representative for such a long period of time. 21
Furthermore, Digiserv does not exercise control over the affected employees. The NLRC highlighted
the fact that Digiserv shared the same Human Resources, Accounting, Audit and Legal Departments
with Digitel which manifested that it was Digitel who exercised control over the performance of the
affected employees. The NLRC also relied on the letters of commendation, plaques of appreciation
and certification issued by Digitel to the Customer Service Representatives as evidence of control.
Considering that Digiserv has been found to be engaged in labor-only contracting, the dismissed
employees are deemed employees of Digitel.
Section 7 of the Implementing Rules holds that labor-only contracting would give rise to: (1) the
creation of an employer-employee relationship between the principal and the employees of the
contractor or sub-contractor; and (2) the solidary liability of the principal and the contractor to the
employees in the event of any violation of the Labor Code.
Accordingly, Digitel is considered the principal employer of respondent employees.

GR No. 182018, October 10, 2012

310
NORKIS TRADING CORPORATION, PETITIONER,
VS.
JOAQUIN BUENAVISTA, HENRY FABROA, RICARDO CAPE, BERTULDO TULOD, WILLY
DONDOYANO AND GLEN VILLARIASA, RESPONDENTS.

Facts:
The respondents were hired by Norkis Trading, a domestic corporation engaged in the
business of manufacturing and marketing of Yamaha motorcycles and multi-purpose vehicles.
Although they worked for Norkis Trading as skilled workers assigned in the operation of industrial
and welding machines owned and used by Norkis Trading for its business, they were not treated as
regular employees by Norkis Trading. Instead, they were regarded by Norkis Trading as members
of PASAKA, a cooperative organized under the Cooperative Code of the Philippines, and which was
deemed an independent contractor that merely deployed the respondents to render services for
Norkis Trading.
The respondents then prompted for the filing of the complaint for labor-only contracting
allegedly led to the suspension of the respondents' membership with PASAKA. On July 22, 1999,
they were served by PASAKA with memoranda charging them with a violation of the rule against
commission of acts injurious or prejudicial to the interest or welfare of the cooperative. The
memoranda cited that the respondents' filing of a case against Norkis Trading had greatly
prejudiced the interest and welfare of the cooperative. In their answer to the memoranda, the
respondents explained that they merely wanted to be recognized as regular employees of Norkis
Trading.
For their part, both Norkis Trading and PASAKA claimed that the respondents were not
employees of Norkis Trading. They insisted that the respondents were members of PASAKA, which
served as an independent contractor that merely supplied services to Norkis International Co., Inc.
(Norkis International) pursuant to a job contract which PASAKA and Norkis International executed
on January 14, 1999 for 121,500 pieces of F/GF-Series Reinforcement Production. After PASAKA
received reports from its coordinator at Norkis International of the respondents' low efficiency and
violation of the cooperative's rules, and after giving said respondents the chance to present their
side, a penalty of suspension was imposed upon them by the cooperative. The illegal suspension
being complained of was then not linked to the respondents' employment, but to their membership
with PASAKA. TEH
Norkis Trading stressed that the respondents were deployed by PASAKA to Norkis International, a
company that is entirely separate and distinct from Norkis Trading.

Issue:
WON PASAKA is a labor-only contractor

Ruling:
YES. Labor-only contracting, a prohibited act, is an arrangement where the contractor or
subcontractor merely recruits, supplies, or places workers to perform a job, work, or service for a
principal. In labor-only contracting, the following elements are present: (a) the contractor or
subcontractor does not have substantial capital or investment to actually perform the job, work, or
service under its own account and responsibility; and (b) the employees recruited, supplied or

311
placed by such contractor or subcontractor perform activities which are directly related to the main
business of the principal. These differentiate it from permissible or legitimate job contracting or
subcontracting, which refers to an arrangement whereby a principal agrees to put out or farm out
with the contractor or subcontractor the performance or completion of a specific job, work, or
service within a definite or predetermined period, regardless of whether such job, work, or service
is to be performed or completed within or outside the premises of the principal. A person is
considered engaged in legitimate job contracting or subcontracting if the following conditions
concur: (a) the contractor carries on a distinct and independent business and partakes the contract
work on his account under his own responsibility according to his own manner and method, free
from the control and direction of his employer or principal in all matters connected with the
performance of his work except as to the results thereof; (b) the contractor has substantial capital
or investment; and (c) the agreement between the principal and the contractor or subcontractor
assures the contractual employees' entitlement to all labor and occupational safety and health
standards, free exercise of the right to self-organization, security of tenure, and social welfare
benefits.
Sec. 4.Definition of terms. — . . .
d). . .
Subject to the provisions of Sections 6, 7 and 8 of this Rule, contracting or subcontracting shall be
legitimate if the following circumstances concur: TAcDHS
i)The contractor or subcontractor carries on a distinct and independent business and undertakes to
perform the job, work or service on its own account and under its own responsibility, according to
its own manner and method, and free from the control and direction of the principal in all
matters connected with the performance of the work except to the results thereof;
ii)The contractor or subcontractor has substantial capital or investment; and
iii)The agreement between the principal and contractor or subcontractor assures the contractual
employees entitlement to all labor and occupational and safety and health standards, free
exercise of the right to self-organization, security of tenure and social and welfare benefits.
Together with his finding that PASAKA evidently lacked substantial capital or investment required
from legitimate job contractors, Regional Director Balanag ruled that the cooperative failed to
dispute the respondents' allegation that officers of Norkis Trading supervised their work and paid
their salaries. In conclusion, PASAKA and Norkis Trading were declared solidarily liable for the
monetary awards made in favor of therein claimants-employees, which included herein
respondents.
irst. PASAKA failed to prove that it has substantial capitalization or investment in the form of tools,
equipment, machineries, work premises, among others, to qualify as an independent contractor.
PASAKA's claim that it has machineries and equipment worth P344,273.02 as reflected in its
Financial Statements and Supplementary Schedules is belied by private respondents' [among them,
herein respondents] evidence which consisted of pictures showing machineries and [equipment]
which were owned [by] and located [at] the premises of petitioner NORKIS TRADING (as earlier
noted, some of the pictures showed some of the private respondents operating said machines).
Indeed it makes one wonder why, if PASAKA indeed had such machineries and equipment worth
P344,273.02, private respondents were using machineries and [equipment] owned [by] and located
at the premises of NORKIS TRADING.

312
Even granting that indeed PASAKA had machineries and equipment worth P344,273.02, it was not
shown that said machineries and equipment were actually used in the performance or completion
of the job, work, or service that it was contracted to render under its supposed job contract.
Second. PASAKA likewise did not carry out an independent business from NORKIS TRADING. While
PASAKA was issued its Certificate of Registration on July 18[,] 1991, all it could show to prove that
it carried out an independent business as a job contractor were the Project Contract dated January
2, 1998 with NORKIS TRADING, and the Project Contract dated December 18, 1998 with NORKIS
INTERNATIONAL. However, as earlier discussed, the Project Contract dated December 18, 1998
with NORKIS INTERNATIONAL is nothing more than an afterthought by the petitioners to confuse
its workers and defeat their rightful claims. The same can be said of the Project Contract with
WICKER and VINE, INC., considering that it was executed only onFebruary 1, 2000. Verily, said
contract was submitted only to strengthen PASAKA's claim that it is a legitimate job
contractor. AIECSD
Third. Private respondents performed activities directly related to the principal business of NORKIS
TRADING. They worked as welders and machine operators engaged in the production of steel
crates which were sent to Japan for use as containers of motorcycles that are then sent back to
NORKIS TRADING. Private respondents['] functions therefore are directly related and vital to
NORKIS TRADING's business of manufacturing of Yamaha motorcycles.
All the foregoing considerations affirm by more than substantial evidence that NORKIS TRADING
and PASAKA engaged in labor-only contracting
Finally, contrary to the insinuations of Norkis Trading, the fact that PASAKA was a duly-registered
cooperative did not preclude the possibility that it was engaged in labor-only contracting, as
confirmed by the findings of the Regional Director. An entity is characterized as a labor-only
contractor based on the elements and guidelines established by law and jurisprudence, judging
primarily on the relationship that the said entity has with the company to which the workers are
deployed, and not on any special arrangement that the entity has with said workers.

G.R. No. 170054 January 21, 2013

GOYA, INC.,
VS
GOYA, INC. EMPLOYEES UNION-FFW

Facts:

Sometime in January 2004, petitioner Goya, Inc. (Company), a domestic corporation engaged in the
manufacture, importation, and wholesale of top quality food products, hired contractual employees
from PESO Resources Development Corporation (PESO) to perform temporary and occasional
services in its factory in Parang, Marikina City. This prompted respondent Goya, Inc. Employees
UnionFFW (Union) to request for a grievance conference on the ground that the contractual
workers do not belong to the categories of employees stipulated in the existing Collective

313
Bargaining Agreement (CBA).5 When the matter remained unresolved, the grievance was referred
to the National Conciliation and Mediation Board (NCMB) for voluntary arbitration.
During the hearing on July 1, 2004, the Company and the Union manifested before Voluntary
Arbitrator (VA) Bienvenido E. Laguesma that amicable settlement was no longer possible; hence,
they agreed to submit for resolution the solitary issue of "[w]hether or not the Company is guilty of
unfair labor acts in engaging the services of PESO, a third party service provider, under the existing
CBA, laws, and jurisprudence."6 Both parties thereafter filed their respective pleadings.
The Union asserted that the hiring of contractual employees from PESO is not a management
prerogative and in gross violation of the CBA tantamount to unfair labor practice (ULP). It noted
that the contractual workers engaged have been assigned to work in positions previously handled
by regular workers and Union members, in effect violating Section 4, Article I of the CBA, which
provides for three categories of employees in the Company, to wit:
Section 4. Categories of Employees. The parties agree on the following categories of employees:
(a) Probationary Employee. One hired to occupy a regular rank-and-file position in the Company
and is serving a probationary period. If the probationary employee is hired or comes from outside
the Company (non-Goya, Inc. employee), he shall be required to undergo a probationary period of
six (6) months, which period, in the sole judgment of management, may be shortened if the
employee has already acquired the knowledge or skills required of the job. If the employee is hired
from the casual pool and has worked in the same position at any time during the past two (2) years,
the probationary period shall be three (3) months.
(b) Regular Employee. An employee who has satisfactorily completed his probationary period and
automatically granted regular employment status in the Company.
(c) Casual Employee, One hired by the Company to perform occasional or seasonal work directly
connected with the regular operations of the Company, or one hired for specific projects of limited
duration not connected directly with the regular operations of the Company.
It was averred that the categories of employees had been a part of the CBA since the 1970s and that
due to this provision, a pool of casual employees had been maintained by the Company from which
it hired workers who then became regular workers when urgently necessary to employ them for
more than a year. Likewise, the Company sometimes hired probationary employees who also later
became regular workers after passing the probationary period. With the hiring of contractual
employees, the Union contended that it would no longer have probationary and casual employees
from which it could obtain additional Union members; thus, rendering inutile Section 1, Article III
(Union Security) of the CBA, which states:
Section 1. Condition of Employment. As a condition of continued employment in the Company, all
regular rank-and-file employees shall remain members of the Union in good standing and that new
employees covered by the appropriate bargaining unit shall automatically become regular
employees of the Company and shall remain members of the Union in good standing as a condition
of continued employment.
The Union moreover advanced that sustaining the Company’s position would easily weaken and
ultimately destroy the former with the latters resort to retrenchment and/or retirement of
employees and not filling up the vacant regular positions through the hiring of contractual workers
from PESO, and that a possible scenario could also be created by the Company wherein it could
"import" workers from PESO during an actual strike.

314
In countering the Unions allegations, the Company argued that: (a) the law expressly allows
contracting and subcontracting arrangements through Department of Labor and Employment
(DOLE) Order No. 18-02; (b) the engagement of contractual employees did not, in any way,
prejudice the Union, since not a single employee was terminated and neither did it result in a
reduction of working hours nor a reduction or splitting of the bargaining unit; and (c) Section 4,
Article I of the CBA merely provides for the definition of the categories of employees and does not
put a limitation on the Company’s right to engage the services of job contractors or its management
prerogative to address temporary/occasional needs in its operation.
On October 26, 2004, VA Laguesma dismissed the Unions charge of ULP for being purely speculative
and for lacking in factual basis, but the Company was directed to observe and comply with its
commitment under the CBA. The VA opined:

While the Union moved for partial reconsideration of the VA Decision, the Company immediately
filed a petition for review before the Court of Appeals (CA) under Rule 43 of the Revised Rules of
Civil Procedure to set aside the directive to observe and comply with the CBA commitment
pertaining to the hiring of casual employees when necessitated by business circumstances.
On June 16, 2005, the CA dismissed the petition.

Issue:

1. WON the contracting arrangement by the company was in keeping with the intent and spirit
of the CBA.
2. WON the contracting out by engaging the services of PESO was within the management
prerogative of the company to justify non-observance of the CBA provision

Ruling:
We confirm that the VA ruled on a matter that is covered by the sole issue submitted for voluntary
arbitration. Resultantly, the CA did not commit serious error when it sustained the ruling that the
hiring of contractual employees from PESO was not in keeping with the intent and spirit of the CBA.
Indeed, the opinion of the VA is germane to, or, in the words of the CA, "interrelated and
intertwined with," the sole issue submitted for resolution by the parties.

The Company kept on harping that both the VA and the CA conceded that its engagement of
contractual workers from PESO was a valid exercise of management prerogative. It is confused. To
emphasize, declaring that a particular act falls within the concept of management prerogative is
significantly different from acknowledging that such act is a valid exercise thereof. What the VA and
the CA correctly ruled was that the Company’s act of contracting out/outsourcing is within the
purview of management prerogative. Both did not say, however, that such act is a valid exercise
thereof. Obviously, this is due to the recognition that the CBA provisions agreed upon by the
Company and the Union delimit the free exercise of management prerogative pertaining to the
hiring of contractual employees. Indeed, the VA opined that "the right of the management to
outsource parts of its operations is not totally eliminated but is merely limited by the CBA," while
the CA held that "this management prerogative of contracting out services, however, is not without

315
limitation. x x x These categories of employees particularly with respect to casual employees serve
as limitation to the Companys prerogative to outsource parts of its operations especially when
hiring contractual employees."???
A collective bargaining agreement is the law between the parties:
It is familiar and fundamental doctrine in labor law that the CBA is the law between the parties and
they are obliged to comply with its provisions. We said so in Honda Phils., Inc. v. Samahan ng
Malayang Manggagawa sa Honda:
A collective bargaining agreement or CBA refers to the negotiated contract between a legitimate
labor organization and the employer concerning wages, hours of work and all other terms and
conditions of employment in a bargaining unit. As in all contracts, the parties in a CBA may
establish such stipulations, clauses, terms and conditions as they may deem convenient provided
these are not contrary to law, morals, good customs, public order or public policy. Thus, where the
CBA is clear and unambiguous, it becomes the law between the parties and compliance therewith is
mandated by the express policy of the law.
Moreover, if the terms of a contract, as in a CBA, are clear and leave no doubt upon the intention of
the contracting parties, the literal meaning of their stipulations shall control. x x x.
In this case, Section 4, Article I (on categories of employees) of the CBA between the Company and
the Union must be read in conjunction with its Section 1, Article III (on union security). Both are
interconnected and must be given full force and effect. Also, these provisions are clear and
unambiguous. The terms are explicit and the language of the CBA is not susceptible to any other
interpretation. Hence, the literal meaning should prevail. As repeatedly held, the exercise of
management prerogative is not unlimited; it is subject to the limitations found in law, collective
bargaining agreement or the general principles of fair play and justice. Evidently, this case has one
of the restrictions- the presence of specific CBA provisions-unlike in San Miguel Corporation
Employees Union-PTGWO v. Bersamira, De Ocampo v. NLRC, Asian Alcohol Corporation v.
NLRC, and Serrano v. NLRC cited by the Company. To reiterate, the CBA is the norm of conduct
between the parties and compliance therewith is mandated by the express policy of the law.l1

G.R. No. 200094 June 10, 2013

BENIGNO M. VIGILLA, ET AL
VS.
PHILIPPINE COLLEGE OF CRIMINOLOGY INC. AND/OR GREGORY ALAN F. BAUTISTA

Facts:

Philippine College of Criminology Inc. (PCCr) is a non-stock educational institution, while the
petitioners were janitors, janitresses and supervisor in the Maintenance Department of PCCr under
the supervision and control of Atty. Florante A. Seril (Atty. Seril), PCCr’s Senior Vice President for
Administration. The petitioners, however, were made to understand, upon application with
respondent school, that they were under MBMSI, a corporation engaged in providing janitorial
services to clients. Atty. Seril is also the President and General Manager of MBMSI.

316
Sometime in 2008, PCCr discovered that the Certificate of Incorporation of MBMSI had been
revoked as of July 2, 2003. On March 16, 2009, PCCr, through its President, respondent Gregory
Alan F. Bautista (Bautista), citing the revocation, terminated the school’s relationship with MBMSI,
resulting in the dismissal of the employees or maintenance personnel under MBMSI, except Alfonso
Bongot (Bongot) who was retired.
In September, 2009, the dismissed employees, led by their supervisor, Benigno Vigilla (Vigilla), filed
their respective complaints for illegal dismissal, reinstatement, back wages, separation pay (for
Bongot), underpayment of salaries, overtime pay, holiday pay, service incentive leave, and 13th
month pay against MBMSI, Atty. Seril, PCCr, and Bautista.
In their complaints, they alleged that it was the school, not MBMSI, which was their real employer
because (a) MBMSI’s certification had been revoked; (b) PCCr had direct control over MBMSI’s
operations; (c) there was no contract between MBMSI and PCCr; and (d) the selection and hiring of
employees were undertaken by PCCr.
On the other hand, PCCr and Bautista contended that (a) PCCr could not have illegally dismissed the
complainants because it was not their direct employer; (b) MBMSI was the one who had complete
and direct control over the complainants; and (c) PCCr had a contractual agreement with MBMSI,
thus, making the latter their direct employer.
On September 11, 2009, PCCr submitted several documents before LA Ronaldo Hernandez,
including releases, waivers and quitclaims in favor of MBMSI executed by the complainants to
prove that they were employees of MBMSI and not PCCr. The said documents appeared to have
been notarized by one Atty. Ramil Gabao.
After due proceedings, the LA handed down his decision, finding that (a) PCCr was the real
principal employer of the complainants ; (b) MBMSI was a mere adjunct or alter ego/labor-only
contractor; (c) the complainants were regular employees of PCCr; and (d) PCCr/Bautista were in
bad faith in dismissing the complainants.
Not satisfied, the respondents filed an appeal before the NLRC. In its Resolution, dated February 11,
2011, the NLRC affirmed the LA’s findings. Nevertheless, the respondents were excused from their
liability by virtue of the releases, waivers and quitclaims executed by the petitioners.
The CA pointed out that based on the principle of solidary liability and Article 1217 of the New Civil
Code, petitioners’ respective releases, waivers and quitclaims in favor of MBMSI and Atty. Seril
redounded to the benefit of the respondents. The CA also upheld the factual findings of the NLRC as
to the authenticity and due execution of the individual releases, waivers and quitclaims because of
the failure of petitioners to substantiate their claim of forgery and to overcome the presumption of
regularity of a notarized document.

Issue:

WON a Labor-only Contractor is Solidarily Liable with the Employer

Ruling:
The NLRC and the CA correctly ruled that the releases, waivers and quitclaims executed by
petitioners in favor of MBMSI redounded to the benefit of PCCr pursuant to Article 1217 of the New

317
Civil Code. The reason is that MBMSI is solidarily liable with the respondents for the valid claims of
petitioners pursuant to Article 109 of the Labor Code.
As correctly pointed out by the respondents, the basis of the solidary liability of the principal with
those engaged in labor-only contracting is the last paragraph of Article 106 of the Labor Code,
which in part provides: "In such cases labor-only contracting, the person or intermediary shall be
considered merely as an agent of the employer who shall be responsible to the workers in the same
manner and extent as if the latter were directly employed by him."
Section 19 of Department Order No. 18-02 issued by the Department of Labor and Employment
(DOLE), which was still in effect at the time of the promulgation of the subject decision and
resolution, interprets Article 106 of the Labor Code in this wise:
Section 19. Solidary liability. The principal shall be deemed as the direct employer of the
contractual employees and therefore, solidarily liable with the contractor or subcontractor for
whatever monetary claims the contractual employees may have against the former in the case of
violations as provided for in Sections 5 (LaborOnly contracting), 6 (Prohibitions), 8 (Rights of
Contractual Employees) and 16 (Delisting) of these Rules. In addition, the principal shall also be
solidarily liable in case the contract between the principal and contractor or subcontractor is
preterminated for reasons not attributable to the fault of the contractor or subcontractor.
The DOLE recognized anew this solidary liability of the principal employer and the labor-only
contractor when it issued Department Order No. 18-A, series of 2011, which is the latest set of rules
implementing Articles 106-109 of the Labor Code. Section 27 thereof reads:
Section 27. Effects of finding of labor-only contracting and/or violation of Sections 7, 8 or 9 of the
Rules. A finding by competent authority of labor-only contracting shall render the principal jointly
and severally liable with the contractor to the latter’s employees, in the same manner and extent
that the principal is liable to employees directly hired by him/her, as provided in Article 106 of the
Labor Code, as amended.
A finding of commission of any of the prohibited activities in Section 7, or violation of either
Sections 8 or 9 hereof, shall render the principal the direct employer of the employees of the
contractor or subcontractor, pursuant to Article 109 of the Labor Code, as amended.
These legislative rules and regulations designed to implement a primary legislation have the force
and effect of law. A rule is binding on the courts so long as the procedure fixed for its promulgation
is followed and its scope is within the statutory authority granted by the legislature.
Jurisprudence is also replete with pronouncements that a job-only contractor is solidarily liable
with the employer. One of these is the case of Philippine Bank of Communications v. NLRC where
this Court explained the legal effects of a job-only contracting, to wit:
Under the general rule set out in the first and second paragraphs of Article 106, an employer who
enters into a contract with a contractor for the performance of work for the employer, does not
thereby create an employer-employees relationship between himself and the employees of the
contractor. Thus, the employees of the contractor remain the contractor's employees and his alone.
Nonetheless when a contractor fails to pay the wages of his employees in accordance with the
Labor Code, the employer who contracted out the job to the contractor becomes jointly and
severally liable with his contractor to the employees of the latter "to the extent of the work
performed under the contract" as such employer were the employer of the contractor's employees.
The law itself, in other words, establishes an employer-employee relationship between the

318
employer and the job contractor's employees for a limited purpose, i.e., in order to ensure that the
latter get paid the wages due to them.
A similar situation obtains where there is "labor only" contracting. The "labor-only" contractor-i.e
"the person or intermediary" - is considered "merely as an agent of the employer." The employer is
made by the statute responsible to the employees of the "labor only" contractor as if such
employees had been directly employed by the employer. Thus, where "labor-only" contracting
exists in a given case, the statute itself implies or establishes an employer-employee relationship
between the employer (the owner of the project) and the employees of the "labor only" contractor,
this time for a comprehensive purpose: "employer for purposes of this Code, to prevent any
violation or circumvention of any provision of this Code." The law in effect holds both the employer
and the "laboronly" contractor responsible to the latter's employees for the more effective
safeguarding of the employees' rights under the Labor Code.
The case of San Miguel Corporation v. MAERC Integrated Services, Inc. also recognized this solidary
liability between a labor-only contractor and the employer. In the said case, this Court gave the
distinctions between solidary liability in legitimate job contracting and in labor-only contracting, to
wit:
In legitimate job contracting, the law creates an employer-employee relationship for a limited
purpose, i.e., to ensure that the employees are paid their wages. The principal employer becomes
jointly and severally liable with the job contractor only for the payment of the employees' wages
whenever the contractor fails to pay the same. Other than that, the principal employer is not
responsible for any claim made by the employees.
On the other hand, in labor-only contracting, the statute creates an employer-employee relationship
for a comprehensive purpose: to prevent a circumvention of labor laws. The contractor is
considered merely an agent of the principal employer and the latter is responsible to the employees
of the labor-only contractor as if such employees had been directly employed by the principal
employer. The principal employer therefore becomes solidarily liable with the labor-only
contractor for all the rightful claims of the employees.
Considering that MBMSI, as the labor-only contractor, is solidarily liable with the respondents, as
the principal employer, then the NLRC and the CA correctly held that the respondents’ solidary
liability was already expunged by virtue of the releases, waivers and quitclaims executed by each of
the petitioners in favor of MBMSI pursuant to Article 1217 of the Civil Code which provides that
"payment made by one of the solidary debtors extinguishes the obligation."
The Court holds that the releases, waivers and quitclaims executed by petitioners in favor of MBMSI
redounded to the respondents' benefit. The liabilities of the respondents to petitioners are now
deemed extinguished. The Court cannot allow petitioners to reap the benefits given to them by
MBMSI in exchange for the releases, waivers and quitclaims and, again, claim the same benefits
from PCCr.
While it is the duty of the courts to prevent the exploitation of employees, it also behooves the
courts to protect the sanctity of contracts that do not contravene the law. The law in protecting the
rights of the laborer authorizes neither oppression nor self-destruction of the employer. While the
Constitution is committed to the policy of social justice and the protection of the working class, it
should not be supposed that every labor dispute will be automatically decided in favor of labor.
Management also has its own rights, which, as such, are entitled to respect and enforcement in the

319
interest of simple fair play. Out of its concern for those with less privileges in life, the Court has
inclined more often than not toward the worker and upheld his cause in his conflicts with the
employer. Such favoritism, however, has not blinded the Court to the rule that justice is in every
case for the deserving, to be dispensed in the light of the established facts and applicable law and
doctrine.

G.R. No. 174912 July 24, 2013

BPI EMPLOYEES UNION-DAVAO CITY-FUBU (BPIEU-DAVAO CITY-FUBU), PETITIONER,


VS.
BANK OF THE PHILIPPINE ISLANDS (BPI), AND BPI OFFICERS CLARO M. REYES, CECIL
CONANAN AND GEMMA VELEZ, RESPONDENTS.

Facts:
BPI Operations Management Corporation (BOMC), which was created pursuant to Central Bank5
Circular No. 1388, Series of 1993 (CBP Circular No. 1388, 1993), and primarily engaged in
providing and/or handling support services for banks and other financial institutions, is a
subsidiary of the Bank of Philippine Islands (BPI) operating and functioning as an entirely separate
and distinct entity.

A service agreement between BPI and BOMC was initially implemented in BPI’s Metro Manila
branches. In this agreement, BOMC undertook to provide services such as check clearing, delivery
of bank statements, fund transfers, card production, operations accounting and control, and cash
servicing, conformably with BSP Circular No. 1388. Not a single BPI employee was displaced and
those performing the functions, which were transferred to BOMC, were given other assignments.
The Manila chapter of BPI Employees Union (BPIEU-Metro ManilaFUBU) then filed a complaint for
unfair labor practice (ULP). The Labor Arbiter (LA) decided the case in favor of the union. The
decision was, however, reversed on appeal by the NLRC. BPIEU-Metro Manila-FUBU filed a petition
for certiorari before the CA which denied it, holding that BPI transferred the employees in the
affected departments in the pursuit of its legitimate business. The employees were neither demoted
nor were their salaries, benefits and other privileges diminished.6

On January 1, 1996, the service agreement was likewise implemented in Davao City. Later, a merger
between BPI and Far East Bank and Trust Company (FEBTC) took effect on April 10, 2000 with BPI
as the surviving corporation. Thereafter, BPI’s cashiering function and FEBTC’s cashiering,
distribution and bookkeeping functions were handled by BOMC. Consequently, twelve (12) former
FEBTC employees were transferred to BOMC to complete the latter’s service complement.

BPI Davao’s rank and file collective bargaining agent, BPI Employees Union-Davao City-FUBU
(Union), objected to the transfer of the functions and the twelve (12) personnel to BOMC
contending that the functions rightfully belonged to the BPI employees and that the Union was
deprived of membership of former FEBTC personnel who, by virtue of the merger, would have

320
formed part of the bargaining unit represented by the Union pursuant to its union shop provision in
the CBA.7

The Union then filed a formal protest on June 14, 2000 addressed to BPI Vice Presidents Claro M.
Reyes and Cecil Conanan reiterating its objection. It requested the BPI management to submit the
BOMC issue to the grievance procedure under the CBA, but BPI did not consider it as "grievable."
Instead, BPI proposed a Labor Management Conference (LMC) between the parties.8

During the LMC, BPI invoked management prerogative stating that the creation of the BOMC was to
preserve more jobs and to designate it as an agency to place employees where they were most
needed. On the other hand, the Union charged that BOMC undermined the existence of the union
since it reduced or divided the bargaining unit. While BOMC employees perform BPI functions, they
were beyond the bargaining unit’s coverage. In contracting out FEBTC functions to BOMC, BPI
effectively deprived the union of the membership of employees handling said functions as well as
curtailed the right of those employees to join the union.

Thereafter, the Union demanded that the matter be submitted to the grievance machinery as the
resort to the LMC was unsuccessful. As BPI allegedly ignored the demand, the Union filed a notice of
strike before the National Conciliation and Mediation Board (NCMB).

BPI then filed a petition for assumption of jurisdiction/certification with the Secretary of the
Department of Labor and Employment (DOLE), who subsequently issued an order certifying the
labor dispute to the NLRC for compulsory arbitration. The DOLE Secretary directed the parties to
cease and desist from committing any act that might exacerbate the situation.

On December 21, 2001, the NLRC came out with a resolution upholding the validity of the service
agreement between BPI and BOMC and dismissing the charge of ULP. It ruled that the engagement
by BPI of BOMC to undertake some of its activities was clearly a valid exercise of its management
prerogative.11 It further stated that the spinning off by BPI to BOMC of certain services and
functions did not interfere with, restrain or coerce employees in the exercise of their right to self-
organization.12 The Union did not present even an iota of evidence showing that BPI had terminated
employees, who were its members. In fact, BPI exerted utmost diligence, care and effort to see to it
that no union member was terminated.

After the denial of its motion for reconsideration, the Union elevated its grievance to the CA via a
petition for certiorari under Rule 65. The CA, however, affirmed the NLRC’s December 21, 2001
Resolution.

Furthermore, the CA ratiocinated that, considering the ramifications of the corporate merger, it was
well within BPI’s prerogatives "to determine what additional tasks should be performed, who
should best perform it and what should be done to meet the exigencies of business."16 It pointed out
that the Union did not, by the mere fact of the merger, become the bargaining agent of the merged
employees17 as the Union’s right to represent said employees did not arise until it was chosen by

321
them.

The Union is of the position that the outsourcing of jobs included in the existing bargaining unit to
BOMC is a breach of the union-shop agreement in the CBA. In transferring the former employees of
FEBTC to BOMC instead of absorbing them in BPI as the surviving corporation in the merger, the
number of positions covered by the bargaining unit was decreased, resulting in the reduction of the
Union’s membership. For the Union, BPI’s act of arbitrarily outsourcing functions formerly
performed by the Union members and, in fact, transferring a number of its members beyond the
ambit of the Union, is a violation of the CBA and interfered with the employees’ right to self
organization. The Union insists that the CBA covers the agreement with respect, not only to wages
and hours of work, but to all other terms and conditions of work. The union shop clause, being part
of these conditions, states that the regular employees belonging to the bargaining unit, including
those absorbed by way of the corporate merger, were required to join the bargaining union "as a
condition for employment." Simply put, the transfer of former FEBTC employees to BOMC removed
them from the coverage of unionized establishment. While the Union admitted that BPI has the
prerogative to determine what should be done to meet the exigencies of business in accordance
with the case of Sime Darby Pilipinas, Inc. v. NLRC,19 it insisted that the exercise of management
prerogative is not absolute, thus, requiring good faith and adherence to the law and the CBA. Citing
the case of Shell Oil Workers’ Union v. Shell Company of the Philippines, Ltd.,20 the Union claims
that it is unfair labor practice for an employer to outsource the positions in the existing bargaining
unit.

For its part, BPI defended the validity of its service agreement with BOMC on three (3) grounds: 1]
that it was pursuant to the prevailing law at that time, CBP Circular No. 1388; 2] that the creation of
BOMC was within management prerogatives intended to streamline the operations and provide
focus for BPI’s core activities; and 3] that the Union recognized, in its CBA, the exclusive right and
prerogative of BPI to conduct the management and operation of its business.21

On the issue of the alleged curtailment of the right of the employees to self-organization, BPI refutes
the Union’s allegation that ULP was committed when the number of positions in the bargaining was
reduced. It cites as correct the CA ruling that the representation of the Union’s prospective
members is contingent on the choice of the employee, that is, whether or not to join the Union.
Hence, it was premature for the Union to claim that the rights of its prospective members to self-
organize were restrained by the transfer of the former FEBTC employees to BOMC.

Issue:
Whether or not the act of BPI to outsource the cashiering, distribution and bookkeeping functions
to BOMC is in conformity with the law and the existing CBA. Particularly in dispute is the validity of
the transfer of twelve (12) former FEBTC employees to BOMC, instead of being absorbed in BPI
after the corporate merger.

Ruling:
The Union’s reliance on the Shell Case is misplaced. The rule now is covered by Article 261 of the

322
Labor Code, which took effect on November 1, 1974.25 Article 261 provides:
ART. 261. Jurisdiction of Voluntary Arbitrators or panel of Voluntary Arbitrators. – x x x
Accordingly, violations of a Collective Bargaining Agreement, except those which are gross in
character, shall no longer be treated as unfair labor practice and shall be resolved as grievances
under the Collective Bargaining Agreement. For purposes of this article, gross violations of
Collective Bargaining Agreement shall mean flagrant and/or malicious refusal to comply with the
economic provisions of such agreement. [Emphases supplied]

Clearly, only gross violations of the economic provisions of the CBA are treated as ULP. Otherwise,
they are mere grievances.

In the present case, the alleged violation of the union shop agreement in the CBA, even assuming it
was malicious and flagrant, is not a violation of an economic provision in the agreement. The
provisions relied upon by the Union were those articles referring to the recognition of the union as
the sole and exclusive bargaining representative of all rank-and-file employees, as well as the
articles on union security, specifically, the maintenance of membership in good standing as a
condition for continued employment and the union shop clause.26 It failed to take into consideration
its recognition of the bank’s exclusive rights and prerogatives, likewise provided in the CBA, which
included the hiring of employees, promotion, transfers, and dismissals for just cause and the
maintenance of order, discipline and efficiency in its operations.27

It is incomprehensible how the "reduction of positions in the collective bargaining unit" interferes
with the employees’ right to self-organization because the employees themselves were neither
transferred nor dismissed from the service. As the NLRC clearly stated:

In the case at hand, the union has not presented even an iota of evidence that petitioner bank has
started to terminate certain employees, members of the union. In fact, what appears is that the
Bank has exerted utmost diligence, care and effort to see to it that no union member has been
terminated. In the process of the consolidation or merger of the two banks which resulted in
increased diversification of functions, some of these non-banking functions were merely
transferred to the BOMC without affecting the union membership.29

BPI stresses that not a single employee or union member was or would be dislocated or terminated
from their employment as a result of the Service Agreement.30 Neither had it resulted in any
diminution of salaries and benefits nor led to any reduction of union membership.31

As far as the twelve (12) former FEBTC employees are concerned, the Union failed to substantially
prove that their transfer, made to complete BOMC’s service complement, was motivated by ill will,
anti-unionism or bad faith so as to affect or interfere with the employees’ right to self-organization.
It is to be emphasized that contracting out of services is not illegal perse. It is an exercise of
business judgment or management prerogative. Absent proof that the management acted in a
malicious or arbitrary manner, the Court will not interfere with the exercise of judgment by an
employer.32 In this case, bad faith cannot be attributed to BPI because its actions were authorized

323
by CBP Circular No. 1388, Series of 199333 issued by the Monetary Board of the then Central Bank of
the Philippines (now Bangko Sentral ng Pilipinas). The circular covered amendments in Book I of
the Manual of Regulations for Banks and Other Financial Intermediaries, particularly on the matter
of bank service contracts. A finding of ULP necessarily requires the alleging party to prove it with
substantial evidence. Unfortunately, the Union failed to discharge this burden.
While D.O. No. 10, Series of 1997, enumerates the permissible contracting or subcontracting
activities, it is to be observed that, particularly in Sec. 6(d) invoked by the Union, the provision is
general in character – "x x x Works or services not directly related or not integral to the main
business or operation of the principal… x x x." This does not limit or prohibit the appropriate
government agency, such as the BSP, to issue rules, regulations or circulars to further and
specifically determine the permissible services to be contracted out. CBP Circular No. 1388 38
enumerated functions which are ancillary to the business of banks, hence, allowed to be
outsourced. Thus, sanctioned by said circular, BPI outsourced the cashiering (i.e., cash-delivery and
deposit pick-up) and accounting requirements of its Davao City branches.

B. WORKER’S PREFERENCE

C. ATTORNEY’S FEES & APPEARANCE OF LAWYERS

G.R. No. 174179, November 16, 2011

KAISAHAN AT KAPATIRAN NG MGA MANGGAGAWA AT KAWANI SA MWC-EAST ZONE UNION


AND EDUARDO BORELA, REPRESENTING ITS MEMBERS, PETITIONERS,
VS
MANILA WATER COMPANY, INC., RESPONDENT.

Facts:
Kaisahan at Kapatiran ng mga Manggagawa at Kawani sa MWC-East Zone Union (Union) is the duly-
recognized bargaining agent of the rank-and-file employees of the respondent Manila Water
Company, Inc. (Company) while Borela is the Union President.[5] On February 21, 1997, the
Metropolitan Waterworks and Sewerage System (MWSS) entered into a Concession Agreement
(Agreement) with the Company to privatize the operations of the MWSS.[6] Article 6.1.3 of the
Agreement provides that “the Concessionaire shall grant [its] employees benefits no less favorable
than those granted to MWSS employees at the time of [their] separation from MWSS.”[7] Among the
benefits enjoyed by the employees of the MWSS were the amelioration allowance (AA) and the cost-
of-living allowance (COLA) granted in August 1979, pursuant to Letter of Implementation No. 97
issued by the Office of the President.

The payment of the AA and the COLA was discontinued pursuant to Republic Act No. 6758,
otherwise known as the “Salary Standardization Law,” which integrated the allowances into the
standardized salary.[9] Nonetheless, in 2001, the Union demanded from the Company the payment

324
of the AA and the COLA during the renegotiation of the parties’ Collective Bargaining Agreement
(CBA).[10] The Company initially turned down this demand, however, it subsequently agreed to an
amendment of the CBA on the matter, which provides:

The Company shall implement the payment of the Amelioration Allowance and Cost of Living
[A]llowance retroactive August 1, 1997 should the MWSS decide to pay its employees and all its
former employees or upon award of a favorable order by the MWSS Regulatory Office or upon
receipt of [a] final court judgment.

Thereafter, the Company integrated the AA into the monthly payroll of all its employees beginning
August 1, 2002, payment of the AA and the COLA after an appropriation was made and approved by
the MWSS Board of Trustees. The Company, however, did not subsequently include the COLA since
the Commission on Audit disapproved its payment because the Company had no funds to cover this
benefit.[12]

As a result, the Union and Borela filed on April 15, 2003 a complaint against the Company for
payment of the AA, COLA, moral and exemplary damages, legal interest, and attorney’s fees before
the National Labor Relations Commission (NLRC).

In his decision of August 20, 2003, Labor Arbiter Aliman D. Mangandog (LA) ruled in favor of the
petitioners and ordered the payment of their AA and COLA, six percent (6%) interest of the total
amount awarded, and ten percent (10%) attorney’s fees.

On appeal by the Company, the NLRC affirmed with modification the LA’s decision.[15] It set aside
the award of the COLA benefits because the claim was not proven and established, but ordered the
Company to pay the petitioners their accrued AA of about P107,300,000.00 in lump sum and to
continue paying the AA starting August 1, 2002. It also upheld the award of 10% attorney’s fees to
the petitioners.

In its Motion for Partial Reconsideration of the NLRC’s December 19, 2003 decision, the Company
pointed out that the award of ten percent (10%) attorney’s fees to the petitioners is already
provided for in their December 19, 2003 Memorandum of Agreement (MOA) which mandated that
attorney’s fees shall be deducted from the AA and CBA receivables. This compromise agreement,
concluded between the parties in connection with a notice of strike filed by the Union in 2003,[17]
provides among others that:[18]
31. Attorney’s fees – 10% to be deducted from AA and CBA receivables.
32. All other issues are considered withdrawn.

In their Opposition, the petitioners argued that the MOA only covered the payment of their share in
the contracted attorney’s fees, but did not include the attorney’s fees awarded by the NLRC. To
support their claim, the petitioners submitted Borela’s affidavit which relevantly stated:
Xxx
4. The 10% attorney’s fees paid by the members/employees is separate and distinct from the

325
obligation of the company to pay the 10% awarded attorney’s fees which we also gave to our
counsel as part of our contingent fee agreement.

In its Decision promulgated on March 6, 2006,[23] the CA modified the assailed NLRC rulings by
deleting “[t]he order for respondent MWCI to pay attorney’s fees equivalent to 10% of the total
judgment awards.” The CA recognized the binding effect of the MOA between the Company and the
Union; it stressed that any further award of attorney’s fees is unfounded considering that it did not
find anything in the Agreement that is contrary to law, morals, good customs, public policy or public
order.

The petitioners argue that in the present case, all the parties’ arguments and evidence relating to
the award of attorney’s fees were carefully studied and weighed by the NLRC. As a result, the NLRC
gave credence to Borela’s affidavit claiming that the attorney’s fees paid by the Union’s members
are separate and distinct from the attorney’s fees awarded by the NLRC. The petitioners stress that
whether the NLRC is correct in giving credence to Borela’s affidavit is a question that the CA cannot
act upon in a petition for certiorari unless grave abuse of discretion can be shown.[27]

Issue:
Whether the NLRC gravely abused its discretion in awarding ten percent (10%) attorney’s fees to
the petitioners.

Held:
On the Award of Attorney’s Fees
Article 111 of the Labor Code, as amended, governs the grant of attorney’s fees in labor cases:
Art. 111. Attorney’s fees.- (a) In cases of unlawful withholding of wages, the culpable party may be
assessed attorney’s fees equivalent to ten percent of the amount of wages recovered.
(b) It shall be unlawful for any person to demand or accept, in any judicial or administrative
proceedings for the recovery of wages, attorney’s fees which exceed ten percent of the amount of
wages recovered.
Section 8, Rule VIII, Book III of its Implementing Rules also provides, viz.:
Section 8. Attorney’s fees. – Attorney’s fees in any judicial or administrative proceedings for
the recovery of wages shall not exceed 10% of the amount awarded. The fees may be deducted
from the total amount due the winning party.
We explained in PCL Shipping Philippines, Inc. v. National Labor Relations Commission[34]that there
are two commonly accepted concepts of attorney’s fees – the ordinary and extraordinary. In its
ordinary concept, an attorney’s fee is the reasonable compensation paid to a lawyer by his client
for the legal services the former renders; compensation is paid for the cost and/or results of legal
services per agreement or as may be assessed. In its extraordinary concept, attorney’s fees are
deemed indemnity for damages ordered by the court to be paid by the losing party to the
winning party. The instances when these may be awarded are enumerated in Article 2208 of the
Civil Code, specifically in its paragraph 7 on actions for recovery of wages, and is payable not to the
lawyer but to the client, unless the client and his lawyer have agreed that the award shall
accrue to the lawyer as additional or part of compensation.[35]

326
We also held in PCL Shipping that Article 111 of the Labor Code, as amended, contemplates the
extraordinary concept of attorney’s fees and that Article 111 is an exception to the declared
policy of strict construction in the award of attorney’s fees. Although an express finding of
facts and law is still necessary to prove the merit of the award, there need not be any
showing that the employer acted maliciously or in bad faith when it withheld the wages. In
carrying out and interpreting the Labor Code's provisions and implementing regulations, the
employee's welfare should be the primary and paramount consideration. This kind of
interpretation gives meaning and substance to the liberal and compassionate spirit of the law as
embodied in Article 4 of the Labor Code (which provides that "[a]ll doubts in the implementation
and interpretation of the provisions of [the Labor Code], including its implementing rules and
regulations, shall be resolved in favor of labor") and Article 1702 of the Civil Code (which provides
that "[i]n case of doubt, all labor legislation and all labor contracts shall be construed in favor of the
safety and decent living for the laborer”).[36]
We similarly so ruled in RTG Construction, Inc. v. Facto[37]and in Ortiz v. San Miguel
Corporation.[38] In RTG Construction, we specifically stated:
Settled is the rule that in actions for recovery of wages, or where an employee was forced to litigate
and, thus, incur expenses to protect his rights and interests, a monetary award by way of attorney’s
fees is justifiable under Article 111 of the Labor Code; Section 8, Rule VIII, Book III of its
Implementing Rules; and paragraph 7, Article 2208 of the Civil Code. The award of attorney’s
fees is proper, and there need not be any showing that the employer acted maliciously or in
bad faith when it withheld the wages. There need only be a showing that the lawful wages
were not paid accordingly.[39] (emphasis ours)
In PCL Shipping, we found the award of attorney’s fees due and appropriate since the respondent
therein incurred legal expenses after he was forced to file an action for recovery of his lawful wages
and other benefits to protect his rights.[40] From this perspective and the above precedents, we
conclude that the CA erred in ruling that a finding of the employer’s malice or bad faith in
withholding wages must precede an award of attorney’s fees under Article 111 of the Labor
Code. To reiterate, a plain showing that the lawful wages were not paid without justification is
sufficient.
In the present case, we find it undisputed that the union members are entitled to their AA benefits
and that these benefits were not paid by the Company. That the Company had no funds is not a
defense as this was not an insuperable cause that was cited and properly invoked. As a
consequence, the union members represented by the Union were compelled to litigate and incur
legal expenses. On these bases, we find no difficulty in upholding the NLRC’s award of ten percent
(10%) attorney’s fees.
The more significant issue in this case is the effect of the MOA provision that attorney’s fees shall be
deducted from the AA and CBA receivables. In this regard, the CA held that the additional grant of
10% attorney’s fees by the NLRC violates Article 111 of the Labor Code, considering that the MOA
between the parties already ensured the payment of 10% attorney’s fees deductible from the AA
and CBA receivables of the Union’s members. In addition, the Company also argues that the Union’s
demand, together with the NLRC award, is unconscionable as it represents 20% of the amount due
or about P21.4 million.
In Traders Royal Bank Employees Union-Independent v. NLRC,[41] we expounded on the

327
concept of attorney’s fees in the context of Article 111 of the Labor Code, as follows:
In the first place, the fees mentioned here are the extraordinary attorney’s fees recoverable as
indemnity for damages sustained by and payable to the prevailing part[y]. In the second
place, the ten percent (10%) attorney’s fees provided for in Article 111 of the Labor Code and
Section 11, Rule VIII, Book III of the Implementing Rules is the maximum of the award that may
thus be granted. Article 111 thus fixes only the limit on the amount of attorney’s fees the
victorious party may recover in any judicial or administrative proceedings and it does not even
prevent the NLRC from fixing an amount lower than the ten percent (10%) ceiling prescribed by the
article when circumstances warrant it.[42] (emphases ours; citation omitted)
In the present case, the ten percent (10%) attorney’s fees awarded by the NLRC on the basis of
Article 111 of the Labor Code accrue to the Union’s members as indemnity for damages and not to
the Union’s counsel as compensation for his legal services, unless, they agreed that the award
shall be given to their counsel as additional or part of his compensation; in this case the
Union bound itself to pay 10% attorney’s fees to its counsel under the MOA and also gave up the
attorney’s fees awarded to the Union’s members in favor of their counsel. This is supported by
Borela’s affidavit which stated that “[t]he 10% attorney’s fees paid by the members/employees is
separate and distinct from the obligation of the company to pay the 10% awarded attorney’s fees
which we also gave to our counsel as part of our contingent fee agreement.”[43] The limit to this
agreement is that the indemnity for damages imposed by the NLRC on the losing party (i.e., the
Company) cannot exceed ten percent (10%).
Properly viewed from this perspective, the award cannot be taken to mean an additional grant of
attorney’s fees, in violation of the ten percent (10%) limit under Article 111 of the Labor Code since
it rests on an entirely different legal obligation than the one contracted under the MOA. Simply
stated, the attorney’s fees contracted under the MOA do not refer to the amount of attorney’s
fees awarded by the NLRC; the MOA provision on attorney’s fees does not have any bearing
at all to the attorney’s fees awarded by the NLRC under Article 111 of the Labor Code. Based
on these considerations, it is clear that the CA erred in ruling that the LA’s award of attorney’s fees
violated the maximum limit of ten percent (10%) fixed by Article 111 of the Labor Code.
Under this interpretation, the Company’s argument that the attorney’s fees are unconscionable as
they represent 20% of the amount due or about P21.4 million is more apparent than real. Since the
attorney’s fees awarded by the LA pertained to the Union’s members as indemnity for damages, it
was totally within their right to waive the amount and give it to their counsel as part of their
contingent fee agreement. Beyond the limit fixed by Article 111 of the Labor Code, such as between
the lawyer and the client, the attorney’s fees may exceed ten percent (10%) on the basis of quantum
meruit, as in the present case.

G.R. No. 183952. September 9, 2013

CZARINA T. MALVAR
VS.
KRAFT FOOD PHILS., INC. AND/OR BIENVENIDO BAUTISTA, KRAFT FOODS INTERNATIONAL

328
Facts:
The case initially concerned the execution of a final decision of the Court of Appeals (CA) in a labor
litigation, but has mutated into a dispute over attorney's fees between the winning employee and
her attorney after she entered into a compromise agreement with her employer under
circumstances that the attorney has bewailed as designed to prevent the recovery of just
professional fees.
Malvar, hired by Kraft in 1988, filed a complaint for illegal suspension and illegal dismissal against
KFPI and Bautista in the National Labor Relations Commission (NLRC) when an investigation for
possible breach of trust and confidence and for willful violation of company rules and regulations
led to her preventive suspension and, eventually, to her termination. She engaged the services of
the Law Firm of Dasal, Llasos and Associates on a contingency basis and pegged the latter’s
contingency fee at 10%.
Malvar’s case was favorably decided by the LA and the NLRC, differing only on the basis of
the computation for the monetary claim. When it was elevated to the CA, the latter issued an
injunction on the execution of the decision, pending a computation of the awards to be directly
done by the Labor Arbiter. While Malvar appealed the injunction of the CA, she entered into a
compromise agreement with Kraft and later terminated the services of the law firm.
Thereafter, Malvar filed an undated Motion to Dismiss/Withdraw Case praying that the appeal be
immediately dismissed/withdrawn in view of the compromise agreement, and that the case be
considered closed and terminated. Before the Court could act on Malvar's Motion to
Dismiss/Withdraw Case, the Court received on February 15, 2011 a so-called Motion for
Intervention to Protect Attorney's Rights from The Law Firm of Dasal, Llasos and Associates,
through its of Counsel Retired Supreme Court Associate Justice Josue N. Bellosillo (Intervenor),
whereby the Intervenor sought, among others, that both Malvar and KFPI be held and ordered to
pay jointly and severally the Intervenor's contingent fees.

Issue:
“The issues for our consideration and determination are twofold, namely: (a) whether or not
Malvar's motion to dismiss the petition on the ground of the execution of the compromise
agreement was proper; and (b) whether or not the Motion for Intervention to protect attorney's
rights can prosper, and, if so, how much could it recover as attorney's fees.

Ruling:
The Supreme Court approved the compromise agreement; granted the Motion for
Intervention to Protect Attorney's Rights; and ordered Czarina T. Malvar and respondents Kraft
Food Philippines, Inc. and Kraft Foods International to jointly and severally pay to Intervenor Law
Firm, represented by Retired Associate Justice Josue N. Bellosillo, its stipulated contingent fees of
10% the further sum equivalent to 10% of the value of the stock option.

1.
Client's right to settle litigation by compromise agreement,
And to terminate counsel; limitations

329
A client has an undoubted right to settle her litigation without the intervention of the attorney, for
the former is generally conceded to have exclusive control over the subject matter of the litigation
and may at any time, if acting in good faith, settle and adjust the cause of action out of court before
judgment, even without the attorney's intervention. 35 It is important for the client to show,
however, that the compromise agreement does not adversely affect third persons who are not
parties to the agreement.
A client may at any time dismiss his attorney or substitute another in his place, but if the contract
between client and attorney has been reduced to writing and the dismissal of the attorney was
without justifiable cause, he shall be entitled to recover from the client the full compensation
stipulated in the contract. However, the attorney may, in the discretion of the court, intervene in the
case to protect his rights. For the payment of his compensation the attorney shall have a lien upon
all judgments for the payment of money, and executions issued in pursuance of such judgment,
rendered in the case wherein his services had been retained by the client.
In fine, it is basic that an attorney is entitled to have and to receive a just and reasonable
compensation for services performed at the special instance and request of his client. The attorney
who has acted in good faith and honesty in representing and serving the interests of the client
should be reasonably compensated for his service.

2.
Compromise agreement is to be approved despite favorable action
on the Intervenor's Motion for Intervention

On considerations of equity and fairness, the Court disapproves of the tendencies of clients
compromising their cases behind the backs of their attorneys for the purpose of unreasonably
reducing or completely setting to naught the stipulated contingent fees. Thus, the Court grants the
Intervenor's Motion for Intervention to Protect Attorney's Rights as a measure of protecting the
Intervenor's right to its stipulated professional fees that would be denied under the compromise
agreement. The Court does so in the interest of protecting the rights of the practicing Bar rendering
professional services on contingent fee basis.
Nonetheless, the claim for attorney's fees does not void or nullify the compromise agreement
between Malvar and the respondents. There being no obstacles to its approval, the Court approves
the compromise agreement. The Court adds, however, that the Intervenor is not left without a
remedy, for the payment of its adequate and reasonable compensation could not be annulled by the
settlement of the litigation without its participation and conformity. It remains entitled to the
compensation, and its right is safeguarded by the Court because its members are officers of the
Court who are as entitled to judicial protection against injustice or imposition of fraud committed
by the client as much as the client is against their abuses as her counsel. In other words, the duty of
the Court is not only to ensure that the attorney acts in a proper and lawful manner, but also to see
to it that the attorney is paid his just fees. Even if the compensation of the attorney is dependent
only on winning the litigation, the subsequent withdrawal of the case upon the client's initiative
would not deprive the attorney of the legitimate compensation for professional services rendered.
But, as earlier pointed out, although a client may dismiss her lawyer at any time, the dismissal must
be for a justifiable cause if a written contract between the lawyer and the client exists. Considering

330
the undisputed existence of the written agreement on contingent fees, the question begging to be
answered is: Was the Intervenor dismissed for a justifiable cause? The Supreme Court did not think
so.
In the absence of the lawyer's fault, consent or waiver, a client cannot deprive the lawyer of his just
fees already earned in the guise of a justifiable reason. Here, Malvar not only downplayed the worth
of the Intervenor's legal service to her but also attempted to camouflage her intent to defraud her
lawyer by offering excuses that were not only inconsistent with her actions but, most importantly,
fell short of being justifiable. The letter Malvar addressed to Retired Justice Bellosillo, who
represented the Intervenor, debunked her allegations of unsatisfactory legal service because she
thereby lavishly lauded the Intervenor for its dedication and devotion to the prosecution of her case
and to the protection of her interests.
Also significant was that the attorney-client relationship between her and the Intervenor was not
severed upon Atty. Dasal's appointment to public office and Atty. Llasos' resignation from the law
firm because “a client who employs a law firm engages the entire law firm; hence, the resignation,
retirement or separation from the law firm of the handling lawyer does not terminate the
relationship, because the law firm is bound to provide a replacement”.
Although the practice of law is not a business, an attorney is entitled to be properly compensated
for the professional services rendered for the client, who is bound by her express agreement to duly
compensate the attorney. The client may not deny her attorney such just compensation.

26. MISCELLANEOUS PROVISIONS

A. SPECIAL TYPES OF WORKERS

B. EMPLOYMENT OF WOMEN

C. EMPLOYMENT OF CHILDREN

D. EMPLOYMENT OF HOUSEHELPER

G.R. No. 195167. November 16, 2011.

FERNANDO CO (FORMERLY DOING BUSINESS UNDER THE NAME “NATHANIEL MAMI HOUSE”)
VS.
LINA B. VARGAS

331
Facts:

On 22 April 2003, respondent Lina B. Vargas (respondent) filed against Nathaniel Bakeshop and its
owner Fernando Co a complaint for underpayment or non-payment of wages and holiday pay. The
complaint was later amended to include illegal dismissal as a cause of action and the non-payment
of service incentive leave.
Respondent alleged that she started working at the bakeshop in October 1994 as a baker and
worked from 8:00 a.m. until 8:30 p.m., Monday to Saturday. Aside from baking, respondent also
served the customers and supervised the other workers in the absence of the owner. Furthermore,
respondent claimed that she sometimes cooked and did the chores of a housemaid whenever the
latter was not available. Respondent had a salary of P220 per day, which she received every
Saturday afternoon.
During the period of her employment, respondent was not given a payslip and she was never asked
to sign a payroll.
On 6 April 2003, petitioner Co's wife, Nely Co, told respondent to cook their lunch because the
housemaid was ironing clothes. Since respondent was busy preparing customers' orders, she lost
track of time and was unable to cook lunch as instructed. Irate at respondent's failure to cook, Nely
Co cussed respondent and told her to leave and never to return because she was not needed
anymore. Respondent was so humiliated and could no longer bear the treatment she received from
her employers that she decided to take her salary and leave that same day. Respondent later filed
the complaint against Nathaniel Bakeshop and its owner Fernando Co.
On the other hand, Co denied that Vargas was hired as a baker. Petitioner alleged that
Vargas was merely employed as a housemaid and on the 6th of April, when the latter failed to cook
lunch, she was reprimanded. Furthermore, Vargas allegedly demanded her salary and never
showed up for work after the incident. She supposedly shouted at their daughter, showed
subordination, and was utterly disrespectful.
The Labor Arbiter ruled in favor of Vargas, noting that: “the place of business of petitioner is
the same as his place of residence and that respondent works for petitioner as well as for his
business which is based in his home. Thus, the Labor Arbiter concluded that "while complainant
may have started her employ doing chores for the [petitioner's] family, she also fulfilled tasks
connected with the [petitioner's] business such as cooking, filling orders, baking orders, and other
clerical work, all of which are usually necessary and desirable in the usual trade or business of the
respondent. Inescapably, complainant is a regular employee and thus, entitled to security of
tenure."
On appeal, the NLRC reversed and set aside the Labor Arbiter's Decision. The NLRC concluded that
respondent was not employed as a baker at petitioner's bakeshop but was merely petitioner's
housemaid who left her employ voluntarily. The NLRC found petitioner not guilty of illegal
dismissal.
In a petition for certiorari with the Court of Appeals, the latter reinstated the decision of the Labor
Arbiter. The CA ruled that, “[I]t is clear that petitioner [Lina B. Vargas] is not a househelper or
domestic servant of private respondents [Nathaniel Bakeshop and Fernando Co]. The evidence
shows that petitioner is working within the premises of the business of private respondent Co and
in relation to or in connection with such business. In the Memorandum of Appeal filed by private

332
respondents before the NLRC, the place of business of respondent Co and his residence is located in
the same place, Brgy. Juliana, San Fernando, Pampanga. Thus, respondent Co exercised control and
supervision over petitioner's functions. Hence, this petition filed by Co with the Supreme Court.

Issue:
“Petitioner raises the sole issue of whether the "Court of Appeals erred in ruling that at the time
Respondent was working with the Co family; the business was being conducted at the residence."

Ruling:
The Supreme Court found the petition without merit. It denied the petition and affirmed the ruling
of the LA and the CA.
In this case, it was only in petitioner's Supplement to the Motion for Reconsideration of the Court of
Appeals' Decision that petitioner raised the issue that contrary to the findings of the Labor Arbiter,
NLRC, and the Court of Appeals, the bakery was not located at his residence at the time respondent
was in their employ. Furthermore, petitioner would even have this Court evaluate additional
documentary evidence which were not offered during the proceedings in the Labor Arbiter, NLRC,
and the Court of Appeals. The additional evidence was only submitted after the Court of Appeals
promulgated its Decision, when petitioner attached the additional evidence in his Supplement to
the Motion for Reconsideration.
The issue raised by petitioner is clearly a question of fact which requires a review of the evidence
presented. The Supreme Court is not a trier of facts. It is not the function of this Court to examine,
review or evaluate the evidence all over again, especially on evidence raised for the first time on
appeal.
As a rule, the findings of fact of the Court of Appeals are final and conclusive and this Court will not
review them on appeal, subject to exceptions such as those enumerated by this Court in
Development Bank of the Philippines v. Traders Royal Bank. Petitioner failed to show that this case
falls under any of the exceptions. The finding of the Labor Arbiter that petitioner's bakery and his
residence are located at the same place was not reversed by the NLRC. Furthermore, the Court of
Appeals upheld this finding of the Labor Arbiter. We find no justifiable reason to deviate from the
findings and ruling of the Court of Appeals.

333
E. EMPLOYMENT OF HOMEWORKERS

F. EMPLOYMENT OF NON-RESIDENT ALIENS

G. EMPLOYMENT OF STUDENTS & WORKING SCHOLAR

H. EMPLOYMENT OF ACADEMIC/NON-ACADEMIC PERSONNEL IN


PRIVATE EDUCATIONAL INSTITUTION

Gr. No. 170388, September 4, 2013

COLEGIO DEL SANTISIMO ROSARIO, ET. AL.


VS.
ROJO
CITING MERCADO, ET.AL. VS AMA- PARANAQUE CITY
GR. NO. 183572, APRIL 13, 2010

Facts:

Petitioner CSR hired respondent as a High School teacher from on probationary basis from 1992-
1995. On April 5, 1995, petitioner thru Sr. Monfada decided not to renew respondent’s
appointment.
Respondent filed a case of illegal dismissal. She contended that she has served the probationary
status for three consecutive school years, the maximum allowed by law and thus should be given a
regular appointment.
Petitioners contended that the probationary status merely expired and was not renewed. They
claimed that the “three years” in the 1970 Manual refers to 36 months not three school years.
Respondent served for 30 months not for 36 months.

Ruling of the LA: In favor of respondent.

Ruling of the NLRC: Affirmed the Labor Arbiter with modification. It ruled that after serving three
consecutive years, respondent attained the status of regular employment especially because CSR
did not make known to respondent the reasonable standard he should meet.

Ruling of the CA: Respondent attained a regular status.

Issue: Whether there was grave abuse of discretion

Held:

334
In Mercado et. al., vs. AMA- Paranaque City, it was ruled that cases dealing with probationary status
of teaching personnel, it is not solely governed by the Labor Code, as it is supplemented by the
Manual of Regulations for Private Schools.
Sec. 92. Probationary Period. – Subject in all instances to compliance with the
Department and the school requirements, the probationary period for academic personnel
shall not be more than three consecutive years of satisfactory service for elementary and
high school, six regular semesters of satisfactory service for those in tertiary level, nine
consecutive trimesters of satisfactory service for tertiary level where collegiate courses are offered
on a trimester basis.
In this case, petitioners’ teachers who were on probationary employment were made to enter into a
contract effective for one school year. Thereafter, it may be renewed for another school year, and
the probationary employment continues. At the end of the second fixed period of probationary
employment, the contract may again be renewed for the last time.
Such employment for fixed terms during the teachers’ probationary period is an accepted practice
in the teaching profession as enunciated in Magis Young Achievers’ Learning Center v. Manalo. Full-
time teachers become regular or permanent employees once they have satisfactorily completed the
probationary period of three school years. The use of the term satisfactorily necessarily connotes
the requirement for schools to set reasonable standards to be followed by teachers on probationary
employment.
In Mercado, this Court, speaking through J. Brion, held that:
The provision on employment on probationary status under the Labor Code is a primary example of
the fine balancing of interests between labor and management.
On the one hand, employment on probationary status affords management the chance to fully
scrutinize the true worth of hired personnel before the full force of the security of tenure guarantee
of the Constitution comes into play. Based on the standards set at the start of the probationary
period, management is given the widest opportunity during the probationary period to reject hirees
who fail to meet its own adopted but reasonable standards.
Labor, for its part, is given the protection during the probationary period of knowing the company
standards the new hires have to meet during the probationary period, and to be judged on the basis
of these standards, aside from the usual standards applicable to employees after they achieve
permanent status. Under the terms of the Labor Code, these standards should be made known to
the teachers on probationary status at the start of their probationary period, or at the very least
under the circumstances of the present case, at the start of the semester or the trimester during
which the probationary standards are to be applied. Of critical importance in invoking a failure to
meet the probationary standards, is that the school should show – as a matter of due process – how
these standards have been applied. This is effectively the second notice in a dismissal situation that
the law requires as a due process guarantee supporting the security of tenure provision, and is in
furtherance, too, of the basic rule in employee dismissal that the employer carries the burden of
justifying a dismissal. These rules ensure compliance with the limited security of tenure guarantee
the law extends to probationary employees.
For teachers on probationary employment, in which case a fixed term contract is not specifically
used for the fixed term it offers, it is incumbent upon the school to have not only set reasonable
standards to be followed by said teachers in determining qualification for regular

335
employment, the same must have also been communicated to the teachers at the start of the
probationary period, or at the very least, at the start of the period when they were to be
applied. These terms, in addition to those expressly provided by the Labor Code, would serve as the
just cause for the termination of the probationary contract. The specific details of this finding of just
cause must be communicated to the affected teachers as a matter of due process.
Corollarily, should the teachers not have been apprised of such reasonable standards at the
time specified above, they shall be deemed regular employees.
In this case, glaringly absent from petitioners’ evidence are the reasonable standards that
respondent was expected to meet that could have served as proper guidelines for purposes of
evaluating his performance. Nowhere in the Teacher’s Contract could such standards be found.
Neither was it mentioned that the same were ever conveyed to respondent. Even assuming that
respondent failed to meet the standards set forth by CSR and made known to the former at the time
he was engaged as a teacher on probationary status, still, the termination was flawed for failure to
give the required notice to respondent.
Hence, petition is denied.

Gr. No. 193897, January 23, 2013

UNIVERSITY OF THE EAST


VS.
PEPANIO

Facts:
In 1992, DECS issued the Revised Manual of Regulations for Private School which requires faculty
members to have a Masters Degree as a minimum requirement to attain a regular status.

In 1994, UE and its Faculty Association entered into a Collective Bargaining Agreement (CBA) that
UE shall extend only semester-to-semester appointments to college faculty staffs who did not
possess the minimum qualifications. Those with such qualifications shall be given probationary
appointments and their performance on a full-time or full-load basis shall be reviewed for four
semesters.

On February 1996, DECS together with other Government agencies issued a joint order mandating
that “teaching or academic personnel who do not meet the minimum academic qualifications shall
not acquire tenure or regular status.”

UE hired respondent Mariti D. Bueno in 1997 and respondent Analiza F. Pepanio in 2000, both on a
semester-to-semester basis to teach in its college. They could not qualify for probationary or
regular status because they lacked postgraduate degrees. Bueno enrolled in six postgraduate
subjects at the Philippine Normal University’s graduate school but there is no evidence that she
finished her course. Pepanio earned 27 units in her graduate studies at the Gregorio Araneta
University Foundation but these could no longer be credited to her because she failed to continue
with her studies within five years.

336
In 2001 UE and the UE Faculty Association entered into a new CBA that would have the school
extend probationary full-time appointments to full-time faculty members who did not yet have the
required postgraduate degrees provided that the latter comply with such requirement within their
probationary period. The CBA granted UE, however, the option to replace these appointees during
their probationary period if a qualified teacher becomes available at the end of the semester.

In 2003, UE CAS Dean sent notices to Pepanio and Bueno reminding them that their probationary
status is about to expire pursuant to the new CBA of those who lack qualification. Pepanio replied
that she was enrolled in PUP Graduate School while Bueno replied that she lacked interest in
acquiring security of tenure.

The CAS Dean wrote Pepanio and Bueno stating that she is extending for two semesters the
probationary appointments to them. Pepanio asked that she be given three semesters extension
while Bueno did not. CAS DEAN did not grant the request of Pepanio. The two teachers did not then
report for work. Bueno later wrote UE, demanding that it consider her a regular employee based on
her six-and-a-half-year service on a full-load basis, given that UE hired her in 1997 when what was
in force was still the 1994 CBA. Pepanio made the same demand, citing her three-and-a-half years of
service on a full-load basis. When UE did not heed their demands, respondents filed cases of illegal
dismissal against the school before the Labor Arbiter’s (LA) office. For its defense, UE countered
that it never regarded respondents as regular employees since they did not hold the required
master’s degree that government rules required as minimum educational qualification for their
kind of work.

Ruling of Labor Arbiter: Bueno and Pepanio were regular employees.

Ruling of the NLRC: Reversed LA

Ruling of CA: Reinstated LA’s decision

ISSUE: Whether the two teachers were illegally dismissed

HELD:
The policy requiring postgraduate degrees of college teachers was provided in the Manual of
Regulations as early as 1992. Indeed, recognizing this, the 1994 CBA provided even then that UE
was to extend only semester- to-semester appointments to college faculty staffs, like respondents,
who did not possess the minimum qualifications for their positions.
A school CBA must be read in conjunction with statutory and administrative regulations governing
faculty qualifications. Such regulations form part of a valid CBA without need for the parties to
make express reference to it. While the contracting parties may establish such stipulations, clauses,
terms and conditions, as they may see fit, the right to contract is still subject to the limitation that
the agreement must not be contrary to law or public policy.
Respondents were each given only semester-to-semester appointments from the beginning of their

337
employment with UE precisely because they lacked the required master's degree. It was only when
UE and the faculty union signed their 2001 CBA that the school extended petitioners a conditional
probationary status subject to their obtaining a master's degree within their probationary period. It
is clear, therefore, that the parties intended to subject respondents' permanent status appointments
to the standards set by the law and the university.
Here, UE gave respondents Bueno and Pepanio more than ample opportunities to acquire the
postgraduate degree required of them. But they did not take advantage of such opportunities.
Justice, fairness, and due process demand that an employer should not be penalized for the
situation.

I. MEDICAL, DENTAL AND OCCUPATIONAL SAFETY (Arts. 156 to 165)

J. MIGRANT WORKER’S ACT/ RECRUITMENT AND PLACEMENT

G.R. No. 179532 May 30, 2011

CLAUDIO S. YAP
VS.
THENAMARIS SHIP'S MANAGEMENT AND INTERMARE MARITIME AGENCIES, INC.

Facts:

Claudio S. Yap was employed as electrician of the vessel, M/T SEASCOUT on 14 August 2001 by
Intermare Maritime Agencies, Inc. in behalf of its principal, Vulture Shipping Limited. On Nov 2001,
however, Yap along with other crewmembers, was informed by the Master of their vessel that the
same was sold and will be scrapped. They were also informed about the Advisory sent by Capt.
Constatinou, which states, among others:
" …PLEASE ASK YR OFFICERS AND RATINGS IF THEY WISH TO BE TRANSFERRED TO OTHER
VESSELS AFTER VESSEL S DELIVERY (GREEK VIA ATHENS-PHILIPINOS VIA MANILA…
…FOR CREW NOT WISH TRANSFER TO DECLARE THEIR PROSPECTED TIME FOR
REEMBARKATION IN ORDER TO SCHEDULE THEM ACCLY…"
Yap received his seniority bonus, vacation bonus, extra bonus along with the scrapping bonus.
However, with respect to the payment of his wage, he refused to accept the payment of one-month
basic wage. He insisted that he was entitled to the payment of the unexpired portion of his contract
since he was illegally dismissed from employment. He alleged that he opted for immediate transfer
but none was made.

Issues:

338
- the constitutionality of the 5th paragraph of Section 10 of R.A. No. 8042 and,
- necessarily, the proper computation of the lump-sum salary to be awarded to petitioner by
reason of his illegal dismissal

Ruling:

Verily, we have already declared in Serrano that the clause "or for three months for every year of
the unexpired term, whichever is less" provided in the 5th paragraph of Section 10 of R.A. No. 8042
is unconstitutional for being violative of the rights of Overseas Filipino Workers (OFWs) to equal
protection of the laws. In an exhaustive discussion of the intricacies and ramifications of the said
clause, this Court, in Serrano, pertinently held:
The Court concludes that the subject clause contains a suspect classification in that, in the
computation of the monetary benefits of fixed-term employees who are illegally discharged, it
imposes a 3-month cap on the claim of OFWs with an unexpired portion of one year or more in their
contracts, but none on the claims of other OFWs or local workers with fixed-term employment. The
subject clause singles out one classification of OFWs and burdens it with a peculiar disadvantage.27
Moreover, this Court held therein that the subject clause does not state or imply any definitive
governmental purpose; hence, the same violates not just therein petitioner’s right to equal
protection, but also his right to substantive due process under Section 1, Article III of the
Constitution.28 Consequently, petitioner therein was accorded his salaries for the entire unexpired
period of nine months and 23 days of his employment contract, pursuant to law and jurisprudence
prior to the enactment of R.A. No. 8042.
As a general rule, an unconstitutional act is not a law; it is inoperative as if it has not been passed at
all. The doctrine of operative fact serves as an exception to the aforementioned general rule. The
doctrine is applicable when a declaration of unconstitutionality will impose an undue burden on
those who have relied on the invalid law.
Following Serrano, we hold that this case should not be included in the aforementioned exception.
After all, it was not the fault of petitioner that he lost his job due to an act of illegal dismissal
committed by respondents. To rule otherwise would be iniquitous to petitioner and other OFWs,
and would, in effect, send a wrong signal that principals/employers and recruitment/manning
agencies may violate an OFW’s security of tenure which an employment contract embodies and
actually profit from such violation based on an unconstitutional provision of law.

G.R. No. 175558, February 8, 2012

SKIPPERS UNITED PACIFIC, INC. AND SKIPPERS MARITIME SERVICES, INC., LTD.
VS.
NATHANIEL DOZA, NAPOLEON DE GRACIA, ISIDRO L. LATA, AND CHARLIE APROSTA

Facts:
This arose from consolidated labor case4 filed by seafarers Napoleon De Gracia (De Gracia), Isidro L.
Lata (Lata), Charlie Aprosta (Aprosta), and Nathaniel Doza (Doza) against local manning agency
Skippers United Pacific, Inc. and its foreign principal, Skippers Maritime Services, Inc., Ltd.

339
(Skippers) for unremitted home allotment for the month of December 1998, salaries for the
unexpired portion of their employment contracts, moral damages, exemplary damages, and
attorney’s fees.

Skippers United Pacific, Inc. deployed, in behalf of Skippers, De Gracia, Lata, and Aprosta to work on
board the vessel MV Wisdom Star. Paragraph 2 of all the employment contracts stated that: "The
terms and conditions of the Revised Employment Contract Governing the Employment of All
Seafarers approved per Department Order No. 33 and Memorandum Circular No. 55, both series of
1996 shall be strictly and faithfully observed."17 No employment contract was submitted for
Nathaniel Doza.

De Gracia, et al. claimed that Skippers failed to remit their respective allotments for almost five
months, compelling them to air their grievances with the Romanian Seafarers Free Union.18 On 16
December 1998, ITF Inspector Adrian Mihalcioiu of the Romanian Seafarers Union sent Captain
Savvas of Cosmos Shipping a fax letter, relaying the complaints of his crew, namely: home allotment
delay, unpaid salaries (only advances), late provisions, lack of laundry services (only one washing
machine), and lack of maintenance of the vessel (perforated and unrepaired deck). 19 To date,
however, Skippers only failed to remit the home allotment for the month of December 1998. 20On 28
January 1999, De Gracia, et al. were unceremoniously discharged from MV Wisdom Stars and
immediately repatriated.21 Upon arrival in the Philippines, De Gracia, et al. filed a complaint for
illegal dismissal with the Labor Arbiter on 4 April 1999 and prayed for payment of their home
allotment for the month of December 1998, salaries for the unexpired portion of their contracts,
moral damages, exemplary damages, and attorney’s fees.22

Skippers, on the other hand, claims that at around 2:00 a.m. on 3 December 1998, De Gracia,
smelling strongly of alcohol, went to the cabin of Gabriel Oleszek, Master of MV Wisdom Stars, and
was rude, shouting noisily to the master.23 De Gracia left the master’s cabin after a few minutes and
was heard shouting very loudly somewhere down the corridors.24 This incident was evidenced by
the Captain’s Report sent via telex to Skippers on said date.25

Skippers also claims that at 12:00 noon on 22 January 1999, four Filipino seafarers, namely
Aprosta, De Gracia, Lata and Doza, arrived in the master’s cabin and demanded immediate
repatriation because they were not satisfied with the ship.26 De Gracia, et al. threatened that they
may become crazy any moment and demanded for all outstanding payments due to them.27 This is
evidenced by a telex of Cosmoship MV Wisdom to Skippers, which however bears conflicting dates
of 22 January 1998 and 22 January 1999.28

Issues:
Skippers, in its Petition for Review on Certiorari, assigned the following errors in the CA Decision:
a) The Court of Appeals seriously erred in not giving due credence to the master’s telex message
showing that the respondents voluntarily requested to be repatriated.

340
b) The Court of Appeals seriously erred in finding petitioners liable to pay backwages and the
alleged unremitted home allotment pay despite the finding of the Labor Arbiter and the NLRC that
the claims are baseless.
c) The Court of Appeals seriously erred in awarding attorney’s fees in favor of respondents despite
its findings that the facts attending in this case do not support the claim for moral and exemplary
damages.55

Ruling:
For a worker’s dismissal to be considered valid, it must comply with both procedural and
substantive due process. The legality of the manner of dismissal constitutes procedural due
process, while the legality of the act of dismissal constitutes substantive due process.56
Procedural due process in dismissal cases consists of the twin requirements of notice and hearing.
The employer must furnish the employee with two written notices before the termination of
employment can be effected: (1) the first notice apprises the employee of the particular acts or
omissions for which his dismissal is sought; and (2) the second notice informs the employee of the
employer’s decision to dismiss him. Before the issuance of the second notice, the requirement of a
hearing must be complied with by giving the worker an opportunity to be heard. It is not necessary
that an actual hearing be conducted.57

Substantive due process, on the other hand, requires that dismissal by the employer be made under
a just or authorized cause under Articles 282 to 284 of the Labor Code.

In this case, there was no written notice furnished to De Gracia, et al. regarding the cause of their
dismissal. Cosmoship furnished a written notice (telex) to Skippers, the local manning agency,
claiming that De Gracia, et al. were repatriated because the latter voluntarily pre-terminated their
contracts. This telex was given credibility and weight by the Labor Arbiter and NLRC in deciding
that there was pre-termination of the employment contract "akin to resignation" and no illegal
dismissal. However, as correctly ruled by the CA, the telex message is "a biased and self-serving
document that does not satisfy the requirement of substantial evidence." If, indeed, De Gracia, et al.
voluntarily pre-terminated their contracts, then De Gracia, et al. should have submitted their
written resignations.

Article 285 of the Labor Code recognizes termination by the employee of the employment contract
by "serving written notice on the employer at least one (1) month in advance." Given that provision,
the law contemplates the requirement of a written notice of resignation. In the absence of a written
resignation, it is safe to presume that the employer terminated the seafarers. In addition, the telex
message relied upon by the Labor Arbiter and NLRC bore conflicting dates of 22 January 1998 and
22 January 1999, giving doubt to the veracity and authenticity of the document. In 22 January 1998,
De Gracia, et al. were not even employed yet by the foreign principal. For these reasons, the
dismissal of De Gracia, et al. was illegal.

On the issue of damages

341
As admitted by Skippers in its Position Paper, the home allotment pay for December 1998 due to De
Gracia, Lata and Aprosta is:
Seafarer Home Allotment Pay

De Gracia US$900.00

Aprosta US$600.00

Lata US$600.00

The monthly salary of De Gracia, according to his employment contract, is only US$800.00.
However, since Skippers admitted in its Position Paper a higher home allotment pay for De Gracia,
we award the higher amount of home allotment pay for De Gracia in the amount of US$900.00.
Since the home allotment pay can be considered as unpaid salaries, the peso equivalent of the dollar
amount should be computed using the prevailing rate at the time of termination since it was due
and demandable to De Gracia, et al. on 28 January 1999.
Section 10 of Republic Act No. 8042 (Migrant Workers Act) provides for money claims in cases of
unjust termination of employment contracts:
In case of termination of overseas employment without just, valid or authorized cause as defined by
law or contract, the workers shall be entitled to the full reimbursement of his placement fee with
interest of twelve percent (12%) per annum, plus his salaries for the unexpired portion of his
employment contract or for three (3) months for every year of the unexpired term, whichever is
less.

The Migrant Workers Act provides that salaries for the unexpired portion of the employent contract
or three (3) months for every year of the unexpired term, whichever is less, shall be awarded to the
overseas Filipino worker, in cases of illegal dismissal. However, in 24 March 2009, Serrano v.
Gallant Maritime Services and Marlow Navigation Co. Inc.,58 the Court, in an En Banc Decision,
declared unconstitutional the clause "or for three months for every year of the unexpired term,
whichever is less" and awarded the entire unexpired portion of the employment contract to the
overseas Filipino worker.

On 8 March 2010, however, Section 7 of Republic Act No. 10022 (RA 10022) amended Section 10 of
the Migrant Workers Act, and once again reiterated the provision of awarding the unexpired
portion of the employent contract or three (3) months for every year of the unexpired term,
whichever is less.

Nevertheless, since the termination occurred on January 1999 before the passage of the
amendatory RA 10022, we shall apply RA 8042, as unamended, without touching on the
constitutionality of Section 7 of RA 10022.
The declaration in March 2009 of the unconstitutionality of the clause "or for three months for
every year of the unexpired term, whichever is less" in RA 8042 shall be given retroactive effect to
the termination that occurred in January 1999 because an unconstitutional clause in the law

342
confers no rights, imposes no duties and affords no protection. The unconstitutional provision is
inoperative, as if it was not passed into law at all.59

As such, we compute the claims as follows:


Seafarer Contract Contract Repatriation Unexpired Monthly Total
Term Date Date Term Salary Claims

De 10 months 17 Jul. 28 Jan. 1999 3 months & 20 US$800 US$2933.34


Gracia 1998 days

Lata 12 months 17 Apr. 28 Jan. 1999 2 months & 20 US$600 US$1600


1998 days

Aprosta 12 months 17 Apr. 28 Jan. 1999 2 months & 20 US$600 US$1600


1998 days
Given the above computation, we modify the CA’s imposition of award, and grant to De Gracia, et al.
salaries representing the unexpired portion of their contracts, instead of salaries for three (3)
months.

GR 197528 September 5, 2012

PERT/CPM MANPOWER EXPONENT CO.


VS
VINUYA ET. AL

FACTS:
Respondents Armando A. Vinuya, Louie M. Ordovez, Arsenio S. Lumanta, Jr., Robelito S. Anipan,
Virgilio R. Alcantara, Marino M. Era, Sandy O. Enjambre and Noel T. Ladea (respondents) filed a
complaint for illegal dismissal against the petitioner Pert/CPM Manpower Exponent Co., Inc.
(AGENCY), and its President Romeo P. Nacino.
The respondents alleged that the AGENCY deployed them between March 29, 2007 and May 12,
2007 to work as aluminum fabricator/installer for the agency’s principal, Modern Metal Solution
LLC/MMS Modern Metal Solution LLC (Modern Metal) in Dubai, United Arab Emirates.
Respondents’ employment contracts were approved by the Philippine Overseas Employment
Administration (POEA). The employment contracts provided for a two-year employment, nine
hours a day, salary of 1,350 AED with overtime pay, food allowance, free and suitable housing (four
to a room), free transportation, free laundry, and free medical and dental services. They each paid
a P 15,000.00 processing fee.
Modern Metal gave respondents, except Era, appointment letters with different terms from those
employment contracts which they signed at the agency’s office in the Philippines. Under these

343
letters of appointment, their employment was increased from 2 years to 3 years and the salary was
reduced from 1,350 AED to 1000-1200 AED with food allowance of 200 AED.
The respondents claimed that they were shocked to find out what their working and living
conditions were in Dubai. They were required to work from 6:30 a.m. to 6:30 p.m. (but their
contract which was approved by POEA provided that they would only work 9 hours a day), with a
break of only one hour to one and a half hours. When they rendered overtime work, they were most
of the time either underpaid or not paid at all. Their housing accommodations were cramped and
were shared with 27 other occupants. The lodging house was in Sharjah, which was far from their
jobsite in Dubai, leaving them only three to four hours of sleep a day because of the long hours of
travel to and from their place of work; there was no potable water and the air was polluted.
Respondents called the AGENCY to complain about their predicament but the agency took no
action.
On May 5, 2007, Modern Metal required respondents to sign new employment contracts which now
contained the terms stated under their appointment letters. Burdened by all the expenses and
financial obligations they incurred, they were left with no choice but to sign these employment
contracts. Again herein petitioner agency took no action on the matter.
On August 5, 2007, respondents expressed to Modern Metal their desire to resign. Due to fear that
they would not be given their salaries and release papers, respondents cited personal/family
problems as reasons for their resignation except Era who mentioned the real reason – “because I
don’t want the company policy.”
It took the agency several weeks to repatriate the respondents to the Philippines. They all returned
to Manila in September 2007. Except for Ordovez and Enjambre, all the respondents shouldered
their own airfare.

ISSUES:
1. Whether or not Modern Metal and petitioner PERT/CPM Manpower Exponent Co. are guilty
of contract substitution
2. Whether or not Modern Metal and petitioner PERT/CPM Manpower Exponent Co.
committed breach of contract
3. Whether or not Modern Metal committed constructive dismissal
4. Whether or not the compromise agreements made before the POEA already settled the
employer-employee related claims of respondents
5. Whether or not the Serrano Ruling should prevail over RA 10022

RULING:
We find no merit in the petition. The CA committed no reversible error and neither did it
commit grave abuse of discretion in affirming the NLRC’s illegal dismissal ruling.
The agency and its principal, Modern Metal, committed flagrant violations of the law on overseas
employment, as well as basic norms of decency and fair play in an employment relationship,
pushing the respondents to look for a better employment and, ultimately, to resign from their jobs.
First. The agency and Modern Metal are guilty of contract substitution. The fact that the
respondents’ contracts were altered or substituted at the workplace had never been denied by the
agency. On the contrary, it admitted that the contract substitution did happen when it argued, "as to

344
their claim for underpayment of salary, their original contract mentioned 1350 AED monthly salary,
which includes allowance while in their Appointment Letters, they were supposed to receive 1,300
AED. While there was a difference of 50 AED monthly, the same could no longer be claimed by
virtue of their Affidavits of Quitclaims and Desistance."
Clearly, the agency and Modern Metal committed a prohibited practice and engaged in illegal
recruitment under the law. Article 34 of the Labor Code provides:
Art. 34. Prohibited Practices. It shall be unlawful for any individual, entity, licensee, or holder of
authority:
xxxx
(i) To substitute or alter employment contracts approved and verified by the Department of Labor
from the time of actual signing thereof by the parties up to and including the periods of expiration
of the same without the approval of the Secretary of Labor.
Further, Article 38 of the Labor Code, as amended by R.A. 8042, defined "illegal recruitment" to
include the following act:
(i) To substitute or alter to the prejudice of the worker, employment contracts approved and
verified by the Department of Labor and Employment from the time of actual signing thereof by the
parties up to and including the period of the expiration of the same without the approval of the
Department of Labor and Employment.
Second. The agency and Modern Metal committed breach of contract. Aggravating the contract
substitution imposed upon them by their employer, the respondents were made to suffer
substandard (shocking, as they put it) working and living arrangements. Both the original
contracts the respondents signed in the Philippines and the appointment letters issued to them by
Modern Metal in Dubai provided for free housing and transportation to and from the jobsite. The
original contract mentioned free and suitable housing. Although no description of the housing was
made in the letters of appointment except: "Accommodation: Provided by the company," it is but
reasonable to think that the housing or accommodation would be "suitable."
As earlier pointed out, the respondents were made to work from 6:30 a.m. to 6:30 p.m., with a meal
break of one to one and a half hours, and their overtime work was mostly not paid or underpaid.
Their living quarters were cramped as they shared them with 27 other workers. The lodging house
was in Sharjah, far from the jobsite in Dubai, leaving them only three to four hours of sleep every
workday because of the long hours of travel to and from their place of work, not to mention that
there was no potable water in the lodging house which was located in an area where the air was
polluted. The respondents complained with the agency about the hardships that they were
suffering, but the agency failed to act on their reports. Significantly, the agency failed to refute their
claim, anchored on the ordeal that they went through while in Modern Metal’s employ.
Third. With their original contracts substituted and their oppressive working and living conditions
unmitigated or unresolved, the respondents’ decision to resign is not surprising. They were
compelled by the dismal state of their employment to give up their jobs; effectively, they were
constructively dismissed. A constructive dismissal or discharge is "a quitting because continued
employment is rendered impossible, unreasonable or unlikely, as, an offer involving a demotion in
rank and a diminution in pay."
The respondents’ position is well-founded. The NLRC itself had the same impression, which we
found worth stressing and hereunder quote:

345
The acts of respondents of requiring the signing of new contracts upon reaching the place of
work and requiring employees to sign quitclaims before they are paid and repatriated to the
Philippines are all too familiar stories of despicable labor practices which our employees are
subjected to abroad. While it is true that quitclaims are generally given weight, however, given the
facts of the case, We are of the opinion that the complainants-appellants executed the same under
duress and fear that they will not be allowed to return to the Philippines.
Fourth. The compromise agreements (with quitclaim and release) between the respondents and
the agency before the POEA did not foreclose their employer-employee relationship claims before
the NLRC. The respondents, except Ordovez and Enjambre, aver in this respect that they all paid for
their own airfare when they returned home and that the compromise agreements settled only their
claim for refund of their airfare, but not their other claims. Again, this submission has not been
refuted or denied by the agency.
On the surface, the compromise agreements appear to confirm the agency’s position, yet a closer
examination of the documents would reveal their true nature. Copy of the compromise agreement
is a standard POEA document, prepared in advance and readily made available to parties who are
involved in disputes before the agency, such as what the respondents filed with the POEA ahead
(filed in 2007) of the illegal dismissal complaint before the NLRC (filed on March 5, 2008).
Under the heading "Post-Deployment," the agency agreed to pay Era and Alcantara P 12,000.00
each, purportedly in satisfaction of the respondents’ claims arising from overseas employment,
consisting of unpaid salaries, salary differentials and other benefits, including money claims with
the NLRC. The last document was signed by (1) Anipan, (2) Lumanta, (3) Ladea, (4) Vinuya, (5)
Jonathan Nangolinola, and (6) Zosimo Gatchalian (the last four signing on the left hand side of the
document; the last two were not among those who filed the illegal dismissal complaint).
The agency agreed to pay them a total of P 72,000.00. Although there was no breakdown of the
entitlement for each of the six, but guided by the compromise agreement signed by Era and
Alcantara, we believe that the agency paid them P 12,000.00 each, just like Era and Alcantara.
The uniform insubstantial amount for each of the signatories to the agreement lends credence to
their contention that the settlement pertained only to their claim for refund of the airfare which
they shouldered when they returned to the Philippines. The compromise agreement, apparently,
was intended by the agency as a settlement with the respondents and others with similar claims,
which explains the inclusion of the two (Nangolinola and Gatchalian) who were not involved in the
case with the NLRC. Under the circumstances, we cannot see how the compromise agreements can
be considered to have fully settled the respondents’ claims before the NLRC — illegal dismissal and
monetary benefits arising from employment. We thus find no reversible error nor grave abuse of
discretion in the rejection by the NLRC and the CA of said agreements.
Fifth. The agency’s objection to the application of the Serrano ruling in the present case is of no
moment. Its argument that the ruling cannot be given retroactive effect, because it is curative and
remedial, is untenable. It points out, in this respect, that the respondents filed the complaint in
2007, while the Serrano ruling was handed down in March 2009. The issue, as the respondents
correctly argue, has been resolved in Yap v. Thenamaris Ship’s Management, where the Court
sustained the retroactive application of the Serrano ruling which declared unconstitutional the
subject clause in Section 10, paragraph 5 of R.A. 8042, limiting to three months the payment of
salaries to illegally dismissed Overseas Filipino Workers.

346
Despite such, petitioner posits that in any event, the Serrano ruling has been nullified by R.A. No.
10022, entitled "An Act Amending Republic Act No. 8042, Otherwise Known as the Migrant
Workers and Overseas Filipinos Act of 1995, As Amended, Further Improving the Standard of
Protection and Promotion of the Welfare of Migrant Workers, Their Families and Overseas Filipinos
in Distress, and For Other Purposes." It argues that R.A. 10022, which lapsed into law (without the
Signature of the President) on March 8, 2010, restored the subject clause in the 5th paragraph,
Section 10 of R.A. 8042. The amendment, contained in Section 7 of R.A. 10022, reads as follows:
In case of termination of overseas employment without just, valid or authorized cause as defined by
law or contract, or any unauthorized deductions from the migrant worker’s salary, the worker shall
be entitled to the full reimbursement "of" his placement fee and the deductions made with interest
at twelve percent (12%) per annum, plus his salaries for the unexpired portion of his employment
contract or for three (3) months for every year of the unexpired term, whichever is less. (emphasis
ours)
This argument fails to persuade us. Laws shall have no retroactive effect, unless the contrary is
provided. By its very nature, the amendment introduced by R.A. 10022 — restoring a provision of
R.A. 8042 declared unconstitutional — cannot be given retroactive effect, not only because there is
no express declaration of retroactivity in the law, but because retroactive application will result in
an impairment of a right that had accrued to the respondents by virtue of the Serrano ruling -
entitlement to their salaries for the unexpired portion of their employment contracts.
All statutes are to be construed as having only a prospective application, unless the purpose and
intention of the legislature to give them a retrospective effect are expressly declared or are
necessarily implied from the language used. We thus see no reason to nullity the application of the
Serrano ruling in the present case. Whether or not R.A. 10022 is constitutional is not for us to rule
upon in the present case as this is an issue that is not squarely before us. In other words, this is an
issue that awaits its proper day in court; in the meanwhile, we make no pronouncement on it.

GR 163657

INTERNATIONAL MANAGEMENT SERVICES/MARILYN PASCUAL


VS
ROEL LOGARTA

FACTS:
Petitioner International Management Services (IMS), owned by Marilyn Pascual, deployed
respondent Roel Logarta to work for Petrocon in connection with services of Petrocon for Saudi
Arabian Oil Company (Saudi Aramco).
Logarta was hired as a Piping Designer for a period of 2 years commencing October 2, 1997.
Saudi Aramco, due to changes in the general engineering services work forecast for 1998, reduced
the originally allotted man-hours by 40%. Consequently, due to the considerable decrease in the
work requirements of Saudi Aramco, Petrocon was constrained to reduce its personnel that were
employed as piping designers, instrument engineers, inside plant engineers, etc., which totaled to
some 73 personnel including herein respondent Logarta.

347
On June 1, 1998, Petrocon gave respondent Logarta a written notice informing the latter that due to
the lack of project works related to his expertise, he is given a 30-day notice of termination, and
that his last day of work with Petrocon will be on July 1, 1998.
Upon his return to the Philippines, respondent filed a complaint with the Regional Arbitration
Branch VII, National Labor Relations Commission (NLRC), Cebu City, against petitioner as the
recruitment agency which employed him for employment abroad. In filing the complaint,
respondent sought to recover his unearned salaries covering the unexpired portion of his
employment contract with Petrocon on the ground that he was illegally dismissed.
Labor Arbiter ruled in favor of respondent. US$ 5,600 awarded to respondent as payment for wages
for the unexpired portion.
NLRC affirmed Labor Arbiter’s decision but modified the amount. US$ 4,800 awarded to
respondent.
CA agreed with the findings of NLRC that retrenchment could be a valid cause to terminate
respondent’s employment with Petrocon. However, although there was a valid retrenchment, the
same was implemented without complying with the requisites of a valid retrenchment. Also, the CA
concluded that although the respondent was given a 30-day notice of his termination, there was no
showing that the Department of Labor and Employment (DOLE) was also sent a copy of the said
notice as required by law. Moreover, the CA found that a perusal of the check payroll details would
readily show that respondent was not paid his separation pay.

ISSUES:
1. Petitioner contended that the 30-day notice of termination as required in Serrano vs NLRC
is not applicable to the case at bar considering that, in fact, respondent was given the 30-day notice.
RA 8042 or the Migrant Worker’s Act and its implementing rules do not require the sending of
notice to DOLE thirty (30) days before the effectivity of the retrenchment of an OFW.
2. Petitioner also posits that the CA should have applied the case of Jariol v. IMS even if the said
case was only decided by the NLRC, a quasi-judicial agency. The said case involved similar facts,
wherein the NLRC categorically ruled that employers of OFWs are not required to furnish the DOLE
in the Philippines a notice if they intend to terminate a Filipino employee.
3. Lastly, petitioner insists that respondent received his separation pay. Moreover, petitioner
contends that Section 10 of R.A. No. 8042 does not apply in the present case, since the termination
of respondent was due to a just, valid or authorized cause. At best, respondent is only entitled to
separation pay in accordance with Article 283 of the Labor Code, i.e., one (1) month pay or at least
one-half (1/2) month pay for every year of service, whichever is higher.

RULING:
1. In the case at bar, despite the fact that respondent was employed by Petrocon as an OFW in
Saudi Arabia, still both he and his employer are subject to the provisions of the Labor Code when
applicable. The basic policy in this jurisdiction is that all Filipino workers, whether employed
locally or overseas, enjoy the protective mantle of Philippine labor and social legislations. In the
case of Royal Crown Internationale v. NLRC, this Court has made the policy pronouncement, thus:
x x x. Whether employed locally or overseas, all Filipino workers enjoy the protective mantle
of Philippine labor and social legislation, contract stipulations to the contrary

348
notwithstanding. This pronouncement is in keeping with the basic public policy of the State
to afford protection to labor, promote full employment, ensure equal work opportunities
regardless of sex, race or creed, and regulate the relations between workers and employers.
xxx
Article 283 of the Labor Code provides that retrenchment is a valid cause for dismissal.
Thus, retrenchment is a valid exercise of management prerogative subject to the strict
requirements set by jurisprudence, to wit:
(1) That the retrenchment is reasonably necessary and likely to prevent business losses which, if
already incurred, are not merely de minimis, but substantial, serious, actual and real, or if only
expected, are reasonably imminent as perceived objectively and in good faith by the employer;
(2) That the employer served written notice both to the employees and to the Department of
Labor and Employment at least one month prior to the intended date of retrenchment;
(3) That the employer pays the retrenched employees separation pay equivalent to one month pay
or at least ½ month pay for every year of service, whichever is higher;
(4) That the employer exercises its prerogative to retrench employees in good faith for the
advancement of its interest and not to defeat or circumvent the employees' right to security of
tenure; and
(5) That the employer used fair and reasonable criteria in ascertaining who would be dismissed
and who would be retained among the employees, such as status, x x x efficiency, seniority, physical
fitness, age, and financial hardship for certain workers.
As for the notice requirement, however, contrary to petitioner’s contention, proper notice to the
DOLE within 30 days prior to the intended date of retrenchment is necessary and must be complied
with despite the fact that respondent is an overseas Filipino worker. In the present case, although
respondent was duly notified of his termination by Petrocon 30 days before its effectivity, no
allegation or proof was advanced by petitioner to establish that Petrocon ever sent a notice to the
DOLE 30 days before the respondent was terminated. Thus, this requirement of the law was not
complied with.
2. Moreover, petitioner’s insistence that the case of Jariol v. IMS should be applied in the
present case is untenable. Being a mere decision of the NLRC, it could not be considered as a
precedent warranting its application in the case at bar. Suffice it to state that although Article 8 of
the Civil Code recognizes judicial decisions, applying or interpreting statutes as part of the legal
system of the country, such level of recognition is not afforded to administrative decisions.
~NLRC decisions are administrative decisions
3. Section 10 of RA 8042 states as follows:
Sec. 10. Money Claims. – x x x In case of termination of overseas employment without just, valid or
authorized cause as defined by law or contract, x x x
In this case, however, despite the fact that respondent’s termination from employment was
procedurally infirm, the same remains to be for a just, valid and authorized cause – retrenchment is
a valid exercise of management prerogative.
Consequently, RA 8042 should not be applied. Instead, it is Art. 283 of Labor Code which is
controlling. Thus, respondent is entitled to payment of separation pay equivalent to one (1) month
pay, or at least one-half (1/2) month pay for every year of service, whichever is higher. Considering
that respondent was employed by Petrocon for a period of eight (8) months, he is entitled to receive

349
one (1) month pay as separation pay. In addition, pursuant to current jurisprudence, for failure to
fully comply with the statutory due process of sufficient notice, respondent is entitled to nominal
damages in the amount P50,000.00.

Petitioner is ORDERED to pay Roel P. Logarta one (1) month salary as separation pay
and P50,000.00 as nominal damages.

[G.R. No. 152642. November 13, 2012]

HON. STO. TOMAS


VS.
SALAC

FACTS:
On June 7, 1995 Congress enacted Republic Act (R.A.) 8042 or the Migrant Workers and Overseas
Filipinos Act of 1995 that, for among other purposes, sets the Government's policies on overseas
employment and establishes a higher standard of protection and promotion of the welfare of
migrant workers, their families, and overseas Filipinos in distress.

G.R. 152642 and G.R. 152710


(Constitutionality of Sections 29 and 30, R.A. 8042)
Sections 29 and 30 of the Act 1 commanded the Department of Labor and Employment (DOLE) to
begin deregulating within one year of its passage the business of handling the recruitment and
migration of overseas Filipino workers and phase out within five years the regulatory functions of
the Philippine Overseas Employment Administration (POEA).
On April 10, 2007 former President Gloria Macapagal-Arroyo signed into law R.A. 9422 which
expressly repealed Sections 29 and 30 of R.A. 8042 and adopted the policy of close government
regulation of the recruitment and deployment of OFWs.

ISSUE: WON Sections 29 and 30 are unconstitutional.

RULING: Issues raised have become moot and academic.

G.R. 167590
(Constitutionality of Sections 6, 7, and 9 of R.A. 8042)
FACTS: Section 6 defines the crime of "illegal recruitment" and enumerates the acts constituting the
same.

The RTC of Manila declared Section 6 unconstitutional after hearing on the ground that its
definition of "illegal recruitment" is vague as it fails to distinguish between licensed and non-
licensed recruiters 11 and for that reason gives undue advantage to the non-licensed recruiters in

350
violation of the right to equal protection of those that operate with government licenses or
authorities.

ISSUE: WON Section 6 is unconstitutional.

RULING: But "illegal recruitment" as defined in Section 6 is clear and unambiguous and, contrary to
the RTC's finding, actually makes a distinction between licensed and non-licensed recruiters. By its
terms, persons who engage in "canvassing, enlisting, contracting, transporting, utilizing, hiring, or
procuring workers" without the appropriate government license or authority are guilty of illegal
recruitment whether or not they commit the wrongful acts enumerated in that section. On the other
hand, recruiters who engage in the canvassing, enlisting, etc. of OFWs, although with the
appropriate government license or authority, are guilty of illegal recruitment only if they commit
any of the wrongful acts enumerated in Section 6.

FACTS:
Section 7 provides the penalties for prohibited acts.
The Manila RTC also declared Section 7 unconstitutional on the ground that its sweeping
application of the penalties failed to make any distinction as to the seriousness of the act committed
for the application of the penalty imposed on such violation.

ISSUE: WON Section 7 is unconstitutional.

RULING: But, in fixing uniform penalties for each of the enumerated acts under Section 6, Congress
was within its prerogative to determine what individual acts are equally reprehensible, consistent
with the State policy of according full protection to labor, and deserving of the same penalties. It is
not within the power of the Court to question the wisdom of this kind of choice.
Obviously, in fixing such tough penalties, the law considered the unsettling fact that OFWs must
work outside the country's borders and beyond its immediate protection. The law must, therefore,
make an effort to somehow protect them from conscienceless individuals within its jurisdiction
who, fueled by greed, are willing to ship them out without clear assurance that their contracted
principals would treat such OFWs fairly and humanely.

FACTS: Finally, Section 9 of R.A. 8042 allowed the filing of criminal actions arising from "illegal
recruitment" before the RTC of the province or city where the offense was committed or where the
offended party actually resides at the time of the commission of the offense.

The Manila RTC also invalidated Section 9 of R.A. 8042 on the ground that allowing the offended
parties to file the criminal case in their place of residence would negate the general rule on venue of
criminal cases which is the place where the crime or any of its essential elements were committed

ISSUE: WON Section 9 is unconstitutional.

351
RULING: Section 9 of R.A. 8042, as an exception to the rule on venue of criminal actions is,
consistent with that law's declared policy 15 of providing a criminal justice system that protects
and serves the best interests of the victims of illegal recruitment.

G.R. 167590, G.R. 182978-79, 16 and G.R. 184298-99 17


(Constitutionality of Section 10, last sentence of 2nd paragraph)
FACTS: They questioned the constitutionality of the last sentence of the second paragraph of
Section 10, R.A. 8042 which holds the corporate directors, officers and partners jointly and
solidarily liable with their company for money claims filed by OFWs against their employers and
the recruitment firms. the Quezon City RTC held as unconstitutional the last sentence of the 2nd
paragraph of Section 10 of R.A. 8042. It pointed out that, absent sufficient proof that the corporate
officers and directors of the erring company had knowledge of and allowed the illegal recruitment,
making them automatically liable would violate their right to due process of law.

ISSUE: WON Section 10 is unconstitutional.

RULING: But the Court has already held, pending adjudication of this case, that the liability of
corporate directors and officers is not automatic. To make them jointly and solidarily liable with
their company, there must be a finding that they were remiss in directing the affairs of that
company, such as sponsoring or tolerating the conduct of illegal activities. 19 In the case of Becmen
and White Falcon, 20 while there is evidence that these companies were at fault in not investigating
the cause of Jasmin's death, there is no mention of any evidence in the case against them that
intervenors Gumabay, et al., Becmen's corporate officers and directors, were personally involved in
their company's particular actions or omissions in Jasmin's case.

As a final note, R.A. 8042 is a police power measure intended to regulate the recruitment and
deployment of OFWs. Hence, in the absence of a clear and unmistakable case that the statute is
unconstitutional, the Court must uphold its validity.

27. SOCIAL LEGISLATION

[G.R. No. 165482 July 23, 2008.]

SOCIAL SECURITY COMMISSION


VS.
FAR S. ALBA

FACTS:

352
Sometime in 1991, petitioner Lamboso filed a claim for retirement benefit before the Social Security
System (SSS). However, his claim was denied on the ground that he could not qualify for monthly
pension under Republic Act (R.A.) No. 1161 5 (the Social Security Act of 1954) as he then had only
thirty-nine (39) paid contributions. On appeal, the commission ordered Respondent Far Alba to pay
to the SSS the delinquent monthly contributions of Apolonio Lamboso from June 18, 1960 to April
1973.
Alba subsequently filed a Petition for Review assigning the following errors allegedly committed by
the Commission: (1) the Order of the Commission was rendered in violation of his constitutional
rights to due process and equal protection; (2) he was not obligated by law to remit contributions
to the SSS prior to 1970 and after 1973 in the absence of employer-employee relationship; and (3)
Lamboso's claim had already prescribed.

ISSUE:
Whether an administrator could be considered an employer within the scope of the Social Security
Act of 1954

RULING:
YES. Section 8 (c), Social Security Act of 1954 (as amended by Presidential Decree [P.D.] No. 1202
and P.D. No. 1636) defines an employer as "any person, natural or juridical, domestic or foreign,
who carries on in the Philippines any trade or business, industry, undertaking, or activity of any
kind and uses the services of another person who is under his orders as regards the employment,
except the Government and any of its political subdivisions, branches or instrumentalities,
including corporations owned or controlled by the Government". Section 8 (d) defines an employee
as "any person who performs services for an employer in which either or both mental and physical
efforts are used and who receives compensation for such services where there is an employer-
employee relationship." 21

First, the Court observes that Far Alba was no ordinary administrator. He was no less than the son
of the hacienda'sowner and as such he was an owner-in-waiting prior to his father's death. He was a
member of the owner's family assigned to actively manage the operations of the hacienda. As he
stood to benefit from the hacienda's successful operation, he ineluctably took his job and his
father's wishes to heart. As emphasized by the Commission his and the owner's interests in the
business were plainly and inextricably linked by filial bond.

Second, nomenclature aside, Far Alba was not merely an administrator of the hacienda. Applying
the control test which is used to determine the existence of employer-employee relationship for
purposes of compulsory coverage under the SSS law, Far Alba is technically Lamboso's
employer. 26

Third, not to be forgotten is the definition of an employer under Article 167 (f) of the Labor Code
which deals with employees' compensation and state insurance fund. The said provision of the law
defines an employer as "any individual, partnership, firm, association, trust, corporation or legal
representative thereof". Plainly, Far Alba, as the hacienda administrator, acts as the legal

353
representative of the employer and is thus an employer within the meaning of the law liable to pay
the SS contributions.

Finally, the Court believes that Section 8 (c) of the Social Security Act of 1954 is broad enough to
include those persons acting directly or indirectly in the interest of the employer.
The Court sustains the jurisdiction of the Commission over disputes under the Social Security Act
"with respect to coverage, benefits, contributions and penalties thereon or any other matter related
thereto."

G.R. No. 162837 July 28, 2008

RODRIN
VS
GSIS ET AL

Facts:
Petitioner Marlene L. Rodrin filed a claim for compensation benefits under PD 626, as
amended, relative to the death of her husband SPO1 Felixberto M. Rodrin before the GSIS. To
bolster her claim, she submitted, among others, 1) the Line of Duty Status of the late SPO1
Felixberto M. Rodrin wherein it was declared that SPO1 Felixberto M. Rodrin, member of Silang
Municipal Police Station, Silang, Cavite and assigned as Intel Operatives was killed on or about
142130 July 2000 at Las Villas Subdivision, Biñan, Laguna while performing his assigned task, and
2) the Investigation Report dated July 17, 2000. The pertinent portion of the findings of which reads
as follows:

"Brothers Anolito Loyola, 45 years old, and Cesar Loyola, 36 years old executed their respective
corroborative sworn statements to this case. Accordingly, OOA 142100H July 2000 they were driving
their respective cars with SPO1 Felixberto Rodrin, their brother-in-law, riding in Cesar Loyola's car.
From Carmona, Cavite intending to go to Pacita Complex, San Pedro, Laguna they decided to pass
through Las Villas de Manila, Brgy San Francisco, Biñan, Laguna. At gate II, they were allegedly
permitted to enter by the duty security guards identified as ERIC MENDOZA Y CARDENAS, 26 years old
and ROGELIO TAGANAP Y DAMASO, 26 years old, upon a favor given to SPO1 Felixberto Rodrin.
However, they were stopped on their exit at gate 1 by the security guards whose service shot guns were
pointed toward their two cars, as follows: VENUSTO DIWA YDEDIL, 50 years old, RODOLFO CREDO Y
DAMASO, 21 years old, and one alias ALLAN VISTO. The Situation prompted SPO1 Felixberto Rodrin to
alight from the car and approached Rodolfo Credo. They were then asked by the security guards why
they persisted to enter gate II, despite the refusal of the guard. At this juncture, while they were
engaged in a heated altercation, Rodolfo Credo shot SPO1 Felixberto Rodrin with a shot gun, hitting
the latter on the left part of the body thereby causing his instantaneous death.”

354
The GSIS denied petitioner's claim for compensation benefits under Presidential Decree 626, as
amended, on the ground that the death of SPO1 Felixberto M. Rodrin did not arise out nor was it in
the course of his employment. Such finding was affirmed by both the ECC and CA.

Issue/s
Whether or not the death of Senior Police Officer (SPO) 1 Rodrin is compensable under the
provisions of Presidential Decree (P.D.) No. 626 as amended.

Held:
Petition granted.

For the compensability of an injury to an employee which results in his disability or death, Section
1(a), Rule III of the Amended Rules on Employees' Compensation imposes the following conditions:

1. The employee must have been injured at the place where his work required him to be;
2. The employee must have been performing his official functions; and
3. If the injury was sustained elsewhere, the employee must have been executing an order of the
employer.

The first condition has been met by petitioner. There was no question that SPO1 Rodrin was a
member of the PNP at the time of his death; and that being so, he was considered to be at his place
of work regardless of whether or not he was "on or off-duty." Both assertions are correctly based on
this Court's ruling in GSIS v. Court of Appeals that members of the national police, unless they are on
official leave, are, by the nature of their functions, technically on duty 24 hours a day, because
policemen are subject to call at any time and may be asked by their superiors or by any distressed
citizen to assist in maintaining the peace and security of the community.
Anent the second and third conditions, the GSIS, ECC and the CA found that SPO1 Rodrin, at the time
of his death, was not in the performance of his official duties pursuant to an official order from his
superior.
It is clear from the Letter-Orders dated July 10, 2000, issued by SPO1 Rodrin's superior, the Chief of
Police of Silang, that SPO1 Rodrin was directed to proceed to Carmona, Cavite and Biñan, Laguna
between July 10, 2000 and July 20, 2000 for the purpose of conducting monitoring, surveillance
and, if possible, arrest of the persons named therein. Being specifically assigned to conduct
intelligence work in Carmona and Biñan, SPO1 Rodrin is presumed to have been performing his
official duty when he was shot to death by a security guard while trying to pass though the Las
Villas de Manila subdivision in Brgy. San Francisco, Biñan, Laguna.
Section 3(m), Rule 131 of the Rules of Court provides the presumption that official duty has been
regularly performed. The said Rule treats this presumption as satisfactory unless contradicted and
overcome by other evidence.
SPO1 Rodrin's reason or reasons for intending to go to San Pedro, Laguna did not involve a purely
private matter. At the time of his death, he came from Carmona, Cavite which is a place specified in
the subject Letter-Orders. Moreover, he was killed in Biñan, which is also a place specified in the
said Letter-Orders. Furthermore, he was killed on July 14, 2000 which is within the period

355
authorized by the subject Letter-Orders for him to conduct surveillance, monitoring and arrest.
Other than the fact the SPO1 Rodrin had intended to go to San Pedro, Laguna, a place which is not
covered by his Letter-Orders, there is no basis to conclude that SPO1 Rodrin's business in going to
San Pedro was private in nature and was not related to his job as an intelligence officer of the PNP.
Intelligence work covers a broad spectrum of activities that, more often than not, would necessarily
involve secret plans or unexpected courses of action to attain its objectives. Moreover, simply
because SPO1 Rodrin was in the company of his brothers-in-law who are not members of any law
enforcement agency does not establish that the business of SPO1 Rodrin at the time of his death
was purely private in character.
With respect to the contention that San Pedro, Laguna was a place which was not covered by the
subject Letter-Orders, the Court takes cognizance of the fact that the nature of work of a police
officer who is an intelligence operative does not confine him to specific places and hours, more so
with respect to a police officer involved in intelligence work. His actions may not be
compartmentalized, as they depend to a large extent on the exigencies of the assignment given him.
Respondents cite GSIS v. CA, wherein the SC denied the grant of death compensation benefits to the
widow of the slain policeman for failing to meet the requirements in the ECC guidelines, as it was
obvious that the matter he was attending to when he was killed, that of ferrying passengers for a
fee, was intrinsically private and unofficial in nature. The same may not be said in the present case.
There is no evidence to prove the claims of respondents that the matter SPO1 Rodrin was attending
to when he was shot to death was intrinsically private and unofficial in nature.
Respondents also raise the argument that the cause of SPO1 Rodrin's death was not in any way
related to his mission as outlined in the subject Letter-Orders is not plausible.
In Employees Compensation Commission v. Court of Appeals, a police officer who was a member of
the Mandaluyong Police Station and assigned to the Pasig Provincial Jail brought his son to the
Mandaluyong Police Station for interview, because the latter was involved in a stabbing incident.
While in front of the said station, the policeman was approached by another policeman and shot
him to death. The claim for death compensation benefits by the widow of the slain police officer
was denied by the GSIS and the ECC on the ground, among others, that he was plainly acting as a
father to his son. However, the CA reversed the denial. In sustaining the CA reversal, this Court
declared that in bringing his son to the police station for questioning to shed light on a stabbing
incident, he was not merely acting as a father but as a peace officer. In the present case, evidence
shows that, at the time that SPO1 Rodrin was gunned down, he was performing his duty as a police
officer. The Investigation Report of the Biñan PNP as well as the Kusang Loob na Salaysay of both
the brothers-in-law of SPO1 Rodrin show that when the latter was shot to death he was in the
course of inquiring why the security guards of the subdivision they were passing through were
aiming their guns at them. At that point, it cannot be denied that he was caught in a situation where
he could not avoid exercising his authority and duty as policeman to maintain peace and security of
the community. While his main mission was to apprehend certain criminal elements named in the
subject Letter-Orders, he was not excused from performing his basic function as a peace officer. His
act of trying to find out the reason why the security guards were acting hostile cannot be said to be
foreign and unrelated to his job as a member of the police force.
Finally it is well to echo the Court's ruling in Employees’ Compensation Commission v. Court of
Appeals, wherein it was held that:

356
x x x in case of doubt, the sympathy of the law on social security is toward its beneficiaries, and the
law, by its own terms, requires a construction of utmost liberality in their favor. For this reason, this
Court lends a very sympathetic ear to the cries of the poor widows and orphans of police officers. If
we must demand - as we ought to – strict accountability from our policemen in safeguarding peace
and order day and night, we must also to the same extent be ready to compensate their loved ones
who, by their untimely death, are left without any means of supporting themselves.

G.R. No. 173430 July 28, 2008

GSIS
VS
CASCO

Facts:
Respondent Felomino Casco was employed as a teacher of the DECS. He joined the
government service on August 14, 1978 on a provisional status. In 1998- up to 1999, he was
assigned at the Mandaluyong East High School wherein he taught Filipino.
In 1994, Casco was diagnosed to be hypertensive and on December 7, 1995 he suffered a
stroke and was admitted at the PGH. On October 14, 1999 he suffered another attack and was
confined at Our Lady of Lourdes Hospital. This forced him to retire from the government service at
an early age.
Casco then applied for disability benefits under Presidential Decree No. 626, as amended. On
October 14, 1999, the GSIS granted him 38 months of permanent partial disability (PPDI).
On December 10, 2000 up to December 19, 2000, Casco was again confined at the
Potenciano Hospital due to his ailments. His confinement within the specified period was likewise
paid by the System.
Casco’s latest physical examination reveals that he still experiences chest pain, which is pricking, in
character, limping accompanied by lapse of memory and vertigo. Thus, he requested the System to
convert his permanent partial disability to permanent total disability (PTD) pursuant to P.D. 626, as
amended, but the same was denied. Such decision was affirmed by the ECC but later reversed and
set aside by the CA.

Issue/s
Whether or not respondent’s claim for conversion of his PPD benefits to PTD benefits
should be granted.

Held:
Petition denied.
There are three types of disability benefits granted under P.D. No. 626: (1) temporary total
disability; (2) permanent total disability; and (3) permanent partial disability. A disability is
considered total and permanent if as a result of the injury or sickness, the employee is unable to
perform any gainful occupation for a continuous period exceeding 120 days. A disability is partial

357
and permanent if, as a result of the injury or sickness, the employee suffers a permanent partial loss
of the use of any part of his body.
In GSIS v. Court of Appeals and Gonzaga v. ECC, et al., the Court declared that disability
should be understood not singly through its medical significance but, more importantly, in terms of
a person’s loss of earning capacity. Permanent total disability means disablement of an employee to
earn wages in the same kind of work, or work of a similar nature that he was trained for or
accustomed to perform, or any kind of work which a person of his mentality and attainment could
do. It does not mean absolute helplessness but rather incapacity to perform gainful work which is
expected to be permanent. Total disability does not require that the employee be absolutely
disabled, or totally paralyzed. What is necessary is that the injury must be such that he cannot
pursue his usual work and earn therefrom.
In this case, respondent was diagnosed to be hypertensive as a result of the physical and mental
stress of his work. His hypertension resulted in two cerebrovascular accidents, the clinical term for
stroke, first in 1995 and again in 1999. As certified by his attending physician, Dr. Fernando F.
Piedad, the degree of his disability is permanent and total. While it may be true that respondent’s
physical condition at the time of his retirement was not considered as a PTD, his condition
subsequently worsened such that in December 2000, he was again confined in a hospital.
Respondent also limps and continues to experience chest pain, vertigo and lapses in memory.
A person’s disability might not emerge at one precise moment in time but rather over a period of
time. It is possible that an injury which at first was considered to be temporary may later on
become permanent, or one who suffers a partial disability becomes totally and permanently
disabled by reason of the same cause. Thus, while respondent had been awarded 38 months of PPD
benefits commensurate to his physical condition at the time of his retirement, this does not
preclude the conversion of the benefits to which he is entitled as a result of the fact that he later on
became permanently and totally disabled. When an employee is constrained to retire at an early
age due to his illness and the illness persists even after retirement, resulting in his continued
unemployment, as in this case, such a condition amounts to total disability which should entitle him
to the maximum benefits allowed by law.
Indeed, denying respondent, who had rendered more than 21 years of service but was forced to
retire due to his ailment, the PTD benefits to which he is indisputably entitled would be contrary to
the spirit of P.D. No. 626 and the social justice principle enshrined in our Constitution.

SSS
V.
GLORIA DE LOS SANTOS

Doctrine: An estranged wife who was not dependent upon her deceased husband for support is not
qualified to be his beneficiary.

Facts:

358
Antonio de los Santos and respondent Gloria de los Santos, both Filipinos, were married on
April 29,1964 in Manila. Less than one year after, Gloria left Antonio and contracted another
marriage with acertain Domingo Talens in Nueva Ecija. Sometime in 1969, Gloria went back to
Antonio and lived withhim until 1983. They had three children: Alain Vincent, Arlene, and
Armine.In 1983, Gloria left Antonio and went to the United States. Later on, she filed for divorce
againstAntonio in California and executed a document waiving all her rights to their conjugal
properties andother matters. The divorce was granted on November 5, 1986.In 1987, Antonio
married Cirila de los Santos in Camalig, Albay. Their union produced one child, May-Ann N. de los
Santos. On the other hand, Gloria married Larry Thomas Constant, an American citizen,on July 11,
1987, in the US.In 1989, Antonio amended his records at SSS and changed his beneficiaries from
Mrs. Margarita delos Santos to Cirila de los Santos; from Gloria de los Santos to May-Ann de los
Santos; and from Erlindade los Santos to Armine de los Santos. Antonio retired from his
employment in 1996, and from then onbegan receiving monthly pension.Antoio died of respiratory
failure on May 15, 1999. Upon his death, Cirila applied for and beganreceiving his SSS pension
benefit, beginning December 1999. On December 21, 1999, Gloria filed a claim for Antonio’s death
benefits with the SSS. Her claim was denied because she was not a qualifiedbeneficiary of Antonio.

Issue:
Whether or not the respondent is still qualified as a primary beneficiary of the deceased SSS
memberAntonio?

Held:
As found by both the SSC and the CA, the divorce obtained by respondent against the
deceased Antoniowas not binding in this jurisdiction. Under Philippine law, only aliens may obtain
divorces abroad,provided they are valid according to their national law. The divorce was obtained
by respondent Gloriawhile she was still a Filipino citizen and thus covered by the policy against
absolute divorces. It did notsever her marriage ties with Antonio.However, although respondent
was the legal spouse of the deceased, We find that she is still disqualified to be his primary
beneficiary under the SS Law. She fails to fulfill the requirement of dependency upon her deceased
husband Antonio.
Social Security System v. Aguas is instructive in determining the extent of the required
“dependency”under the SS Law. In Aguas, the Court ruled that although a husband and wife are
obliged to support each other, whether one is actually dependent for support upon the other
cannot be presumed from the fact of marriage alone.”
Further, Aguas pointed out that a wife who left her family until her husband died and lived
with other men, was not dependent upon her husband for support, financial or otherwise, during
the entire period.

Said the Court:


In a parallel case involving a claim for benefits under the GSIS law, the Court defined
a dependent as “one who derives his or her main support from another. Meaning, relying on, or
subject to, someone else for support; not able to exist or sustain oneself, or to perform anything
without the will, power, or aid of someone else.” It should be noted that the GSIS law likewise

359
defines a dependent spouse as “the legitimate spouse dependent for support upon the member or
pensioner.” In that case, the Court found it obvious that a wife who abandoned the family for more
than 17 years until her husband died, and lived with other men, was not dependent on her husband
for support, financial or otherwise, during that entire period. Hence, the Court denied her claim for
death benefits.

The obvious conclusion then is that a wife who is already separated de facto from her
husband cannot be said to be “dependent for support” upon the husband, absent any showing to the
contrary. Conversely, if it is proved that the husband and wife were still living together at the time
of his death, it would be safe to presume that she was dependent on the husband for support, unless
it is shown that she is capable of providing for herself.
Respondent herself admits that she left the conjugal abode on two (2) separate occasions, to
live with two different men. The first was in 1965, less than one year after their marriage, when she
contracted a second marriage to Domingo Talens. The second time she left Antonio was in 1983
when she went to the US, obtained a divorce, and later married an American citizen.

In fine, these uncontroverted facts remove her from qualifying as a primary beneficiary of her
deceased husband.
WHEREFORE, the petition is GRANTED and the appealed Decision REVERSED and SET
ASIDE. The Resolution of the Social Security Commission is REINSTATED.

G.R. Nos. 182978-79 April 7, 2009

BECMEN SERVICE EXPORTER AND PROMOTION, INC.,PETITIONER,


VS.
SPOUSES SIMPLICIO AND MILA CUARESMA (FOR AND IN BEHALF OF THEIR DAUGHTER,
JASMIN G. CUARESMA), WHITE FALCON SERVICES, INC. AND JAIME ORTIZ
(PRESIDENT,WHITE FALCON SERVICES, INC.),RESPONDENTS.

Facts:
On January 6, 1997, Jasmin Cuaresma (Jasmin) was deployed by Becmen Service Exporter and
Promotion, Inc. (Becmen) to serve as assistant nurse in Al-Birk Hospital in the Kingdom of Saudi
Arabia (KSA), for a contract duration of three years, with a corresponding salary of US$247.00 per
month. Over a year later, she died allegedly of poisoning. Jessie Fajardo, a co-worker of Jasmin,
narrated that on June 21, 1998, Jasmin was found dead by a female cleaner lying on the floor inside
her dormitory room with her mouth foaming and smelling of poison.
Based on the police report and the medical report of the examining physician of the Al-Birk
Hospital, who conducted an autopsy of Jasmin’s body, the likely cause of her death was poisoning.
Jasmin’s body was repatriated to Manila on September 3, 1998. The following day, the City
Health Officer of Cabanatuan City conducted an autopsy and the resulting medical report indicated
that Jasmin died under violent circumstances, and not poisoning as originally found by the KSA
examining physician. The toxicology report of the NBI, however, tested negative for non-volatile,
metallic poison and insecticides.

360
Simplicio and Mila Cuaresma (the Cuaresmas), Jasmin’s parents and her surviving heirs,
received from the Overseas Workers Welfare Administration (OWWA) the following amounts:
P50,000.00 for death benefits; P50,000.00 for loss of life; P20,000.00 for funeral expenses; and
P10,000.00 for medical reimbursement.
On November 22, 1999, the Cuaresmas filed a complaint against Becmen and its principal in the
KSA, Rajab & Silsilah Company (Rajab), claiming death and insurance benefits, as well as moral and
exemplary damages for Jasmin’s death, Jasmin’s death was work-related, having occurred at the
employer’s premises; that under Jasmin’s contract with Becmen, she is entitled to “iqama
insurance” coverage; that Jasmin is entitled to compensatory damages in the amount of
US$103,740.00, which is the sum total of her monthly salary of US$247.00 per month under her
employment contract, multiplied by 35 years (or the remaining years of her productive life had
death not supervened at age 25, assuming that she lived and would have retired at age 60).
In their position paper, Becmen and Rajab insist that Jasmin committed suicide, citing a prior
unsuccessful suicide attempt sometime in March or April 1998 and relying on the medical report of
the examining physician of the Al-Birk Hospital. They likewise deny liability because the Cuaresmas
already recovered death and other benefits totaling P130,000.00 from the OWWA. They insist that
the Cuaresmas are not entitled to “iqama insurance” because this refers to the “issuance” – not
insurance – of iqama, or residency/work permit required in the KSA. On the issue of moral and
exemplary damages, they claim that the Cuaresmas are not entitled to the same because they have
not acted with fraud, nor have they been in bad faith in handling Jasmin’s case.
While the case was pending, Becmen filed a manifestation and motion for substitution alleging
that Rajab terminated their agency relationship and had appointed White Falcon Services, Inc.
(White Falcon) as its new recruitment agent in the Philippines. Thus, White Falcon was impleaded
as respondent as well, and it adopted and reiterated Becmen’s arguments in the position paper it
subsequently filed.

Issue:
1. whether the Cuaresmas are entitled to monetary claims, by way of benefits and damages, for
the death of their daughter Jasmin.
2. whether or not Jasmin’s death be considered as work-connected and thus compensable
even while she was not on duty

Ruling:
Article 19 of the Civil Code provides that every person must, in the exercise of his rights and in
the performance of his duties, act with justice, give everyone his due, and observe honesty and good
faith. Article 21 of the Code states that any person who wilfully causes loss or injury to another in a
manner that is contrary to morals, good customs or public policy shall compensate the latter for the
damage. And, lastly, Article 24 requires that in all contractual, property or other relations, when one
of the parties is at a disadvantage on account of his moral dependence, ignorance, indigence, mental
weakness, tender age or other handicap, the courts must be vigilant for his protection.
Clearly, Rajab, Becmen and White Falcon’s acts and omissions are against public policy because
they undermine and subvert the interest and general welfare of our OFWs abroad, who are entitled
to full protection under the law. They set an awful example of how foreign employers and

361
recruitment agencies should treat and act with respect to their distressed employees and workers
abroad. Their shabby and callous treatment of Jasmin’s case; their uncaring attitude; their
unjustified failure and refusal to assist in the determination of the true circumstances surrounding
her mysterious death, and instead finding satisfaction in the unreasonable insistence that she
committed suicide just so they can conveniently avoid pecuniary liability; placing their own
corporate interests above of the welfare of their employee’s – all these are contrary to morals, good
customs and public policy, and constitute taking advantage of the poor employee and her family’s
ignorance, helplessness, indigence and lack of power and resources to seek the truth and obtain
justice for the death of a loved one.
Giving in handily to the idea that Jasmin committed suicide, and adamantly insisting on it just
to protect Rajab and Becmen’s material interest – despite evidence to the contrary – is against the
moral law and runs contrary to the good custom of not denouncing one’s fellowmen for alleged
grave wrongdoings that undermine their good name and honor.
Whether employed locally or overseas, all Filipino workers enjoy the protective mantle of
Philippine labor and social legislation, contract stipulations to the contrary notwithstanding. This
pronouncement is in keeping with the basic public policy of the State to afford protection to labor,
promote full employment, ensure equal work opportunities regardless of sex, race or creed, and
regulate the relations between workers and employers. This ruling is likewise rendered imperative
by Article 17 of the Civil Code which states that laws which have for their object public order, public
policy and good customs shall not be rendered ineffective by laws or judgments promulgated, or by
determinations or conventions agreed upon in a foreign country.
The relations between capital and labor are so impressed with public interest,and neither shall
act oppressively against the other, or impair the interest or convenience of the public. In case of
doubt, all labor legislation and all labor contracts shall be construed in favor of the safety and
decent living for the laborer.
The grant of moral damages to the employee by reason of misconduct on the part of the
employer is sanctioned by Article 2219 (10) of the Civil Code, which allows recovery of such
damages in actions referred to in Article 21.
Thus, in view of the foregoing, the Court holds that the Cuaresmas are entitled to moral
damages, which Becmen and White Falcon are jointly and solidarily liable to pay, together with
exemplary damages for wanton and oppressive behavior, and by way of example for the public good.
On the second issue:
While the “employer’s premises” may be defined very broadly not only to include premises
owned by it, but also premises it leases, hires, supplies or uses, we are not prepared to rule that the
dormitory wherein Jasmin stayed should constitute employer’s premises as would allow a finding
that death or injury therein is considered to have been incurred or sustained in the course of or
arose out of her employment. There are certainly exceptions, but they do not appear to apply here.
Moreover, a complete determination would have to depend on the unique circumstances obtaining
and the overall factual environment of the case, which are here lacking.
WHEREFORE, Rajab & Silsilah Company, White Falcon Services, Inc., Becmen Service Exporter
and Promotion, Inc., and their corporate directors and officers are found jointly and solidarily liable
and ORDERED to indemnify the heirs of Jasmin Cuaresma, spouses Simplicio and Mila Cuaresma,
the following amounts: (1) TWO MILLION FIVE HUNDRED THOUSAND PESOS (P2,500,000.00) as

362
moral damages; (2) TWO MILLION FIVE HUNDRED THOUSAND PESOS (P2,500,000.00) as
exemplary damages; (3)Attorney’s fees equivalent to ten percent (10%) of the total monetary
award.

G.R. NO. 183646

GREAT SOUTHERN MARITIME SERVICES CORP. AND IMC SHIPPING CP., PTE LTD
VS.
LEONILA SURIGAO

Facts :
[Respondent Leonila Surigao’s] husband, the late Salvador M. Surigao, was hired as Fitter by
[petitioner] Great Southern Maritime Services Corporation, for and in behalf of [co-
petitioner] IMCShipping Co. Pte., Ltd. (Singapore) for a period of ten (10) months. In his pre-
employment medical examination, he was found fit for sea duty. Thus, on April 29, 2001, he
commenced his work aboard MV Selendang Nilam.

However, on August 22, 2001, as per Ship Master’s advice, a doctor was sent on board the vessel to
medically attend to Salvador due to complaints of extensive neuro dermatitis, neck region viral,
aetiology, urticaria, maculo popular, rash extending to the face, chest and abdomen. After
examination, Salvador was advised to take a blood test. His condition having worsened, he was
confined at the Seven Hills Hospital. Not long thereafter, the Ship Master decided to sign him off
from the vessel on August 25, 2001 for treatment in the hospital and for repatriation upon
certification of the doctor that he was fit to travel.

Prior to his repatriation, though, or on August 26, 2001, at around seven o’clock in the
morning, Salvador was found dead inside the bathroom of his hospital room. Later, his body was
transferred to a government hospital, the Ling George Hospital Mortuary Hall, for post-mortem
examination. The Post-Mortem Certificate issued by the Department of Forensic
Medicine, Visakhapatnam City, stated that the cause of death of Salvador was asphyxia due to
hanging.

As an heir of the deceased seaman, petitioner, for in behalf of her minor children, filed for death
compensation benefits under the terms of the standard employment contract, but her claims were
denied by the [petitioners]. Labor Arbiter rendered his decision on favor of the respondent
ordering Great Southern Maritime Services Corporation and/or IMC Shipping Co., PTE LTD.,
Singapore to pay complainants Leonila S. Surigao, Miriam Surigao and Kaye Angeli Surigao the
amount of SEVENTY ONE THOUSAND FIVE HUNDRED DOLLARS ($71,500.00) or its equivalent in
Philippine pesos at the prevailing rate of exchange at the time of actual payment representing the
death benefits, burial expenses of the deceased Salvador M. Surigao and attorney’s fees. All other
claims are DISMISSED for lack of merit.

363
On appeal, the NLRC reversed and set aside the decision of the Labor Arbiter and declared
[petitioners] not liable for death benefits. In lieu thereof, however, the commission directed the
[petitioners] to grant financial assistance to the [respondent] in the amount of Five Thousand
Dollars ($5,000.00) at the prevailing rate at the time of payment.

Respondent thereafter elevated the case to the appellate court which reversed the decision of the
NLRC and reinstated that of the Labor Arbiter in its herein assailed February 14, 2008
Decision. The appellate court found that Salvador did not commit suicide; hence, respondents are
entitled to receive death benefits.

Petitioners insist that respondents are not entitled to death benefits because Salvador committed
suicide. As proof, they presented the Death Certificate issued by Dr. Butchi Raju stating that
Salvador was suspected to have committed suicide; the post-mortem examination results stating
that the deceased appeared to have died of “ASPHYXIA DUE TO HANGING”; the Indian Police Inquest
Report also stating that he died due to hanging; the affidavit of the nurse on duty of Seven Hills
hospital, Ms. P. V. Ramanamma, wherein she stated that as the entrance doors to the bathroom main
room was bolted from the inside and no other person was in the near physical vicinity of the deceased,
it was concluded that seafarer committed suicide; as well as photos taken immediately after the
discovery of the body with a belt around his neck. They contend that the appellate court erred in
disregarding these pieces of evidence which convincingly rule out suspicions of foul play.
The pertinent provisions of the Standard Terms and Conditions Governing the Employment of
Filipino Seafarers On-Board Ocean-Going Vessels, or the POEA Standard Employment Contract,
which Salvador and the petitioners incorporated into their contract, provide that:

SECTION 20. COMPENSATION AND BENEFITS


A. COMPENSATION AND BENEFITS FOR DEATH
1. In case of death of the seafarer during the term of his contract, the employer shall pay his
beneficiaries the Philippine Currency equivalent to the amount of Fifty Thousand US dollars
(US$50,000) and an additional amount of Seven Thousand US dollars (US$7,000) to each child
under the age of twenty-one (21) but not exceeding four (4) children at the exchange rate
prevailing during the time of payment.
xxxx
D. No compensation and benefits shall be payable in respect of any injury, incapacity, disability
or death of the seafarer resulting from his willful or criminal act or intentional breach of his duties,
provided however, that the employer can prove that such injury, incapacity, disability or death is
directly attributable to the seafarer.

Issues :

1. Whether or not private respondent is entitled to death benefits for the death of her husband
under the POEA standard employment contract for seafarers.
2. Whether or not private respondent is entitled to damaged and attorney’s fees.

364
Held :
The general rule is that the employer is liable to pay the heirs of the deceased seafarer for death
benefits once it is established that he died during the effectivity of his employment
contract. However, the employer may be exempted from liability if he can successfully prove that
the seafarer’s death was caused by an injury directly attributable to his deliberate or willful
act.[6] In sum, respondents’ entitlement to any death benefits depends on whether the evidence of
the petitioners suffices to prove that the deceased committed suicide; the burden of proof rests on
his employer.[7]
The post-mortem examination conclusively established that the true cause of death was
asphyxia or suffocation. The appellate court’s ruling that while it may be consistent with the theory
that the deceased hanged himself but it does not rule out the possibility that he might have died of
other causes,[18] does not persuade. Aside from being purely speculative, we find it hard to believe
that someone strangled Salvador inside the bathroom then locked the door thereof on his way out
undetected. As shown by the evidence presented by the petitioners, the bathroom door was locked
or bolted from the inside and could not be opened from outside. In order to gain entrance, the
hospital staff had to pass through a closed door with a mess leading to the ceiling of the
bathroom. Entry could not likewise be effected through the bathroom window as it has grills.
Moreover, the conclusion that Salvador could not have hanged himself to the showerhead as he was
found lying on the floor with a belt tied around his neck; or that he could not have died since the
pipe broke down and he fell therefrom,[19] are based on speculations and hypothetical in
nature. This confusion could have been avoided had both the Court of Appeals and the Labor
Arbiter considered the most logical possibility that Salvador died hanging on the showerhead
before the pipe broke down due to his body weight, and thus, explaining why he was found on the
floor with the belt still on his neck and broken pipe and showerhead near his lifeless body. That the
post-mortem examination, the Certification of Dr. Raju and the police inquest report, all stated
that Salvador’s cause of death was asphyxia due to hanging, and not due to any other injury, lead to
a fair and just conclusion that Salvador was already dead before the showerhead broke.
Indeed, we are not unaware of our ruling in Becmen Service Exporter and Promotion, Inc. v.
Cuaresma,[20] where we held that Jasmin Cuaresma, also an overseas Filipino worker, did not
commit suicide; that Filipinos are resilient people, willing to take on sacrifices for the good of their
family; and that we do not easily succumb to hardships and difficulties. Nevertheless, the
circumstances prevailing in said case are totally different from this case. In Becmen, the
postmortem examination and the police report did not state with specificity that poisoning or
suicide was the cause of Jasmin’s death. In fact, both reports mentioned that the cause of death of
Jasmin was still under investigation. In contrast, the postmortem examination and the police report
in this case, categorically mentioned that Salvador died of asphyxia due to hanging. It was also
shown that no other individual could have caused the death of Salvador because the bathroom door
was locked or bolted from the inside and could not be opened from outside.
In Mabuhay Shipping Services, Inc. v. National Labor Relations Commission,[21] the Court held
that the death of a seaman even during the term of employment does not automatically give rise to
compensation. The circumstances which led to the death as well as the provisions of the contract,
and the right and obligation of the employer and the seaman must be taken into consideration, in
consonance with the due process and equal protection clauses of the Constitution.

365
It is true that the beneficent provisions of the Standard Employment Contract are liberally
construed in favor of Filipino seafarers and their dependents.[22] We commiserate with
respondents for the unfortunate fate that befell their loved one; however, we find that the factual
circumstances in this case do not justify the grant of death benefits as prayed for by them as
beneficiaries of Salvador.

G.R. No, 185035

GOVERNMENT SERVICE INSURANCE SYSTEM


VS.
SALVADOR DE CASTO

Respondent Salvador De Castro (De Castro) rendered service in the Philippine Air Force (PAF)
from April 1, 1974 until his retirement on March 2, 2006.
On December 22, 2004, De Castro was admitted at the V. Luna General
Hospital, AFP Medical Center due to chest pains. He underwent on January 21, 2005 a 2-D
echocardiography which revealed that he had “dilated left atrium eccentric left ventricular
hyperthropy and left ventricular dysfunction.” His full diagnosis consisted of hypertensive
cardiovascular disease, dilated atrium, eccentric left ventricular hypertrophy and left ventricular
dysfunction, and old anterior wall myocardial infarction. He also underwent coronary angiogram
procedure which showed that he had significant simple vessel coronary artery disease (CAD).
On August 15, 2005, De Castro was confined in the same hospital and was diagnosed to be
suffering from (1) 41X-D21 – Coronary artery disease and (2) 400-533 – Hypertensive
cardiovascular disease.
De Castro retired from the service on March 2, 2006 with a “Certificate of Disability
Discharge.”[4] On this basis, he filed a claim for permanent total disability benefits with the GSIS.
In a decision dated June 20, 2006, the GSIS denied De Castro’s claim based on the finding that
De Castro's illnesses were non-occupational. De Castro appealed to the Employees’ Compensation
Commission (ECC).
The ECC, however, also held that, contrary to the ruling of the GSIS, CAD is a form of
cardiovascular disease included in the list of occupational diseases. The ECC still denied the claim
despite this observation because of “the presence of factors which are not work-related, such as
smoking and alcohol consumption.”[6] It likewise noted that manifestations of Cardiomyopathy in De
Castro’s 2-D echocardiography examination results could be related to his drinking habits.
De Castro sought relief from the CA through a petition for review under Rule 43 of the Rules of
Court. Relying on Dominga A. Salmone v. ECC,[7] De Castro argued that the causal relation between
his illness and the resultant disability, on the one hand, and his work, on the other, is not that
essential; it is enough that his illness is listed as an occupational disease. He disputed the findings of
the ECC that hypertension or high blood pressure (which causes CAD) may have been caused by his
cigarette smoking and drinking habits. He posited that other factors, such as stress brought about
by the nature of his work, could have caused his illness. He claimed that the positions he held in the
PAF, the last being First Sergeant, were comparable to a managerial position in the civilian business

366
community because it served as an extension of the office of his commanding officer in the
management, administration, and supervision of his fellow enlisted personnel within the unit.
The CA granted the petition.[8] It noted that, as found by the ECC itself, De Castro’s illnesses are
listed as occupational diseases in Annex “A” of the Amended Rules of the Employees’ Compensation
Commission (Amended ECC Rules). It explained that under the same rules, the sickness must be the
result of an occupational disease under Annex “A” in order for the illness and the resulting disability
or death to be compensable.[9]
The CA further explained that it is not necessary that there be proof of causal relation between the
work and the illness which resulted in De Castro’s disability. Citing GSIS v. Baul,[10] it held that in
general, a covered claimant suffering from an occupational disease is automatically paid
benefits. While it noted that the exact etiology of hypertension which led to De Castro’s
cardiovascular ailments cannot be accurately traced, it stressed that medical experiments tracing
the etiology of essential hypertension show a relationship between this illness and the nature and
conditions of work. The CA found significant the statement in De Castro’s Certificate of Disability
Discharge that his CAD and hypertensive cardiovascular diseases were aggravated during active
service; were not incurred while on AWOL; did not exist prior to entry into service; were incident to
service; were not incurred by private avocation; were not due to misconduct; and, were incurred
while in line of duty. The appellate court, therefore, brushed aside the findings a quo that De
Castro’s illnesses might have been caused by his smoking and drinking habits.

Issues :
(1) whether the CA erred in reversing the decisions of the ECC and the GSIS that denied De Castro’s
claim for disability benefits; and
(2) whether De Castro proved that his heart ailments are work-related and/or have been
precipitated by his duties with the Armed Forces of the Philippines (AFP).

Held :
Other than the given facts, another undisputed aspect of the case is the status of the ailments that
precipitated De Castro’s separation from the military service – CAD and hypertensive
cardiovascular disease. These are occupational diseases.[26] No less than the ECC itself confirmed
the status of these ailments when it declared that “Contrary to the ruling of the System, CAD is a form
of cardiovascular disease which is included in the list of Occupational Diseases.”[27] Essential
hypertension is also listed under Item 29 in Annex “A” of the Amended ECC Rules as an
occupational disease.
Despite the compensable character of his ailments, both the GSIS and the ECC found De
Castro’s CAD to be non-work related and, therefore, non-compensable. To use the wording of the
ECC decision, it denied De Castro’s claim “due to the presence of factors which are not work-related,
such as smoking and alcohol consumption.”[28] De Castro’s own military records triggered this
conclusion as his Admitting Notes,[29] made when he entered the V. Luna General Hospital due to
chest pains and hypertension, were that he was a smoker and a drinker.
As the CA did, we cannot accept the validity of this conclusion at face value because it considers
only one side – the purely medical side – of De Castro’s case and even then may not be completely
correct. The ECC itself, in its decision,[30] recites that CAD is caused, among others, by

367
atherosclerosis of the coronary arteries that in turn, and lists the following major causes:
increasing age; male gender; cigarette smoking; lipid disorder due to accumulation of too much fats
in the body; hypertension or high blood pressure; insulin resistance due to diabetes; family history
of CAD. The minor factors are: obesity; physical inactivity; stress; menopausal estrogen deficiency;
high carbohydrate intake; and alcohol.
We find it strange that both the ECC and the GSIS singled out the presence of smoking and drinking
as the factors that rendered De Castro’s ailments, otherwise listed as occupational, to be non-
compensable. To be sure, the causes of CAD and hypertension that the ECC listed and explained in
its decision cannot be denied; smoking and drinking are undeniably among these causes. However,
they are not the sole causes of CAD and hypertension and, at least, not under the circumstances of
the present case. For this reason, we fear for the implication of the ECC ruling if it will prevail and
be read as definitive on the effects of smoking and drinking on compensability issues, even on
diseases that are listed as occupational in character. The ruling raises the possible reading that
smoking and drinking, by themselves, are factors that can bar compensability.
We ask the question of whether these factors can be sole determinants of compensability as the ECC
has apparently failed to consider other factors such as age and gender from among those that the
ECC itself listed as major and minor causes of atherosclerosis and, ultimately, of CAD. While age
and gender are characteristics inherent in the person (and thereby may be considered non-work
related factors), they also do affect a worker’s job performance and may in this sense, together with
stresses of the job, significantly contribute to illnesses such as CAD and hypertension. To cite an
example, some workplace activities are appropriate only for the young (such as the lifting of heavy
objects although these may simply be office files), and when repeatedly undertaken by older
workers, may lead to ailments and disability. Thus, age coupled with an age-affected work activity
may lead to compensability. From this perspective, none of the ECC’s listed factors should be
disregarded to the exclusion of others in determining compensability.
In any determination of compensability, the nature and characteristics of the job are as important
as raw medical findings and a claimant’s personal and social history. This is a basic legal reality in
workers’ compensation law.[31] We are therefore surprised that the ECC and the GSIS simply
brushed aside the disability certification that the military issued with respect to De Castro’s
disability, based mainly on their primacy as the agencies with expertise on workers’ compensation
and disability issues.
While ECC and GSIS are admittedly the government entities with jurisdiction over the
administration of workers’ disability compensation and can thus claim primacy in these areas, they
cannot however claim infallibility, particularly when they use wrong or limited considerations in
determining compensability.
In the present case, they should at least have considered the very same standards that they stated
in their own decisions, and should not have simply brushed aside as incorrect the basis for
disability that the AFP, as home agency, used in passing upon De Castro’s separation from the
service and discharge for disability. In saying this, we are not unmindful that neither the GSIS nor
the ECC conducted a medical examination of De Castro on their own; they merely relied on the
results of De Castro's medical examination conducted at the V. Luna General Hospital, a government
military hospital. It was from these same medical findings that the GSIS and ECC derived their
conclusion that De Castro's drinking and smoking habits and personal lifestyle caused his

368
ailments. We are aware, too, that De Castro’s discharge based on disability was not the sole result
of the AFP medical findings; the medical findings were further reviewed and deliberated upon by
the AFP’s DSB which certified on the causes of De Castro’s separation from the service and his
disability.
The military’s disability certification clearly states that De Castro’s ailments were: (1) aggravated
by active service, (2) incident to service, (3) not incurred while on AWOL, (4) never existed prior
to entry to military service, (5) not due to misconduct, (6) not incurred by private avocation and,
(7) in line of duty. De Castro further stated in the course of this case that the positions he occupied
as the PAF-Non-Commissioned Officer-in-Charge for Operational Security, Asst. First Sergeant and
First Sergeant of the 577th CS, 570th CTW stationed at Puerto Princesa, Palawan were positions
comparable to managerial positions in the private business sector; he served as the extension of his
commanding officer in the management, administration, and supervision of the activities of his
fellow enlisted soldiers within the unit – tasks whose urgency and sensitivity resulted in job
stress. While the task before the GSIS and the ECC was to determine compensability, not merely the
fact of disability that justifies a separation from the service, still, these agencies should not have
simply glossed over the findings of the military on the matters they certified to, as these are the
same facts that are material to compensability. The health of De Castro upon entry into the service
and how his work affected his health are very relevant facts that should not have been disregarded
in favor of singled out facts that the GSIS and the ECC considered as conclusive indicators of
incompensability. The ECC and the GSIS, in short, did not seriously look at all the relevant factors
determinative of compensability and thereby decided De Castro’s case based on incomplete, if not
wrong, considerations. This is a reversible error that requires rectification.
In contrast, the assailed CA ruling was sensitive to all these concerns and found reasonable work
connection between De Castro’s ailments and his duties as a soldier for 32 years without at all
disregarding De Castro’s drinking and smoking habits that could have contributed to his
afflictions. On the latter concerns, we quote with approval the following CA observations:
Intoxication which does not incapacitate the employee from following his occupation is not
sufficient to defeat the recovery of compensation, although intoxication may be a contributory
cause to his injury. While smoking may contribute to the development of a heart ailment, heart
ailment may be cause by other factors such as working and living under stressful conditions. Thus,
the peremptory presumption that petitioner’s habit of smoking heavily was the wilfull act which
causes his illness and resulting disability, without more, cannot suffice to bar petitioner’s claim for
disability benefits.[32]
We consider it significant that De Castro entered military service as a fit and healthy new soldier.
We note, too, De Castro’s service record and the medals, awards, and commendations he
earned,[33] all attesting to 32 years of very active and productive service in the military. Thus, the
CAD and the hypertension came while he was engaged in these endeavors. To say, as the GSIS and
the ECC did, that his ailments are conclusively non-work related because he smoked and drank, is to
close our eyes to the rigors of military service and to the demands of De Castro’s specific positions
in the military service, and to single out factors that would deny the respondent’s claim. This is far
from the balancing that the GSIS invokes between sympathy for the workingman and the equally vital
interest of denying underserving claims.[34] Thus, based on the totality of the circumstances
surrounding De Castro’s case, we are convinced that his long years of military service, with its

369
attendant stresses and pressures, contributed in no small measure to the ailments that led to his
disability retirement. We, therefore, agree with the CA when it concluded that De Castro's “illness
was contracted during and by reason of his employment, and any non-work related factor that
contributed to its aggravation is immaterial.”
We close by reiterating that what the law requires is a reasonable work connection and not
direct causal relation.[35] Probability, not the ultimate degree of certainty, is the test of proof in
compensation proceedings.[36] For, in interpreting and carrying out the provisions
of the Labor Code and its Implementing Rules and Regulations, the primordial
and paramount consideration is the employee's welfare. To safeguard the worker's rights, any
doubt on the proper interpretation and application must be resolved in favor of labor.[37]
We reiterate these same principles in the present case. Accordingly, we hold that De Castro's
ailments – CAD and hypertensive cardiovascular disease – are work-connected under the
circumstances of the present case and are, therefore, compensable.

G.R. No. 198501 January 30, 2013

KESTREL SHIPPING CO., INC./ CAPT. AMADOR P. SERVILLON AND ATLANTIC MANNING
LTD., PETITIONERS,
VS.
FRANCISCO D. MUNAR, RESPONDENT.

On March 23, 2006, petitioner Kestrel Shipping, Inc. (Kestrel), on behalf of its principal, petitioner
Atlantic Manning, Ltd., and respondent Munar forged a six (6)-month employment contract
designating Munar as pump man for M/V Southern Unity. The heavy lifting that forms part of his
daily work duties has contributed significantly to the abnormalities demonstrated on his lumbar
spine MRI scans.
On April 17, 2007, Munar filed with the labor arbiter (LA) a complaint for total and permanent
disability benefits. Munar claimed that the mere fact that his medical condition, which incapacitated
him to engage in any gainful employment, persisted for more than 120 days automatically entitles
him to total and permanent disability benefits.
During the mandatory mediation and conciliation conferences, petitioners invoked Dr. Chua’s
(company physician) assessment per his medical report and offered to pay Munar the benefit
corresponding to Grade 8 disabilities or $16,795.00. Munar rejected petitioners’ offer and
maintained that his disability should be rated as Grade 1, as assessed by Dr. Chiu, an orthopedic
surgeon at Lorma Medical Center.
The LA awarded Munar with total and permanent disability benefits in the amount of US$60,000.00
and attorney’s fees equivalent to ten percent (10%) of the former, the same is maximum
compensation benefit as provided under the POEA Standard Employment Contract. As between the
assessment of Dr. Chua and that of Dr. Chiu, LA Guerrero gave more weight to the latter.
On appeal by petitioners, the NLRC affirmed LA’s Decision dated May 30, 2008. In a Decision dated
June 30, 2009, the NLRC ruled that Dr. Chiu’s categorical and definite assessment should prevail
over that of Dr. Chua, which failed to approximate the period needed by Munar to fully recover and
lacked clear basis.

370
After the NLRC denied petitioners’ motion for reconsideration, petitioners filed a petition for
certiorari with the CA, alleging that the NLRC acted with grave abuse of discretion in characterizing
Munar’s disability as total and permanent. Denying said petition, CA said despite Munar’s having
undergone surgeries, treatment and physical therapy of more than seven months from the injury,
Munar is still found by all physicians involved to continue to suffer from weakness, tenderness and
pain that prevent him from doing strenuous activities. Should he even try, Munar is certain to get
disqualified as seafarer since such an employment will require him to undergo rigorous physical
examinations which he is sure to fail because of the sorry state of his physical health.

ISSUE
The only issue left for resolution is the amount of disability payments due to Munar. Whether
Munar’s disability entitles him to claim the benefits corresponding to Grade 1 disabilities, as he has
not yet fully recovered after the lapse of 120 days from the time he signed-off from M/V Southern
Unity.

RULING
Petition is denied.
Under Section 32 of the POEA-SEC, only those injuries or disabilities that are classified as Grade 1
may be considered as total and permanent. However, if those injuries or disabilities with a
disability grading from 2 to 14, hence, partial and permanent, would incapacitate a seafarer from
performing his usual sea duties for a period of more than 120 or 240 days, depending on the need
for further medical treatment, then he is, under legal contemplation, totally and permanently
disabled. In other words, an impediment should be characterized as partial and permanent not only
under the Schedule of Disabilities found in Section 32 of the POEA-SEC but should be so under the
relevant provisions of the Labor Code and the Amended Rules on Employee Compensation (AREC)
implementing Title II, Book IV of the Labor Code. That while the seafarer is partially injured or
disabled, he is not precluded from earning doing the same work he had before his injury or
disability or that he is accustomed or trained to do. Otherwise, if his illness or injury prevents him
from engaging in gainful employment for more than 120 or 240 days, as the case may be, he shall be
deemed totally and permanently disabled.
The Labor Code’s concept of permanent total disability is applicable to the case at bar. Petitioner
claims to have suffered from permanent total disability as defined under Article 192(c)(1) of the
Labor Code, viz:
Art. 192 (c). The following disabilities shall be deemed total and permanent:
(1) Temporary total disability lasting continuously for more than one hundred twenty days, except
as otherwise provided in the Rules; x x x
Petitioner likewise cites Vicente v. ECC and Abaya, Jr. v. ECC, both of which were decided applying
the Labor Code provisions on disability benefits. Private respondents, on the other hand, contend
that petitioner erred in applying the definition of "permanent total disability" under the Labor Code
and cases decided under the ECC as the instant case involves a contractual claim under the 1996
POEA SEC.The Court said "disability should not be understood more on its medical significance but
on the loss of earning capacity. Permanent total disability means disablement of an employee to
earn wages in the same kind of work, or work of similar nature that he was trained for or

371
accustomed to perform, or any kind of work which a person of hismentality and attainment could
do. It does not mean absolute helplessness." It likewise cited Bejerano v. ECC, that in a disability
compensation, it is not the injury which is compensated, but rather it is the incapacity to work
resulting in the impairment of one’s earning capacity.
Consequently, if after the lapse of the stated periods, the seafarer is still incapacitated to perform
his usual sea duties and the company-designated physician had not yet declared him fit to work or
permanently disabled, whether total or permanent, the conclusive presumption that the latter is
totally and permanently disabled arises. On the other hand, if the company-designated physician
declares the seaman fit to work within the said periods, such declaration should be respected
unless the physician chosen by the seaman and the doctor selected by both the seaman and his
employer declare otherwise.
What is important is that he was unable to perform his customary work for more than 120 days
which constitutes permanent total disability. An award of a total and permanent disability benefit
would be germane to the purpose of the benefit, which is to help the employee in making ends meet
at the time when he is unable to work.

G.R. No. 191740 February 11, 2013

SUSANA R. SY, PETITIONER,


VS.
PHILIPPINE TRANSMARINE CARRIERS, INC., AND/OR SSC SHIP MANAGEMENT PTE.,
LTD., RESPONDENTS.

FACTS
On June 23, 2003, Alfonso N. Sy (Sy) was hired by respondent Philippine Transmarine Carriers
Incorporated for and in behalf of its foreign principal, co-respondent SSC Ship Management Pte. Ltd.
In their contract of employment Sy was assigned to work as Able Seaman (AB) on board the vessel
M/V Chekiang for the duration of ten months, with a basic monthly salary of US$512.00. Considered
incorporated in AB Sy's Philippine Overseas Employment Administration-Standard Employment
Contract (POEA-SEC) is a set of standard provisions established and implemented by the POEA,
called the Amended Standard Terms and Conditions Governing the Employment of Filipino Seafarers
on Board Ocean-Going Vessels.

On October 1, 2005, while the vessel was at the Port of Jakarta, Indonesia, AB Sy went on shore
leave and left the vessel at about 1300 hours. At 1925 hours, the vessel's agent from Jardine
received an advice from the local police that one of the vessel's crew members died ashore. At 1935
hours, the agent advised the vessel's master, Capt. Norman C. Marquez, about the incident. At 2050
hrs., Capt. Marquez and his 3 crew members went to Cipto Mangunkusumo Hospital where they
confirmed the cadaver to be that of AB Sy. A forensic pathologist certified that AB Sy's death was an
accident due to drowning, and that there was "alcohol 20mg%" in his urine.6

Petitioner Susana R. Sy, widow of AB Sy, demanded from respondents payment of her husband's
death benefits and compensation.

372
On August 28, 2007, the Labor Arbiter (LA) ordered respondent to pay complainant the Philippine
Currency equivalent to Fifty Thousand US Dollars (US$50,000.00) as death benefit and an
additional amount of Philippine Currency equivalent to One Thousand U.S. Dollars (US$1,000.00) as
burial expenses at the exchange rate prevailing at the time of payment
Respondents filed their appeal with the National Labor Relations Commission (NLRC), reiterating
that AB Sy's death was not work-related, hence, there was no basis for the LA's award. Petitioner
also filed her appeal claiming that she was entitled to attorney's fees as well as moral and
exemplary damages.
On October 17, 2008, the NLRC rendered its Resolution dismissing respondent’s appeal and
granting the complainant’s.
Reversing the NLRC, the CA found AB Sy's death not work-related.

ISSUE
WHETHER OR NOT THE HONORABLE COURT OF APPEALS COMMITTED GRAVE ABUSE OF
DISCRETION IN GRANTING RESPONDENTS' PETITION FOR CERTIORARI AND DENYING
PETITIONER'S MOTION FOR RECONSIDERATION BY REVERSING AND SETTING ASIDE THE
NATIONAL LABOR RELATIONS [COMMISSION'S] DECISION IN AWARDING DEATH BENEFITS
UNDER THE POEA STANDARD CONTRACT

RULING
The petition is devoid of merit.
The terms and conditions of a seafarer's employment is governed by the provisions of the contract
he signs with the employer at the time of his hiring, and deemed integrated in his contract is a set of
standard provisions set and implemented by the POEA, called the Standard Terms and Conditions
Governing the Employment of Filipino Seafarers on Board Ocean-Going Vessels, which provisions are
considered to be the minimum requirements acceptable to the government for the employment of
Filipino seafarers on board foreign ocean-going vessels. The issue raised of whether petitioner is
entitled to death compensation benefits from respondents is best resolved by the provisions of
their Employment Contract which incorporated the 2000 Standard Terms and Conditions Governing
the Employment of Filipino Seafarers on Board Ocean-Going Vessels. Section 20 (A) of the Contract
provides:
SECTION 20. COMPENSATION AND BENEFITS
A. COMPENSATION AND BENEFITS FOR DEATH
1. In the case of work-related death of the seafarer during the term of his contract, the employer
shall pay his beneficiaries the Philippine Currency equivalent to the amount of Fifty Thousand US
dollars (US$50,000) and an additional amount of Seven Thousand US dollars (US$7,000) to each
child under the age of twenty-one (21) but not exceeding four (4) children, at the exchange rate
prevailing during the time of payment.
x x x.
Clearly, to be entitled for death compensation benefits from the employer, the death of the seafarer
(1) must be work-related; and (2) must happen during the term of the employment contract. Under
the Amended POEA Contract, work-relatedness is now an important requirement. The qualification

373
that death must be work-related has made it necessary to show a causal connection between a
seafarer’s work and his death to be compensable.
Under the 2000 POEA Amended Employment Contract, work-related injury is defined as an
injury(ies) resulting in disability or death arising out of and in the course of employment. Thus,
there is a need to show that the injury resulting to disability or death must arise (1) out of
employment, and (2) in the course of employment.
Notably, at the time of the accident, AB Sy was on shore leave and there was no showing that he was
doing an act in relation to his duty as a seaman or engaged in the performance of any act incidental
thereto. It was not also established that, at the time of the accident, he was doing work which was
ordered by his superior ship officers to be done for the advancement of his employer's interest. On
the contrary, it was established that he was on shore leave when he drowned and because of the
20% alcohol found in his urine upon autopsy of his body, it can be safely presumed that he just
came from a personal social function which was not related at all to his job as a seaman.
While AB Sy's employment relationship with respondents did not stop but continues to be in force
even when he was on shore leave, their contract clearly provides that it is not enough that death
occurred during the term of the employment contract, but must be work-related to be
compensable. There is a need to show the connection of AB Sy's death with the performance of his
duty as a seaman. As we found, AB Sy was not in the performance of his duty as a seaman, but was
doing an act for his own personal benefit at the time of the accident.
While it is true that labor contracts are impressed with public interest and the provisions of the
POEA-SEC must be construed logically and liberally in favor of Filipino seaman in the pursuit of
their employment on board ocean-going vessels, still the rule is that justice is in every case for the
deserving to be dispensed with in the light of established facts, the applicable law, and existing
jurisprudence.

G.R. No. 192601 June 3, 2013

PHILIPPINE JOURNALISTS, INC., PETITIONER,


VS.
JOURNAL EMPLOYEES UNION (JEU), FOR ITS UNION MEMBER, MICHAEL ALFANTE,
RESPONDENTS.

FACTS:
Private respondents sued the petitioner for illegality of dismissal, the non-payment of rest days,
and the violation of Minimum Wage Order No. 9 and denial of funeral and bereavement aid. Except
the last issue, all others have already been decided and submissions of the private respondent was
already well taken.

Issue
The sole remaining issue is whether or not petitioner’s denial of respondents’ claims for funeral and
bereavement aid granted under Section 4, Article XIII of their CBA constituted a diminution of
benefits in violation of Article 100 of the Labor Code.

374
Ruling
The petition for review lacks merit.
The coverage of the term legal dependent as used in a stipulation in a collective bargaining
agreement (CBA) granting funeral or bereavement benefit to a regular employee for the death of a
legal dependent, if the CBA is silent about it, is to be construed as similar to the meaning that
contemporaneous social legislations have set. This is because the terms of such social legislations
are deemed incorporated in or adopted by the CBA.
Here, a conflict has arisen regarding the interpretation of the term legal dependent in connection
with the grant of funeral and bereavement aid to a regular employee under Section 4, Article XIII of
the CBA.
Petitioner insists that notwithstanding the silence of the CBA, the term legal dependent should
follow the definition of it under Republic Act (R.A.) No. 8282 (Social Security Law),24 so that in the
case of a married regular employee, his or her legal dependents include only his or her spouse and
children, and in the case of a single regular employee, his or her legal dependents include only his
or her parents and siblings, 18 years old and below; and that the term dependents has the same
meaning as beneficiaries as used in Section 5, Article XIII of the CBA.
We cannot agree with petitioner’s insistence.
Social legislations contemporaneous with the execution of the CBA have given a meaning to the
term legal dependent. First of all, Section 8(e) of the Social Security Law provides that a dependent
shall be the following, namely: (a) the legal spouse entitled by law to receive support from the
member; (b) the legitimate, legitimated, or legally adopted, and illegitimate child who is unmarried,
not gainfully employed and has not reached 21 of age, or, if over 21 years of age, is congenitally or
while still a minor has been permanently incapacitated and incapable of self-support, physically or
mentally; and (c) the parent who is receiving regular support from the member. Secondly, Section
4(f) of R.A. No. 7875, as amended by R.A. No. 9241,25 enumerates who are the legal dependents, to
wit: (a) the legitimate spouse who is not a member; (b) the unmarried and unemployed legitimate,
legitimated, illegitimate, acknowledged children as appearing in the birth certificate; legally
adopted or step-children below 21 years of age; (c) children who are 21 years old and order but
suffering from congenital disability, either physical or mental, or any disability acquired that
renders them totally dependent on the member of our support; and (d) the parents who are 60
years old or older whose monthly income is below an amount to be determined by the Philippine
Health Insurance Corporation in accordance with the guiding principles set forth in Article I of R.A.
No. 7875. And, thirdly, Section 2(f) of Presidential Decree No. 1146, as amended by R.A. No.
8291,dependent for support upon the member or pensioner; (b) the legitimate, legitimated, legally
adopted child, including the illegitimate child, who is unmarried, not gainfully employed, not over
the age of majority, or is over the age of majority but incapacitated and incapable of self-support
due to a mental or physical defect acquired prior to age of majority; and (c) the parents dependent
upon the member for support.1âwphi1
It is clear from these statutory definitions of dependent that the civil status of the employee as
either married or single is not the controlling consideration in order that a person may qualify as
the employee’s legal dependent. What is rather decidedly controlling is the fact that the spouse,
child, or parent is actually dependent for support upon the employee.

375
The application of the prohibition against the diminution of benefits presupposes that a company
practice, policy or tradition favorable to the employees has been clearly established; and that the
payments made by the employer pursuant to the practice, policy, or tradition have ripened into
benefits enjoyed by them.30 To be considered as a practice, policy or tradition, however, the giving
of the benefits should have been done over a long period of time, and must be shown to have been
consistent and deliberate.31 It is relevant to mention that we have not yet settled on the specific
minimum number of years as the length of time sufficient to ripen the practice, policy or tradition
into a benefit that the employer cannot unilaterally withdraw.
WHEREFORE, the Court AFFIRMS the decision promulgated on February 5, 201 0; and ORDERS
petitioner to pay the costs of suit.

RAMON G. NAZARENO, PETITIONER,


VS.
MAERSK FILIPINAS CREWING INC., AND ELITE SHIPPING A/S, RESPONDENTS.

FACTS

The factual and procedural antecedents are as follows:

Ramon G. Nazareno was hired by Maersk Filipinas Crewing Inc. (MCI) as Chief Officer for and in
behalf of its foreign principal Elite Shipping A/S (Elite) on board its vessel M/V Artkis Hope .
On March 25, 2001, the vessel was berthed at Port Belem, Brazil to load timber. While petitioner
was checking the last bundle of timber to be loaded, he suddenly lost his balance and fell at a height
of two (2) meters. He landed on the timber and injured his right shoulder. Due to the pain he felt in
his right shoulder, he was later examined at Philadelphia, U.S.A. and was considered not fit for
work. It was recommended that petitioner should be confined for thorough evaluation and further
tests, such as MRI. Petitioner was also advised to see an Orthopedic Surgeon and/or a Neurologist.3
However, petitioner was not permitted to disembark as there was no one available to replace him.
The same finding was told in Ulsan, South Korea when petitioner was brought at the Ulsan Hyundai
Hospital where he was treated and given medication for his "frozen right shoulder."4
In Manila, he was examined by the company physician who said he ws fit to work. However, upon
further consultation, one Dr. Santiago concluded that petitioner will no longer be able to function as
in his previous disease-free state and that his condition would hamper him from operating as chief
officer of a ship. Meanwhile, petitioner was also examined by Dr. Efren R. Vicaldo who, in a Medical
Certificate8 dated January 29, 2002, diagnosed petitioner to be suffering from Parkinson’s disease
and a frozen right shoulder (secondary), with an "Impediment Grade VII (41.8%). He concluded
that petitioner is unfit to work as a seafarer.
On the basis of the findings of his doctors, petitioner sought payment of his disability benefits and
medical allowance from respondents, but was refused. Petitioner therefore instituted the present
Complaint9 against the respondents docketed as NLRC OFW Case No. (M) 02-03-0660-00.

376
ISSUES
THE HONORABLE COURT OF APPEALS COMMITTED GRAVE ERROR IN REVERSING AND SETTING
ASIDE THE DECISIONS OF BOTH THE LABOR ARBITER A QUO AND THE NATIONAL LABOR
RELATIONS COMMISSION FINDING PETITIONER ALREADY UNFIT TO WORK AS A RESULT OF THE
INJURY HE SUSTAINED DURING THE ACCIDENT ON BOARD THE RESPONDENT’S VESSEL AND
THEREFORE ENTITLED TO DISABILITY BENEFITS.

RULING
The petition is meritorious.
The Court pronounced citing the following case:
Respecting the appellate court's ruling that it is POEA Memo Circular No. 55, series of 1996 which is
applicable and not Memo Circular No. 9, series of 2000, apropos is the ruling in Seagull Maritime
Corporation v. Dee involving employment contract entered into in 1999, before the promulgation of
POEA Memo Circular No. 9, series of 2000 or the use of the new POEA Standard Employment
Contract, like that involved in the present case. In said case, the Court applied the 2000 Circular in
holding that while it is the company-designated physician who must declare that the seaman
suffered permanent disability during employment, it does not deprive the seafarer of his right to
seek a second opinion which can then be used by the labor tribunals in awarding disability
claims.24
The case of Maunlad Transport, Inc. v. Manigo, Jr.28 is also worthy of note. ,,,,”in submitting himself
to examination by the company-designated physician, a claimant does not automatically bind
himself to the medical report issued by the company-designated physician; neither are the labor
tribunals and the courts bound by said medical report. Its inherent merit will be weighed and duly
considered. Moreover, the claimant may dispute the medical report issued by the
companydesignated physician by seasonably consulting another physician. The medical report
issued by said physician will also be evaluated by the labor tribunal and the court based on its
inherent merits.29
In the recent case of Daniel M. Ison v. Crewserve, Inc., et al.,30 although ruling against the claimant
therein, the Court upheld the abovecited view and evaluated the findings of the seafarer’s doctors
vis-à-vis the findings of the company-designated physician. A seafarer is, thus, not precluded from
consulting a physician of his choice. Consequently, the findings of petitioner’s own physician can be
the basis in determining whether he is entitled to his disability claims.
In any case, the bottomline is this: the certification of the companydesignated physician would
defeat petitioner’s claim while the opinion of the independent physicians would uphold such claim.
In such a situation, the Court adopts the findings favorable to petitioner. The law looks tenderly on
the laborer. Where the evidence may be reasonably interpreted in two divergent ways, one
prejudicial and the other favorable to him, the balance must be tilted in his favor consistent with
the principle of social justice.34
Anent the question of whether or not petitioner is indeed entitled to disability benefits based on the
findings and conclusions, not only of his personal doctors, but also on the findings of the doctors
whom he consulted abroad, the Court rules in the affirmative.

377
GR No. 175773 June 17, 2013

MITSUBISHI MOTORS PHILIPPINES SALARIED EMPLOYEES UNION (MMPSEU)


VS.
MITSUBISHI MOTORS PHILIPPINES INCORPORATED

The CBA between MMPSEU and Mitsubishi Motors (Mitsubishi) provides that the Mitsubishi will
shoulder the hospitalization expenses of the dependents of covered employees subject to certain
limitations and restrictions. Employees pay part of the hospitalization insurance premiums through
monthly salary deduction.
The conflict arose when a portion of the hospitalization expenses of the covered employees’
dependents were paid/shouldered by the dependent’s own health insurance. While the company
refused to pay the portion of the hospital expenses already shouldered by the dependents’ own
health insurance, MMPSEU insists that the covered employees are entitled to the whole amount
representing hospital expenses.
Mitsubishi’s continued refusal led to MMPSEU referring the case to the NCMB and requested for
preventive mediation.
MMPSEU: nothing in the CBA prohibits the employees from obtaining other insurance or declares
that medical expenses can be reimbursed only upon presentation of original receipts. Mitsubishi
would be unjustly benefited from the monthly premiums contributed by the employees though
salary deduction.
Mitsubishi: reimbursements of entire amounts being claimed, including that already paid by other
insurance companies would constitute to double indemnity or double insurance which is
circumscribed under the Insurance Code. A contract of insurance is a contract of indemnity and the
employees cannot be allowed to profit from their dependent’s loss.
MMPSEU acquired an opinion from the Claims Adjudication Division of the Insurance Commission:
in cases of claims for reimbursement of medical expenses where there are two contracts providing
benefits to that effect, recovery may be had on both simultaneously. Absent any provision from the
other insurance, courts uniformly hold that an insured is entitled without regard to the amount of
total benefits provided by other insurance. This is consistent with public policy underlying the
collateral source rule where courts have usually concluded that the liability of a health or accident
insurer is not reduced by other possible sources of indemnification or compensation.
Voluntary Arbitrator: in FAVOR of MMSEU relying heavily on the Insurance Commission’s opinion.
Court of Appeals: REVERSED the Voluntary Arbitrator. Despite the lack of a provision which bars
recovery in case of payment by other insurers, the wordings of the subject provision of the CBA
shows that the parties intended to make Mitsubishi liable only for expenses actually incurred by an
employee’s qualified dependent. The provision stipulates that payment should be made directly to
the hospital and that the claim should be supported by actual hospital and doctor’s bills. These
mean only amounts not covered by other health insurance will be paid. This is more consistent with
the principle of indemnity of insurance contracts. A contrary interpretation would allow
unscrupulous employees to unduly profit from the benefits and shall open the floodgates to
questionable claims.
MMPSEU then petitions the Supreme Court.

378
Issue:
Whether or not MMPSEU is entitled to the full amount medical expenses even with the
indemnification from other insurance of qualified employees’ dependents.

Held:
Supreme Court: petition has NO MERIT.
Collateral Source Rule: applicable only to place the responsibility for losses on the party causing
them. Its application is justified so that the wrongdoer should not benefit from the expenditures
made by the injured party or take advantage of contracts or other relations that may exist between
the injured party and third persons.
In this case, Mitsubishi is a no-fault insurer. It cannot then be obliged to pay the hospitalization
expenses of the dependents of its employees already paid by separate health insurance providers.
The conditions set in the CBA indicates the intention to limit Mitsubishi’s liability only to actual
expenses incurred by the employees’ dependents, excluding the amounts paid by dependents’ other
health insurance providers. SC agrees with the CA that the condition to pay directly to the hospital
and doctor implies to limit Mitsubishi’s liability to pay medical expenses actually shouldered by the
employees’ dependents. The condition is intended to thwart not only fraudulent claims but also
double claims for the same loss of the dependents of covered employees.
To allow reimbursement of amounts paid under other insurance policies shall constitute double
recovery which is not sanctioned by law. To constitute unjust enrichment, it should be shown that a
party was unjustly enriched in the sense that the term unjustly could mean illegally or unlawfully.
The CBA provides for Mitsubishi’s limited liability which extends only to the amount paid to the
hospital and doctor by the employees’ dependents, excluding that paid by other insurers.
Consequently, covered employees will not receive more than what is due them; neither is
Mitsubishi under any obligation to give more than what is due under the CBA.
Since the subject CBA provision is an insurance contract, the rights and obligations of parities must
be determined in accordance with the general principles of insurance law. Being a non-life
insurance contract and essentially a contract of indemnity, the CBA obligates only expenses which
are actually incurred. This is consistent with the principle of indemnity which proscribes the
insured from recovering greater than the loss.

G.R. No. 186509

PHILMAN MARINE AGENCY, INC. (NOW DOHLE-PHILMAN MANNING AGENCY, INC.) AND/OR
DOHLE (10M) LIMITED, PETITIONERS,
VS.
ARMANDO S. CABANBAN, RESPONDENT.

FACTS:
On September 15, 2002, respondent entered into a nine-month contract of employment with
DOHLE, through its local agent PTCI. He was assigned to work as a 2nd mate on board the vessel
“INGA-S.” On Sept. 9, 2002, Armando underwent the requisite pre-employment medical
examination (PEME) at PTCI’s accredited medical clinic, which found him fit for sea service. During
his medical examination, he declared that he had no history of high blood pressure and heart

379
trouble, and had not previously consulted any doctor relative to any disease. Armando was
deployed on Oct. 14, 2002.
On Feb. 9, 2003, while on board the vessel “INGA-S,” Armando felt dizzy and complained of chest
pain. He was immediately brought to the Fujairah Port Clinic, UAE, and was admitted to the
Coronary Care Unit after an initial diagnosis of “Unstable Angina.” The final diagnosis of Armando’s
illness stated Microvascular Unstable Angina Class III B established on medical treatment. One of
the recommendations was the Repatriation of Armando on Medical ground. Armando arrived in the
Philippines on Feb. 23, 2003, he proceeded to Dr. Natalio Alegre II, PTCI’s company designated
physician, at St. Luke’s Medical Center. His condition was closely monitored by Dr. Alegre, he
underwent various laboratory tests. ON May 12, 2003, Dr. Alegre declared Armando “fit to work”.
Despite the certification of Dr. Alegre as to Armando’s fitness to resume work, Armando
nevertheless claimed otherwise. In a letter dated June 25, 2003, Armando demanded from PTCI
payment of permanent disability benefits under the Philippine Overseas Employment Agency
Standard Employment Contract (POEA-SEC).
The petitioners did not heed Armando’s demand, prompting Armando to file, on July 4, 2003, a
complaint against the petitioners for injury/illness compensation benefit under a disability grade of
7, according to the POEA- SEC, in the amount of US$20,900.00. In the complaint, he indicated
“Coronary Artery Disease” (CAD) as the ground for his claim for disability benefits. Armando also
sought payment of the balance of his sickness allowance equivalent to two months,
unpaid/underpaid salary amounting to US$966.00, vacation leave pay, sick leave pay, moral and
exemplary damages, and attorney’s fees. On September 9, 2003, Armando amended his complaint
to include “hypertension, hyperlipidemia, obesity and alcoholism” as grounds for his disability
benefits claim.
Armando consulted various doctors which diagnosis him with hypertension and coronary heart
disease and similarly gave him a disability grade of “7” based on the POEA disability grading
schedule under the POEA-SEC. Armando subsequently presented these medical certificates before
the LA.
LA dismissed Armando’s claims except for the balance of the latter’s sickness allowance in the
amount of P68,560.30. In ruling for the petitioners, the LA declared that the petitioners had fully
complied with their liabilities to Armando for the work-related injury/illness suffered by the latter
during the term of the contract, pursuant to the POEA-SEC. Armando appealed the decision with
the NLRC. In its February 29, 2008 decision, the NLRC dismissed Armando’s appeal for lack of
merit. Armando filed with the CA a petition for certiorari under Rule 65 of the Rules of Court.
The Ruling of the CA
In its December 10, 2008 decision, the CA reversed the NLRC’s decision and ordered the petitioners
to pay Armando the following: (1) total and permanent disability benefits in the amount of
US$20,900.00 at its peso equivalent at the time of actual payment; (2) the balance of the sickness
allowance in the amount of US$2,189.60 at its peso equivalent at the time of actual payment; and
(3) attorney’s fees.
Petitioners filed a motion for reconsideration but denied by CA. Hence, this petition.

ISSUE:

380
WON Armando is entitled to total and permanent disability benefits on account of his medical
condition?

RULING:
The Court ruled in the negative.
The entitlement of a seafarer on overseas employment to disability benefits is governed by the
medical findings, by law and by the parties’ contract. By law, the governing provisions are Articles
191 to 193, Chapter VI (Disability Benefits) of the Labor Code, in relation to Section 2, Rule X of the
Rules and Regulations Implementing Book IV of the Labor Code. By contract, the provisions of the
POEA-SEC incorporating Department Order No. 4, series of 2000 of the Department of Labor and
Employment (the POEA-SEC) govern.
Since the present controversy centers on Armando’s claim for total permanent disability, we find it
necessary to define total and permanent disability as provided under Article 192(3)(1) of the Labor
Code:
(3) The following disabilities shall be deemed total and permanent:
(1) Temporary total disability lasting continuously for more than one hundred twenty days, except
as otherwise provided for in the Rules[.] [emphasis ours]
In relation to this Labor Code provision, we also refer to Section 2, Rule X of the Rules and
Regulations Implementing Book IV of the Labor Code:
SEC. 2. Period of entitlement – (a) The income benefit shall be paid beginning on the first day of
such disability. If caused by an injury or sickness it shall not be paid longer than 120 consecutive
days except where such injury or sickness still requires medical attendance beyond 120 days but
not to exceed 240 days from onset of disability in which case benefit for temporary total disability
shall be paid.
However, the System may declare the total and permanent status at any time after 120 days of
continuous temporary total disability as may be warranted by the degree of actual loss or
impairment of physical or mental functions as determined by the System. [emphases ours]
By contract, pertinent to the issue of compensability in the event of the seafarer’s illness or
disability is Section 20-B of the POEA-SEC. It reads:
SECTION 20. COMPENSATION AND BENEFITS
xxxx
B. COMPENSATION AND BENEFITS FOR INJURY OR ILLNESS
The liabilities of the employer when the seafarer suffers work- related injury or illness during the
term of his contract are as follows:
xxxx
3. Upon sign-off from the vessel for medical treatment, the seafarer is entitled to sickness allowance
equivalent to his basic wage until he is declared fit to work or the degree of permanent disability
has been assessed by the company-designated physician but in no case shall this period exceed
one hundred twenty (120) days.
For this purpose, the seafarer shall submit himself to a post- employment medical examination by a
company-designated physician within three working days upon his return except when he is
physically incapacitated to do so, in which case, a written notice to the agency within the same

381
period is deemed as compliance. Failure of the seafarer to comply with the mandatory reporting
requirement shall result in his forfeiture of the right to claim the above benefits.
If a doctor appointed by the seafarer disagrees with the assessment, a third doctor may be
agreed jointly between the Employer and the seafarer. The third doctor’s decision shall be
final and binding on both parties. [emphases ours]
Section 20-B of the POEA-SEC, in plain terms, laid out two primary conditions which the seafarer
must meet in order for him to claim disability benefits – that the injury or illness is work-related
and that it occurred during the term of the contract. It also spelled out the procedure to be followed
in assessing the seafarer’s disability - whether total or partial and whether temporary or
permanent - resulting from either injury or illness during the term of the contract, in addition to
specifying the employer’s liabilities on account of such injury or illness.
When read together with Articles 191 to 193, Chapter VI (Disability Benefits) of the Labor Code and
Section 2, Rule X of the Rules and Regulations Implementing Book IV of the Labor Code, and
following our various pronouncements, Section 20-B of the POEA-SEC evidently shows that it is the
company-designated physician who primarily assesses the degree of the seafarer’s disability. Upon
the seafarer’s repatriation for medical treatment, and during the course of such treatment, the
seafarer is under total temporary disability and receives medical allowance until the company-
designated physician declares his fitness to work resumption or determines the degree of the
seafarer’s permanent disability - either total or partial. The company-designated physician should,
however, make the declaration or determination within 120 days, otherwise, the law considers the
seafarer’s disability as total and permanent and the latter shall be entitled to disability benefits.
Should the seafarer still require medical treatment for more than 120 days, the period granted to
the company-designated physician to make the declaration of the fitness to work or determination
of the permanent disability may be extended, but not to exceed 240 days. At anytime during this
latter period, the company-designated physician may make the declaration or determination: either
the seafarer will no longer be entitled to any sickness allowance as he is already declared fit to
work, or he shall be entitled to receive disability benefits depending on the degree of his permanent
disability.
The seafarer is not, of course, irretrievably bound by the findings of the company-designated
physician as the above provisions allow him to seek a second opinion and consult a doctor of his
choice. In case of disagreement between the findings of the company-designated physician and the
seafarer’s appointed physician, the parties shall jointly agree to refer the matter to a third doctor
whose findings shall be final and binding on both.
In the present petition, the petitioners’ designated physician – Dr. Alegre – declared Armando fit for
sea service on May 12, 2003 or 92 days from the time he disembarked or signed off from the vessel
on February 10, 2003. As defined under Article 192(c)(1) of the Labor Code, total and permanent
disability means total temporary disability lasting for more than 120 days (unless the seafarer is
still under treatment up to a maximum period of 240 days as the Court held in Vergara v.
Hammonia Maritime Services, Inc.). While Armando was initially under temporary total disability,
Dr. Alegre declared him fit to work well within the 120-day mark.
Armando is entitled to sickness allowance only until the company- designated physician declared
him fit to work

382
To recall, the company-designated physician declared Armando fit to work on May 12, 2003.
Armando disembarked or signed/off from the vessel on February 10, 2003. Thus, following our
discussion above and pursuant to Section 20-B, paragraph 3 of the POEA-SEC, Armando’s sickness
allowance should be counted only at 92 days, that is from February 10, 2003 when he disembarked
form the vessel, until May 12, 2003 when Dr. Alegre declared him fit to work.
As a final note, while the Court adheres to the principle of liberality in favor of the seafarer in
construing the POEA-SEC, it cannot allow claims for compensation based on surmises. Liberal
construction is not a license to disregard the evidence on record or to misapply our laws.

383

Anda mungkin juga menyukai